You are on page 1of 241

12/15/09

1. Febrile neutropenia (chemotherapy for tumor patient) is a medical emergency. Admit the
patient to the hospital, obtain blood cultures, then start broad-spectrum antibiotics such as IV
ceftazidime or cefepime. The most common site of mucositis si the gastrointestinal tract, and
the frequently identified organism are G-, Particular P.aeruginosa.
2. In patient with acute ‘mechanical’ back pain (disc herniation) without significant neurologic
deficit, conservative approach is preferred for a period of 4-6 weeks. This includes early
mobilization, muscle relaxants, and NSAIDs. Best rest and physical therapy has not been
shown to be help. If the pain persists after 4-6 weeks of conservative treatment or progressive
neurologic deficit evolves, high-resolution diagnosis modalities are usually employed: MRI
and CT with or without contrast myelography.
3. Advanced age is probably the single most important risk factor for breast cancer. That is why
women of more than 40-50 years of age would need regular annual mammograms.
Interestingly, only 10 percent of women diagnosed with breast cancer have a positive family
history.
4. When questionable ulcer is visualized by flexible sigmoidoscopy, a biopsy is recommended
for two purposes: (1) to rule out possible cancerous lesions, and (2) distinguish ulcerative
colitis from Crohn’s disease, since the treatment of each given conditions is different.
5. Mitral valve prolapse (MVP) typically causes a mid-systolic click over the cardiac apex, with
a short systolic murmur at the apex after the click if mitral regurgitation is present. Squatting
decreases the murmur. MVP is sometimes associated with chest pain, anxiety, palpitations,
and/or hayperventilation.
6. The most common cause of overflow incontinence in elderly males is an enlarged prostate,
which is usually revealed by rectal examination, and is characterized by a high post-void
residual volume.
7. The typical symptoms of aortic stenosis are exertional dyspnea, syncopy, and angina. The
physical exam will reveal a systolic ejection murmur radiating to the apex and carotid arteries.
8. A prolonged QRS interval (i.e., typically bundle branch block) suggests a bradyarrhythmia,
whereas a prolonged QT interval suggests a tachyarrhythmia.
When the heart rate is lower than 30bmp, cerebral perfusion is impaired, even if there is a
compensatory increase in stroke volume. A patient with a disease of the sinus node (SSS) or
conduction system can have intermittent bradycardia and typically manifest with recurrent
syncopy episodes. Ventrical beats, by themselves, are not associated with syncope. The most
important differential diagnosis of syncope is generalized seizures. Distinguishing features of
a seizure include the presence of sustained clonic-tonic movement, tongue biting, and
postictal confusion.
9. Palliate radiation, along with anti-androgen therapy, is the treatment of choice for metastatic
prostate cancer. Anti-androgen therapy consists of Leuprolide (LHRH analogue). Flutamide is
considered inferior to LHRH analogues.
10. While effective in the treatment of many bacterial infections, aminoglycosides can also cause
nephrotoxicity and ototoxicity. Gentamicin sometimes causes severe vestibulotoxicity,
resulting in potentially permanent vertigo and ataxia.
Benign positional vertigo occurs when calcium crystals within the inner ear shift position.

1
Patients present with recurrent, acute-onset vertigo that is precipitated by head movement.
Nystagmus, nausea and vomiting are common associated findings.
11. Suspect alpha-a antitrypsin (AAT) deficiency in non-smoking adults <45 years of age with
panlobular emphysema on chest x-ray. Patients with AAT deficiency are also at risk for liver
disease (neonatal hepatitis, cirrhosis, and/or liver failure). Measurement of the serum alpha-
antitrypsin level is minimally invasive but can be highly diagnosis.
12. Acute mitral regurgitation can occur due to papillary muscle dysfunction in patients with
acute myocardial infarction. Acute mitral regurgitation characteristically causes a rise in left
atrial pressure without significant changes in left atrial size or left ventricular size or left
ventricular ejection fraction.
13. In patient presenting with significant peripheral vascular disease, calcium channel blockers
are preferred as anti-hypertensive agents.
14. Normally, tachypnea is the mode of respiratory compensation for metabolic acidosis, as it
allows for increased CO2 losses in exhale air. Relative hypoventilation in patients with COPD
can impair this respiratory compensation. (cause lathargy).
Winter’s formula: pCO2=1.5[HCO3]+8
Defective tubular bicarbonate reabsorption is the defect seen in type II renal tubular acidosis
(RTA). Type II RTA is oftentimes inherited, and may be a component of Fanconi syndrome. Type
II RTA can also occur with use of drugs like carbonic anhydrase inhibitors.
Low tubular ammonium production is a component of Type I RTA. The primary defect in type
I RTA is failure to excrete sufficient H+ in the urine. Without sufficient hydrogen ions in the
tubular fluid, ammonium cannot be produced.
15. Atrial flutter is recognized by a sawtooth pattern on EKG. In atrial flutter, the ventricular rate
is determined by AV node conduction. A 2:1, 3:1 or 4:1 heart block is often seen. (The
symptom is not caused by atrial ectopy—P wave).
16. The serum albumin level should always be measured simultaneously with the serum calcium
level in order to calculate the correct total serum calcium value. With every 1 g/dL change in
serum albumin, serum calcium changes by 0.8 mg/dL.
17. Suspect multiple myeloma in an elderly patient with anemia, renal failure, and hypercalcemia.
Dx: Serum immunoelectrophoresis. Sx: back pain—most common
18. Indicators of a severe asthma attack include normal to increase PCO2 values (indicates CO2
retention because of severe airway obstruction (air trapping) and /or respiratory muscle
fatigue.), speech difficulty, diaphoresis, altered sensorium, cyanosis, and ‘silent’ lungs.
Lung hyperinflation and tachycardia are typical findings during an asthma attack and not
indicators of severity.
19. Typical and atypical antipsychotics are good for treating acute agitation in elderly and
demented patients. Atypical antipsychotics like quetiapine and risperidone may also be used.
It is important to know that prolonged use of both typical and atypical antipsychotics can
increase mortality in elderly, but they are safe in the acute setting. (Lewy body dementia is
one exception where typical antipsychotics should not be used, as these patients tend to be
very sensitive to these medicines)
Lorazepam and other benzodiazepines may be used to treat agitation in young patients. They
are also beneficial in treating alcohol withdrawal. However, benzodiazepines are typically
contraindicated in older patients for several reasons. First, older patients tend to metabolize these

2
drugs slowly, making their effects very long-lasting. Plus, in the elderly, benzodiazepines often
worsen confusion. Finally, the elderly are at increase risk for all adverse events associated with
benzodiazepines including withdrawal, dependence, and motor impairment.
Memantine is an agent used to treat moderate to severe Alzheimer’s disease. It works by
blocking glutamate’s actions on the NMDA receptor.
20. Anticoagulation with warfarin places patients at risk for hemorrhage. Retroperitoneal
hematoma may occur even without a supratherapeutic INR. Back pain and signs and
symptoms of hemodynamic compromise should raise suspicious for retroperitoneal
hematoma. (know CT scan)
21. The diagnosis of acute limb ischemia should be considered in patients who present with the
‘five’ P’s: pain, pulselessness, paresthesias, poikilothermia (coldness), and pallor.
Angiography will typically show an abrupt cutoff of arterial blood flow. IV heparin should be
started immediately upon suspicion. The definitive treatment is surgical embolectomy or
intra-arterial fibronolysis/mechanical embolectomy via inteventional radiology. Streotokinase
is a fibronolytic agent. While IV infusion of a fibrinolytic agent is an option for STEMI
patients, patients with acute limb ischemia typically are treated with direct intra-arterial, not
IV, administration of a fibrinolytic agent on the clot using an angiographic catheter.
22. Staphylococcal scalded skin syndrome is a disease of children. It presents with sudden onset
diffuse erythema, skin tenderness, fever, flaccid bullae, facial edema and perioral crusting.
Toxic epidermal necrolysis is more benign than staph scalded skin syndrome. Hot, tender,
erythematous morbilliform or discrete macules and loose patches of skin are characteristic. The
oral mucosa is frequently involved, with blistering and erosions. Patients give a history of taking
sulfonamides, phenytoin, barbiturates, or NSAIDs.
Scarlet fever is a group A beta-hemolytic Strep infection. Exudative tonsillitis precedes the
condition. The rash typically appears as scarlet macules over generalized erythema.
Erysipelas begin as a small erythematous patch that progresses to a red, indurated, tense and shiny
plaque. It usually occurs over the cheek, and there is often a history of trauma or pharyngitis. The
presence of ‘raised, sharply demarcated’ advancing margins is a classic feature. Local signs of
inflammation are universal. Overlying skin streaking and regional lymphadenopathy indicate
lymphatic involvement. Over 80% of the cases are due to Strep, thus penicillin is the drug of
choice.
Impetigo is a contagious infection of the skin that is caused by Staph, or strep or both. The lesions
appear as macules, vesicles, and bullae with honey colored crusts.
23. Acyclovir can cause crystalluria with renal tubular obstruction during high-dose parenteral
therapy, especially in inadequately hydrate patients.
24. Eczema herpeticum is a form of primary herpes simplex virus infection associated with atopic
dermatitis. Numerous vesicles over the area of atopic dermatitis are typical. The infection can
be life-threatening in infants; thus prompt treatment with acyclovir should be initiated.
25. The known risk factors for the development of pancreatic cancer include: family history,
chronic pancreatitis, smoking, DM, obesity, and a diet high in fat. Alcoholism is not a risk
factor pancreatic cancer.
26. 3 clinical findings are characteristic of aortic dissection: abrupt onset ‘tearing’ pain in the
chest or back, variation in pulse or blood pressure between the right and left arms, and
widened mediastinum on CXR. When two or more of these findings are present, the

3
incidence of dissection exceeds 80%.
27. Iron deficiency anemia is one of the most common presentations of celiac sprue (intermittent
abdominal distention, flatulence, greasy stools—malabsorption syndrom). Up to 24% of cases
of celiac disease is associated with dermatitis herpetiformis.
28. OHS (Obesity hypoventilation syndrome) is defined by severe obesity (>150% of ideal body
weight) and alveolar hypoventilation during wakefulness. The ABG will demonstrate
hypercapnia, hypoxemia and respiratory acidosis (pH 7.3, pO2 60mmHg, pCO2 69 mmHg) as
a consequence of decreased lung compliance. Weight loss, ventilator support, oxygen therapy,
and progestins (a respiratory stimulant) are all potential therapies for these patients.
29. The basic pathology in myasthenia gravis lies at the neuromuscular junction (that’s why it’s
called myasthenia) and is mediated by auto-antibodies against the acetylcholine receptor.
30. HIV patients are at high risk for tuberculosis. A positive PPD test (>5mm in HIV) require
prophylaxis with isoniazid (+pyridoxine) for 9 months. Alternatives to the use of isoniazid
include the use of pyrazinamide with rifamipin or rifabutin for 2 months. If a patients if
intolerant ot pyrazinamide, rifampin is given alone for 4 months. A combination of 3 or 4
drugs is used to treat active tuberculosis infection.
31. Situational syncope should be considered in the differential diagnosis of syncopal episodes.
The typical scenario would include a middle age or older male, who loses his consciousness
immediately after urination, or a man who loses his consciousness during coughing fits.
32. Hypertension in patients with thyrotoxicosis is predominantly systolic and caused by
hyperdynamic circulation.
33. False negative
34. Lymphogranuloma venereum (LGV) is a sexually transmitted disease caused by Chlamydia
thrachomatis serotypes L1,2 and 3. The disease starts to 1 to 4 weeks after initial contact and
manifests with generalized malaise, headaches and fever. A papule appears and subsequently
transforms into an ulcer, typically located in the vulvovaginal region. The ulcer is painless and
the disease may go unnoticed until inguinal adenitis develops about a month later. In women,
however, the lymphadenopathy involves more commonly the deep nodes around the rectum
and anus. If untreated at this stage, LGV progresses into a severe and chronic disease causing
ulceration, proctocolitis, rectal stricture, rectovaginal fistulas and elephantiasis.
Granuloma inguinale (GI) is a distinct condition that presents similarly but is due to Donovania
granulomatis. Unlike LGV, the ulcer and lymphadenopathy of granuloma inguinale present at the
same time. Moreover, the ulcer of GI has irregular borders and is characterized by a beefy red
granular base.
35. Once the specific causative bacterium responsible for native valve bacterial endocarditis is
identified, the antibiotic coverage can be narrowed from empiric to specific. Native valve
bacterial endocarditis due to Strep viridans highly susceptible to penicillin is well-treated with
IV penicillin G or IV ceftriaxone.
36. While most live vaccines (BCG, varicella, anthrax, oral typhoid, intranasal influenze, oral
polio, and yellow fever) are C/I in patients with HIV, the MMR vaccine is an exception and
can be given to patients who have CD4 counts greater than 200/mL and not evidence or
history of an AIDS-defining illness.
37. Hyperparathyroidism predisposes to the development of pseudogout. Pseudo tends to present
as an acute onset, painful monoarthritis affecting the knee. Rhomboid shaped, positively

4
birefringent crystals are diagnosic.
Struvite crystals have the shape of coffin lids and are seen in nephrolithiasis causes by chronic
urinary tract infection with urease-producing organisms.
38. The null hypothesis is always the statement of no relationship between the exposure and the
outcome. To state the null hypothesis correctly, you should recognize the study design first.
Cross-section: no association; cohort study: the same
39. The most important causes of thyrotoxicosis with low radioactive iodine uptake include: 1.
subacute painless thyroiditis; 2. subacute granulomatous thyroiditis—De Quervain’s, intense
pain; 3.iodine-induced thyroid toxicosis; 4.levothyroxine overdose; 5. Struma ovarii
40. Leydig cell tumors, the estrogen production can be increased with secondary inhitibition of
LH and FSH. High testosterone, high estrogen
41. Howell-Jolly bodies (single round, blue inclusions on Wright stain) are nuclear remnants
within RBC typically removed by the spleen. Their presence strongly suggests physical or
functional hyposplenism.
42. Hypertrophic cardiomyopathy is inherited in an autosomal dominat pattern.
43. Hypertension can be one of the presentating signs of polycythemia (know the various
presentations).
44. Steroid-induced myopathy is a well described result of long-term corticosteroid use. It is
characterized by painless proximal muscle weakness. Steroid myopathy will improve slowly
once the offending medication is discontinued. Serum Ck and ESR are normal. Insidious,
acute and chronic.

12/16/09
1. Lead-time bias should always be considered while evaluating any screening test. This bias
occurs when there is an incorrect assumption or conclusion of prolonged apparent survival
and better prognosis due to a screening test. What actually happens is that detection of the
disease was made at an earlier point in time, but the disease course itself or the prognosis
didn’t change. So the screened patients appeared to live longer from the time of diagnosis to
the time of death. (USMLE tip: think of lead-time bias when you see ‘a new screening test’
for poor prognosis diseased like lung or pancreatic cancer.)
2. The main causes of metabolic alkalosis include exogenous administration of alkali, removal
of acidic gastric secretions due to vomiting or NG tube aspiration, renal hydrogen ion loss due
to mineralocorticoid excess and contraction alkalosis.
3. Hyperemesis gravidarum typically causes metabolic alkalosis with respiratory compensation
via hypercapnia. PE causes a primary respiratory alkalosis due to hyperventilation.
4. Blood PH 7.43, PaO2 100mmHg, PaCO2 25mmHg, HCO3- 16 mEq/L. Aspirin intoxication
causes a mixed respiratory alkalosis and metabolic acidosis. Respiratory alkalosis results from
increased respiratory drive, while metabolic acidosis results from the accumulation of
salicylate and organic acids.
5. Memorize!
Lesion Manifestation
posterior limb of the unilateral motor hemiparesis of the face, arm and leg without any
internal capsule (lacunar higher cortical dysfunction and visual field abnormalities.
infact)
Middle cerebral artery contralateral hemiplegia, conjugate eye deviation toward the side

5
occlusion of the infarct, hemianesthesia and homonymous hemianopia.
*aphasia (dominant hemisphere), *hemineglect (non-dominant
hemisphere lesions).
Anterior cerebral artery contralateral weakness that predominantly affects the lower
occlusion extremity. Associated findings include abulia, akinetic mutism,
emotional disturbances, deviation of the head and eyes toward the
lesion, and sphincter incontinence.
vertebrobasilar system ‘alternate’ syndromes, with contralateral hemiplegia and ipsilateral
lesion (*supplying the brain cranial nerve involvement.
stem)

6. Toxic adenoma presents as symptoms suggestive of thyroid toxicosis. There is radioactive


iodine uptake in the nodule, and suppression of uptake in the rest of the thyroid gland. Patients
with toxic nodule do not have infiltrative ophalmopathy.
Patients with Hashimoto’s thyroiditis are hypothyroid or euthyroid. The thyroid scan shows
heteogenous pattern.
Grave’s –diffusely increased radioactive iodine uptake + infiltrative ophalmopathy+pretibial
myxoedema.
The radioactive iodine uptake in patients with painless thyroiditis is markedly reduced.
7. Esophagoscopy is indicated when a patient with GERD fails to repond to empiric treatment,
or when a patient had features of complicated disease.
8. Nephritic syndrome is characterized by dependent edema, hypertension, and hematuria
(dysmorphic red blood cells or red blood cell cast). Rash, low-grade fevers and proteinuria
may also be present.
9. Agioedema is characterized by the rapid onset non-inflammatory edema of the face, acral
extremities, genitals, trachea and abdominal organs. It is due to a deficiency in C1 esterase
inhibitor, which results in elevated levels of the edema-producing factors, C2b and
bradykinin.
Low C1q levels are associated with familial systemic lupus erythematous.
10. Fluphenazine is a high potency ‘typical’ antipsychotic medication that occasionally causes
hypothermia by disrupting thermoregulation and the body’s shivering mechanism. Patients
taking antipsychotics should be advised to avoid prolonged exposure to extreme temperatures.
11. Any male adolescent who presents with epistaxis (nosebleeds), a localized mass, and a bony
erosion on the back of the nose had an angiofibroma until proven otherwise. This is typically
found in the back of the nose or upper throat (nasopharynx). Reactive nasal polyps normally
do not cause bony erosions. These are usually associated with chronic infections or allergies,
and the main complaint is obstruction rather than bleeding.
12. Osteoarthritis is a non-inflammatory arthritis presenting with pain that is worse with activity
and improved with rest. X-ray findings include joint space narrowing and osteophytes.
Synovial fluid analysis will reveal fewer than 2000 WBC/ml, no organisms, and no crystals.
Gouty arthritis—punched out erosions with a rim of cortical bone. 2,000-50,000 WBC/ml,
negative birefringent crystals.
RA—periarticular osteopenia and joint margin erosions. 2,000-50,000 WBC/ml
Infectious arthritis—normal joint space with soft tissue swelling, >50,000 cells/ml Gram staining
Psudogout, or calcium pyrophosphate dyhydrate (CPPD) deposition—calsification of
cartilaginous structures (chondrocalcinosis).

6
13. indications for upper endoscopy in the management of GERD: 1-Nausea/vomiting; 2-Weight
loss, anemia or melena/blood in the stool; 3-Long duration of symptoms (>1-2 years), esp in
Caucasian males >45 years old; 4-Failure to respond to proton pump inhibitors.
14. Vitiligo is an autoimmune condition characterized by area of depigmentation lacking
melanocytes. Vitiligo is sometimes associated with other autoimmune conditions such as
pernicious anemia, autoimmune thyroid disease, type I diabetes mellitus, primary adrenal
insufficiency, hypopituitarism, and alopecia areata.
15. hemochromatosis is an autosomal recessive disorder characterized by increased skin
pigmentation, diabetes, cirrhosis and arthralgia in the later stages.
16. CDC vaccination schedules on a yearly basis:
Tetanus and diphtheria (Td): every 10 years after 18. A single tetanus, diphtheria, and acellular
pertussis (Tdap) booster between 19-64.
Influenze: all adults agen 50 years and older. It should also be given to all adults with chronic
cardiovascular, pulmonary, hepatic, renal, or metabolic (e.g.diabetes) disease, immunosupprssion,
or pregnancy. Healthcare workers, close contact of children aged 0-59 months, and those in
nursing homes should also be vaccinated. Healthy, non-pregnant persons aged 5-49 years may
receive the intranasal influenza vaccine.
Pneumococcal vaccine (PCV): all adults aged 65 years and older. It should also be given to all
adults with chronic cardiovascular, pulmonary, hepatic, renal, or metabolic (e.g. diatetes) disease,
or immunosuppression. Persons vaccinated before age 65 need a booster in 5 years.
17. Dubin-Johnson and Rotor syndrome are two familial disorders of hepatic bile secretion that
result in conjugated hyperbilirubinemia. A dark granular pigment is present in the hepatocytes
of patients with Dubin-Johanson, but not Rotor. For diagnosis of Dubin-Johnsom, conjugated
hyperbilirubinemia with a direct bilirubin fraction of at least 50% and an otherwise normal
liver function profile must be present. Confirmation can be obtained by evaluating the urinary
coproporphyrin for usually high levels of coproporphyrin I80% (normal 80% III) .
18. Fresh frozen plasma is the therapeutic agent of choice for the coagulopathy in patients with
liver failure.
19. Primary HIV infection causes a febrile illness that can closely resemble infectious
mononucleosis. The key distinctions: rash (unless antibiotics have been administered),
diarrhea—HIV; a tonsillar exudates –EBV
20. The sudden onset of vertigo, vomiting and occipital headache in a hypertensive patient is
strongly suggestive of cerebellar hemorrhage. Other manifestation are 6th nerve paralysis,
conjugate deviation, blepharospasm and coma.
Heat stroke—failure of body thermoregulation upon exposure to high environmental temperatures.
High core body temperature (>105F), dehydration, confusion, coma, and dry, flushed skin. “heat
exhaustion” less severe.
Vestibular neuronitis—acute onset of vertigo and nystagmus without any other neurological deficit
Meiiere disease—increase pressure of the endolymph, vertigo, tinnitus, hearing problem. Vomiting
is not prominent.
21. V/Q scan is a useful tool in diagnosis pulmonary thromboembolism. Mismatched perfusion
defect is characteristic, but it is found in less than 50% of the patients.
22. Actinic keratoses—sun –exposed, erythemarous papules with a central scale and a ‘sand
paper-like’ texture, squamous cell carcinoma.

7
Atopic dermatitis –light microscopy—spongiosis (edema of the epidermis)
23. The typical VitK deficiency –NPO for a prolonged period of time + broad spectrum
antibiotics. PT is elevated more than PTT.
Similar in newborn who had not received prophylactic vitK (home born)
24. Hypercalcemia and intractable ulceration –MEN syndromes
25. ARDS-acute onset, bilateral patchy airspace disease on CXR, PCWP<18 or no clinical
evidence of increase LVEDP, and PaO2/FiO2 <200.
26. Suspect rosacea in 30- to 60- year old pts with telangiectasia over the cheecks, nose and chin.
Flushing of these areas is typically precipitated by hot drinks, heat, emotion and other causes
of rapid body temperature changes. Topical antibiotic such as metronidazole is the most
frequently prescribed initial therapy. DD: Acne vulgaris, seborrheic dermatitis, carcinoid
syndrome, SLE
27. Infective endocarditis –progressive subacute fevers, chills, malaise, and dyspnea. It is
diagnosed using Duke’s criteria. Complications include vascular (i.e.embolic) and immune
complex phemomena.
28. P.jiroveci is an opportunistic pathogen, and an important cause of pneumonia in
immunocompromised hosts (chemotherapy). Bilateral diffuse interstitial infiltrates beginning
in the perihilar region is a characteristic finding on CXR.
29. Cohort study—incidence. Cross-section—prevalence.
30. TB CXR
31. Viral arthritis (parvovirus, hepatitis, HIV, mumps and rubella) can present with symmetric
small joint inflammatory arthritis. It is distinguished for other causes of symmetric
inflammatory arthritis by the fact that it tends to resolve with two months. Positive
inflammatory markers such as ANA and RF may occur. Tx involves use of NSAIDs for
resolution of symptoms. Antiviral therapy is unnecessary as the symptoms are self-limited.
32. Repeated nocturnal awakenings associated with dyspnea, cough and a chocking sensation, esp
in an overweight or obese pt.—GERD. Central sleep apnea—not classically choking, cough
33. De Quervain tenosynovitis –mother hold baby with thumb outstretched (abducted/extended)
34. Patient with an inferior MI may suffer right ventricle infarction, leading to right sided heat
failure. Hypotension, jugular venous distention and clear lung fields should raise suspicion for
right ventriclar infarction. Tx—IV fluids and avoidance of preload reducing medications such
as nitrates and diuretics.
35. Fatty liver (steatosis), alcoholic hepatitis and early fibrosis of the liver can be reversible with
the cessation of alcohol intake. True cirrhosis (with regenerative nodules) is irreversible,
regardless of alcohol abstinence.
36. Individual who received blood transfusions before 1992 should screen HCV; before 1986-
HBV.
37. Herpetic whitlow is the most common viral infection of the hand. HSV-1 &2, self-limiting.
Increase risk in health care workers.
38. Toxic megacolon—MCC is ulcerative colitis. Tx—IV steroids, nasogastric decompression
and fluid management.
39. A compression fracture of the vertebrae is a common complication of advanced osteoporosis
—acute back pain without an obvious preceding trauma in a predisposed patient. Neurologic
exam will be normal. Note: absent ankle reflex can be seen in elderly patients as part of the

8
normal aging process.
Lumbago is usually related to physical strain and has less dramatic onset. Usually paravertebral
muscle tenderness rather than spinal tenderness is the feature.
40. Rupture of the ventricular free wall is a mechanical complication of MI with peak incidence
3-7 days after anterior wall MI. Acute decompensation and PEA (pulseless electrical activity)
secondary to rapid development of tamponade is typical.
41. Digitalis toxicity –increase ectopy and vagal tone. Specific—Atrial tachycardia with AV block
42. Oral isotretinoin –moderate to severe acne that is predominantly nodulocystic form, and those
developed scars. High yield!!!
oral antibiotics (tetracyclines)—moderate to severe inflammatory acne.
Mild, inflammroty—topically benzyl peroxide, antibiotics (erythromycin, clindamycin,
tetracyclins
Initial, non-inflammatory comedones—topical retinoids
43. Ankylosing spondylitis—HLA B27
44. Primary hyperaldosteronism (Conn’s syndrome)—hypertension, mild hypernatremia,
hypokalemia and metabolic alkalosis. Dx—a low rennin level and an elevated aldosterone
level.

12/17
1. Acute vertigo (sensation of spinning of head) is often due to the dysfunction of the labyrinth.
BPPV (benign paroxysmal positional vertigo)—vertigo related to head positioning. Global
cerebral ischemia results in presyncope (lightheadedness with visual blurring, not spinning)
and syncope.
2. SLE—young woman with joint pain, fatigue, ankle edema and facial rash. Kidney disease—
immune complex-mediated, serum C3 level is decreased. IgE-mediated reaction is important
in allergic interstitial nephritis like methicillin-induced interstitial nephritis. Cytotoxic
antibodies cause Goodpasture’s syndrome. Delayed hypersenditivity reaction may be
important in non-specific chronic glomerulonephritis. Non-immune kidney damage may be
related to toxins, hypertension, diabetes mellitus or other factors.
3. MS usually presents during the 3rd or the 4th decade with recurrent focal neurologic
dysfunction. The attacks are non-predictable and erratic in presentation. The symptoms may
last a few weeks with variable recovery. MS is one of the few conditions that may present
with bilateral trigeminal neuralgia. Another clue is the presentation of optic neuritis.
4. Theophylline toxicity – CNS stimulation (headache, insomnia), GI disturbances (nausea,
vomiting), and cardiac toxicity (arrhythmia). The mechanisms responsible for the toxicity
may include phosphodiesterase inhibition, adenosine antagonism, and stimulation of
epinephrine release. * Remember the interaction with ciprofloxacin and erythromycin.
5. Pressors such as norepimephrine can cause ischemia of the distal fingers and toes sencondary
to vasospasm. The diagnosis is suggested by symmetric duskiness and coolness of all
fingertips. Raynaud’s phenomenon—pallor to cyanosis to erythema. Cholesterol emboli
—“blue toe syndrome”, symmetrical involvement of all digits unusual.
6. AF is the MCC chromic arrhythmia in adults. HR–irregularly irregular, rapid rate. Rhythm
control or rate control plus anticoagulation may be used to decrease the risk of embolic stroke
in patients with atrial fibrillation.

9
7. The first step in Tx fo acute variceal bleeding is to establish vascular access with two large
bore IV needles or a central line.
8. In HIV-infected Pt, diarrhea can be due to a variety of organisms. Therefore, and attempt
should be made to identify the causal organism prior to instituting therapy so that appropriate
antibiotic may be chosen.
9. Pt who have had an episode of rheumatic fever should receive antibiotic prophylaxis with
penicillin to prevent further attacks. The duration of treatment is not firmly established, but
most physicians will treat until the pt reaches the age of 18.
10. The ATP 3 (adult treatment Panel 3) recommends lifestyle and /or medications to decrease
the risk of coronary artery disease or coronary artery disease-related outcomes based upon
risk factors and LDL cholesterol levels.
Risk category LDL Goal LDL levels at which LDL level at which
lifestyle modification drug thrapy starts
starts
CAD or CAD risk <100 mg/dL ≥100 mg/dL ≥130 mg/dL
equivalent
2+ risk factors <130 mg/dL ≥130 mg/dL ≥160 mg/dL
0-1 risk factors <160 mg/dL ≥160 mg/dL ≥190 mg/dL
Risk factors: men ≥45 years, women ≥55 years hypertension,smoking, HDL < 40 mg/dL, and
family history of premature CAD (male <55, femail <65). HDL≥60 negates one risk factor. CAD
risk equivalent: DM, carotid artery disease, abdominal aortic aneurysm, peripheral arterial disease,
or a 10-year risk of CAD of ≥ 20%.
11. The classic triad of RCC (flank pain, hematuria, and a palpable abdominal renal mass) is
uncommon (10%), when present, it strongly suggests advanced/metastatic disease. Hematuria
is seen only in about 40% pt. Scrotal varicocele, (the majority are on left-side), may be
observed in a few Pt (<10%). Variceles typically fail to empty when the pt is recumbent.
Presence of this finding should always raise suspicion for a RCC, which is most commonly
due to obstruction of the gonadal vein where it enters the renal vein. 20% of pt may also have
constitutional symptoms like fever, night sweats, anorexia, weight loss, or an easy
fatigability. Increased erythropoietin by kidney mass can produce polycythemia and
thrombocytosis. CT scan of the abdomen is most sensitive and specific for diagnosis the
RCC.
12. Suspect Alport’s syndrome in pt with recurrent episodes of hematuria, sensorineural deafness
and a family history of renal failure. EM—alternating areas of thinned and thickened
capillary loops with splitting of the glomerular basement membrane.
13. Serum sickness-like reaction may occur 1-2 weeks after administration of certain drugs, such
as penicillin, amoxicillin or cefaclor in the setting of a viral illness. Prominent symptoms
include fever, urticarial rash, polyarthragia and lymphadenopathy. The condition derives its
name from its similarities to true serum sickness, an immune-complex mediated
huypersensitivity reaction to non-human proteins. Serum-sickness like reaction is clinical
diagnosis, and should resolve with withdrawal of the offending agent; it does not represent a
true drug allergy.
14. Light’s criteria may be used to distinguish transudative and exudative pleural effusions.
Exudates have either an effusion to serum total protein ratio greater than 0.5, an effusion to
serum LDH ratio greater than 0.6, or and effusion LDH concentration greater than 2/3 the

10
upper limit of normal for serum LDH. Complicated effusions (PH<7.2, glucose <60 mg/dL,
or gram stain or culture) require chest tube drainage.
15. Vanishing bile duct syndrome is a rare condition characterized by progressive ductopenia, or
loss of intrahepatic bile ducts. The most common cause of ductopenia in adults in primary
biliary cirrhoiss. Other causes include failing liver transplantation, hodgkin’s disease, graft-
versus-host disease, sarcoid, CMV infection, HIV, and medication toxicity. PSC—are
periductal portal tract fibrosis and segmental stenosis of extrahepatic and intrahepatic bile
ducts. Bile duct loss is not classically observed.
16. Isonizid causes idiosyncratic liver injury with histological features similar to those seen in pt
with viral hepatitis.
17. Tinea corporis is a superficial fungal infection characterized by an erythematous, scaly,
pruritic rash with central clearing. It’s treated with topical antifungals—terbinafine.
18. The appropriate blood product to administer for anemia is packed RBC. In pt with
preexisting heart disease, a hemoglobin level of 10g/L or higher
(hematocrit ≥30%) should be maintained. Blood is collected from donors as whole blood,
but hen is separated into packed RBC, platelets, fresh frozen plasma, and cyoprecipitate. FFP
contains all clotting factors and plasma proteins. Thansfusion of FFP is generally used to
reverse a coagulopathy (e.g. to rapidly lower a high INR in a patient taking wafarin).
Cryoprecipitate is rich in factor VIII, fibrinogen deficiency, and VWF disease. It’s very
useful in volume sensitive patients.
19. The MCC of hypernatremia is hypovolemia. Mild cases can be treated with 5% dextrose in
0.45% saline. Severe cases should be initially treated with 0.9% saline.
20. Oral trimethoprim-sulfamethoxazole is effective in preventing PCP infection in transplant pt.
These pts should also be vaccinated against influenza, pneumococcus, and Hepatitis B.
Ganciclovir or vilgancivlovir can be used to prevent CMV infections.
21. Elevated BNP (brain natriuretic peptide) levels and an audible S3 are both signs of increased
cardiac filling pressure. Brain BNP is secreted in response to ventrical stretch and wall
tension when cardiac filling pressures are elevated. Specifically, cardiac myocytes secrete the
BNP precursor, proBNP, which is then cleaved into biologically active c- terminal BNP and
inactive n-terminal proBNP. It is common practice in the emergency department to measure
the BNP level to differential dyspnea of the heart failure from dyspnea of non-cardiac
etiology. The level of circulating BNP correlates both with the severity of left ventricular
filling pressure elevation as well as with mortality. A low BNP level argues against acute
heart failure with a very high negative predictive value.
22. Sarcoidosis is diagnosis in the setting of a compatible history, CXR findings of hilar
adenopathy with or without reticulonodular infiltrates, and biopsy of noncaseating
granulomas, Elevated serum ACE levels also provides support for the diagnosis. For
symptomatic disease, administration of system glusocorticoids is indicated.
23. One of the known complications of ERCP is an iatrogenic biliary enteric fistula characterized
by the presence of air in the biliary tree. Other complications included pancreatitis, biliary
peritonitis, sepsis, hemorrhage, and adverse effects from the contrast, sedative, or
anticholinergic agents.
24. Chronic Myelogenous leukemia (CML) is characterized by leukocytosis, anemia, and
increased number of mature granulocytic forms, such as segmented neutrophils and band

11
forms on peripheral blood film. The disease is mostly seen after the age of 50 and presents
with fatigue, malaise, low-grade fever, anorexia, weight loss and bone pains. Night sweats
and fever associated with increased metabolism due to granulocytic cell tumor turn over may
occur. Examination of a bone marrow biopsy specimen reveals hypercellularity with
prominent granulocytic hyperplasia. The leukocyte alkaline phosphate score is low. The only
other diseases where this may happen are hypophosphatemia and paroxysmal nocturnal
hemoglobinuria (PNH).
Elevated leukocyte alkaline phosphatase is characteristic of Leukemoid Reaction. Presence of
Philadelphia chromosome and the low leukocyte alkaline phosphatase makes the CML more likely
than the leukemoid reaction.
Increased bone marrow iron is seen in hemachromatosis, anemia of chronic disease and
sideroblastosis.
Absent of measurable erythropoietin in Urine is an important diagnostic feature of polycythemia
vera.
Auer rods are seen in AML.
Tartrate resistant acid phosphatase is present in hairy cell leukemia.
25. Indications for hemodialysis: 1-refractory hyperkalemia; 2-vulume overload or pulmonary
edema not responding to diauretics; 3-refratory metabolic acidosis (PH<7.2); 4-Uremic
pericarditis; 5-Uremic encephalopathy or neuropathy; 6-coagulopathy due to renal failure.
26. Hypocalcemia can occur during or immediately after surgery in parients undergoing major
surgery and requiring extensive transfusions. Hperactive deep tendon reflexes may be the
initial manifestation.
27. papilledema usually presents with transient loss of vision lasting a few seconds with changes
in head position and can be confirmed with ophthalmologic examination. Norma individual
have a blind spot in there visual fields at the location of the optic nerve head, but this blind
spot enlarges in pts with papilledema. Papilledema is caused by increased intracranial
pressure, which has its own associated symptoms such as morning headaches or changes in
headache intensity with position.
Optic neuritis is commonly associated with multiple sclerosis, but can be seen with other disorders
as well. It typically presents as unilateral eye pain and visual loss with an associated afferent
papillary defect.
Amaurosis fugax is transient monocular blindness that lasts only a few minutes and is usually
vascular in origin. It would be unlikely in the is pt given young age and low risk of
atherosclerosis.
Uveitis can present with eye pain and redness if it is in the anterior chamber, or painless visual
changes secondary to floaters or reduced visual acuity if the posterior chamber is affected.
28. Vitreous hemorrhage typically presents as a sudden loss of vision and onset of floaters. It is
characterized by a fundus that is hard to visualize, an even if it visualized, details may be
obscured. It’s the MCC in diabetic retinopathy. For pt with underlying medical conditions,
conservative treatment (i.e. upright position during sleep, which enhances settling of the
hemorrhage) is recommended.
Retinal detachment—pt typically complain of photopsia with showers of floaters.
Central retinal vein occlusion presents as a sudden, painless, unilateral loss of vision. It is also
noted in pt with history of hypertension. Opthalmoscopic sighs are disk swelling, venous dilation

12
and tortuosity, retinal hemorrhage, and cotton wool spots.
Diabetic retinopathy pt are usually asymptomatic, even though changes in fundoscopy are seen.
Vitreous hemorrhage frequently occurs in pt with proliferative diabetic retinopathy. Once a
diabetic patients presents with a sudden onset of visual loss with numerous floaters, a vitreous
hemorrhage is most likely to have occurred.
Pt with macular degeneration typically present with painless progressive blurring of central vision,
which can be acute or insidious. It occurs bilaterally.
29. HAV is the most common vaccine-preventable disease among travelers. The HAV vaccine
should be considered for people anticipating travel to developing countries. CDC
recommends—HAV, HBV, typhoid vaccine as well as a polio booster vaccine, HAV—Asia
and African countries. Yellow fever—sub-Saharan African and equatorial South American
sountries. Rabies—outdoors or in rural areas, children.
30. Acute post-strep glomerulonephritis occurs 10-20 days after strep throat or skin infections. It
presents with hematuria, hypertension, red cell casts, and mild proteinuria.
31. CT scan of the head without contrasts is the initial diagnostic test of choice when a patient
presents with impaired consciousness or seizures or focal neurologic signs.
32. Spinal cord compression is characterized by signs and symptoms of upper motor neuron
dysfunction distal to the site of compression, such as weakness, hyperreflexia, decreased
sensation, and bowel/bladder dysfunction (saddle anesthesia and compromised bladder or
bowel control). Cord compression is a medical emergency requiring prompt diagnosis via
spinal MRI.
33. Vancomycin + ceftrixone—community-acquired bacterial meningitis in adults and children
since it covers the 3 most frequent etiologic agents: Strep. Pneumoniae, H. influenzae, and
Neisseria meningitides. Many pneumococcal stains have become resistant ot penicillin and
cephalosporins, thus, vancomycin is needed in addition to ceftriaxone. Vancomycin alone
does not sufficiently penetrate the blood-brain barrier.
Ampicillin is included in the empiric regimen to cover Listeria monocytogenes, which is also an
important cause of meningitis in pt older than 55. Other pt who are at risk include
immunocompromised pt, pt with malignancies (esp lymphoma), and pt taking corticosteroids.
IV cefotaxime + ampicillin – pt less than 3 months of age. Cefotaxime covers the 3 common
community-acquired pathogens and ampicillin covers Listeria. Although ceftrixone can be used in
neonates, cefotaxime is generally preferred because ceftrizxone have been associated with biliary
sludging.
IV ceftazidime+vancomycin –hospitalized pt who develop meningitis, esp after neurosurgery,
These drugs cover pseudomonas and Strap aureus respectively.
34. hypertestrogenism in cirrhosis leads to gynemastia, testicular atrophy, decreased body hair,
spider angiomas and palmar erythema.
35. Herpes zoster ophthalmicus is characterized by dendriform corneal ulcers and a vesicular
rash in the trigeminal distribution.
Herpes simplex keratitis presents with pain, photophobia, and decrease vision. Dendritic ulcer is
the most common presentation. There may also be minute clear vesicles in the corneal epithelium.
The typical vesicular rash of herpes zoster and system symptoms are not seen.
Dacryocystitis is an infection of the lacrimal sac due to obstruction of the nasolacrimal duct. It is
characterized by pain, swelling, tenderness, and redness in the tear sac area. Mucous or pus can be

13
expressed.
Bacterial keratitis is usually seen in contact lens wearers, and following corneal trauma. The
cornea appears hazy with a central ulcer and adjacent stromal abscess. Hypopyon may be present.
The history is the only factor by which a diagnosis of trigeminal neuralgia is made clinically. It is
characterized by a stabbing or shock-like pain over the distribution of the maxillary or mandubular
division of the trigeminal nerve. The pain may be triggered by light tough or vibration. Rash is not
seen.
36. Always suspect parenchymal brain hemorrhage whenever a patient presents with a history of
uncontrolled systemic hypertension and focal neurological signs.
37. Asymptomatic patients with Paget’s disease generally do not require any treatment.
Symptomatic pt are best treated with oral or IV biophosphonates (decrease bone resorption,
persists for months to years after tx is stopped. Superior to clacitonin—inhibits osteoclastic
bone absorption and reduces disease activity, weaker than bisphosphonates.)
Indications for tx: 1—bone pain; 2-Hypercalcemia of immobilization; 3-neurological deficit; 4-
high output cardiac failure; 5—preparatio ofr orthopedic surgery; 6—involvement of weight-
bearing bones (to prevent deformities).
38. HCV RNA is the single most sensitive serological marker used in screening marker used in
screening for HCV infection.
39. Chromic lymphocytic leukemia—LN biopsy confirms the diagnosis.
40. A essential tremor is characterized by a postural tremor (not a resting tremor unlike
Parkinson’s disease) and usually disturbs the performance of fine motor tasks. Sometimes, it
runs in families with AD inheritance. The pathophysiologic basis of the this condition is
unclear. The inhibition of the tremor by a small amount of alcohol is typical. Propranolol, a
non-selective beta-blocker, is effective in reducing the tremor probably by blocking beta-2
receptors.
41. Atracurium is a neuromuscular blocking agent that is metabolized in plasma and hydrolyzed
by serum esterases. Its use is safe in pt with renal and liver dysfunction. Pancuronium and
mivacurium are excreted mostly unchanged in the urine. Therefore, their effect is prolonged
in pt with renal dysfunction. Rocuronium is cleared mostly by liver. Succinylcholine is a
depolarizing agent with ultra-short duration of action. In pt with renal dysfunction,
succinylcholine may cause apnea and hyperkalemia.
42. Poroxysmal supraventricular tychycardia (PSVT) most commonly results from accessory
conduction pathway through the AV node. Vagal maneuvers and medications that decrease
conduction through AV nodes often resolve the PSVT.
43. Campylobacter jejuni is the most frequent precipitant of Guillain-Barre syndrome.
E.Coli (O157:H7) –hemolytic uremic syndrome
Samonella, Shigella, Campylobacter, Yersinia and Chlamydia—reactive arthritis.
44. Confidence interval (CI) of the mean is calculated by the following formula. CI
(mean)=mean ± standard score (z) ’standard error of mean (SE).

12/18/09
1. pt suffering from panic disorder have an increase incidence of depression, agoraphobia,
generalized anxiety and substance abuse. Dx criteria: 1- At least 3 attacks in a 3 week period.

14
2-No clear circumscribed stimulus. 3- Abrupt onset of symptoms which peak within 10
minutes. * extremely important question!!
2. Hyperpigmentation of the skin and mucous membranes is characteristic of primary
adrenocortical deficiency, and is due to the increased levels of ACTH. This clinical feature is
not seen in patients with secondary adrenal insufficiency, which is due to hypothalamo-
pituitary failure.
3. Diabetes insipidus presents as polyuria, polydipsia and excretion of dilute urine in the
presence of elevated serum osmolality. Primary polydipsia is due to excessive water drinking;
both plasm and urine are diluted. SIADH results in hyponatremia, low serum osmolality and
inappropriately high urine osmolality.
4. classic symptoms for carbon monoxide (CO) poisoning—polycythemia, nausea, dizziness,
and headaches, works in enclosed space where he is constantly exposed to CO from
automobile exhaust. Always consider CO poisoning in pt with environmental risk factor who
present with headache, nausea, and dizziness.
5. Ludwig’s angina is infection of the submaxillary and sublingual glands (bilateral cellulites,
typically Strep and anaerobes). The source of infection is most commonly an infected tooth,
usually the second or third mandibular molar. Asphyxiation is the most common cause of
death in this disease.
6. Congenital toxoplasmosis triad—chorioretinitis, hydrocephalus, and intracranial calcificatons.
Congenital rubella syndrome (CRS) triad—deafness, cataracts, and cardiac defects.
HSV—most infections are acquired during passage of the fetus through the birth canal—
encephalitis, chorioretinitis and disseminated disease.
7. Glasgow coma scale (GCS) is a component of the primary survey in all trauma pts.
Eye opening
Spontaneous 4
To verbal command 3
To pain 2
None 1
Verbal response
Oriented 5
Disoriented/confused 4
Inappropriate words 3
Incomprehensible sound 2
None 1
Motor response
Obeys 6
Localizes 5
Withdraws 4
Flexion posturing (Decorticate) 3
Extension posturing (Decerebrate) 2
None 1
Head injuries can be classified according to GCS score as minor (13-14), moderate (9-12) and
severe (<9).
8. Transfusion reactions occur acutely during or immediately following the transfusion of blood
products. Such reactions are immune-mediated; perform host antibody reacts with antigens on
transfused blood products causing the release of inflammatory mediators and activation of
complement. Acute reactions can be hemolytic, a severe reaction that may cause death, or

15
nonhemolytic, a dose-dependent self-limited reaction characterized by fever and rigors.
Malignant hyperthermia is caused by inhaled anesthetics. Patients typically have fever,
tachycardia, acidosis, rhabdomyolysis and are at high risk for cardiac arrest and death.
Drug fever most commonly occurs one to two weeks following initiation of therapy. It is more
commonly associated with the use of anticonvulsants and trimethoprim/sulfamethoxazole, though
the list of possible drugs is lengthy.
9. The abrupt onset of hypoxia with respiratory failure, cardiogenic shock, and DIC should raise
suspicion for amniotic fluid embolism (AFE) in a pregnant female who has undergone
amniocentesis or delivery. Respiratory support is always first step in management.
10. CVS is the best test for detection of fetal chromosomal abnormalities in the first trimester of
pregnancy.
11. The most significant risk factor for distal limb reduction defects associated with CVS
sampling is the gestational age of the fetus. CVS before 10 weeks gestational age is associated
with greater risk.
12. Allergic contact dermatitis presents with erythema, edema, pruritus, tiny vesicles and weepy
or crusted lesions 24-48 hours after contact with the allergen. It is due to cell-mediated (delay,
type IV) hypersensitivity.
13. Symmetric distal sensorimotor polyneuropathy is the most common type of diabetic
neuropathy, and is characterized by the classic ‘stocking glove’ pattern of sensory loss. Aside
from symmetric distal sensorimotor polyneuropathy, diabetes can also cause
mononeuropathies of cranial and peripheral nerves.
14. pulsus paradoxus is defined as a difference in systolic pressure of >12 mmHg during
inspiration. Increase venous return to the right heart during inspiration impedes left
ventricular filling and thus causes a drop in systolic pressure. Causes include cardiac disease
(tamponade, pericardial effusion ), and pulmonary disease (tension pneumothorax, and severe
asthma).
15. Lesch-Nyhan syndrome is secondary to a deficiency in hypocanthine-guanine phosphoribosyl
transferase (HPRT). Symptoms include self-mutilation, neurologic features (e.g. mental
retardation, dystonia, choreoathetosis, spasticity), gouty arthritis, and tophus formation. Gout
is usually seen in pts above 50 years of age, therefore, suspect Lesch-Nyhan syndrome if you
see a boy with gout.
16. Intellectualization is the transformation of an unpleasant event into a purely intellectual
problem with no emotional component.
17. Primary hyperparathyroidism is the most common cause of hypercalcemia in ambulatory
patients. Most patients are asymptomatic, and are identified only through routine chemistry
testing.
Familiar hypocalciuric hypercalcemia is a rare AD disorder. Pt are usually asymptomatic, but have
a positive family history and very low urinary calcium level. The underlying pathology involves
the presence of abnormal calcium-sensing receptors on the parathyroid cells.
18. Every case of leukocoria (a white reflex in the eye) is considered a retinoblastoma, until
proven otherwise; therefore, such cases should be promptly referred to an ophthalmologist. *
Extremely high yield question!!!
19. Child development—USMLE frequently testes:
Language development:

16
Social smile: 2 months
Babbles: 6 months
2 words, obeys 1-step command: 1 year
2-3 word phrases, obey 2-step command: 2 years

Gross motor development:


Holds head: 3 months
Rolls back to front and front to back: 4 months
Sits well unsupported: 6 months
Walk alone: 12 months
Walk up and down stairs without help: 24 months

Fine motor development:


Raking grasp: 6 months
Throw object: 12 months
Builds tower of 2 blocks: 15 months
Builds tower of 6 blocks/turn pages of books: 24 months

Social development:
Recognizes parents: 2 months
Recognizes strangers (stranger anxiety): 6 months
Imitates action/come when called: 12 months
Plays with other children: 18 months
Parallel play: 24 months

20. Iron deficiency anemia is common in infants and toddlers who drink excessive amount of
cow’s milk. Empiric iron therapy is appropriated for children with microcytic, hypochromic
anemia.
21. Tamoxifen acts as an estrogen antagonist on breast tissue, and is used in the tx and prevention
of breast cancer. It acts as an estrogen receptor agonist on the endometrium and increase the
risk of endometrial carcinoma. Tamoxifen has an overall mortality benefit, and also decrease
risk for osteoporosis.
22. When a pt is interested in alternative therapy, the physician should first inquiry as to why.
23. The hyperdynamic type of septic shock is characterized by (1) and elevated cardiac output, (2)
low systemic vascular resistance, right atrial pressure and pulmonary capillary wedge
pressure, and (3) a frequently normal mixed venous oxygen concentraton—results from
hyperdynamic circulation and improper distribution of the cardiac output. It is not indicative
of normal tissue perfusion in this case!
24. Atrial fibrillation (AF) is identified on EKG by an irregularly irregular R-R interval with
absent P waves and narrow QRS complexes. In unstable pts, immediate cardioversion should
be performed. In stable pts who have been in AF < 48 hours, cardioversion is appropriate
(Beta-blocker or CCB). In stable pts in AF> 48 hours, 3-4 weeks of rate control and
anticoagulation should precede cardioversion.
25. A muffled voice should make on consider a diagnosis other than uncomplicated pharyngitis or

17
tonsillitis. A peritonsillar abscess is potential complication of tonsillitis and requires both IV
antibiotic therapy and urgent drainage of the abscess. Deviation of the uvula and unilateral
lymphadenopathy can be helpful in distinguishing a peritonsillar abscess from epiglottitis.
26. Dihydropyridine Ca-channel antagonists can cause peripheral edema and should always be
considered in the differential diagnosis of this condition, along with other causes, such as
heart failure, renal disease and venous insufficiency. These medications can cause significant
edema due to the property to dilate peripheral blood vessels; it is NOT an allergic reaction!
27. Dermaromyositis—classic cutaneous findings accompanied by proximal muscle weakness
(violaceous poikiloderma: periorbital edema— heliotrope sign; on the chest and lateral neck
—shawl sign; elbows and knees—Gottron’s sign). Over 10% of adult pts will develop an
internal malignancy, most commonly an ovarian cancer. Regular age-appropriate cancer
screening is essential in these pts.
Inflammatory diseases associated with aortic aneurysms: Behcet syndrome, Takayasu arteritis,
giant cell arteritis, ankylosing spondylitis, rheumatoid arthritis, psoriatic arthritis, relapsing
polychondritis and Reiter syndrome.
Inflammatory diseases associated with renal failure: SLE, scleroderma, Goodpasture syndrome,
relapsing polychondritis and Wegener granulomatosis.
Inflammatory diseases associated with alveolar hemorrhage: Goodpasture, Wegener
granulomatosis, polyarteritis, Churg-strauss syndrome, Bechet syndrome and antiphospholipid
antibody syndrome, DM causes pulmonary fibrosis.
Inflammatory diseases associated with carpal tunnel syndrome : RA, sarcoidosis and amyloidosis.
28. Perioperative beta-blockers lower the incidence of perioperative myocardial ischemic events
in pts at risk for cardiac complications following noncardiac surgery.
29. Risk factors for the development of diabetic foot ulcers include diabetic neuropathy,
peripheral vascular disease, poor glycemic control, bony abnormalities of the foot, male sex,
chronic diabetes (>10 years), and a history of previous ulcer or amputation. Neuropathy is
found in approximately 80% of diabetics with foot ulcers. High yield!!!
30. Pt with chronic HCV who show persistently normal liver enzymes on multiple occasions have
minimal histological abnormalities, therefore they do not need to be treated with interferon or
antiviral drugs at this stage.
31. Exertional heat stroke occurs in otherwise healthy individuals undergoing conditioning in
extreme heat and humidity as a result of the thermoregulatory center’s inability to adequately
dissipate heat.
32. Generalized anxiety disorder (GAD)—excessive anxiety and concern regarding multiple
events or activities for a minimum of six months. 3 or more of the following symptoms must
also be present: sleep impairment, easy fatigability, restlessness, muscle tension, poor
concentration, or irritability. Bispirone is a first-line drug for treating GAD. Also benzo
(alprazolam) or antidepressants (venlafaxine, escitalopram). Bispirone does not cause the
physical dependence and withdrawal symptoms.
33. Reactive arthritis is a seronegative spondyloarthropathy resulting from enteric or
genitourinary infection. Findings in reactive arthritis may include urethritis, conjunctivitis,
mucocutaneous lesions, enthesitis (Achilles tendon pain) and asymmetric oligoarthritis. Not
all extra-articular manifestations need be present to suspect the Dx. NSAIDs are the first line
therapy.

18
34. Confounding refers to the bias that can result when the exposure-disease relationship is mixed
with the effect of extraneous factores (i.e.confounders). Selection bias results from the manner
in which people are selected for the study, or from the selective losses from follow-up.
Obsever’s bias and measurement bias distort the measure of association by misclassigying
exposed /unexposed and /or disease/non-diseased subjects. Recalled results from the
inaccurate recall of past exposure by people in the study.
35. Hypercalcemia, hyperphosphatemia, and increased parathyroid hormone levels are
characteristic biochemical abnormalities of secondary hyperparathyroidism in chronic renal
failure.
36. Loop diuretics—reversible or permanent hearing loss and /or tinnitus. These ototoxic effects
typically occur in pts taking high doses of loop diuretics, those with coexistent renal failure,
or in pts who are also being treated with other known ototoxic medications: aminoglycosides,
aspirin, chemotherapeutic agents.
37. Brain abscess can present with fever, headache, focal neurologic chagens, seizure, spacticity,
or signs of increased intracranial pressure. Congenital heart disease, heart trauma, infections
of the jaw or mouth, infections of the face or scalp, meningitis, and cranila instrumentation
increase the risk of developing a brain abscess.
38. The classical pathophysiology mechanism responsible for hypercalcemia of malignancy is the
production of PTH related peptide (PTHrP), which is structurally homologous with PTH and
can stimulate PTH receptors. The serum calcium level is increased, and the serum phosphate
level is depressed. PTHrP is present n very low concentrations in normal people, and is
thought to be responsible for calcium excretion in breast milk and transplacental calcium
transport. PTHrP can be measured in the circulation by using a PTHrP specific assay. The
elevated calcium level suppresses PTH release, thereby resulting in low serum PHT levels,
however, ectopic PTH secretion have been describe in a small number of cases.
Hypercalcemia of lung malignancy is most commonly due to the production of PTHrP by
malignant cells.
39. The history of loss of small amounts of urine simultaneously occurring with activities that
increase intraabdominal pressure, along with a physical examination demonstrating pelvic
floor weakness, is diagnosis of stress incontinence. Urine analysis, cystometry and postvoid
residual volume are normal
Detrusor instability, bladder irritation from a neoplasm, and interstitial cystitis result in urge
incontinence, which causes sudden and frequent loss of moderate to large amounts of urine. This
is often accompanied by nocturia and frequency.
Diabetic neuropathy causes overflow incontinence, which is characterized by loss of small amount
of urine from an over distended bladder, and a markedly increased residual volume. Patients
usually have a long history of diabetes that is not well controlled.
40. Pts with recurrent hypercalciuric renal stones should be treated with increased fluid intake,
sodium restriction, and a thiazide diuretic. Calcium restriction is not advised. The Tx of a first
uncomplicated calcium stone is hydration and observation.
Renal stones are predominantly of 4 types: 1-calcium oxalate/phosphate (75%), 2-uric acid (10-
15%), 3-struvite/triple phosphate (10-15%), 4- cysteine (<1%).
41. Serum CK level are used for screening the muscular dystrophies. Muscle biopsy can confirm
the diagnosis in most cases. The gold standard is genetic studies, which is required in atypical

19
cases.
42. Vitiligo is characterized by macular depigmentation what involves that acral and peri-orificial
areas. It is caused by the autoimmune destruction of melanocytes.
43. The most common adverse effect of inhaled corticosteroid therapy is oropharyngeal thrush
(oral candidiasis).
44. All patients who have the myocardial infarction should receive secondary prevention. The
following medications have shown to have a mortality benefit when given as secondary
prevention: 1-Aspirin (75-325 mg/day). 2-Beta-blockers. 3-ACEIs. 4-lipid-lowering statin
drugs. Clopidogrel should be included as secondary prevention following unstable
angina/NSTEMI for at least 12 months. It should also be prescribed for thirty days (bare metal
stents) to one year (drug eluting stents) following PCI, as it has been shown to help prevent
subacute stent thrombosis.
Low molecular weight heparin (LMWH) is usually given post-MI for the first 48 hours or until
angiography is performed, but it is not necessary after the pt has been discharged from hospital.
Wafarin is indicated if the pt has evidence of ventricular thrombus, which is usually seen in
individuals with apical infarcts and dyskinetic wall motion abnormalities.

12/19/09
1. Primary sclerosing cholangitis (PSC) is caused by inflammation and fibrosis of the
intrahepatic and extrahepatic biliary ducts. It is frequently associated with ulcerative coitis.
Microscopic polyangiitis is a pauci-immune, necrotizing vasculitis of small vessels. While it is
often p-ANCA positive, it is not associated with IBD and typically presents with a wide range of
organ-specific manifestations (eg. Skin rash, hemoptysis, chest pain, GI bleeding, arthralgias)
PBC—anti-mitochondrial antibodies are typically positive.
2. Antibiotic therapy is the most accepted and recommended management of for the eradication
of H.pylori in pts with gastric mucosa-associated lymphoid tissue (MALT) lymphoma without
any metastasis.
3. Always suspect malignant hypertension in pt with very high BP (≥200/140 mmHg). Presence
of papilledema on pphothalmoscopy confirms the diagnosis. The pathologic change
responsible for end-organ damage in malignant hypertension is fibrinoid necrosis of small
arterioles. Renal failure with elevated creatinine level and oliguria can develop rapidily with
tx, but is not required to diagnose.
4. So far, there have been no case reports of congenital rubella syndrome in womrn inadvertently
vaccinated during early pregnancy. Mx—reassurance.
5. Torsades de points is a polymorphic ventricular tachycardia which occurs in the setting of a
prolonged QT interval and is seen in pts with familiar long QT syndrome, malnourished pts
predisposed to hypomagnesemia (such as alcoholic), and in pts taking certain drugs (tricyclic
antidepressants), certain antiarrhythmics (amidarone, sotalol), and anti-infective agents
(moxifloxacin, fluconazole). TX—cessation of any offending agents, and initiation of
magnesium sulfate.
Sodium bicarbonate is used in tricyclic antidepressant and aspirin overdose.
6. Severe vomiting produces metabolic alkalosis due to the unbalanced loss of hydrogen ions in
the gastric fluid. Contraction alkalosis also occurs in the is setting due to ECF volume
contraction resulting from vomiting. The decrease in ECV volume activates the rennin-

20
angiotensin-aldosterone system causing water retention at the expense of hydrogen ion losses.
7. Subdural hematomas are serious intracranial hemorrhages that occur due to tearing of
bridging veins. Blunt trauma is the most common cause, and surgical evacuation is usually
require.
Small vessel hyalinosis is believed to contribute to the phathophysiology of lacunar infarction.
Lacunar strokes occur due to occlusion of small penetrating arteries and account for 25% of
ischemic strokes. Hypertension and diabetes are two major risk factors. Lacunar strokes often
affect the internal capsule and result in pure motor dysfunction. Small vessel hyalinosis has no
association with subdural hematoma,
8. Temporal arteritis should be suspected in pts over 50 years of age with new-onset temporal
headache, jaw claudication, vision loss, or symptom of polymyalgia rheumatica (CK, EMG
normal, ESR ↑). Once the diagnosis is suspected, immediate initiation of high dose steroids is
indicated to prevent damage to the retinal artery and other vessels. The ESR is elevated in
temporal arteritis and diagnosis is confirmed by temporal artery biopsy.
9. All pts with cirrhosis of the liver should be screened for esophageal varices by endoscopy.
Primary prophylaxis should include beta-blockersm which will significantly reduce the risk of
bleeding.
10. Amaurosis 黑朦 fugax is characterized by visual loss that is usually monocular, transient, and
described as ‘like a curtain falling down’. Ophthalmoscopy reveals zones of whitened,
edematous retina following the distribution of the retinal arterioles. Non-invasive evaluation
of the carotids is useful in providing information regarding the degree of carotid artery
stenosis.
Central retinal artery occlusion (CRAO) –sudden painless loss of vision in one eye; however, the
funduscopic findings differ. Ophthalmoscopy in pts with CRAO reveals pallor of the optic disc,
cherry red fovea, and boxcar segmentation of blood in both the retinal arteries and veins.
Central retinal vein occlusion –sudden , painless, unilateral loss of vision. It is also noted in pts
with a history of hypertension. Ophthalmoscopic signs are disk swelling, venous dilation,
tortuosity, retinal hemorrhages and cotton wool spots.
Vitreous hemorrhage presents with a sudden loss of vision and floaters in the visual field. Diabetic
retinopathy is the most common cause. An important clue to the diagnosis is a fundus that is hard
to visualize with obscure details.
Pts with hypertensive retinopathy usually do not show any symptoms associated with visual
impaiement. There is initially focal spasm of arterioles, followed by progressive sclerosis and
narrowing. Fundoscopy findings may reveal AV nicking, copper wiring or silver wiring, exudates
and hemorrhages (specific finding depends on the grade of retinopathy.)
11. The presence of a systolic-diastolic abdominal bruit in a pt with hypertension and
atherosclerosis is strongly suggestive of renal artery stenosis. Arortic coarctations are
congenital strictures along the aorta. They can cause hypertension and differences in blood
pressures between the upper and lower extremities. Pts are most often diagnosed in infancy or
childhood.
12. In the US, sarcoidosis most commonly affects young- to middle- aged African females,
causing insidious-onset hyspnea and dry cough in the absence of constitutional symptoms.
Sarcoidosis also typically affects the skin (protean manifestations, most commonly erythema
nodosum) and the eye (uveitis).

21
Idiopathic pulmonary fibrosis presents insidiously with progressive dyspnea on exertion and a dry,
nonproductive cough. Auscultation reveals ‘Velcro-like’ inspiratory crackles. Fever and chest pain
are typically absent and uveitis is not a assocated feature.
13. Fat-soluble vitamins are stored in the body for long periods of time, and hence have the
potential to accumulate to toxic levels when supplements are taken. VitD toxicity leads to
hypercalcemia, constipation, abdominal pain, weight loss, polyuria, and polydipsia. In cases
of toxicity, both VitD and calcium should be withheld until symptoms improve and levels
return to normal.
VitA toxicity may cause dry skin, headache, abdominal pain, and blurry vision. Can also cause
psudotumor ceribri.
14. Loss to follow-up in prospective studies creates a potential for selection bias.
15. Ultrasound or other imaging techniques (CT scan) is indicated when pts with pyelonephritis
do not respond after 48-72 hours of the appropriate antibiotic therapy, searching for any
underlying pathologies (eg. obstruction) or complications (eg. Renal, perirenal abscess).
16. Polymyositis and dermatomuyositis are two forms of inflammatory myopathy and present
proximal muscle weakness. A diagnosis of inflammatory myopathy is supported by elevated
ESR and CK, and is confirmed by muscle biopsy. Tx—high dose corticosteroids.
Riluzole is used to slow the progress of amyotrophic lateral sclerosis (ALS). ALS presents with
propressive weakness of the arms, legs and cranial nerves, with a mixture of upper and lower
motor neuron signs.
Amitriptyline--fibromyalgia
17. Monoclonal gammapathy of undermined significance (MGUS) are usually Mx initially iwht
reassurance and regular follow-up. Labs—an M component (IgA, G or M) <3000mg/dL, and
fewer than 10% plasma cells in the BM. 25% risk of development of a serious disease, 1% yr
—multiple myeloma.
18. The combination of arterial/venous thrombosis and thrombocytopenia in pts receiving heparin
therapy is highly suggestive of heparin-induced thrombocytopenia. Antibodies against
heparin-platelet factor-4 complex are responsible for the disease.
19. The BUN levels is elevated in pts with upper GI bleeds bz the bacterial breakdown of
hemoglobin in the GI tract results in the absorption of urea. A BUN level > 40 in the presence
of a normal serum creatinine level may indicate an upper GI bleed, The other common
scenario where you can see elevated BUN without increased creatinine levels is the
administration of steroids. Extremely important!!
20. know how to interpret the strength of association and dose response relationship from a study.
21. “lone atrial fibrillation” is atrial fibrillation in the absence of other risk factors for stroke, such
as previous stroke or TIA, DM, hypertension, heart failure, age over 65 years, or valvular
heart disease, Aspirin is the only antithrombotic therapy required for pts with lone atrial
fibrillation.
Warfarin with a goal INR of 2-3 is the recommendation for pts who have AF and other risk factors
for stroke. When starting warfarin therapy, it takes at least 3-4 days to achieve appropriate levels
of anticoagularion. In fact, warfarin therapy may actually cause hypercoagulability initially. (This
is bz the first VitK dependent factors to be lost are protein C and S, which are both
anticoagulants.) In order to provide anticoagulation during this window of time, pts are often
‘bridged’ with a heparin product.

22
22. Enthesitis is a condition in which inflammation and pain occur at the site of tendon and
ligment attachment to bone and is a common finding in anklosing spondilitis (AS). Typical
sites of enthesitis include the heels, tibial tuberosities and iliac crests.
Proteinuria + arthritis—lupus
Subcutaneous nodules—RA, gout.
23. Torticollis 斜 颈 is a common form of focal dystonia involving the sternocleidomastoid
muscle. It can occur idiopathically but is very often medication-related. Discontinuation of
causative agents may improve symptoms.
Akathisia 静坐不能 is a sensation of restlessness that causes the pt to move frequently.
Athetosis 手足徐动症 refers to slow, writhing movements that typically affect the hands and feet.
Athetoid movements are characteristic of Huntington’s disease. Chorea and athetosis often occur
together.
24. A normal distribution is symmetric and bell-shaped. All its measures of central tendency are
equal: mean=median=mode. Positively skewed (tail on the right), mean>median>mode.
Negatively skewed (tail on the left), mean<median<mode.
25. Acute angle closure glaucoma presents as unilateral eye pain, redness, and a dilated pupil with
poor light response, It’s important to distinguish acute angle closure glaucoma from migraine,
cluster headache, temporal arteritis, and keratoconjunctivitis bz failure to correctly diagnose
this disease can lead to blindness.
26. Know how to interpret the confidence interval. Know the relationship between confidence
interval and ‘p’ value.
27. Electrical alternans is when the amplitudes of the QRS complexes vary from beat to beat. It is
fairly specific for pericardial effusion. Enlargement of the cardiac silhouette can be seen on
CXR as well. Echocardiogram can more definitely demonstrate a pericardial effusion.
New-onset right bundle branch block—PE
28. Hypersensitivity pneumonitis (HP) si inflammation of the lung parenchyma caused by antigen
exposure. Acute episodes present with cough, breathlessness, fever and malaise that occur
within 4-6 hours of antigenic exposure. Chronic exposure may cause weight loss, clubbing,
and honeycombing of the lung. The cornerstone of HP Mx is avoidance of the responsible
antigen.
29. One of the most common causes of hyperkalemia is medications—ACEI, angiotensin receptor
blockers and spironolactone—by blocking aldosterone synthesis or action. Other medications
—trimethoprim and pentamidine. Pt with hyperkalemia—Mx—review all the medication
first.
30. Baker cysts develop as a result of excessive fluid production by an inflamed synovium, as
occurs in cases of RA, osteoarthritis and cartilage tears. The excess fluid accumulates in the
popliteal bursa which expands, creating a tender mass in the popliteal fossa. Baker cysts
occasionally burst and release their contents into the calf, resulting in an appearance similar to
a DVT.
31. Acanthosis nigricans—symmetrical, hyperpigmented, velvety plaques in the axilla, groin, and
neck. It is associated with DM (insulin resistance) in younger patients, and GI malignancy in
older individuals.
32. HIV screening-- ELISA
33. Pseudomonas aeruginosa is a common cause of severe, pulmonary infections in cystic fibrosis

23
pts. Prefered therapy—combination of an aminoglycoside (tobramycin) and and
antipseudomonal penicillin (piperacillin). Ciprofloxacin is C/I in children as it may cause
cartilage destruction and growth retardation.
34. Pulmonary neoplasm in superior sulcus—apical lung tumor cause compression of the
sympathetic trunck (Horner syndrome), the brachial plexus (pancoast syndrome), the right
recurrent laryngeal nerve (hoarse voice), and the superior vena cava (SVC syndrome).
Pancoast syndrome is characterized by shoulder pain radiating into the arm in an ulnar
distribution and is caused by tumor invasion of the 8th cervical and 1st thoracic nerves.
35. Insulin resistance typical for pt with central-type obesity is the key pathogenic factor in the
development of type-2 DM and associated abnormalities (hypertension, dyslipidemia).
Extremely high yield!!
36. Bronchiectasis is marked by bronchial dilation, impaired clearance of secretion and airway
obstruction. High resolution chest CT is used to make the definitive Dx. Massive and
potentially lethal hemoptysis is a feared complication.
37. Adrenal TB—MCC of primary adrenal insufficiency in developing countries; autoimmune
adrenalitis—developed countries.
38. Women produce androgens, such as androstenedione (AS), dehydroepiandrosterone (DHEA),
testosterone (T) and dehydroepiandrosterone sulfate (DHEA-S). AS, DHEA and T are
produced by the ovaries and adrenals, whereas DHEA-S is predominantly produced in the
adrenal glands only. AS, DHEA and DHEA-S are not true androgens cz they do not interact
with the androgen receptor. They can be converted to testosterone, and overproduction of
these hormones can lead to clinical features of androgen excess (adrenal tumor).
39. Pulmoary embolism should be suspected in all pt at risk for DVT who present with acute-
onset tachypnea, dyspnea and chest pain. Dx—helical CT scanning of the chest.
40. Hyperlipidemia (decreased clearance of LDL due to decreased LDL receptors), unexplained
hyponatremia (ISADH) and elevated serum muscle enzymes(myopathy) are indicationa for
thyroid function tests—hypothyroidism.
41. Untreated hyperthyroid pts are at risk for rapid bone loss resulting from increased osteoclastic
activity in the bone cells, also at risk for cardiac tachyarrhythmias, including AF.
42. Chagas disease is a chronic disease that can cause megaesophagus, megacolon, and /or
cardiac dysfunction. The protozoan trypanosome cruzi, endemic to Latin America.
43. Heparin-induced thrombocytopenia is seen in about 5-15 of pts taking therapeutic heparin
with onset between 3-15 days after initiation of heparin and resolution with 4-5 days of
discontinuation of heparin. While the elevated PTT is a therapeutic effect of heparin, the
thrombocytopenia is an adverse effect of the same.
44. In ARDS, mechanical ventilation with low tidal volumes (prevent lung damage) and PEEP
(up to 15cm H2O) can improve oxygenation. Potential complications of PEEP include
barotraumas and tension pneumothorax.
12/20/09
1. Cerebellar dysfunction is common among chronic alcohol abuser. Symptoms include gait
instability, difficulty with rapid movements, and intention tremor
The ‘clasp knife’ phenomenon refers to s sunned reduction in muscle tone and increase in laxity
after initial resistance to passive movement. It is seen in pts with hypertonia and is not
characteristic of cerebellar disease.

24
2. Obstructive sleep apnea is an under-diagnose condition common in the obese population.
Snoring, daytime sleepiness, and apneic episodes during sleep should raise suspicion of OSA.
Affected pts are at risk for hypertension, heart disease, cor pulmonale, and accidents.
Polysomnography is used for diagnosis.
3. Mitral regurgitation can cause a holosystolic murmur that radiates to the apex. Mitral valve
prolapse is the MCC of mitral regurgitation (developed country). Mitral regurgitation can lead
to left atrial dilation and atrial fibrillation.
4. Hydatid cysts in the liver are due to infection with Echinococcus granulosus. ‘Eggshell’
calcification of a hepatic cyst on CT scan is highly suggestive of hydatid cyst. Aspiration of
the cyst is generally not indicated due to the risk of anaphylactic shock sen\condary to spilling
of cyst contents. Tx is generally surgical resection under the cover of albendazole.
Cysticercosis is secondary to infection with tenia solium and generally results in cysts in the brain
or in muscle.
5. Beta-blockade is the most appropriate initial intervention for acute aortic dissection. Type A
dissections involve the ascending aorta and are treated with medical therapy and surgery,
while Type B dissections involve only the descending aorta and are usually treated with
medical therapy alone.
Hydralazien, nifedipine, and vasodilates would increase shear stress on the aorta, should be avoid.
Heparin and fibinolytisc, like streptokinase should never be used if aortic dissection is suspected
—inhibit the body’s efforts to contain the dissection and could greatly exacerbate the condition.
6. The presentation of cryoglobulinemia (mixed essential cryoglobulinemia) includes palpable
purpura, glomerulonephritis, non-specific systemic symptoms, arthralgias,
hepatosplenomeagly, peripheral neuropathy, and hypocomplementemia. Most patients also
have HCV. The demonstration of circulating cryoblobulin is confirmatory. All such pts should
be tested for HCV antibody.
Henoch-Schonlein purpura usually presents in childhood as palpable purpura on the buttocks,
abdominal pain, arthralgias, proteinuria and hematuria with RBC casts on urinalysis. Serum
complement levels are normal. HCV infection is not associated with this diease.
7. A fixed upper airway obstruction will decrease the airflow rate during inspiration, active
expiration and passive expiration. (see pictures in notes)
8. Cerebral emboli are one of the most dreaded consequences of endocarditis. They occur when
pieces of infected valvular vegetations break off and enter the CNS circulation. Tx is with
antibiotics.
Todd’s palsy: a focal neurologic defecit following a seizure. EEG can aid in the dx of an epileptic
seizure.
9. Shy-Dragger syndrome—1.parkinsonism; 2-Autonomic dysfunction (postural hypotension,
abnormal sweating, disturbance of bowel or bladder control, abnormal salivation or
lacrimation, impotence, gastroparesis, etc); 3- widespread neurological sings (cerebellar,
pyramidal or lower motor neuron). Always consider Shy-Dragger syndrome when a patient
with Parkinsonism experiences orthostatic hypotension, impotence, incontinence, or other
autonomic symptoms. Anti- Parkinsonism drugs are generally ineffective, and tx is aimed at
intravascular volume expansion with fludrocortisone, salt supplementation, alpha-adrenergic
agonists, and applications of constrictive garments to the lower body.
Idiopathic orthostatic hypotension is seen secondary to the degeneration of postganglionic

25
sympathetic neurons; however, the pathology is confined to the autonomic nervous system, and
there are no signs Parkinsonism or cerebellar involvement.
Riley-Day syndrome or familiar dysautonomia is an autosomal-recessive disease seen
predominantly in children of Ashkenazi Jewish ancestry. It is characterized by gross dysfunction
of the autonomic nervous system with sever orthostatic hypotension.
10. immobilization of an individual with a high bone turnover results in increased osteoclastic
activation that can lead to hypercalcemia. Bisphosphonate therapy in pts who are immobilized
is helpful in reducing hypercalcemia and preventing osteopenia.
11. GI manifestations of diabetic autonomic neuropathy include postpranial bloating, early satiety,
constipation and diarrhea. Mx: 1-improved glycemic control; 2—Small, frequent meals; 3—A
dopamine antagonist (eg. Metoclopramide, domperidone) before meals; 4—Bethanechol; 5—
erythromycin (interacts with motilin receptors and promote gastric emptying); 6—Cisapride is
effective in diabetic gestroparesis, but it is currently available only through the manufacturer.
Fisk factors—cardiac arrhythmias.
12. Heat stroke is defined as a temperature above 40.5 (105F). Exertional heat stroke occurs in
otherwise healthy individual exercising in extreme hear. Dehydration, hypotension,
tachycardia and tachypnea are common. Systemic effects like seizure, ARDS, DIC, and
hepatic/renal failure may also occur.
13. Local therapy (eg. Resection of metastases, local irradiation) is rarely curative in pts with
metastatic breast cancer, but it can be tried in pts with a respectable solitary metastatic focus
without signs of systemic involvement. Criteria for pulmonary metastasectomy: 1) the
primary tumor is controlled (resected); 2) no other sites of disease exist; 3) the operative risk
is low (no significant co-morbid condition) and 4) complete resection is possible.
14. SLE’s effect on the kidneys can range from minimal mesangial lupus nephritis to advance
sclerosing lupus nephritis and are graded class I-IV. The tx and outcomes are different
depending on the class of lupus nephritis. Therefore, renal biopsy is required in all pts with
new onset lupus nephritis. Immunosuppressive therapy may be given once the lupus nephritis
is classified.
15. American Cancer Association urges physician to offer yearly PAS blood tests and digital
rectal examination (DRE) beginning at age: 1—50 years for men who have at least a 10-year
life expectancy; 2—45 years for men at risk *African Americans *men with a first-degree
relative diagnosed with prostate cancer at an early age (younger than age 65); 3—40 years for
men at even higher risk* men with several first degree relatives who had prostate cancer at an
early age. Depending on the results of this initial test, further testing might not be needed until
age 45.
16. Initial hematuria suggests urethral damage. Terminal hematuria indicates bladder or prostatic
damage, and total hematuria (the entire urinary cycle) reflects damage in the kidney or ureters.
Clots are not usually seen with renal causes of hematuria. Pt has painless terminal hematuria
with clots and should be evaluated for bladder cancer by cystoscopy.
17. The C/I of triptans: 1-familial hemiplegic migraine; 2-uncontrolled hypertension; 3-coronary
artery disease; 4-prinzmetal angina, 5-pregnacy, 6-ischemic stroke, 7-basilar migraine
18. Right mainstem bronchus intubation is a relatively common complication of endotracheal
intubation. It causes asymmetric breath sound immediately after intubation and is fixed by
withdrawal of the tube (ideal position—between the vocal cords and the carina for greatest

26
safety and effect.)
19. Prolonged placement of central lines can lead to subclavian vein thrombosis and result in arm
swelling. Catheters should be removed and duplex ordered to document the thrombus and for
the need of anticoagulation,
20. Obstructive sleep apnea (OSA) is the most common medical cause of excessive daytime
sleepiness in the US. It occurs due to poor oropharyngeal tone and results in daytime
sleepiness, morning headaches, and depression.
Narcolepsy is a sleep disorder characterized by poorly regulated rapid eye movement (REM)
sleep. It, like OSA, is characterized by excessive daytime sleepiness, but pts also suffer from
cataplexy, sleep attacks, hypnagogic 入 睡 前 发 生 的 /hypnopompic 半 醒 的 hallucinations, and
sleep paralysis.
21. Ventrical aneurysm is a late complication of MI characterized by persistent ST elevations
(only infarcted leads) on ECG. Pts may have CHF, ventricular arrhythmias, and/or thrombus
formation long with mitral regurgitation (a pansystolic murmur at the apex with radiation to
the axilla). Echocardiography demonstrating dyskinetic wall motion of a portion of te left
ventricle may be used to confirm the Dx.
Interventricular wall rupture—5 days after MI, causes an acute left-to-right shunt with right sided
heart failure and new-onset systolic murmur head best at the left lower sternal border.
Ventricular free wall rupture—5 days after MI, causes acute pericardial tamponade and rapid
decompensation with pulseless electrical activity.
Papillary muscle rupture—3-7 days after MI, causes severe acute mitral regurgitation and
pulmonary edema. It does not typically cause persistent ST elevation on ECG.
22. Stepwise approach to abort acute attacks of migraine: simple analgesics (eg. acetaminophen),
NSAIDs, triptans and ergot derivatives. Serotonin agonist (triptans, Rizatriptan, zolmitriptan,
sumatriptan, frovatriptan and almotriptan) are generally preferred over ergotamines bz it has
fewer side effects and greater efficacy than ergotamine.
Ergotamine derivatives are considered when acute attacks of migraine last for longer than 48
hours and are frequently recurrent. The use of these drugs has been shown to be associated with
rebound headache and vascular occlusion. For this reason, ergotamine derivatives are generally
avoided in pts with coronary artery disease, hypertention, peripheral vascular disease, liver
disease, renal disease and complicated migraine,
Propranolol and verapamil are used ofr prophylaxis of migraine, These drugs are not used to abort
acute attackes.
23. Myasthenia crisis is life-threatening condition hat is characterized by weakness of the
respiratory and pharyngeal muscles. Tx of myasthenia crisis consists of endotracheal
intubation and withdrawal of anticholinesterases for several days. The MCC is an intercurrent
infection and in such cases antibiotics are an important part of Mx. DD—cholinergic crisis.
Physicians used to perform the Tensilon (pyridostimine) test. However, since cholinergic
crisis has become uncommon, and since the tx of both conditions is to withdraw
anticholinesterases, physicians are no linger doing this test.
24. Infectious diarrhea is classified into bloody/inflammatory and non-bloody. Bloody—E. coli
O157:H7, enteroinvasive E. coli, Shigella, Salmonella, Campylobacter, Yersinia, Entamoeba,
and C.difficile. E. coli O157:H7 had been transmitted via the intake of undercooked
hamburger meat from fast food chains,

27
25. Basal cell carcinoma presents as a slow-growing papule with pearly, rolled borders and
overlying telangiectasia. It is the most common malignant tumor of the eyelid.
Keratoacanthoma is a rapid growing ‘vocano-like’ nodule with a central keratotic plug. While
these lesions are classically self-limited, many are treated as well-differentiated squamous cell
carcinomas. Early tx is indicated is the lesion is near an important structure, such as the eye.
Seborrheic keratoses are most often greasy, brown, crust-like lesions with a stuck-on appearance.
26. Suspect dermatitis herpetiformis in a pt with (1)GI symptoms suggestive of malabsorption,
and (2) pruritic papules and vesicles over the extensor surfaces of the elbows, knees, posterior
aspect of neck and scalp. The presence of aniti-endomysial antibodies is characteristic. Pts
also suffer from celiac sprue or gluten sensitive enteropathy. There is an increased risk for
gastro-intestinal lymphomas, which can be reduced by following a gluten-free diet.
Erythema multiforme—a sudden onset of symmetric erythematous skin lesions. Target skin
lesions with a clear center and concentric erythematous rings may be noted. HSV infection,
systemic diseases, drug reactions are often assocated with this condition.
Acantholytic dermatosis is also know as Grover disease. Pts present with pruritis and
erythematous to brown keratotic papules over the anterior chest, upper back, and lower rib cage.
The etiology is unknown.
27. Hypothyroidism should always be considered in pts with an unexplained elevation of serum
CK concentration and myopathy—check serum TSH.
28. Methotrexate is a first line tx for RA and its side-effects include stomatitis, nausea, anemia
and hepatotoxicity—Tx folic acid.
Felty syndrome is an uncommon form of RA accompanied by splenomegaly and granulocytopenia
(neutrophil count <2,000/mm3). Granulocytopenia generally does not occur until symptoms of
arthritis have been present for over 10 years. Splenomegaly is a common finding in pts with RA
without granulocytopenia.
29. Increased extracellular PH levels (eg. Respiratory alkalosis) can cause in the affinity of serum
albumin to calcium, thereby increasing the levels of albumin-bound calcium, and
consequently decreasing the levels of ionized calcium. The level of calcium bound to organic
and inorganic anions remains unaltered. Ionized calcium is the only physiologically active
form—manifestation of hypocalcemia.
30. Small cervical LNs are a commons observation in children and young adults (upper
respiratory infection). Pts with asymptomatic, soft (rubbery) LN can be observed for node
growth or the development of symptoms. A nodal diameter >2.0—biopsy; <1cm, almost
benign.
31. The pH of the pleural fliud <7.2—an empyema—drained, removal of the fluid by
thoracostomy. Glucose <60mg/dL—indication for tube thoracostomy. Infected pleural space
is usually initially drained with a chest tube.
32. In mechanically ventilation pts with respiratory alkalosis in the setting of an appropriate tidal
volume (6ml/kg of idea body weight), the respiratory rate should be lowered. (Reductions in
tidal volume can trigger an increased ventilatory rate, potentially exacerbating the situation.)
33. The association between DM and mucormycosis is frequently tested on the USMLE. The
MCC cause of mucormycosis is Rhizopus. Pseudomonas is a cause fo malignant otitis externa
in DM pts; it may cause black necrotic lesions in the ear.
34. The sign and symptoms of PE are nonspecific and highly variable. It is a diagnosis that should

28
be suspected in any pt who presents with some combination of sudden-onset shortness of
breath, pleuritic chest pain, low-grade fever, and hemoptysis. Tachycardia, and hypoxemia are
common exam findings. Calf swelling and Virchow’s triad (stasis, endothelial injury, and
hypercoagulable state) are not always present in PE.
35. Elderly and critically ill pts with immobility, poor nutrition and sensory impairment are at risk
of developing decubitus 褥疮 ulcers. Ulcers form as a result of uninterrupted pressure on the
tissue overlying bony prominences. The elbow, coccyx, hips and heels are sites at expecially
high risk. Pt repositioning every 2 hours and pressure reducing devices (air/foam mattresses)
are important methods of prevention.
36. Epidural abscesses must be considered in pts with fever and back pain, particularly in those
who are immunosuppressed or use injection drugs. The diagnostic test of choice is an MRI of
the spine. Tx consists of antibiotics and surgical decompression.
37. Non-invasive positive pressure ventilation (NIPPV) is an excellent option for pts with COPD
exacerbation. It should be tried before intubation (more complications, includes infections)
and mechanical ventilation in COPD pts with CO2 retention. NIPPV is recommended in a pt
of respiratory distress with a pH <7.35 or PaCO2 >45mmHg or respiratory rate >25 per min.
C/I: septic, hypotensivem or dysrhythmic pts.
38. Always remember that the ABC’s come first in the tx of any pt. The tx of choice for RMSF is
doxycycline for both adults and children.
39. Hypercalcemia is a common finding in a pt with multiple myeloma. Hypercalcemia may
cause severe constipation, anorexia, renal tubular dysfunction, and neurologic symptoms. *
other scenario could be metastatic bone cancers such as lung, breast, renal, thyroid, etc.
Always look for hypercalcemia as a cause of constipation. Extremely high yield!!
40. Corrected Ca++ = 0.8(normal albumin 4.0-measured albumin) + measured Ca++
hypocalcemia caused by hypoalbuminemia is not required to be treated.
41. In hyperkalemia, removal of K+ from the body can be achieved with dialysis, cation exchange
resins (kayexalate) or diuretics.
42. Paget’s disease is caused by osteoclast dysfunction which results in a ‘mosaic’ pattern of
lamellar bone. ↑alkaline phosphatase level, and characteristic x-ray findings like femoral
bowing. Bone and joint pain, skeletal deformities and hearing loss (enlarging cranial bones
leads to entrapment of cranial nerve 8) are common symptoms.
43. Nasal eosinophilia –allergic rhinitis, although this finding is not specific. Nasal eosinophilia is
absent in pts with infectious causes and vasomotor rhinitis. Rhinitis Dx—nasal cytology.
44. Pt who develop serous bleeding (eg. Intracerebral hemorrhage) due to excess anticoagulation
with warfarin should be given FFP for the rapid reversal of anticoagulation. Vit K reverses the
action of warfarin, but takes 8-12 hours to be effective. Rapid reduction of blood pressure in
the presence of an acute intracerebral hemorrhage is not recommended. Giving nitroprusside
to rapidly reduce the BP may lead to cerebral hypoperfusion and make matters worse. The
goal systolic BP in such pts is 140-160mmHg.
INR <5, no significant bleeding Omit next warfarin dose
INR 5-9, no significant bleeding Stop warfarin temporarily
INR >9 Stop warfarin, give oral vitamin K

12/20/09

29
1. Pt with PID should be treated with appropriate antibiotics, counseled on safe-sex practices,
and encouraged to inform any sexual partners from the past 60 days of their condition so that
they can be treated as well. Pts with PID should also be screened for HIV, syphilis, HBV,
cervical cancer (pap smear), and HCV if they have a history of IV drug use.
2. Hypothyroidism is an important cause of reversible changes in memory and mentation. It will
be accompanied by systemic changes such as weight gain, fatigue, and constipation.
3. Elevated PT/INR levels in a patient with hepatic failure many be due to vitK deficiency or
liver cirrhosis. Regardless of the cause, the first step in this setting is empiric administration
of vit K, since there is usually an underlying vitK deficiency due to several comorbidities.
FFP is indicated if the pt is actively bleeding, or if the pt needs immediate surgery or an
invasive procedure. Cryoprecipitate is required for coagulation factor deficiencies such as
factor VIII deficiency, Platelet transfusion is indicated only if the pt is actively bleeding, and
the platelet count is less than 20,000 to 30,000/mm3. Packed red cells are usually indicated if
the hemoglobin concentration is <8 in asymptomatic pts, or <10 in symptomatic pts.
4. Multiple observational studies have demonstrated that pts with hypertension have
approximately for times the risk of stroke when compared to non-hypertensive subjects. *
Know the concept of multiple risk factors. Know the association between hypertwnsion and
strokes. This is a very common scenario for USMLE 2 & 3.
5. If the outcome of a case-control study is not common in the population, the odds ratio is close
to the relative risk.
6. In pts who present with stable angina and hypertension, a beta-block is the drug of choice.
7. Bupropion is FDA-approved for smoking cessation. It should be used in conjunction with
counseling and nicotine replacement.
8. Rubella—erythematous and maculopapular. It stars on the face and progresses to the trunk
and extremities. Prodomal symptoms include fever, lymphadenopathy, and malaise. Occipital
and posterior cervical lymphadenopathy are suggestive of the Dx. Adult women usually have
associated arthtitis, which is another diagnostic clue, Some patients may have mile coryza and
conjunctivitis.
measles is also erythematous and maculopapular, and similarly progresses from the head to the
trunk and extremities. There is usually a prodrome of fever, cough, coryza, and conjunctivitis. The
presence of Koplik’s spots is suggestive. Arthritis is not commonly seen.
The rash of chicken pox is pruiritic and usually develops after a prodrome of fever and malaise.
The lesions appear in consecutive crops, so lesions of several different stages are often visible on
examinations (ie. Popular, vesicular and crusted lesions).
Posterior cervical lymphadenopathy and a maculopapular rash man be seen in infectious
mononucleosis; however, these pts usually also complain of a sore throat ad pharyngitis would be
seen on exam. Leukocytosis is common. In infectious mononucleosis, rash ofter develops after the
administration of ampicillin.
Rash with generalized nontender lyphadenopathy is also seen in secondary syphilis. This rash is
classically a symmetric maculopapular eruption involving the entire trunk and extremities,
including the palms and soles.
9. Osteophytes are the most common findings on cervical radiography in pts with cervical
spondylosis, but specificity of these findings is low. Cervical spondylosis-- chronic neck pain,
limited neck rotation and lateral bending is due to osteoarthritis and secondary muscle spasm.

30
Typical radiographic findings include bony spurs and sclerotic facet jounts.
10. alpha-1 antitrypsin deficiency is associated with panacinar emphysema and cirrhosis.
11. Metaprolol is a selective beta1-adrenergic antagonist that when administered in high doses,
can block bronchodilatory beta2-adrenergic receptors causing bronchoconstriction in
susceptible individual. (History of eczema—may mild undiagnosed intermittent asthma.
12. The peripheral skeleton is more commonly affected by RA than the axial skeleton. The
cervical spine is most commonly affected in RA, resulting in C1-C2 instability subaxial
subluxation.
13. Organophosphate poisoning (Farmer, suicide)—bradycardia, miosis, rhonchi, muscle
fasciculations, salivation, lacrimation, urimation and defecation, Atropine administration can
reverse these effects. Of equal importance is the removal of any clothes (which may be
contaminated with pesticides) and washing of the skin to prevent further transcutaneous
absorption.
14. Aminoglycosides are antibiotics used to treat serious gram-negative infections (multi-drug
resistant organism). They are potentially nephrotoxic and drugs levels and renal function must
be monitored closely during therapy.
Nafcillin is a common cause of acute renal failure due to acute interstitial nephritis (AIN). This
drug is used to treat infections caused by methicillin-sensitive Staph aureus (MSSA). AIN—
eosinophils and white blood cell casts present in urine.
15. Seborrheic keratoses are very common benign growths that have a warty or cerebriform,
stuck-on appearance. They are typically pigmented, but may lack pigment in many cases.
An acrochordon is a skin tag. These are usually flesh-colored or pedunculated papules in regions
of the body subjected to friction such as the neck, axillae, and inner thighs.
16. Urinary infection most commonly arises by an ascending route. Sexual intercourse is one of
the most important risk factor for developing uncomplicated UTIs in women due to its
mechanical effect of introducing uropathogens into the bladder (honeymoon cystitis).
17. Blastomycosis is a broad-based budding yeast which is endemic in the south-central and
north-central US. It usually affects the lungs, skin, bones, joint, and prostate. Infection in
immunocompromised hosts is uncommon. Primary pulmonary infection is asymptomatic, or
presents with flu-like symptoms. Cutaneous disease is either verrucous or ulcerative.
Verrucous lesions are initially papulopustular, and then progressively become crusted, heaped
up and warty, with a violaceous hue. These lesions have sharp borders, and may be
surrounded by microabscesses. Wet preparation of purulent material expressed from these
lesions shows the yeast form f the organism.
Histoplasmosis—acute pneumonia, chronic pulmonary histoplasmosis and disseminated
histoplasmosis (HIV).
Coccidioides—pulmonary infection. Cutaneous findings such as erythema multiforme and
erythema nodosum are common.
Invasive aspergillosis—immunocompromised pts, esp transplant pts or those taking cytotoxic
medications.
Nocardia—a subacute pneumonia mimics TB, immunocompromised pts. Actinomyces causes
abscess near the head and neck that drain sulfur granules.
18. Strep pneumoniae is the most common pathogen causing pneumonia in nursing home pts, also
MCC of community-acquired pneumonia in adults. Pts with neurologic disorders (eg.

31
Advanced dementia, parkinson’s disease, and stroke) are at increased risk of aspiration
pneumonia caused by anaerobes.
19. Fever and sore throat in any patient taking antithyroid drug (propylthiouracil, methimazole)
suggests agranulocytosis (immune destruction of granulocytes). Most cases occur within 90
days of tx. Antithyroid drugs should be stopped and WBC count checked. A total WBC count
less than 1,000/cubic mm warrants permanent discontinuation of the drug. Routine WBC
count measurement in pts taking antithyroid drugs is not cost effective.
20. uncomplicated pseudopancreatic cysts smaller than 5cm should be observed for 6 weeks
before any further therapeutic intervention.
21. The MCC of lead poisoning in adults is work exposure. Typical symptoms include GI
complaints, poor concentration, and anemia. The tx is exposure reduction and a lead chelator
such as EDTA or succimer.
22. Hyperparathyroidism is an uncommon cause of secondary hypertension. Asymptomatic
hypercalcemia in hypertensive pt or the presence of ‘stones, bones, groans, and psychiatric
overtones’ should raise suspicion for this condition. DD renal parenchymal disease, renal
artery stenosis, aortic dissections, hypothyroidism, adrenal cortex, medulla
23. Supplementation with folic acid is recommended in all pts with sickle cell anemia to prevent
the occurrence of aplastic crisis. Hydroxyurea produces its beneficial effect by increasing HbF
levels. It is used in pts with sickle cell anemia when their painful episodes occur very
frequently.
24. The odds ratio (OR) is used to interpret a case control study.
OR value interpretation
>1 The factor being studied is a risk factor for the outcome
<1 The factor being studied is a protective factor in respect to the outcome
=1 There is no significant difference in outcome in either exposed or unexposed group

25. Non-invasive testing is critical in the work-up of DVT. For pts at moderate to high risk,
compression ultrasonograph is used to determine the need for anticoagulation. In pts at low
risk, the D-dimer level should be checked. If negative, no anticoagulation is necessary. If
elevated, compression ultrasonography is used to determine the need for anticoagulation.
26. COPD is characterized by progressive expiratory airflow limitation which causes air trapping
and increased lung volumes. An FEV1/FVC ration of less than 0.7 on spirometry is
diagnostic.
27. Acute monocytic (FAB M5) leukemia—leukocytosis with many monoblasts, promonocytes
and monocytes. Monoblasts lack Auer rods, and are peroxidasae negative and nonspecific
(alpha-naphthyl) esterase positive.
Acute myeloblastic leukemia with maturation (M2) –predominance of myeloblasts on the
peripheral blood film.
APML (promyelocytic)—many hypergranular promyelocytes. Each cell had many Auer rods.
There is a high incidence of DIC in these pts
Acute lymphoblastic leukemia—predominance of lymphoblasts on the peripheral blood film.
These lymphoblasts are mostly PAS-positive.
Acute erythroleukemia—erythroblasts, which have an irregular outline and a high nuclear-
cytoplasmic ratio.

32
28. The most beneficial therapy to reduce the progessio of diabetic nephropathy in the presence of
renal insufficiency is strict blood pressure control. The data on dietary restriction and
progression of nephropathy are controversial.
29. Massive doses of aspirin and NSAIDs an cause acute erosive gastritis and upper GI bleeding.
Alcohol can aggravate their effect.
30. Know the complication of ventilation with a high PEEP: alveolar damage, tension
pneumothorax and hypotension. Tension pneumothorax may present with sudden-onset
shortness of breath, hypotension, tachycardia, tracheal deviation, and unilateral absence of
breath sounds.
31. Vit B12 deficiency
Schilling test radiolabeled B12
A dose of radiolabeled oral B12 with intrinsic factor
An intramuscular injection of unlabeled
B12
Urinary excretion of radiolabeled B12
Low dietary intake Normal
Pernicious anemia diminished Normal
Malabsorption (celiac sprue, Low excretion
diphyllobothrium latum
infestation)

32. office-based anoscopy/proctoscopy should be the initial procedure in pts less than 50 years old
who present with minimal bright red blood per rectum (bright red blood on toilet paper after
wiping. A few drops of blood in the toilet bowl after defecation, and small amounts of blood
on the surface of the stool) and do not have any risk factors for colon cancer. Blood
intermixed with stool is not included in this category. MCC: hemorrhoids, anal fissure,
polyps, proctitis, rectal ulcers and cancer.
33. Constrictive pericarditis causes hepatomegaly, ascites and increased JVP due to decreased
diastolic filling and impairment of cardiac output. Common causes include radiation therapy
(Hodgkin lymphoma), viral pericarditis, and cardiac surgery.
34. Allergic contact dermatitis is an inflammatory skin condition caused by allergens such as
poison sumac, cosmetics and nickel. It presents days to weeks after exposure with an intensely
pruritic erythematous rash with vesicles at the site of exposure. Secondary infection as a result
of excessive screatching is possible, and is suggested by pus filled vesicles.
Suppurative hidradenitis is a chronic follicular occlusive disease most commonly affecting the
intertriginous skin, including the axillary, groin and inframammary regions. It appears as painful
inflammatory nodules and draining sinus tracts that can last weeks or longer.
35. Any elderly pt who presents with pneumonia, abdominal pain, confusion and hyponatremia
should be suspected for Legionella pneumonia. This should also be suspected in pts who fail
to respond to beta-lactam antibiotics and Gram stain showing many polymorphs with few
visible organisms. Tx of choice is high dose erythromycin or azithromycin.
36. Know the epidemiology of these classic fungal infections. A pt from Arizona/California
should make you think of coccidiodomycosis.
37. Gallstones are common in the ‘fat, female, fertile woman of forty.’ 80% cholesterol + mixed
(radialucent); 3rd type—calcium bilirubinate. The gallbladder hypomotility and stasis

33
associated with pregnancy lead to the formation of biliary sludge, a precursor to gallstones.
Pregnant pts who experience repeated attacks of biliary colic can undergo cholecystectomy,
preferably during the second trimester. Those pts who prefer not to undergo cholecystectomy
may be given a bile salt such as ursodeoxycholic acid to dissolve the cholesterol stones. The
gallstones frequently recur once the treatment is stopped, however. Gallstones are far more
common in the US and other Western countries than in Aisa.
38. An elevated biliary sphincter pressure is highly specific for sphincter of Oddi dysfunction.
Sphincter dysfunction is characterized by right upper quadrant pain and elevation of liver
enzymes during the pain attack, which resolve when the attacks subside. ERCP with a
sphincterotomy is the most appropriate treatment.
39. Portal hypertention is the MCC of ascites. Portal hypertension is usually due to cirrhosis from
chronic liver disease (alcoholic or viral). Intravenous drug abuse predisposes to cirrhosis by
putting individuals at increased risk for chronic infection with HBV or HCV.
40. In the evaluation and management of acute renal failure (ARF—acute oliguria <250cc urine
in 12 hours; azotemia and increased serum creatinine), foley cathererization is a critical first
step. It can resolve various post-renal obstructions in addition to facilitating monitoring of
urine output in pre-renal and intrinsic renal failure.
41. Hypothyroidism is the most common side effect of radioiodine therapy, but it is easily treated
with levothyroxine therapy. The pt’s eye disease may worsen at the beginning of radioiodine
therapy. Extremely high yield!!
42. Eaton-Lambert syndrome is associated with small cell carcinoma of the lung, and results from
autoantibodies directed against the voltage-gated calcium channels in the presynaptic motor
nerve terminal. Electrophysiological studies confirm the diagnosis (the muscle response to
motor nerve stimulation should increase with repetitive stimulation). Tx consists of
plasmapheresis and immunosuppressive drug therapy.
Autoantibodies against postsynaptic receptors cause myasthenia gravis.
Immune-mediated muscle inflammation suggests polymyositis.
Multicentric CNS inflammation and demyelination are indicative of multiple sclerosis.
43. The 3 main categories of diabetic retinopathy are background or simple (microaneurysms,
hemorrhages, exudates, retinal edema), pre-proliferative (cotton wool spots), and proliferative
or malignant (neovascularization). Visual impairment occurs with the development of macular
edema. Argon laser photocoagulation is preformed for the prevention of complications.
44. Mohs surgery is a special type of surgery wherein microscopic shaving is done, such that 1-
2mm of clear margins are excised. This technique currently has the highest cure rate for basal
cell cancer, but is indicated only in pts with high-risk features, as well as those with lesions in
functionally critical areas (e.g. perioral region, nose, lips , ears).

12/21/09
1. Ventricular remodeling in the weeks to months following myocardial infarction can lead to
dilation of the ventricle. This process is lessened by ACEIs—should be initiated within 24
hours of myocardial infarction in all pts without a contiaindiction.
2. Bacillus cereus causes nausea and vomiting after eating rice. Symptom onset is within 1-6
hours. Staph aureus toxin is present in foods such as dairy, salad, meat, and eggs. Symptoms
include nausea, vomiting, diarrhea, and abdominal pain. Symptom onset is within 1-6 hours.

34
3. Disseminated gonococcal infection often presents with a triad of polyarthralgias,
tenosynovitis (pain elicited along the tendon sheaths), and vesiculopustular skin lesions.
4. Suspect babesiosis in any pt from an endemic area who presents with a tick bite. This illness
is caused by the parasite Babesia and is transmitted by the Ixodes tick. It endemic in the
northeastern US. Following a tick bite, the parasite enters the pt’s RBC and causes hemolysis.
Clinical manifestations vary from asymptomatic infection to hemolytic anemia associated
with jaundice, hemoglobinuria, renal failure, and death. Unlike other tick-borne illness, rash is
not a feature of babesiosis, except in severe infection where thrombocytopenia many cause a
secondary petechial or purpuric rash. Clinical significant illness usually occurs in person over
age 40, pts without a spleen, or immunocompromised individuals. Dx: Giemsa-stained thick
and thin blood smear. Labs—intravascular hemolysis, anemia, thrombocytopenia, mild
leucopenia, atypical lymphocytosis, elevated ESR, abnormal LFT, and decreased serum
complement levels. The two most widely used drug regimens are quinine-clindamycin and
atovaquone-azithromycin.
Ehrlichiosis, or “spotless RMSF”, is another tick-borne illness. Clinical features include fever,
malaise, headache, nausea, and vomiting. Labs show leuckopenia and thrombocytopenia.
Hemolysis and jaundice are not common.
Q fever is a zoonosis caused by Coxiella burnetii. The main sources of human infection are
infected cattle, goat, and sheep. People at risk include meat processing workers and veterinarians.
Infection due to C. burnetii occurs in most areas of the world. Manifestation of Q fever many
include a flu-like syndrome, hepatitis, or pneumonia.
5. Zinc deficiency may result form chronic total parenteral nutrition (TPN) or malabsorption.
Symptoms include alopecia, skin lesions, abnormal taste, and impaired wound healing.
Vit A deficiency many be caused by nutritional deficiency or malabsorption. Symptoms include
blindness, dry skin, and impaired immunity.
Selenium deficiency, like zinc deficiency, may result from chronic TPN. It may also result from
malabsorption or malnutrition (food source include nuts, meat, and fish). The most important
feature is cardiomyopathy.
6. Edema in congestive heart failure is due to both mechanical impedance to fluid removal and
increase renal sodium retention.
7. Esophageal cancer may mimic achalasia. Features that favor malignancy over achalasia are: a
short history, rapid weight loss, and inability of the esophagoscope to pass through the lower
esophageal sphincter.
8. Abdominal ultrasound is the study of choice for diagnosis and follow-up of abdominal aortic
aneurysms, as it has nearly 100% sensitivity and specificity of this condition. Drawbacks to
CT imaging as compared to ultrasound include higher cost, requirement for contrast (which
would be C/I in this pt with chronic renal insufficiency), and poor visualization of aortic
branch origin. CT scan can add valuable information when the differential diagnosis of a pt’s
abdominal pain is broad, as it provides a better overall examination of the abdomen. MRI is a
poor imaging choice for pts with renal insufficiency given the association of nephrogenic
dermatopathy with gadolinium administration. It is also much more expensive than
ultrasound.
9. Any pt who return from a developing country and has symptoms suggestive of malabsorption
should be considered for empirical tx with metronidazole for giadiasis. Remember for the

35
pathophysiology (adhesive disks and malabsorption). Extremely high yield!!
10. Warfarin is commonly used for anticoagulation bz it can be administrated orally. However,
during first few days that it’s given, it can have paradoxically pro-coagulant effects in
susceptible pts by inhibiting protein C production. Affected pts may present with skin necrosis
secondary to thrombus formation.
11. It is important to be able to distinguish between different types of tremors. Resting tremor is
often the presenting symptom Parkinson’s disease. It usually starts in one hand before
involving the other extremities.
Essential tremor is the most common type of tremor, affecting up to 5% of the population. It is
familial in up to 50% of cases. It typically presents as an intention tremor in the upper extremities
that is worst at the end of goal-directed activities (eg. reaching for a pen). The head, chin, voice,
and trunk may also be affected.
Cerebellar dysfunction can cause postural and intention tremors and , when severe, resting
tremors. The tremors have a lower frequency than those in Parkinson (5-7 Hz), typically 3-4Hz.
Cerebellar tremors can affect the extremities and whole head. Nystagmus and other signs of
cerebellar dysfunction are usually also present.
Large fiber peripheral neuropathy may be accompanied by tremor. Tremor is more common in
hereditary neuropathies (eg.Charcot-Marie-Tooth), chronic inflammatory demyelinating
polyneuropathy (CIDP), and the recovery phase of Guillain-Barre syndrome. Weakness, loss of
proprioception, and loss of sensation are often also seen.
12. Acute back pain education—strengthening the supporting muscles (including abdominal
muscles) by regular exercise, choosing an appropriate sleeping posture (eg. Avoiding sleeping
on the stomach), and learning proper techniques for bending and lifting objects. Exercises
with repetitive twisting and bending should be avoided. It is important to bend at the knees,
not at the waist. While lifting an object, one should also bend the knees, keeping the back
straight, this techniques is very useful in preventing strains and back injuries. Warm-up
exercises should be done before any sporting activities.
13. Tick-borne paralysis is characterized by rapidly progressive ascending paralysis, absence of
fever, absence of sensory abnormalities and normal CSF examination.
Guillain-Barre—ascending paralysis over days to weeks. Sensation is either mildly or grossly
abnormal. CSF-albumino-cytologic dissocation. Tx-IVIG and plasmapheresis.
Spinal cord tumor may also present with an ascending paralysis over days to weeks. Sensation is
either mildly or grossly abnormal. MRI of the spine may be used to confirm the Dx. Tx-IV
steroids.
14. All pts who are diagnosed with HIV infection should receive the following: 1) detailed history
and physical examination. 2) Routine chemistry and hematology. 3) Two plasma HIV RNA
levels. 4) CD4 T lymphocyte count. 5) VDRL test for syphilis. 6) PPD skin test. 7) Anti-
Toxoplasma antibody titer. 8) Mini Mental Status Examination. 9) Pneumococcal
polysaccharide vaccine, unless the CD4 counts less than 200cm3. 10) HAV, HBV serology.
11) HAV, HBV vaccine if seronegative. 12) HIV counseling. 13) information and assistance
for those who might have been infected by the subject.
Studies before starting antiretroviral tx: 1) Complete history and physical examination. 2) CBC,
complete blood chemistry, serum transaminases, and lipid profile. 3) CD4 T lymphocyte count. 4)
Plasma HIV RNA titer.

36
PCP prophylaxis: CD4<200/cm3
15. Painless jaundice in an elderly pt should make you think about pancreatic head carcinoma.
DD: both AlT and AST should be high incase of Viral hepatitis and hepato-cellular carcinoma.
16. Dacryocystitis is an infection of the lacrimal sac in the medial canthal region of the eye. Staph
aureus and beta-hemolytic strep are the usual infection organisms. Tx-systemic antibiotic.
Episcleritis 巩膜外层炎 is an infection of the episcleral tissue between the conjunctiva and sclera.
Pts complain of acute onset of mild to moderate discomfort, photophobia, and watery discharge.
Examination reveals diffuse or localized bulbar conjunctival injection.
Hordeolum 麦粒肿(stye) refers to an abscess located over the upper or lower eyelid. It is usually
caused by staph aureus. It appears as a localized red, tender swelling over the eyelid.
Chalazion 睑板腺囊肿 presents as lid discomfort. It is a chronic, granulomatous inflammation of
the meibomian gland. It appears as a hard, painless lid nodule.
17. Predominantly found in children who were given aspirin for virus-induced fever, Reye
syndrome can lead to fulminant hepatic failure. Elevation of ammonia levels and
transaminases, vomiting, and mental status changes are the more common clinical
manifestation of this syndrome (diffuse mitochrondrial injury.)
Balloon degeration with polymorphic cellular infiltrates –acute alcoholic hepatitis.
18. Matching is frequently used in case-control studies bz it is an efficient method to control
confounding. Remember: matching variables should always be the potential confounders of
the study (eg. Age race). Cases and controls are then selected based on the matching variables,
such that both groups have a similar distribution in accordance with the variables.
19. The MCC of iron deficiency anemia in an elderly pt is GI blood loss. A single negative fecal
occult blood test is not sufficient to diagnosis occult blood loss. The next step in evaluation
would be colonoscopy. If negative, upper GI endoscopy. Another indication for performing an
upper GI endoscopy is a current or significant history of NSAIDs and/or history of peptic
ulcer disease. If both negative, capsule endoscopy is preformed to look for small intesitine
disease (eg. Arteriovenous malformations).
20. VitB12 deficiency is common after a total or partial gestrectomy. VitB12 is a necessary
cofactor in the pathway leading to purine synthesis. B12 deficiency decrease DNA synthesis
and erythropoiesis, leading to a megaloblastic anemia.
21. Fibromuscular dysplasia can present as new onset hypertension in children. Bruit or venous
hum may be heard at the costovertebral angle. Angiogram reveals the “string of beads” sign.
22. Neurofibromatosis 1—café-su-lait spots, macrocephaly, feeding problems, short stature, and
learning disabilities. Pts may later develop fibromas. Neurofibromas or different tumors.
Neurofibromatosis 2—bilateral acoustic neuromas and cataracts.
23. Helical CT angiogram of the chest is currently the investigation of choice for dx PE in pts
with normal serum creatinine. Ventilation perfusion scan is ordered in pts where iv contrast is
C/I, such as renal failure pts.
24. Gout is a common manifestation of myeloproliferative disorder-induced increases in uric acid
production. Polycythemia vera commonly causes splenomegaly and characteristic pruritis
with hot baths due to histamine release from an increased number of circulating basophils. Up
to 40% pt suffer from gout.
25. Subdural hematoma results from the rupture of bridging veins. Epidural hematoma results
from the rupture of the middle meningeal artery.

37
26. Suspect ischemic colitis in pts who have evidence of atherosclerotic vascular disease, present
with abdominal pain followed by bloody diarrhea, and have minimal abdominal exam
findings. The most commonly involved segment of the colon is the splenic flexure, because it
is supplied by end arteries (watershed). Another watershed zones in colon is the recto-sigmoid
junction, which is supplied by narrow terminal branches of the inferior mesenteric artery.
27. Mx of nephrolithiasis: 1) imaging study—CT scan of the abdomen without contrast is the
investigation of choice bz of its high sensitivity and specificity. Better than KUB in detecting
radiolucent stones. 2) Narcotics and NSAIDs—equally effective in relieving the pain of acute
renal colic, however, in pts with normal renal function, NSAIDs are preferred over narcotics
bx the latter can exacerbate nausea and vomiting. 3) stones <5mm in diameter typically pass
spontaneously with conservative Mx. This includes a fluid intake of greater than 2L daily.
Increased hydration increases the urinary flow rate and lowers the urinary solute
concentration, thus preventing stone formation. 4) Urology referral—urgent urologic
evaluation is warranted in pts with anuria, urosepsis, or abut renal failure. Hydration is the
cornerstone of therapy for renal stone disease. A detailed metabolic evaluation is not needed
when a pt present with his first renal stone.
28. Massive pulmonary embolism is likely in a postoperative pt with jugular venous distension
and new onset right bundle branch block (right heart stain).
29. Cause of widened mediastinum on CXR include aortic dissection (no hypotension),
mediastinal mass, esophageal rupture, SVC syndrome (dyspnea and facial swelling), anthrax
exposure, and mediastinal hemorrhage (hypotension). The clinical history is critical to making
the Dx.
30. A 50-year-old man at average risk for colon cancer, CAD, and prostate cancer should have
colon cancer screening using any number of modalities, cholesterol measurement, and discuss
about prostate cancer screening with PSA measurement.
31. Hyposthenuria 低渗尿 is common in pts with sickle cell anemia and sickle cell trait.
32. Radioiodine therapy is more likely to cause permanent hypothyroidism in pts with Grave’s
disease than those with multinodular goiter or toxic adenoma (bz the whole thyroid glands is
hyperfunctional).
33. Irritable bowel syndrome is a functional disorder of the GI tract characterized by abdominal
pain with diarrhea and/or constipation. There is no lab or pathologic hallmark.
34. Amiodarone is an effective antiarrhythmic but its use is associated with several potential side
effects. In particular, it can cause pulmonary fibrosis, hepatotoxicity, and thyroid dysfunction.
Pulmonary function tests, LFTs and thyroid function tests should be monitored in pts on
chronic amiodarone therapy.
Hydralazine—salt retention, reflex tachycardia, and a lupus-like syndrome.
Metoprolol-impotence, brahycardia, and AV blockade.
Verapamil—constipation, dizziness, and flushing.
Enalapril—hyperkalemia, cough, decrease GFR, rash and angioedema.
35. Duodenal ulcer disease typically presents with epigastric pain that improves with eating. Over
90% pts with duodenal ulcer disease are infected with H. pylori. Tx is acid suppression and
organism eradication with antibiotics.
36. The nocturnal penile tumescence helps differentiate psychogenic from organic causes of male
erectile dysfunction. It is positive in psychogenic causes and negative in organic cuases.

38
37. Think of malignant melanoma in a pt with a metastatic mass that is bleeding inside brain. 4
tumors in the body not been reported to metastasize to the brain—1) non-melanomatous skin
cancer, 2) oropharyngeal cancer, 3) esophageal carcinoma, 4) prostate cancer
38. Increase intragastric pressure during vomiting can cause tears in the mucosa of the cardia 贲
门, an sometimes of the distal esophagus—Mallory-Weiss tears.
39. Acidosis accompanied by hypercarbia and a normal or elevated serum bicarbonate is
diagnositic of a respiratory acidosis. Hypovantilation is a major cause fo respiratory acidosis
and may occur in the post-ictal state.
40. Suspect celiac disease in any pt who presents with malabsorption and iron deficiency anemia,
Celiac disease is associated with anti-endomysial antibodies (IgA).
Symptoms:
1) characteristically bulky, foul-smelling, floating stool
2) loss of muscle mass or subcutaneous fat
3) pallor-iron deficiency anemia
4) born pain-osteomalacia
5) easy bruising-vitK deficiency
6) hyperkeratosis-vitA deficiency
41. MENII consists of medullary carcinoma of the thyroid, hyperparathyroidism and
pheochromocytoma. The serum calcitonin level is elevated in pts with medullary thyroid
cancer.
42. In pts with fever and a cough productive of foul-smelling sputum after an upper GI endoscopy
or any other instrumentation of the upper airway or esophagus suspect an anaerobic lung
infection. Clindamycin is the most commonly used agent for this type of infection.
43. Amaurosis fugax is the painless loss of vision from emboli, and is a warning sign of an
impending stroke. Most emboli occur from the carotid bifurcation; hence, a duplex untrasound
of the neck should be performed.
44. One of the most practical methods of primary prevention of melanoma is wearing protective
clothing such as tightly woven fabrics, hats, and long sleeved shirts. Currently, the available
data shows little to no protection against melanoma with the use of sunscreen lotions with
SPF 15-30; however, these can protect from non-melanoma skin cancers such as squamous
cell cancer.

12/22/09
1. Ankylosing spondylitis is a seronegative spondyloarthropathy that presents with progressive
inflammatory back pain and stiffness. In the setting of suggestive findings on history and
physical, AS is confirmed by pain film x-ray demonstrating fused sacroiliac joints and/or
bamboo spine. Pts often complaint of morning stiffness lasting >30 minutes, and their back
pain typically improves with exercise. Any young pt presenting with progressive low back
pain and spinal stiffness of > 3 months duration should be investigated for this condition.
2. Biphosphonates (Zoledronic acid) are the drugs of choice for mild to moderate hypercalcemia
due to malignancy (bone metastatic breast cancer, multiple myeloma and radiographic lytic
bone disease.) Corticosteroids are used in pts with hypercalcemia due to extra renal
production of 1,25 hydroxy vitamins D (calcitriol) such as carcoidosis.
3. Alcohol hepatitis is characterized by an AST:ALT>2. It is thought that this transaminase

39
imbalance occurs in alcoholic secondary to a hepatic deficiency of pyridoxal-6-phosphate (a
cofactor for ALT enzymatic activity). The absolute values of serum AST and ALT are almost
always less than 500 IU/L in alcoholic liver disease. If higher, concurrent hepatic injury
secondary to viral, ischemia, or acetaminophen usage. Females are more susceptible than
males to hepatic injury form alcohol ingestion. Fatty liver is present in 80-100% of binge or
heavy drinkers .Classical findings—ballooning degeneration, PMN cellular infiltration,
fibrosis, and necrosis. Mallory bodies are often observed in severe cases, but are not specific
for alcoholic hepatitis.
4. The most typical findings of a PE on arterial blood gas sampling are hypoxemia (increased
A-a gradient) and hypocarbia (tahcypnea-respiratory alkalosis).
5. In all pts with COPD, the two modalities that have been shown to decrease mortality are
home oxygen therapy and smoking cessation.
6. The indication for parathyroidectomy is asymptomatic pts with primary
huyperparathyroidism are : 1) serum calcium level at least 1 mg/dL above the upper limit of
normal. 2) 24-hr uninary calcium >400mg. 3) yong age <50. 4) BMD (bone mineral density)
<T-2.5 at any site. 5) difficulty in follow up of the pt.
7. Pancreatic cancer is the primary differential diagnosis in pts with chronic pancreatitis.
Abdominal ultrosonogram is the initial investigation of choice in pts with jaundice; however,
CT scan has a higher sensitivity for suspected pancreatic cancer. ERCP is most useful for pts
in whom there is a high suspicion for chronic prancreatitis, or in cases where CT or USG fails
to reveal a mass lesion within the pancreas. MRI is the investigation of choice in pts in whom
ERCP is not possible due to pancreatic duct obstruction.
8. Acute pyelonephritis can potentially result in gram-negative sepsis. Urine and blood cultures
should be routinely obtained prior to administering antibiotics (ciprofloxacin or ceftriaxone).
9. Hyperkalemia is a medical emergency. Therapy involves three steps: membrane stabilization
with calcium, shifting potassium intracellularly, and decreasing the total body potassium
content. Insulin/glucose administration is the quickest way to decrease the serum potassium
concentration.
10. Reassurance is the most appropriate course of action for a pts with known immunity to HBV
who is exposed to the disease. The HBIG and the HBV vaccination series should be given to
pts with unknown immunity after exposure.
11. Finasteride acts on epithelium and alpha-1 blockers act on smooth muscles of prostate and
bladder base.
12. Diarrhea (either watery or bloody) due to Vibrio parahaemolyticus is usually transmitted by
the ingestion of seafood (including shrimp, crab, and raw oysters). Campylobacter—
undercooked infected poultry. Watery or hemorrhagic diarrhea, along with severe abdominal
pain.
13. The A-a gradient is increased in interstitial lung disease due to reduced diffusing capacity and
V/Q mismatch. In restrictive lung disease, the total lung capacity (TLC), functional residual
capacity and residual volume are all reduced. Flow volumes are also reduced but the ratio of
FEV1/FVC is either normal or increased.
14. Pts with alpha-thalassemia (target cell) minor or beta-thalassemia minor will have an
impressive microcytosis but only modest anemia. Reassurance is the only intervention
necessary for these pts. In contrast, pts with thalassemia major are generally severely

40
symptomatic and transfusion-dependent.
15. Mucosal neuromas are the most distinct feature of multiple endocrine neoplasia type 2b, and
are present in more than 90% of cases.
MEN I MEN-2a MEN 2b
Pituitary adenoma Medullary thyroid cancer Medullary thyroid cancer
Pancreatic islet cell tumor Pheochromocytoma Pheochromocytoma
hyperparathyroidism Primary parathyroid hyperplasia Mucosal neuromas and
Marfanoid habitus

16. The majority of human infections with E.granulosus are asymptomatic. Hydatid cysts can be
found in almost any part of the body but liver (MC) is involved in 2/3 of pts and the lungs are
involved in 25% of pts. It is most commonly seen in areas where sheep are raised.
17. The kidney compensates for respiratory alkalosis by preferentially excreting bicarbonate in
the urine. The result is an alkalinized urine (increased urine pH).
18. Amyotrophic lateral sclerosis (ALS) is characterized by the presence of both upper
(spasticity, bulbar symptoms, hyperreflexia) and lower (fasciculations) motor neuron lesions.
19. ACE inhibitors, ARBs, beta-blockers, and spironolactone all confer a survival benefit in
CHF. While digoxin and furosemide (loop diuretics) can reduce CHF symptoms and
hospitalizations, they do not improve survival.
20. Hepatorenal syndrome is a deadly complication of cirrhosis characterized by renal failure that
does not respond to volume resuscitation (1.5L normal saline bolus). It is best treat with liver
transplantation.
21. Microcytic anemia and basophilic stippling in a child are important clues to the Dx of lead
poisoning.
22. Elderly pts are particularly predisposed to dehydration after even minor insults (eg. a minor
febrile illness). Know the classic signs of dehydration (ie. Dry mucosa, marginally high
values for hematocrit and serum electrolytes, BUN/creatinin ration >20). The Tx is
administration of iv sodium containing crystalloid solutions (usually 0.9% NaCl= normal
saline). Colloid solutions (eg. albumin) are usually used in burns or conditions accompanied
by hypoproteinemia.
23. Normal pressure hydrocephalus (NPH) is thought to result from decreased CSF absorption or
transient increases in intracranial pressure that cause permanent ventricular enlargement
without chronically increasing intracranial pressure. It is characterized clinically by dementia,
gait disturbances, and incontinence. Increase in CSF production are not strongly associated
with a particular disease process.
24. Fibrolytic therapy improves neurologic outcomes in pts with ischemic strokes when given
within 3 hours of symptom onset. Before administering tPAm a non-contrast head CT should
be performed to rule out hemorrhage stroke and the pt should be screened for other
contraindications to therapy. Urgent anticoagulation with Heparin is not recommended for
acute ischemic stroke, even when the etiology is cardioembolic, because of the increased the
risk for intracereral hemorrhage. However, anticoagulation with warfarin can be started two
weeks after an acute cardioembolic stroke to prevent recurrence.
25. Proper Tx of mild persistent asthma includes a PRN albuterol inhaler and inhaled
corticosteroids.
26. In general, membranous nephropathy is the most common nephropathy associated with

41
carcinoma, however, nephritic syndrome is a well-known complication of Hodgkin’s
lymphoma, and is usually caused by minimal change disease. Extremely high yield!!
27. Factors independently associated with high rates of liver fibrosis in chronic HCV are: 1) male
gender. 2) Acquiring infection after age of 40. 3) Alcohol intake: alcohol intake in any
amount can hasten the progression of fibrosis.
28. Acute exacerbations of MS are treated with corticosteroids. Beta-interferon or glatiramer
acetate is used to decrease the frequency of exacerbations in pts with relapsing-remitting or
secondary progressive form of MS.
29. DM causes a number of complications, with neuropathy being very common. Diabetic most
commonly have (symmetric) peripheral neuropathy, mononeuropathy, and autonomic
neuropathy. Mononeuropathies are divided into cranial and somatic-among cranial nerves.
CNIII is affected the most often. In diabetes, oculomotor nerve (CNIII) neuropathy is
ischemic. Somatic and parasympathetic fibers of CNIII have separate blood supplies. For this
reason, diabetic CNIII neuropathy affects only somatic fibers, leaving parasympathetic fibers
intact. Symptoms are ptosis and a “down and out” gaze. Accommodation and the pupil’s
response to light remain intact. (Alternatively, if nerve compression causes CNIII neuropathy,
both somatic a parasympathetic fibers will be affected. Pts will have ptosis, a “down and out”
gaze, a fixed, dilated pupil, and no accommodation reflex.)
CNIII (oculomotor nerve) has the following functions:
1) Its somatic component innervates the inferior, superior, and medial rectus, inferior
oblique and levator palpebrae muscles. Ptosis occurs due to paralysis of the levator
palpebrae; whereas, the unopposed action of the lateral rectus (CN VI) and superior
oblique (CN IV) muscle lead to a “down and out” gaze.
2) Parasympathetic fibers of CNIII innervated the sphincter of the iris and the cilliary
muscle. Paralysis of parasympathetic fibers causes a fixed, dilated pupil and causes an
inability of the lens to change shape (loss of accomodation).
30. Suspect toxoplasmosis in AIDS pt with multiple ring-enhancing lesion on CT. Administration
of empirical sulfadiazine and pyrimethamine is both diagnostic and therapeutic.
31. Suspect Creutzfeldt-Jacob disease in an old pt (50-70 yeas old) with rapidly progressive
dementia, myoclonus and periodic synchronous bi-triphasic sharp wave complexes on EEG.
DSM-IV criteria for dx of Alzheimer’s disease:
1) Gradual impairment of cognitive function resulting in social or occupational dysfunction.
2) Impaired recent memory with one or more of the following: impaired executive function,
impaired visual processing, and impairment of skilled motor activities.
3) Absent of other psychiatric, neurologic or systemic diseases.
4) Occurrence of deficits not exclusively in the setting of delirium.
32. Nephrotic syndrome is a hypercoagulable condition which manifests as venous or arterial
thrombosis, and even pulmonary embolism. Renal vein thrombosis is the most frequent
manifestation. Complications of nephritic syndrome include: protein malnutrition, iron-
resistant microcytic hypochromic anemia (transferrin loss), increased susceptibility to
infection, and vitamin D deficiency (increase urinary excretion of cholecalciferol-binding
protein).
33. Xanthelasma are cholesterol-filled yellow plaques that appear on the medial aspects of the
eyelids bilaterally. The upper and lower eyelids may be affected. Xanthelasma may occur in

42
the setting of primary biliary cirrhosis though these lesions are most commonly idiopathic.
34. H1 histamine receptor antagonists decrease nasal inflammation and post-nasal drip in pts
with allergic rhinitis. 5-20% pts treated with ACEIs develop dry cough. This is bc decreased
degradation of bradykinin and substance P. Accumulation of these substances causes cough.
35. In otherwise young healthy pts who develop CHF, myocarditis should be considered thigh on
the differential. Vital infection, especially with Coxsachie B virus, is the MCC.
36. Scabies is a high contagious disease which presents with generalized itching and pruritic
papules over the penis and scrotum in males, and on the areolas and breasts in females. For
adults, the tx is 5% permethrin cream, which is applied from the neck down and left
overnight.
37. Suspect leprosy in an Asian patient with an insensate, hypopigmented patch of skin. Dx is
made by demonstration acid –fast bacilli on skin biopsy. Pregrassive peripheral nerve damage
results in muscle atrophy, with consequent crippling deformities of the hands. The most
common affected sites are the face, ears, wrists, buttocks, knees, and eyebrows.
38. The US Preventive Services Task Force (USPSTF) strongly recommends routine screening
for chlamydial infection in all sexually active women age 24 years and younger, and in other
asymptomatic women at increased risk for this infection.
39. The development of clubbing and sudden-onset joint arthropathy in a chronic smoker is
suggestive of hupertrophic osteroarthropathy. This condition if often associated with lung
cancer, and therefore a chest x-ray is indicated to rule out malignancy and /or other lung
pathology.
40. Gouty arthritis cannot be reliably distinguished from septic arthritis or pseudogout base on
history and exam alone; therefore, synovial fluid analysis is critical to diagnosis.
41. Suspect tropical sprue in pts with malabsorption, along with a history of living in endemic
areas for more than one month. Tropical sprue involves the small intestine; the typical biopsy
is characterized by blunting of villi with infiltration of chronic inflammation cells, including
lymphocytes, plasma cells and eosinophils.
42. Ticks should be removed promptly in order to decrease the risk of developing a tick-borne
illness. The recommended technique is to grasp the tick’s mouth parts with tweezers and
remove the tick using slow, constant pressure.
43. Tumor lysis syndrome is characterized by huperphosphatemia, hypocalcemia, hyperkalemia,
and hyperuricemia. Tumors with high cell turnovers (eg, poorly differentiated lymphoma and
leukemias) are the frequent culprits. Both potassium and phosphate are intracellular ions, and
the breakdown of the cells release excess amounts of these. Release phosphate binds calcium
and causes hypocalcemia. Hypocalcemia is also due to the release of intracellular products by
cell lysis. Degeneration of cellular proteins causes elevation of uric acid levels. Allopurinol
may greatly reduce the possibility of acute urate nephropathy.
44. Alcohol abuse is the MCC of nutritional folate deficiency in US and would cause
megaloblastic anemia.

12/24/09
1. In any dog bite, an attempt is made to capture the dog.
1) If the dog is not captured, it is assumed rabies, and post-exposure prophylaxis is
indicated.

43
2) If the dog is captured and does not show features of rabies, it is kept for observation for
the development of rabies for 10 days. If the dog develops any features of rabies, post-
exposure prophylaxis should be started immediately. The dog’s diagnosis is confirmed by
fluorescent antibody (FA) examination of the brain.
3) Post-exposure prophylaxis should be started immediately for exposures involving the
head and neck.
2. Hypoglycemia is associated with multiple differential diagnoses. Helpful tests used in the
evaluation of hypoglycemia patients are measurements of c-peptide, proinsulin and
sulfonylurea levels (surreptitious use of insulin or sulfonulurea). Hypoglycemia secondary to
insulinoma is assocated with elevated sulin, c-peptide and proinsulin levels.
Non-beta cell tumors, typically large mesenchymal tumors, can lead to hypoglycemia independent
of insulin. Such tumors produce insulin-like growth factor II (IGF II), which has an
insulinomimetic action after binding to insulin receptors. In pts with suspected non-beta cell
tumors, the serum IGF II level can be measured. Pts with this condition characteristically have
suppressed insulin and c-peptide levels.
3. Acute bacterial prostatitis—In young pts, the MCC organisms are sexually transmitted
organisms such as Chlamydia and gonococci. In elderly pts, E.coli is the MCC. Prostatic
massage and urethral catheterization are C/I due to the risk of septicemia.
4. Choriocarcinoma is a metastatic form of gestational trophoblastic disease. It may occur after
molar pregnancy or normal gestation, and the lungs are the most frequent site of metastatic
spread. Suspect choriocarcinoma in any postpartum woman with pulmonary symptoms and
multiple nodules on CXR. An elevated beta hCG helps to confirm the diagnosis.
5. Elderly pts are particularly sensitive to fluid loss, and even mild hypovolemia may predispose
them to orthostatic syncopy, especially upon getting up in the morning. BUN/creatinine ratio
is a useful indicator of dehydration.
6. Precision is the measure of random error (not scattered widely). The tighter the confidence
interval, the more precise the result. Increasing the sample size increases precision. Validity
(Specificity and sensitivity) and accuracy are measurements of systemic error (bias).Accuracy
is reduced if the result does not reflect the true value of the parameter measured.
7. The three major mechanical complications of MI include mitral regurgitation due to papillary
muscle rupture, left ventricle free wall rupture, and interventricular septum rupture. While all
3 of these developments can results in hypotension, a pansystolic murmur heard loudest at the
apex with radiation to the axilla is the classic characterization of mitral regurgitation. Left
ventricle free wall rupture classically results in pericardial tamponade. While this condition
often results in sudden death, pts who survive the initial insult may develop hypotension,
jugular venous distension, distant heart sounds, pericardial rub, and pulsus paradoxus. Like
papillary muscle rupture, interventricular septum rupture results in a pansystolic murmur. It
differs in that the murmur is heard best at the left sternal border, often has an accompanying
thrill, and is less likely to radiate to the axilla.
8. Metaclopramide is a prokinetic agent used to treat nausea, vomiting, and gastroparesis. Pts
taking this medication should be monitored closely for the development of drug-induced
extrapyramidal symptoms—tardive dyskinesia, dystonic reactions, and Parkinsonism (neck
pain an neck muscle are noted to be stiff and tender). Tx for a dystonic reaction involves
discontinuation of the medication and administration of benztropine or diphenhydramine.

44
9. External hordeolum or stye is a common staph abscess of the eyelid. It is treated with warm
compresses. Incision and drainage is performed if resolution does not begin in the next 48
hours. Incision and curettage is the Tx of choice for chalazion, which is a granulomatous
inflammation of the meibomian gland.
10. Pts who have received the equivalent of more than one blood volume of blood transfusions or
packed RBC over 24 hours may develop elevated plasma level of citrate (a substance added to
stored blood). Citrate chelates calcium and magnesium and may reduce their plasma levels,
causing parestesias. Stored RBC gradually loose intracellular potassium to the surrounding
solution. This mechanism has the potential to cause hyperkalemia, but not hypocalcemia.
11. Hypokalemia is a common electrolyte abnormality that causes weakness, fatigue, and muscle
cramps. When severe, it can lead to paralysis and arrhythmia. The ECG may show U waves,
flat and broad T waves, and premature ventricular beats.
12. Water restriction is the first step in managing hyponatremia in pts with SIADH. If the serum
sodium is below 120 or if the pt is seizing, emergency tx is asministration of 3% sodium
chloride solution to raise the serum sodium to 125. If the pt has evidence of fluid overloading,
a history of CHF, or is resistant to tx, loop diuretics may be added as well. Chronic tx may
involve administration of lithium or demeclocycline, which inhibit ADH action.
13. Chemical in the eye deserve immediate attention. The best primary course of action is to flush
the eye with water—best achieved under a faucet of running water—for at least 15 minutes.
Obtaining medical care is also appropriate once this treatment is initiated. For foreign bodies
or cuts/scratches of the eye, obtaining medical care is the first priority.
14. The classic physical exam findings of cardiac tamponade are pulsus paradoxus and Beck’s
triad: hypotension, jugular venous distention, and muffled heart sounds. When the pericardial
space filled with fluid, the ventricles are less able to expand to accept venous return to the
heart. As a result, overall preload to the heart decreases.
15. Hereditary spherocytosis is an autosomal dominant disorder of spectrin that leads to increased
RBC membrane fragility. It is characterized by a positive family history, splenomegaly,
spherocytosis, and can cause cholecystitis due to pigmented gallstones (tenderness in the right
upper quadrant exacerbated by deep inspiration.)
Gaucher’s disease can result in anemia, thrombocytopenia, and hepatosplenomegaly. Pts can also
have painful cortical bone erosions. Pts typically present at an earlier age and their pain is more
commonly ‘bone’ pain. Gaucher cells with eccentric nucleoli and PAS inclusions are diagnostic.
16. Acute pancreatitis is most commonly secondary to gallstones or alcohol abuse. However, it
may be also secondary to drug toxicity, idiopathic etiology, heypertriglyceridemia, infection,
severe trauma or burns, hypercalcemia, pancreatic abnormality, tumors, toxins or due to post
operative or post ERCP etiology. Hypertriglyceridemia>1000mg/dl can cause acute
pancreatitis. Hyperlipidemia types 1 and 5 are associated with acute pancreatitis. Acute
pancreatitis sencondary to hypertriglyceridemia often has normal levels of amylase.
17. Paracentesis provides the most useful diagnostic information in pts with ascites. It may reveal
signs suggestive of an exudates: turbid fluid, >250 leukocytes/mL; protein >2.5 g/L; serum-
to-ascitic albumin gradient <1.1; and specific gravity >1.016 g/L. Gram stain of the peritoneal
fluid should be done. Cultures of the blood and peritoneal fluid should also be done. Empiric
antibiotic therapy with third generation cephalosporins (eg. cefotaxime or ceftriaxone) should
be started immediately after obtaining cultures.

45
18. Analgestic nephropathy (eg. aspirin and naproxen, female, 50-55 years) is the most common
form of drug-induced chronic renal failure. Papillary necrosis and chronic tubulointestitial
nephritis are the most common pathologies seen. Pts with chronic analgesic abuse are also
more likely to develop premature aging, atherosclerotic vascular disease, and urinary tract
cancer. Polyuria and sterile pyuria (WBC casts may also be seen) are early manifestations.
Microscopic hematuria and renal colic may occur following sloughing of renal papilla.
Hyertension, mild proteinuria, and impaired urinary concentration commonly occur as the
disease advances.
19. Anticholinergics (benztropine) are useful for Parkinsonism pts younger than 70 years with
disturbing tremors and minimal bradykinesia. Parkinsonism tremor is a resting tremor.
Anticholinergics should be discouraged in pts who are elderly, have dementia or do not have
any tremor. The side effects are similar to those of other anticholinergics. Amantadine is an
antiviral agent, and its mechanism of action is unknown. It has relatively lower efficacy, and
is used for pts with mild disease and no disability. It has the advantage of low toxicity, and
improves all the features of Parkinsonism. Its side effects include confusion, restlessness, skin
rash, depression, edema, nausea, anorexia and postural hypotension; however, these side
effects are uncommon with the usual dose. Selegiline is a selective monoamine oxidase B
inhibitor and neuroprotective agent that may slow down the progression of the disease. It has
a mile symptomatic action, and may be used as an adjunctive treatment in pts receiving L-
dopa. Clozapine is an atypical antipsychotic agent, and is used when pts with Parkinsonism
have hallucinations. Propranolol is used for the treatment of benign essential familial tremors.
Pts with this condiction may have a family history of tremors, and their tremors are usually
absent at rest and appear with emotional stress. * Recognize the different types of tremor and
know how to evaluate and treat them. This is a big time topic for the USMLE!!
20. Presence of hematuria in a pt with irritative or obstructive voiding should alert the physician
to the possibility of bladder cancer. A long history of analgesic use and cigarette smoking aer
risk factors for the development of bladder cancer. Pts with prostate cancer have induration of
their gland on rectal examination and PSA is greater than 4ng/ml in such pts.
21. Chalazion initially presents as a painful swelling that progresses to nodular rubbery lesion.
This is a chronic granulomatous condition that develops when a meibomian gland becomes
obstructed. Persistent or recurrent chalazion may be due to meibomian gland carcinoma
(sebaceous carcinoma). Additionally, basal cell carcinoma frequently presents as a solitary
nodule on the lid margin, and may initially be clinically difficult to distinguish from a
chalazions. Recurrent chalazion requires histopathologic examination to rule out malignancy.
22. In a pt presenting with sudden onset of severe shortness of breath with stridor and coarse
upper respiratory sounds, laryngeal edema is the most likely diagnosis and the pt should be
managed as such.
23. The sudden onset of a sharply-demarcated, erythematous, edematous, tender skin lesion with
raised borders in a febrile pt suggests erysipelas. The most frequently implicated organism is
group A beta-hemolytic streptococcus.
24. There is a lack of physical exam findings and laboratory values specific for osteoarthritis
(OA). Therefore, the diagnosis of OA is often made based upon the overall clinical impression
and supplemented by radiographic findings. There are six criteria used to establish the
diagnosis of OA in the setting of a painful knee: age >50, crepitus, bony enlargement, bony

46
tenderness, and lack of warmth/morning stiffness. If 3 or more criteria are met, the specificity
for OA is 69%.
25. Whipple’s disease can mimic many illnesses, such hyperthyroidism, connective tissue
diseases, alcoholism and AIDS. It should be suspected in all pts with fever of unknown origin,
generalized lymphadenopathy, arthralgias, weight loss, abdominal pain and diarrhea. The
diagnosis of Whipple’s disease can be readily made using upper GI endoscopy and PAS-
staining of the obtained small intestinal biopsies. The classical findings are PAS-positive
material in the lamina propria and villous strophy. In doubtful cases, electron microscopy may
be required to demonstrate the Whipple bacillus for confirmation of diagnosis.
26. High-grade fever and bony tenderness in an IV drug abuser is considered osteomyelitis until
proven otherwise. Empiric antibiotic therapy should be instituted immediately after blood
cultures are obtained. Plain x-ray film are the initial radiologic study in the evaluation of
possible osteomyelitis; however, x-ray are usually normal in the first 2-3 weeks. If the x-rays
are normal, a three phase technetium bone scan is the next test of choice, except in the
evaluation of the spine or foot where MRI is preferred.
27. Marijuana is the most commonly used illicit drug in the US and is involved in 10-50% of car
accidents. Marijuana use can be recognized based on a history of slowed reaction time,
impaired short term memory and increased appetite. In addition, look for conjunctival
injection and dry mouth, as well as hypertension and tachycardia.
Cocaine intoxication is characterized by elevated blood pressure and heart rate and feelings of
euphoria. Unlike marijuana, it causes increased alertness and does not produce increased appetite,
slowed reaction time or conjunctival injection. Stimulant withdrawal, on the other hand, is
associated with increased appetite.
28. Pts with Meniere’s disease typically present with a combination of vertigo, ear fullness,
tinnitus, and hearing loss. First line therapy consists of environmental and dietary
modifications, including maintenance of a low-salt diet. Medical therapy with diuretics,
antihistamines, or anticholinergics is usually considered if lifestyle modifications are
unsuccessful.
29. Iron poisoning disrupts basic processes, causing systemic manifestations including abdominal
pain, hematemesis, shock and metabolic acidosis. It commonly occurs in children of pregnant
women taking prenatal vitamins because children often confuse brightly colored ion pills for
candy (abdominal imaging shows radioopaque tablets in the stomach). Tx involves
deferoxamine, which binds ferric iron, allowing urinary excretion.
30. It is better to keep systolic pressure < 130 and diastolic <80 mmHg to slow end-organ damage
in pts with diabetes and chronic renal failure.
31. Radiation therapy is useful in managing bone pain in pts with prostate cancer who have
undergone orchiectomy. Flutamide combined with a LHRH agonist, prolongs the survival of
pts with limited disease. However, there have been no demonstrable benefits in pts who
underwent orchiectomy.
32. Aortic dissection causes chest pain that is classically described as sudden, tearing, and
radiating to the back. Hypertension is the most common predisposing factor. The decrescendo
diastolic murmur heard here suggests aortic regurgitation, which can be a consequence of
aortic dissection involving the aortic root. Marfan’s syndrome or Ehlers-Danlos syndrome is
not the most common cause of aortic dissection and would tend to present in younger pts.

47
33. Chlamydial urethritis is suggested by mucopurulent urethral discharge, absent bacteriuria
(urine culture shows less than 100 colonies/ml), and history of multiple sexual partners.
34. Renal vein thrombosis is an important complication of nephritic syndrome, which is most
commonly caused by membranous glomerulonephritis in adults.
35. Breast fibrocystic disease is treated with aspiration of the cyst, which should yield clear fluid
and results in the disappearance of the mass. Afterwards, pts are typically observed for 4-6
weeks. Cytology is warranted if the aspirated fluid is bloody or foul smelling.
36. Anticoagulation with warfarin is the most effective way to decrease the risk of embolic
phenomena in pts with atrial fibrillation. Emboli can cause stroke and limb ischemia. Aspirin
and clopidogrel are less effective than warfarin in decreasing embolic complications in pts
with artial fibrillation.
37. Dihydropyridine calcium channel blockers such as nifedipine can worsen cardiac ischemia
since they cause peripheral vasodilation and reflex tachycardia; as such they are C/I in pts
with acute coronary syndrome.
38. When a seizure has been ongoing for more than 30 min (status epilepticus) and has failed to
respond to the usual anti-seizure medications (iv lorazepam, phenytoin, glucose), general
anesthesia and intubation are the next steps. A neuromascular blocker will greatly aid the pt,
an may also be included in his Mx. Work-up is indicated once the seizure is controlled.
39. Central or secondary hypogonadism (uncontrolled diabetes) in males is characterized by low
levels of testosterone with low or inappropriately normal serum gonagotropin levels.
Measurement of serum prolactin levels is the most important biochemical test to perform in
pts with suspected central hypogonadism. Regardless of the cause, high serum prolactin levels
inhibit the release of GnRH, thereby resulting in hypogonadism. Prolactin-secreting pituitary
tumor is one of the most important causes of elevated prolactin levels. Primary (testicular)
hypogonadism is characterized by elevated serum gonadotrophin levels.
40. Plasma aldosterone to plasma renin activity (PA:PRA) is used as an initial screening test for
primary hyperaldosteronism; a ratio over 30 is suggestive of the diagnosis. Non-
suppressibility of aldosterone levels after iv saline or oral salt loading is used to confirm
autonomous production of aldosterone in primary hyperaldosteronism. Once primary
huperaldosteronism is confirmed, adrenal CT is performed to look for adrenal adenoma.
41. Neuroleptic Malignant Syndrome (NMS) is caused by the initiation of dopaminergic
antagonists and typically presents with fever, muscle rigidity (lead pipe rigidity), autonomic
instability and mental status change. An elevated creatine kinase, leukocytosis and electrolyte
abnormalities are also common.
42. Chronic bronchitis is defined as a chronic productive cough that has been present for at least
three months in each of the past 2 years without an alternative diagnosis. Productive cough
and exertional dyspnea are the most common complaints, and symptoms can worsen with
increased mucous buildup in the airways. Therefore, the dx should be considered in pts who
report waking up around the same time each night with dyspnea that resolves with (yellowish)
sputum production. Pts with left ventricle failure often become dyspneic at night, but this is a
result of lying flat (orthopnea) as opposed to a lack of sputum clearance. Pts with pulmonary
edema would also have pink frothy sputum.
43. The pancreatic pseudocyst is an encapsulated area comprised of enzyme-rich fluid, tissue, and
debris that accumulates within the pancreas and causes an inflammatory response. A

48
complication of both acute and chronic pancreatitis, the pseudocyst is best diagnosed by
ultrasound and tends to resolve spontaneously. Hemorrhagic pancreatitis is a sever, life-
threatening illness characterized by retroperitoneal hemorrhage and pancreatic necrosis.
Physical exam findings may include a bluish discoloration of the flanks (Grey Turner sign) or
periumbilical region (Cullen sign) resulting from blood accumulation in abdominal fascial
planes.
44. Symptoms of giant cell arteritis include headache, jaw claudication, muscle fatigue, and visual
disturbance. On exam, pts have scalp tenderness and decreased temporal artery pulse. ESR is
generally more than 50mm/hr. Aortic aneurysms are well-known complications of giant cell
or temporal arteritis due to the involvement of the branches of the aorta. For this reason, pts
should be followed with serial CXR.

12/25/09
1. the stepwise approach of the tx for ascites: 1) sodium and water restriction. 2) spironolactone.
3) Loop diuretic (not more than 1L/day of diuretics due to the risk of hepato-renal syndrome).
4) frequent abdominal paracentesis (2-4 L/day, renal function should be frequently monitored.
Less aggressive paracentesis is recommended in pts with borderline renal function.
Vascular shunts are generally indicated for symptomatic varices (ie. After first bleeding). Distal
spleno-renal shunting will not improve the ascties, and may actually worsen it. Side-to-side portal-
caval shunt will improve the ascite, but may worsen the encephalopathy. Peritoneo-jugular shunt
is designed for the tx of ascites only. Peritonitis, sepsis, and disseminated intravascular
coagulation are its major drawbacks.
2. MEN 2a is a syndrome of pheochromocytoma, medullary carcinoma of thyroid and
hyperparathyroidism. 1) ↑ serum calcium 2) ↑ calcitonin 3) ↓serum phosphorous 4) ↑ urine
metanephrine 5) ↑ serum alkaline phosphatase 6) ↑ urine catecholamines 7) MRI of abdomen
showing adrenal mass 8) thyroid biopsy showing medullary carcinoma cell 9) adrenal biopsy
showing pheochromocytoma 10) parathyroid biopsy showing tumor or hyperplasia 11)
Radioimmune assay of parathyroid hormone showing increased level
3. Porcelain gallbladder results from the intramural deposition of calcium salts and usually
diagnosed on abdominal rediograph. The condition is associated with an increase risk of
gallbladder carcinoma and requires surgical resetion.
4. Diabetics have a high risk for erectile dysfunction—vascular complications, neuropathy and
medications. The first-line drug of tx are the phosphodiesterase inhibitors (sildenafil).
Remember to consider: 1) Sildenadil is C/I in pts being treated with nitrates, and in those who
are hypersensitive to sildenafil. 2) Sildenafil is used with precaution in conditions
predisposing to priapism. 3) concurrent us of drugs which interfere with metabolism of
sildenafil (eg. erythromycin, cimetidine) may predispose to adverse reactions by prolonging
its plasma half life. 4) While combining with an alpha-blocker (tx for BPH), it is important to
give the drugs with at least 4-hour interval to reduce the risk of hypotension.
5. The patient’s symptoms (ie. Spontaneous pain, odynophagia for cold and hot food) are
suggestive of diffuse esophageal spasm. Resolution of her chest pain after taking nitroglycerin
is also consistent with the diagnosis, because nitrates (and Ca channels blockers) relax not
only the myocytes in the vessels, but also all others, including those in esophagus, thereby
alleviating the pain. Esophagography may or may not show other anomalies (eg. corkscrew

49
shape). Esophageal manometry should reveal repetitive, non-peristaltic, high amplitude
contractions, either spontaneously or after ergonovine stimulation.
6. Sporotrichosis is the gardener’s disease. The initial lesion, reddish nodule that later ulcerates,
appear at the site of the thorn prick or other skin injury. From the site of inoculation, the
fungus spreads along the lymphatics forming subcutaneous nodules and ulcers.
7. Contrast administration has the potential to cause contrast-induced nephropathy, particularly
in pts with renal insufficiency (Cr>1.5) and/or diabetes. Non-ionic contrast agents are
associated with lower incidence of nephropathy than the older ionic hyperosmolar agents. In
addition to using non-ionic contrast agents, adequate IV hydration and acetylcysteine can
decrease the incidence of nephropathy. Predinison can be used to prevent hypersensitivity
reactions to contrast media in pts with known dye allergies. Prednisone does not prevent
contrast-induced nephropathy, and in pts with renal insufficiency, prednisone can actually
cause fluid retention. Prednisone can also cause elevations in blood pressure. NSAIDS should
be withheld before the procedure, as they can cause renal vasoconstriction. Statins are known
to improve endothelial function and reduce oxidative stress and may therefore have some
benefit in preventing contrast nephropathy.
8. Fasting blood glucose measurement is now the recommended screening test for DM. A fasting
blood glucose level of 126mg/dL on two separate occasions is diagnositic of DM.
9. Pts with dressler’s syndrome present weeks after a myocardial infarction with chest pain that
is improved by leaning forward. NSAIDs are the tx of choice. Anticoagulation should be
avoided to prevent development of a hemorrhagic pericardial effusion.
10. Recurrent pneumonia in the same anatomic location is a red flag for lung cancer. Furthermore,
smoking is the primary risk factor for bronchogenic carcinoma. When lung cancer is
suspected, CT scan of the chest is warranted. Bronchoscopic (pericronchial) or CT-guided
(peripheral) biopsy may then be performed depending on whether the lesion had a
pericronchial or peripheral location. Bronchoscopy is an invasive procedure that should be
reserved for pts with masses documented by CT scan.
11. Secondary (AA) amyloidosis is frequently the result of chronic systemic inflammation as may
occur in autoummune disorders, chronic infections, IV drug abuse and various other dieases
(psoriasis, IBD, RA). Amyloidosis is a systemic disease, affecting the kidneys (nephritic
syndrome), GI tract (hepatomegaly), heart (cardiomyopathy), musculoskeletal system
(pseudohypertrophy) and nervous system (peripheral neuropathy).
12. Chronic supraphysiological doses of glucocorticoids suppress CRH release from the
hypothalamus, thereby causing adrenal insufficiency. ACTH acts on the adrenal cortex, and is
responsible for the secretion of cortisol and androgen from the zona fasciculate and zona
reticularis. Aldosterone secretion is relatively preserved. Laboratory studies typically show
low ACTH and low cortisol levels.
13. Radioactive iodine therapy is the preferred tx of most patients with Grave’s disease. The C/I
of radioactive iodine tx are pregnancy and very severe ophthalmopathy.
14. Toxic shock syndrome (TSS) is a potential fetal condition caused by toxins produced by
specific strains of Staph, including epidermal exfoliating toxin. The condition is related to
females using tampons and other intravaginal articles. Symptoms include a sudden onset of a
flu-like syndrome, high fever, hypotension, erythematous rash, and signs of systemic failure.
15. TCA overdose is characterized by CNS depression, hypotension, hyperthermia, and

50
anticholinergic effects including dilated pupils, dry flushed skin, and intestinal ileus. It also
causes QRS prolongation on EKG, leaving the patients susceptible to ventricular arrhythmias.
By obtaining an EKG and evaluating the QRS complex, the physician can obtain valuable
prognostic nformation,; namely the patient’s likelihood of developing arrhythmia and seizure.
Sodium bicarbonate shortens the QRS interval in TCA overdose, decreasing the likelihood of
arrhythmia.
16. The tx of choice for PCP is trimethoprim-sulfamethoxazole, and the alternative drug is
pentamidine. Pentamidine has a number of side effects, with 50% of pts developing
hypoglycemia, hyperglycemia, hypocalcemia, azotemia, or liver dysfunction.
17. Several medications can cause hyperkalemia. Examples of these are: ACEIs, NSAIDs, and
potassium-sparing diuretics, such as spironolatone and amiloride. Immediate tx is indicated if
there is cardiac toxicity (as evidenced by EKG changes), muscle paralysis, or K+ >6.5. 10ml
10% calcium gluconate-insulin and /or B2 agonists-Sodium bicarbonate-loop and thiazide
diurectics-cation exchange. Dialysis should be reserved for pts with renal failure and those
with severe life-threatening hyperkalemia that is unresponsive to the more conservative
measures. In pt is symptomatic and with normal EKG findings (ie. Without indications for
immediate drug therapy), may be treated less aggressively by simply stopping usage of
causative drug (ie. amiloride) and rechecking the blood levels after one week.
Pseudohyperkalemia should always be considered in a symptomatic pts, as occasionally the
lab sample can become hemolysed during venipunctue.
18. Hemarthroses are the most common manifestation of hemophilia. The tx of hemophilia
consists of replacing the missing coagulation factor.
19. Guillain-Barre syndrome (GBS) can lead to respiratory muscle weakness and may
occasionally result in respiratory failure. The best way to monitor respiratory function in such
cases is through serial measurements of bedside vital capacity. * Extremely important
question for USMLE!!
20. Pts with chronic liver disease should receive a number of vaccinations. In addition to being
up-to-date with childhood immunizations, they should be vaccinated against HAV, HBV,
influenza (intramuscular inactivated influenza vaccine is preferred over the live attenuated
intranasal vaccine), and invasive pneumococcal disease. Like all adults, they should get a Td
booster at least every 10 years.
21. Neurofibromatosis—decreased hearing and several subcutaneous nodules, hyperpigmented
spots on the back. Chromosome 22. Nonsense and frameshift mutations are typically more
severe that missense mutations. Silent (same sense) mutation do not affect the structure of the
protein.
22. Acute-onset epigastric pain that radiates to the back and is accompanied by nausea, vomiting,
and tenderness is strongly suggestive of acute pancreastitis. The hypotension that can
complicate acute pancreatitis is thought to arise from intravascular volume loss secondary to
local and systemic vascular endothelial injury, which increases vascular permeability and
transudation of plasma. Systemic vasodilation may also contribute.
23. Increase serum calcium and increased PTH levels (8-24 pg/ml) are suspicious for either
primary hyperparathyroidism or familial hypocalciuric hypercalcemia (FHH). Primary
hyperparathyroidism presents with normal or elevated urinary calcium excretion, while FHH
presents with decreased calcium excretion. Pts with VitD intoxication, multiple myeloma,

51
renal cell carcinoma and sarcoidosis may also present with hypercalcemia, but their PTH
levels are typically low due to feedback inhibition.
24. Metabolic acidosis observed during diabetic ketoacidosis is typically accompanied by
hyperkalemia; this is sometimes called paradoxical hyperkalemia because the body potassium
reserves are actually depleted. All hyperkalemic pts should start receiving potassium once the
serum potassium level goes below 4.5mEq/L. In pts with normal or low potassium levels,
potassium replacement should be started with initiation of iv fluid therapy. Extremely high
yield!!
25. Hyperglycemic, hyperosmolar non-ketotic coma is characterized by very high blood glucose
level, plasma hyperosmolality, normal anion gap and negative serum ketones. Serum
osmolarity can be calcultated using the following formula: 1(serum Na) + blood glucose/18 +
BUN/2.8.
26. The most likely diagnosis in a patient presenting with metabolic acidosis, polyuria,
dehydration, decreased level of consciousness, and diffuse abdominal pain in the setting of an
ongoing infection is diabetic ketoacidosis (DKA). DKA is characterized by an osmotic
diuresis that reduces total body potassium stores, even though the serum potassium level may
be elevated.
27. Ischemic hepatic injury occurs in the setting of hypotension and manifests as acute, massive
increases in the AST and ALT with milder associated increases in the total bilirubin and
alkaline phosphatase. In pts who survive the inciting condition, liver enzymes typically return
to normal within a few weeks. HAV or HBV can present with large AST and ALT elevations,
typically accompanied by significant huperbilirubinemia, nausea, and vomiting.
28. The above patient is most likely suffering from MS. Her history is consistent with patchy
neurological problems (eg. right upper extremity, left lower extremity and optic neuritis),
whick is characteristic for MS. Optic neuritis presents as a painful loss of vision, and is an
important presentation. Patients usually have a central visual field defect, and fundoscopy is
normal. Sensory abnormalities may also occur. MRI is the test of choice to support the clinical
diagnosis of MS.
29. Suspect DKA in stuporous pts with rapid breathing and a history of weight loss, polydipsia
and polyuria.
30. Nucleoside reverse transcriptase inhibitors (particularly zidovudine) may cause lactic acidosis
(low pH, primary decrease in HCO3-, compensartory decrease in PaCO2, AG=26) and
steatosis (elevated transaminases and could be further proven by imaging or liver biopsy) due
to inhitibition of DNA polymerase gamma and depletion of mitochondrial DNA (reduced
oxygen utilization by tissues)—acidosis-steatosis syndrome (LASS).
31. Multiple myeloma is a plasma cell disorder which often presents with lytic lesions,
hypercalcemia and renal failure. Marrow plasmacytosis.
32. The goal of tx of pseudotumor cerebri (idiopathic benign intracranial hypertension) include
prevention of visual loss, reduction of intracranial pressure and symptomatic relief. For early
detection of visual loss, there must be routine monitoring of visual acuity, visual fields and
fundoscopy. Intracranial pressure can be reduced by medical or surgical means. Medical
measures are employed firsht and if they fail, surgery is the next option. Surgical options are
optic nerve sheath decompression and lumboperitoneal shunting. Medical tx includes tx of
underlying disorder, pharmacotherapy with acetazolamide (first line, inhibits choroid plexus

52
carbonic anhydrase and thus reduces CSF production and intracranial hypertension) or
corticosteroids and repeated lumbar punctures.
33. The symptoms of BPH are weak urinary stream, urgency, frequency, and sensation of
incompletes voiding. Unlike prostate cancer, BPH starts in the center of the prostate.
Placement of a Foley catheter is the most appropriate initial step in pts who present with acute
renal failure. The obstruction should be reversed within the first 2 weeks to prevent permanent
kidney damage.
34. Decrease in the synthesis of non-cholesterol products may be responsible for some adverse
effects of statin therapy, for example, reduced CoQ10 production is implicated in the
pathogenesis of statin-induced myopathy.
35. in VF and pulseless VT, defibrillation is of primary importance. Time to defibrillation is
strongly correlated with survival. Once defibrillation has been attempted, epinephrine should
be given. Epinephrine administration is then repeated every 3 minutes while CPR is onging.
After a repeated attempt at defibrillation, use of antiarrhythmics (amiodarone, lidocaine, and
magnesium) is warranted.
36. Infection with HEV had a high rate of progression to fulminant hepatitis in pregnant women,
especially in the 3rd trimester.
37. HIV-infected pts should follow the same immunization regimen for teranus and diphtheria
(Td) as the general population. This means they should receive a Td booster at least every 10
years. Also influenza, S.pneumoniae, HAV and HBV.
38. Pts with DVT in whom anticoagulation is C/I (those with recent surgery, hemorrhagic stroke,
bleeding diathesis, or active bleeding) require placement of an inferior vena cava filter to
prevent PE. Compression stocking decrease venous stasis in the lower extremities and are
effective in reducing postoperative venous thrombosis in low risk surgical patients. While
they may be useful in preventing DVTs, compression stockings offer no benefit in the tx of
existing DVTs.
39. Folic acid supplement is encouraged in pts with high risk for developing aplastic crisis
(spherocytosis, AD). Splenectomy is usually indicated for pts who have moderate to severe
disease in order to prevent the complications of the disorder such as aplastic crisis,
cholelithiasis and chronic leg ulcers. Usually splenectomy should be delayed four to five years
to prevent post splenectomy sepsis. All pts prior to splenectomy should receive vaccinations
against encapsulated organisms such as Harmophilus, pneumococci, and meningococci.
40. Recognize the cutaneous lesions of Kaposi sarcoma. In HIV pts it’s caused by Human
herpesvirus 8. Asymptomatic, elliptical, arranged linearly in legs, face, oral cavity, genitalia,
light brown to violet.
41. Laboratory test used in the evaluation of liver disease either assesss liver functionality (eg.
prothombin time, bilirubin, albumin, cholesterol) or structural integrity and cellular intactness
(eg. transaminases, gamma glutaryl transferase, alkaline phosphatase). A progressive decrease
in transaminase levels signals either recovery from liver injury or that few hepatocytes are
functional.
42. Routine urine cultures are not indicated in wormen with incomplicated cystitis. Oral
trimethoprim-sulfamethoxazole is the preferred empiric tx. Ciprofloxacin or nitrofurantoin is
not a first-line empiric therapy, except when there is resistance or an allergy to trimethoprim-
sulfamethoxazole.

53
43. Focal neurologic deficits in a young adult with asymmetric periventricular white matter signal
changes on MRI are most likely due to MS. Glatiramer acetate is a long-term disease
modifying tx for MS that works by modulating T-cell-mediated autoimmunity to myelin basic
protein. Argatroban is an iv anticoagulant that selectively and reversibly inhibits thrombin. Its
main use is to prevent thrombosis in pts with heparin-induced thrombocytopenia who require
anticoagulation.
44. Cancer pts with new or progressive back pain should have detailed neurological examination
for signs of myelopathy. Mx 1) iv steroids (dexamethasone usually) 2) MRI of spine, if MRI
spine is not available or C/I (pts with pacemaker), CT myelogram should be done. 3) Once the
diagnosis is confirmed—radiotherapy. Extremely high yield!!

12/25/09
1. Pts with streptococcal pharyngitis often have a fever, exudative pharyntitis, and tender
cervical lymphadenopathy in the absence of a rash or rhinorrhea. A rapid streptococcal antigen
test can confirm the diagnosis. Tx with penicillin is recommended to hasten recovery, reduce
the risk of rheumatic fever and prevent transmission to close contacts.
2. Paroxysmal nocturnal hemoglobinuria (PNH) is an acquired disorder of hematopoietic cells. It
is characterized by an intravascular hemolytic anemia, a hypercoagulable state, and bone
marrow aplasia. PNH should be considered in the following situations: 1) pancytopenia
(hypocellular bone marrow) accompanied by hemolytic anemia (increased reticulocyte count
and LDH and low haptoglobin levels). Loss of iron in the urine may result in iron deficiency
anemia. 2) Recurrent thrombosis at unusual sites eg. portal vein thrombosis or Budd Chiari
syndrome. Sucrose lysis test and Ham acid hemolysis test were used in the past as screening
and confirmatory tests, respectively. Currently flow cytometry had replaced—CD55 and
CD59 can be analyzed using monoclonal antibodies.
Aplastic anemia is characterized by a hypocellular bone marrow. The peripheral blood smear does
not show any abnormal morphology of cells. RBCs are normocytic or macrocytic. There is neither
hemolyis of RBCs nor splenomegaly. Reticulocyte count will be very low.
3. This patient’s syphilis infection suggests that he may be involved in high-risk sexual activity,
also putting him at risk for HIV exposure. After proper counseling, HIV screening using
ELISA should be offered.
4. If a test result is negative, the probability of having the disease is 1-negative predictive value.
Other cases and diagnostic tests which are high-yield for USMLE are 1) pulmonary embolism
and perfusion-ventilation scanning; 2) prostate cancer and serum PSA level
5. The usual cause of renal artery stenosis in young adults is fibromuscular dysplasia. In older
pts, the cause is usually an atheromatous plaque. The tx of choice for fibromuscular dysplasia
is percutaneous angioplasty with stent placement. * Extremely high yield!!
6. Salmonella is the MCC of osteomyelitis in pts with sickle cell disease (responsible for up to
70% of cases); Staphylococcus aureus accounts for less than 25% of cases. * whether
Salmonella or S. aureus is the MCC in sickle cell pts had been a source of debate; however,
meta-anlysis of the available studies supported that the majority of cases are due to
Salmonella.
7. The cause of vitB12 deficiency include a strict vegetarian diet (after 3-4 years) and pernicious
anemia. Anti-TB drugs such as INH can cause B6 or pyridoxine deficiency.

54
8. Sodium bicarbonate narrows the QRS complex, preventing the development of arrhythmia in
pts with TCA toxicity by alleviating the cardio-depressant action on sodium channels. Sodium
bicarbonate is a critical part of tx in TCA intoxication. Urine alkalinization is a mechanism by
which Sodium bicarbonate increases salicylate excretion and thus, alleviates aspirin overdose.
9. Pts with end stage renal disease have only two tx options: dialysis or renal transplantation.
Renal transplantation is always preferred due to better survival rates and quality of life. A
living related donor is always preferred, followed by a living non-related donor, and lastly, a
cadaveric donor.
10. Most colon cancers develop from polyps. The risk factors for a polyp progressing into
malignancy are villous adenoma, seeeile adenoma, and size >2.5cm. Only adenomatous
polyps are clearly premalignant, but <1% of such lesions progress to malignancy.
Hyperplastic polyps are non-neoplastic and do not require further work-up. Extremely high
yield!!
11. Whenever a healthcare worker is exposed to HIV, baseline HIV testing should be performed
immediately. Postexposure prophylaxis with two or three antiretroviral drugs should be started
without delay.
12. COPD causes airflow limitation and increase lung volumes. The latter results in
diaphragmatic flattening, evident on CXR, which further exacerbates pts’s dypnea and work
of breathing.
13. A tea and toast type of diet is assocated with folic acid deficiency. Folic acid is heat sensitive.
Folic acid deficiency causes macrocytic anemia.
14. Premature atrial beats are benign and neither requires any follow up nor treatment.
15. MI is one of the differential diagnoses of acute abdominal pain and should be ruled out in pts
with risk factors.
16. There are 3 methods to control confounders in the design stage of a study: matching,
restriction and randomization (clinical trial, control known risk factors, as well as unknown
and difficult-to-meaure confounders). The methods to control confounding in the analysis
stage of a study are stratification and modeling.
Selection bias can be controlled by choosing a representative sample of the population for the
study and achieving high rates of follow-up.
Observer’s bias is controlled by the blinding technique.
Ascertainment bias can be avoided by selecting a strict protocol of case ascertainment.
17. The measurement of serum BNP can help distinguish between CHF and other causes of
dyspnea. A value >100pg/mL diagnoses CHF with a sensitivity, specificity, and predictive
accuracy of 90, 76, and 83 percent, respectively.
18. MCC of anionic gap metabolic acidosis are: 1) Lactic acidosis: hypoxia, poor tissue perfusion,
mitochondrial dysfunction 2) ketoacidosis: Type I DM, starvation or alcoholism 3) Methanol
ingestion: Formic acid accumulation 4) Ethylene glycol ingestion: Glycolic and oxalix acid
accumulation 5) Salicylate poisoning: Causes concomitant respiratory alkalosis 6) Uremia
(ESRD): Failure to excrete H+ as NH4+
19. In anion gap acidosis, the anion gap is increased by the abnormal presence of non-chlorinated
acids in the serum.
20. 3 key principles to the use of pain medications in pts with terminal stages of cancer are as
follows: 1) try non-narcotic measures first, unless you are sure (using your clinical judgement)

55
that the pt is in severe pain. 2) Do not be afraid to give narcotic analgesics. 3) Prescribe
adequate amounts of medication. Short-acting morphine → long-acting
21. Bechet’s syndrome is a multi-systemic inflammatory condition characterized by recurrent oral
and genital ulcers, skin lesions; seen most commonly in the Turkish, Asian and Middle
Eastern population. Some pts have involvemen of the GI, skeletal and vascular systems.
Corticosteroids offer relief from most of the symptoms but do not protect from progression to
dementia or blindness.
22. The proxima deep leg veins (illiofemoral vein) are the MCC of symptomatic pulmonary
embolism. Other less common sources of emboli include the calf veins, renal veins, pelvic
veins, upper extremityh veins, and right heart.
23. Propranolol is the drug of choice for benign essential tremor (head and upper expremities are
affected, legs are usually spared. AD, ↑ at the very end of goal-directed activities).
24. DD for thrombocytopenia: Bone marrow disorders—aplastic anemia, hematologic
malignancies, megaloblastic anemia, alcoholism, myelodysplasia. Non BM—ITP, heparin-
induced thrombocytopenia, hupersplenism, DIC, sepsis, viral infection, HIV, SLE, TTp, and
HUS. Thrombocytopenia is the initial presentation of HIV infection in up to 10% of pts. An
HIV antibody test is the first step in diagnosing this disease.
25. Lactulose and a reduced-protein diet are recommended for the tx of acute hepatic
encephalopathy. Non responder—ornithine-aspartate infusion or oral sodium benzoate; A
further alternative—neomycin (long-term use →ototoxicity and nephrotoxicity).
26. In the mx of pts with hyperthyroidism, propanolol is generally used for symptomatic relief
until the underlying cause is identify and definitively treated. *Extremely important!!
27. Factitous thyrotoxicosis results form ingestion of exogenous thyroid hormone. Pts present
with signs and symptoms of hyperthyroidism, but not goiter or exophthalmos. Thyroid
function tests demonstrate low TSH and elevated T3 and T4. The diagnosis many be
confirmed by a 24-hour radioiodine uotake test showing diffusely decreased iodine uptatke by
the thyroid. Biopsy of the thyroid demonstrates follicular atrophy.
28. Vertigo is a sensation of exceeeive motion compared to physical reality. It is most commonly
due to dysfunction within the vestibular system. Disease of the posterior column of the spinal
cord can occur in syphilis (tabes dorsalis) and vit B12 deficiency. Typical symptoms include
ataxia, devreased proprioception and vibratory sense, and hyporeflexia. Cerebellar
dysfunction presents as ataxia, imbalance, incoordination, and nystagmus. Pts with cerebellar
dysfunction will be unable to perform tandem walking due to their imbalance.
29. Suspect aspirin intoxication in a pt with the triad of fever, tinnitus and tachypnea. Adults with
aspirin toxicity develop a mixed respiratory alkalosis and anion gap metabolic acidosis—
characterized by a near-normal pH (7.36), a primary decrase in PaCO2 , and a concurrent
primary decrease in HCO3-.
30. Beta-blockers can worsen severe peripheral vascular disease by causing beta2-mediated
vasoconstriction of the peripheral arteries.
31. Suspect secondary pneumothorax (dilated apical airspaces) in COPD pts presenting with
catastrophic worsening of there respiratory symptoms.
32. Admimistration of DDAVP after water depression can distinguish between central and
nephrogenic diabetic indipidus. Pts with central DI will have an increase of urine osmolality
by at least 50% following the administration of AVP or DDAVP, whereas pts with

56
nephrogenic DI will not have such an increase. The tx of choice for central DI is
desmopressin, which is usually administered intranasally.
33. The absence of forehead furrows indicates peripheral facial nerve palsy (Bell’s palsy) and
excludes central causes.
34. Congenital cataract is the MCC of while reflex in the pediatric population (more than
retinoblastoma).
35. Pts with generalized resistance to thyroid hormones (receptor defects on the peripheral
tissues) have high serum T4 and T3 levels with normal to mildly elevated TSH levels. Pts
typically have features of hypothyroidism despite having elevated free thyroid hormones. Pts
usually present at an early age with growth and mental retardation, however, pts with milder
defects can present later in life.
36. Approximately 10-20% of pts on isoniazid will develop mild aminotransferse elevation within
first few weeks of treatment. This hepatic injury is typically self-limited and will resolve
without intervention. INH has beeb associated with severe INH hepatitis and a much milder
hepatotoxicity. INH hepatitis similar to those seen with viral hepatitis→discountinue the drug
immediately. Most of the first –line antiTB drugs are hepatotoxic and should be replaced by
second-line drugs in pts who develop significant hepatotoxicity with the initial regimen.
37. Lewy body dementia is characterized by flunctuating cognitive impairment, recurrent visual
hallucinations and motor features of Parkinsonism. Pick’s disease is a fronto-temporal
dementia. The typical features include personality change, (euphoria, disinhibition, apathy),
compulsive behaviors (peculiar eating habits, huperorality), and impaired memory. Visual-
spatial functions usually remain intact.
38. A pt with familiar colonic polyposis has a 100% risk of cancer, if not treated appropriately
with a proctocolectomy at the time of diagnosis.
39. Antihypertensive mx should be the first step in pts with aortic dissection with hypertension.
40. Dopamine-agonists such as bromocriptine or cabergoine (new drug with fewer side effects,
more effective) are the mainstay of tx for most pts with prolactinoma
(microadenoma<10mm).
41. GERD is present in up to 75% of asthma pts, and may be the primary trigger in many. Pts
with adult-onset asthma and symptoms that are worse after meals, exercise, or laying down
are likely to have GERD-induced disease. Where GERD is suspected, a trial of PPI can be
both diagnostic and therapeutic.
42. Decrease free water intake will cause hupernatremia due to obligate renal free water losses
and sodium reabsorption by the kidney. This is particularly common in elderly demented pts.
43. Senroineural hearing loss that occurs with aging is referred to as presbycusis. It is usually first
noticed in the sixth decade of life, and presents with high-frequency, bilateral hearing loss. Pts
with presbycusis often have difficulty hearing in noisy, crowded environments. Otosclerosis is
a type of chronic conductive hearing loss associated with bony overgrowth of the stapes. It
typically begins with low-frequency hearing loss and is often found in middle-aged
individuals.
44. Electromyography and conduction studies are the best diagnositic tests for polyneuropathy.
Repetitive stimulation electromyography can be used for the diagnosis for myasthenia gravis.
Electroencephalography detects brain electrical activity. Evoked potentials are very small
electrical impulses monitored from the scalp ather a motor or sensory peripheral, visual or

57
hearing impulse.

12/26/09
1. It is essential to distinguish between a seizure and a syncopal event. A seizure resulting in loss
of consciousness will often be followed by a post-ictal state of clouded sensorium, whereas
people rapidly return to their baseline mental status after a syncopla event.
2. The MCC of folic acid deficiency is nutritional due to poor diet and/or alcoholism. Folic acid
deficiency can be caused by some drugs, these drugs can impair the absorption of folic acid
(eg, phenytoin, primidone and penobarbital) or antagonize its physiologic effects (eg.
methotrexate, trimethoprim). Extremely high yield!!
3. Cauda equina syndrome (CES) is a seious neurologic disorder of the spinal nerve root. CES
can result from any disorder that comprsses the nerves of the cauda equine, include trauma,
lumbar disk disease, malignancy and abscesses. The cauda quina is particularly sensitive to
damage because the nerves have a poorly developed epineurium. Symptoms of CES include
low back pain, bowl and bladder dysfunction, saddle anesthesia, sciatica, and low extremity
sensory and motor loss. Exam will reveal perineal anesthesia, poor rectal tone, low extremity
muscle weakness, reflex abnormalities, and and elevated post-void residual volume. An
emergent MRI of the spine must be obtained when entertaining a diagnosis of CES.
Peripheral nerve disease presents with lower motor neuron symptoms (weakness, hyporeflexia,
decreased muscle tone, fasciculations) and/or decreased sensation. The most common form of
peripheral neuropathy are due to diabetes and alcohol. Bowl and bladder dysfunction, saddle
anesthesia, and back pain suggest that this is CES and not a generalized peripheral neuropathy.
The presence of upper motor neuron findings (such as hyperreflex, hypertonia, spasticity, and
positive Babinski sign) would indicate that the lesion was in the brain or spinal cord. CES
involves spinal nerve roots, which carry lower rather that upper motor neuron fibers.
4. Transfusion reactions remain relatively common and canbe life-treatening. An acute
hemolytic transfusion reaction is a life-threatening reaction resulting form transfusion of
mismatch blood, typically ABO mismatch. Patients rapidly develop fever, hemolysis, shock
and DIC (bleeding around the iv catheta site).
Reaction to cytokines stored in the transfused blood products is thought to be the leading cause of
febrile nonhemolytic reaction, the most common transfusion reaction. During blood storage,
leukocytes release cytokines, whick when transfused cause transient fevers, chills and malaise.
This reaction can largely be prevented by using leukoreduced blood products, particularly if the
RBCs are washed.
IgA deficiency is a major risk factor for developing an anaphylactic reaction to blood products.
Anaphylaxis typically occurs within seconds to minutes of the infusion start time. It develops
because some IgA deficient pts hava anti-IgA IgG antibodies. It can be prevented by using IgA-
deficient blood products or performing extra washes of the blood product.
Despite rigorous screening standards, a vey small percentage of blood products are contaminatd
iwht infectious organisms. The presentation is variable but most pts develop fever within 24 hours.
Delayed hemolytic transfusion reactions result from an anamnestic antibody response to an RBC
antigen to which the pts has previously encountered. Exposure may occur via pregnancy,
transplant, or transfusion. The antibody is undetectable prior to transfusion but reappears rapidly
following transfusion. It typically causes low-grade hemolysis 2-10 days after the transfusion.

58
5. Prophylactic lidocaine decreases the frequency of VPBs, and diminishes the risk of ventricular
fibrillation, but the overall prognosis is unaffected. The problem is that lidocaine can increase
the risk of asystole in these pts. Lidocaine has little effect on myocardial contractility. So
Lidocaine is not used prophylactically in pts with acute coronary syndrome.
6. Lactose intolerance may result in chronic diarrhea, but fecal fat will not be high in such cause.
Bacterial overgrowth, pancreatic insufficiency, Whipple’s and Crohn’s disease all result in chronic
diarrhea with increase fecal fat content. D-xylose is a simple sugar. It does not need to undergo
any digestive process before it can be absorbed. Its absorption requires an intact mucosa only.
In normal individuals, after ingestion of 25 grams of D-xylose, its urinary excretion will be greater
than 4.5 gram in 5 hours. In pancreatic insufficiency, there is deficiency of pancreatic enzymes,
which lead to impaired digestion of fats. D-xylose absorption remains unimpaired, as there is no
damage to small intestinal mucosa.
In terminal ilea disease, bile salts are decrease due to impaired enterohepatic circulation, and as a
result absorption of fats will be impaired. In bacterial overgrowth there will be both impaired fat
absorption due to bacterial de-conjugation of bile as well as decreased excretion of xylose due to
bacterial consumption of the simple sugar. In such cases, urinary excretion of xylose returns to
normal after treatment with antibiotics, while Whipple’s disease does not.
7. Continued pressure on a bony preminence for more than 2 hours can result in ischemic
necrosis of overlying muscle, subcutaneous tissue and skin leading to formation of a pressue
(decubitus) ulcer.
8. When treating pts with pheochromocytoma, do not give beta-blockers without alpha-blockers
(unopposed). Always give an alpha-blocker first, followed by a beta-blocker; doing this in the
wrong order can precipitate a very dangerous increase in blood pressure.
9. Coarctation can present with rib notching on the CXR. A ‘3’ sign is typically seen with
coarctation of longer duration, implying proximal aortic dilation, constriction and descending
aorta dilatation.
10. Nearly 20% of cocaine oversodoses are complicated by rhabdomyolysis, as indicated by
marked elevations on serum creastine phosphokinease (CPK). The main danger associated
wht CPK levels greater than 20,000U/L is acute renal tubular necrosis due to myoglobinuria.
11. Airway disease is the MCC of both mild and massive hemoptysis. Chronic bronchitis is a
frequent cause of mild hemoptyiss in pts with significant smoking history. A low suspicion of
respiratory tract cancer must be maintained, so obtaining a chest X-ray in this pt would be
reasonable. Bronchiectasis is a well-known cause of hemoptysis but much less common than
chronic bronchitis.
12. In a pt wht increased reticulocyte count, increased bilirubin, negative Cooms test and
spherocytes on peripheral blood films (anisocytosis, some spherocytosis, and
polychromatophilia), the diagnosis of Hereditary Spherocytosis is top on the list. MCV is
normal or slightly decreased while MCH is normal but MCHC is generally greater than 36%.
The osmotic fragility test is the next best test for the dignosis of the pt.
13. Squamouse cell carcinoma of the mucosa of the head and neck is common in people with a
significant history of alcohol and tobacco use. The first manifestation may be a palpable
cervical lymph node. The best initial test is panendoscopy (triple endoscopy=esophagoscopy,
bronchoscopy, laryngoscopy) to detect the primary tumor.
14. In US, acute liver failure and fulminant hepatitis are most commonly due to acetaminophen

59
toxity. Remember that acute hepatic failure is defined as the development of liver failure
within 8 weeks of hepatocellular injury onset. If hepatic encephalopathy is also seen, then the
symptom is described as fulminant hepatic failure.
15. In clinical trials, randomization is said to be successful when a similarity of baseline
characteristics of the pts in the tx and placebo groups is seen.
16. The most common site of colon cancer metastasis is the liver. Liver metastases manifest as
right upper quadrant pain, mildly elevated liver enzyme, and firm hepatomegaly. The
diagnosis is confirmed by CT of the abdomen.
17. Sexually active women with a cevix may have Pap smears every 2-3 years if they have had 3
consecutive negative Pap smears and have the same monogamous partner. Screening may stop
at age 70.
18. No matter what the underlying disease of the pt is, remember that the mx of any pt arriving in
the ER begins with assessment of the pt’s “ABC’s”. Somatostatin or octreotide can be used
when endoscopy is unsuccessful, contraindicated, unavailable, or as an adjuncti before
endoscopy.
19. The diagnosis of Gilbert’s syndrome is suggested in those pts with no apparent liver disease
who have mild unconjugated huperbilirubinemia (a familial disorder of bilirubin
glucuronidation in which the production of UDP glucuronul transferases is reduced—enzymes
that mediate glucuronidation of various substances) thought to be provoked by one of the
classic triggers (hemilysis, fasting or consuming a fat-free diet, physical exertion, febrile
illness, stress, or fatigue). Presumptive diagnosis can be made when the unconjugated
huperbilirubinemia persists with repeat testing, but liver function test, complete blood count,
blood smear, and reticulocyte count are normal. Tx generally considered to be unnecessary in
pts with Gilbert’s syndrome. However, its mode of inheritance should be discussed with pts as
a means of preventing needless testing in similarly affected family members.
Crigler-Najjar syndrome type 1 is an autosomal recessive disorder of bilirubin metabolism
characterized by severe jaundice and neurologic impairment due to kernicterus (bilirubin
encephaopathy). In these infants, the indirect bilirubin levels are typically 20-25mg/dL but can rise
to as high as 50mg/dL. Liver enzymes and histology are normal. If iv phenobarbiral is
administered, the serum bilirubin remains unchanged. Phototheray or plasmapheresis are typically
helpful in the short term, though liver transplant is the only curative option.
Crigler-Najjar syndrome type 2 is an mild autosomal recessive disorder of bilirubin metabolism
characterized by low serum bilirubin levels ( < 20 mg/dL) and survival into adulthood with no
kernicterus or neurologic impairment. Liver enzymes and histology are normal. If iv phenobarbiral
is administered, the serum bilirubin is reduced. Tx is often unnecessary in milder cases, though
periodic administration of phenobarbital or clofibrate can reduce serum bilirubin levels if
necessary.
20. Dipsticks are commercially available kits that detect the presence of leukocyte esterase and
nitrie in the urine of pts with suspected UTI. Positive leukocyte esterase signifies significant
pyuria and positive nitrites indicate the presence of Enterobacteriaceae. However, dipsticks
are associated with a high false positive rate and high false negative rate. For this reason, a
negative dipstick test in a pt with symptoms of UTI should still have urine cultures done.
21. Cardiogenic pulmonary edema results from left heart failure and is characterized by crackles
on pulmonary exam. Nitroglycerin is the most rapid acting medication to relieve the

60
symptoms of pulmonary edema (quicker than morphine or loop diuretics). Beyond NTG, loop
diuretics are the mainstay of therapy for decompensated heart failure and principally work by
reducing total body volume.
22. Hypovolemic shock is characterized by hypotension, tachycardia, decreased central venous
pressure (or pulmonary capillary wedge pressure), decreased cardiac output, and increased
peripheral vascular resistance. *Extremely important!!
23. Dilated cardiomyopathy mayh be seen following viral myocarditis (postpartum), particularly
following Coxsackie B infection. Diagnosis is made by echocardiogram, which typically
shows dialated ventricles and diffuse hypokinesia resulting in systolic dysfunction (low
ejection fraction).
24. Thrombocytopenia and hypercoagulation within days of initiating anticoagulant therapy are
most likely caused by unfractionated heparin. Heparin-induced thrombocytopenia is often
associated wth abnormal intravascular thrombosis. The aPTT can become elevated due to
thrombin consumption. Parenteral enoxaparin and other LMWHs may be used in place of
unfractionated heparin in the tx of DVT. However, enoxaparin does not prolong the aPTT.
25. Differentiating preseptal cellulites and orbital cellulites is important in any patient who
presents with a swollen and painful eye accompanied by fever. Pain with eye movement is
more suggestive of orbital cellulites, and proptosis in addition to decreased visual acuity are
almost always seen in pts with orbital cellulites. CT can be used to confirm the extent of
infection and to identify a potential abscess. Cavernous sinus thrombosis (CST) is also
characterized by periorbital edema, exophthalmos, and chemosis, but fundoscopy typically
reveals papilledema and dilated tortuous retinal veins, Although it is often difficult to
differentiate CST from orbital cellulites, the symptoms of CST are often bilateral, and there is
often involvement of cranial nerves III (resulting in ptosis) and V as well as early visual loss.
26. Consider CMV infection in a pt with mononucleosis-like symptoms, typical lymphocytes on
the blood smear, and a negative monospot test. Unlike EBV-associated mononucleosis, sore
throat and lymphadenopathy are uncommon in CMV infection. Peripheral blood smear shows
large basophilic lymphocytes with a vacuolated appearance.
27. Suspect glucagonoma in a pt with huperglycemia, necrolytic migraroey erythema (an
erythematous, scaly plaque) and diarrhea. It is usually malignant, and metastasis to the liver
may occur. The excess glucagon causes symptoms such as glucose intolerance and
hyperglycemia. The diagnosis is confirmed by elevated levels of fasting glucose and serum
glucagons, as well as pancreatic tumor revealed by CT scan. The preferred treatment is
surgical removal of the tumor. The tumor does not respond to chemotherapy.
28. Herpes zoster (shingles) develops when there is reactivation of VZV. Pts experience pain and
a vesicular rash in a dermatomal distribution, The pain may precede the rash by several days
or they may appear concurrently.
29. Expressive aphasia results from the lesions in dominant frontal lobe (motor area of Broca).
Frontal lobe also contains motor cortex whose damage results in contralateral paresis.
Sensory cortex is present in the parietal lobe and damage to sensory cortex results in contralateral
hemianesthesia. Damage to dominant parietal lobe also results in dysgraphia, dyscalculia and
right/left confusion. Damage to nondominant parietal lobe causes construction apraxia,
contralateral sensory neglect and anosognosia. Unilateral lesion of occipital lobe causes
contralateral hemianopia.

61
30. Damage caused by lye (sodium hydroxide) ingestion occurs nearly nstantaneously and its
effects are most pronounced in the esophagus, where liquefactive necrosis occurs. Efforts to
neutralize the alkali, induce vomiting, or administer charcoal do not improve outcomes and
will only complicate management. Early upper gastrointestinal contrast studies and/or
endoscopy are critical for evaluating the extent of damage and determining further tx.
31. Several well-designed clinical trials have proven that interferon-beta decreases the frequency
of relapse, and reduces disability in pts with the relapsing-remitting form of MS.
32. The diagnosis of hypertension requires at least 3 separate blood pressure readings greater than
140/90 mmHg, preferably measured over a period of months. Once the diagnosis is made, the
goal of therapy is to maintain BP below 140/90 mmHg in pts with uncomplicated
hypertension and below 130/80 in pts with DM or renal disease. For pts with a BP ranging
from 120-139/80-89 (pre-hypertension), lifestyle modification in the form of weight loss,
exercise, and decreased salt intake is recommended. Pts with a BP in excess of 140/90 should
also undergo a trial period of lifestyle modification; if the BP remains elevated despite such
measurement, then it is appropriate to initiate antihypertensive therapy. When the BP is in the
range of 140-159/90-99 (stage I), single-drug therapy should be attempt. Hydrochlorothiazide
is the initial antihypertensive of choice in the absent of specific indications for other
antihypertensives. Thiazide diuretics are particularly beneficial in older pts, African-
Americans, and those afflicted with osteoporosis (↓urinary calcium excretion). Tow-drug
combinations are appropriate when the BP exceeds 160/100 (stage II).
Beta-blocker: young, Caucasian pts, hypertension due to systolic heart failure, post-myocardial
infarction, angina, essential tremor, migraine, huperthyroidism, and atrial fibrillation.
ACEIs: hypertension pts with co-existing DM, proteinuric renal failure, systolic heart failure, or a
history of myocardial infarction.
33. Whipple disease is a multisystem disorder with a varied presentation caused by infection with
the gram+ bacillus Tropheryma Whippelii. Chronic malabsorptive diarrhea, weight loss,
migratory non-deforming arthritis, lymphadenopathy and a low-grade fever are the most
common presenting symptoms.
34. Disseminated histoplasmisis is characterized by palatal ulcers, hepatosplenomegaly, and
pancytopenia.
35. The third heart sound (S3) is a low frequency diastolic sound heard just after S2 that is
associated with left ventricular failure. IV diuretics provide symptomatic benefits ot pts with
decompensated heart failure.
36. The low glucose concentration in exudative effusion is due to the high metabolic activity of
leukocytes within the pleural fliud.
37. Scatter plots are useful for crude analysis of data. These can demonstrate the type of
association (linear, non-linear), if any present.
38. Cervicofacial actinomycosis classically presents as a slowly pregressive, non-tender,
indurated mass, which evolves into multiples abscesses, fistulae, and draining sinus tracts
with sulfur granules, which appear yellow. The tx is high-dose penicillin for 6-12 weeks.
39. The gold standard test for making a diagnosis of acromegaly is measurement of growth
hormone levels following an oral glucose load. Most pts with acromagaly are unable to
suppress growth hormone levels following an oral glucose load. GH leads to increase IGF1
formation and secretion mainly by the liver. Most of effects of GH are mediated through

62
IGF1. IGF1 has a very long half-life, and its level does not fluctuate like GH levels. IGF1 is
therefore a good screening test for acromagely. Nontheless, IGF1 is an indirect measure of
GH, diseases of the liver, kidney, and some other systemic illness can lead to spuriously low
IGF1 levels. So IGF1 only for screening.
40. The most common malignancy of the liver is metastasis form another primary source (normal
AFP, multiple nodules of varying sizes). Hepatocellular carcinoma is far less common.
Imaging of HCC usually reveals one lesion with poorly defined margins instead of several
masses of varying sizes.
41. Drugs with anticholinergic properties (amitryptilin) can cause acute urinary retention by
preventing detrusor muscle contraction and urinary sphincter relaxation. The tx is medication
discontinuation and urinary catheterization.
42. Membranous glomerulonephritis is the most likely diagnosis in pts with both HBV infection
and nephrotic syndrome. HCV-cryoglobulinemia, membranoproliferatvie glomerulonephritis.
Minimal change disease—Hodgkin’s lymphoma. HIV-collapsing and focal segmental
glomerulosclerosis. SLE-diffuse proliferative glomerulonephritis.
43. Encapsulated bateria, esp pheumococcus, are the most common cause of pneumonia in HIV
pts.
44. Spironolactone is the diuretic of choice in treating cirrhotic ascites. Tx of cirrhotic ascites: 1)
diagnostic paracentesis, SAAG (serum albumin minus ascetic albumin) >1.1 suggest portal
hupertension. 2) Salt-restricted diet 3) spironolactone, generally in conjunction with
furosemide. 4) large volume paracentesis. 5) transjugular intrahepatic portosystemic shunt
(TIPS) procedure.

12/27/09
1. Screening for ovarian cancer with annual CA125 and transvaginal ultrasound is a reasonable
option for pts who are at higher than average risk fo developing the disease. CA19-9 is a
marker of GI cancers such as pancreatic, gastric, and colon cancers. Pts with who are positive
for BRCA 1 or 2 mutations are typically recommoned to undergo a prophylactic
oophorectomy by age 35 or after completing child bearing. Prophylactic oophorectomy
decreases mortality from ovarian cancer in such pts.
2. Risk is the probability of getting a disease over a certain period of time. To calculate the risk,
divide the numer of disease subjects by the total number of subjects in the study (all people at
risk). a/a+c
3. Always suspect Crohn’s disease in a young pt with chronic bloody diarrhea. Crohn’s disease
can involve the GI tract anywhere, from the esophagus to anus. Ulcerative colitis nealy always
involves the rectal mucosa.
4. A complex partial seizure is characterized by brief (ie. Lasting for a few minutes) episodes of
impaired consciousness, failure to respond to various stimuli during the episode, staring
spells, automatisms (eg. lip smacking, swallowing, picking movements of the hands), and
post-ictla confusion. The EEG pattern is usually normal or may show brief diacharges. Unlike
in absence seizure, hupervantilation during the EEG cannot simulate a complex partial
seizure.
A typical absence seizure is characterized by brief (ie typically lasting for a few seconds) periods
of impaired consciousness. Pts may also present with automatism, but do not have a post-ictal

63
state. Huperventilation during the EEG reveals a generalized 3Hz spike-and –wave patern on a
normal background.
A atypical absence seizure lasts longer. The characteristic EEG patern is slow apike-and-wave
activity with a frequency less than 2.5 Hz.
Juvenile muoclonic epilepsy presents in adolescents with unilateral or bilateral myoclonic jerks.
The symptoms take place in the morning, and may be precipitated by sleep deprivation.
Lennox-Gastaut syndrome usually presents in childhood as seizure of multiple types, impaired
cognitive function, and slow spike-and-wave activity on EEG.
5. The first step in the evaluation of a pt with dizziness is to classify the symptoms as vertigo
(spinning accompanied by nausea), presyncope or disequilibrium. The next step is to classify
the vertigo as either central (A lesion of VIII cranial nerve) or peripheral (inner ear).
Meniere’s disease is the likely cause of vertigo if the pt also has a sensation of ear fullness
(abnormal accumulation of endolymph within the inner ear). Pt with Meniere’s disease will
often complain of hearing loss and tinnitus.
A lesion of VIII cranial nerve can lead to central vertigo, but not lead to ear fullness.
Pts with cerebellar lesions may have vertigo, but it is usually accompanied by other signs of
cerebellar dysfunction, such as incoordination or imbalance.
Lateral medullary infarction, also known as Wallenberg syndrome, can present with intense
vertigo. However, other nsurologic problems such as gaze abnormalities, limb ataxia, sensory loss,
and Horner’s syndrome are often present as well.
6. Tumors that are metastatic to bone cause local osteolysis by production of cytokines such as
IL-1 and tumor necrosis factors (TNF). The most frequent tumors that produce hypercalcemia
by this mechanism are lung cancer and breast cancer.
The most common cause of hupercalcemia in pts with nonmetastatic solid tumors is production of
parathyroid hormone related peptide (PTHrP). In such cases, serum PTH is typically low.
Ectopic PTH production by tumor cell is a very rare cause of hypercalcemia and has been reported
in ovarian tumors, lung cancer and neuroectodermal tumors.
Hypercalcemia is cases of Hodgkin’s disease is almost always produced by calcitriol.
7. All adults should be immunized against diphtheria and tenanus every 10 year. Adult age 50
and older as well as youger adults with certain comorbidities should also hav annual influenza
vaccinations. Pneumococcal vaccine is recommended for all adults age 65 years as older as
well as in younger adults who are at least risk for invasive pneumococcal disease or its
complications. (This includes pts with chronic illness and immunosuppression as well as
cigarette smokers.)
8. Cardiac tamponade is grave condition characterized by hypotension, tachycardia, and
distended jugular veins. The ECG reveals electrical alternans—the amplitude of the QRS
alternates as the heart moves in the fluid-filled pericardial sac. Tx consists of massive volume
resuscitation and emergent pericardiocentesis.
9. PVCs (premature ventricular complexes) in pts post-myocardial infarction and can be
recognized by there widened QRS (>120msec), bizarre morphology, and compensatory pause.
Even though they may indicate a worse prognosis, tx is not indicated unless the pt is
symptomatic. Suppressing PVCs with antiarrhythmis medications has actually been shown to
worsen survival.
10. High-dose niacin therapy that is used to treat liped abnormalities frequently products

64
cutaneous flushing and prutitis. This side effect is explained by prostaglandin-inducec
peripheral vasodilatation and can be reduced by low-does aspirin.
11. Polymyositis is an inflammatory muscle disease of unknown etiology that presents initially
with slowly progressive proximal weakness of the lower extremities characterized by
difficulty with stair climbing or rising from a seated postion. Muscle tenderness also occurs in
some pts (severe tenderness or tenderness in the sbsence of weakness should lead to the
consideration of other dx such as polymyalgia rheumatica). There is no skin findings in PM,
which helps to distinguish it from dermatomuositis. A muscle biopsy is the best diagnosis
study for polymyositis. Biopsy in PM shows a mononuclear infiltration surrounding necrotic
and regenerating muscle fibers. In more advanced disease, an inflammation infiltrate may be
absent and replacement of muscle with fibrotic tissue may be evident.
12. A recently changed mole is a very strong risk factor for malignancy, and is associated with a
relative risk of at least 10.
13. Immunocomplex disease is primarily responsible for IE (Infective endocarditis) -associated
glomerulonephritis, Roth spots, and Osler’s nodes. Janeway lesions (macular, erythematous,
blanching, non-painful lesions on the palm and soles) result from septic embolism.
Embolization of begetation fragements lead to infection or infarction of remote tissues (eg.
pulmonary and splenic infarction).
14. Placing a consolidated lung segment in the dependent position can increase the right-to-left
shunt in a pt with pneumonia. In an upright individual, both ventilation and perfusion are
greatest at the lung bases (the most dependent regions of the lungs) and leasr at the apices.
When lying on one’s righ or left side, the dependent lung gets the greatest ventilation and
perfusion. In condolidated lung segments the alveoli are filled with exudates and do not
participate in pulmonary gas exchange, so their ventilation is essentially zero, ie, blood
perfusing consolidated areas is not oxygenated.
An example of a situation that would cause hypoxia due to increased dead space would be a
pulmonary embolism, where the lung is properly ventilated, but segments of lung are not perfused.
15. Compression fractures typically occur in the setting of decreased bone mineralization or
density. Diagnosis the underlying cause of compression fractures is important in decreasing
the risk of future fracture. Local tenderness is evident upon percussion of the fourth lumbar
vertebra.
Ligamentous sprains of the back can often be traced to a specific event or action. However, they
are more painful with movement and should feel their best in the morning. Pain is typically
perispinal, not directly over a vertebral body.
16. Folic acid and cobalamin deficiencies both cause a macrocytic anemia with hupersegmented
neutrophils. Deficiencies in either nutrient will result in increased homocysteine levels
because both are involved in homocysteine metabolism. Only cobalamin deficiency, however,
will cause in increase in methylmalonic acid concentration (cobalamin is involved in the
conversion of methylmalonyl-CoA to succinyl-CoA).
17. Development of nephropathy is preceded by development of excessive protein excretion, the
initial stages of which is termed microalbuminuria (30-300 mg/24hr, Normal<30mg/24hr).
Spot urine collection and timed urine collection for the measurement of urine microalbumin to
creatinine ratio are generally accepted as good screening methods. Although 24-hour urine
collection is slightly more accurate in screening for microalbuminuria, its inconvenience to

65
pts make it less preferred by physicians. Routine dipstick testing is not recommended during
the initial stages of nephropathy. It only detect macroalbuminiuria >300mg/24hr.\
18. Iron deficiency is the MCC of anemia in elderly pts. Inflammatory joint disease, and not
degenerative joint disease, causes the anemia of chronic disease. A diet poor in green leafy
vegetables and meat can cause folate or VitB12 deficiency, respectively. *Extremely high
yield!!
19. Slurred speech, unsteady gait and drowdiness can be seen in the overdose of multiple drugs.
Benzodiazepine overdose can be distinguished from opioid overdose by the lack of severe
respiratory depression and the lack of papillary constriction. Furthermore, while alcohol
(nystagmus in addition to slurred speech, sedation, and unsteady gait) and phenitoin
intoxication (horizontal nystagmus, cerebellar ataxia and confusion) also share similarities
with benzodiazepine overdose, they can be distinguished by the presence of nystagmus.
20. Cradle cap, or seborrheic dermatitis, is a common pediatric skin condition. This popular, scaly
rash tends to affect the eyebrows, nasolabial folds, and scalp. Tx consists of moisturizers,
antifungals, and topic steroids. DD: Atopic dermatitis, contact dermatitis, tinea capitis,
psoriasis.
21. Incidental discovery of a pulmonary nodule on radiographic imaging is common. The first
step in the work-up of such nodules is to obtain any previous radiographic lung images.
Absence of growth over time rules out malignancy. If previous films are not available, then
the nodules is assigned either a low, intermediate or high probability of malignancy based on
clinical and radiographic data. Low probability nodules are followed by serial resolution chest
CT (3,6,9,12,18, and 24 months), intermediate probability nodules are further investigated
with PET scan and/or biopsy, and high probability nodules are removed surgically.
22. Klebsiella is an encapsulated gram-negative bacillus and can cause pnrumonia in subjects
with debilitating conditions, esp. alcoholics. Friedlander’s pneumonia generally affects the
upper lobe and is characterized by currant jelly-like sputum.
Pseudomonas is also encapsulated gram-negative bacillus and a very common cause of
nosocomial infection.
23. In hyperthyroid pts not pretreated with antythyroid medications, radioactive iodine treatment
may initially cause exacerbations of the thyrotoxic state (dying tryroid cells release excess
thyroid hormone into the circulation and can temporarily aggravate the hyperthyroid state--
new onset atrial fibrillation and hand tremor). Iopanoic acid (telepaque) is a contrast medium
used in radiography. It has antithyroid effects by virtue of its iodine content. It decreases
thyroid hormone release from the overactive thyroid gland and decrease peripheral conversion
of T4 to T3.
24. pts with CHF due to alcoholic dilated cardiomyopathy: thrombocytopenia, macrocytosis,
elevated transminase, coronary angiography excludes ischemia. Total abstinence from alcohol
is the mainstay of alcoholic cardiomyopathy mx and it may reverse this condition if it is
employed earlier in the course of the disease.
25. CO ↓ PCWP ↑: left ventricle failure
CO N PCWP ↑: volume overload
PCWP ↓: hypovolemia-blood loss, intravascular volume redistribution;
Non-cardiogenic pulmonary edema (ARDS) usually had normal or low PCWP.
* extremely high yield!!

66
26. ARDS causes hupoxemia regractory to high inspired oxygen concentrations. Adequate
oxygenation in ARDS often requires positive end-expiratory pressure (PEEP) delivered via
mechanical ventilation.
27. All pts suspected of having Zollinger-Ellison syndrome (prominent gastric folds, ulcer located
beyond the duodenal bulb) should have fasting serum gastrin levels done. A serum gastrin
value greater than 1000pg/ml is diagnositic of the disorder. Pts with non-diagnostic fasting
serum gastrin levels should have a secretin stimulation test done. Secretin stimulates the
release of gastrin by gastrinoma cells. Normal gastric G cells are inhibited by secretin.
28. Valacyclovir is the drug of choice in the tx of herpes zoster, acyclovir is an alternative.
Postherpetic neuralgia can be prevented and /or treated with tricyclic antidepressants such as
amitriptyline or nortriptyline along with acute antiviral therapy.
29. Cherry hemangiomas (senile hemangiomas) are small red cutaneous papules common in
aging adults. They do not regress spontaneously and typically increase in number with age.
Light microscopy of these lesions reveals proliferation of capillaries and postcapillary venules
in the papillary dermis. They are always cutaneous and are not found on the mucosa or deep
tissues.
Strawberry (infantile) hemangiomas appear during the first week of life, initially grow rapidly,
then frequently regress spontaneously by 5-8 years of age. They are bright red when neart the
epidermis and more violaceous when deeper.
Spider angiomas consist of a bright red central papule surroundied by several ourwardly radiating
vessels. They occur due to dilatation of a central arteriole and its superficial capillary network and
are estrogen-dependent.
Cavernous hemangiomas consist of dilated vascular spaces with thin-walled endothelial cells.
They present as soft blue compressible masses up to a few centimeters in size. They may appear
on the skin, mucosa, deep tissue and viscera; when they appear on the skin, they are most
frequently based in the dermis. They are less likely to regress spontaneously than capillary
hemangiomas. Cavernous hemangiomas of the brain and viscera are associated with von Hippel-
Lindau disease.
Cystic hygromas consist of lymphatic cysts lined by a thin endothelium. These benign tumors are
often present at birth, and are most commonly located on the neck and lateral chest wall. Cystic
hygromas are frequently found in neonates with Turner syndrome and Down syndrome.
30. Ursodeoxycholic acid is the most commonly used drug in the tx of PBC as it relieves
symptoms and lengthens transplant-free survival time. Associated conditions: Sjogren’s
syndrome, Raynaud’s syndrome, scleroderma, autoimmune thyroid disease, hypothyroidism,
and celiac disease. Liver transplantation is the sole definitive cure for progressive PBC, but
indicated only in severe liver damage or cirrhosis.
31. A pt’s data may be used for research purposes only after obtaining informed consent.
32. Vaccination against HBV decrease the incidence of hepatocellular carcinoma, esp in regions
with high level of HBV infection such as Asia and Africa.
33. Syncopal episode (caused by Tordades de pointes) without following disorientation (post-
episode confusion is more characteristic for a seizure), hearing impairment, normal physical
exam, and family history of sudden cardia death should make you think of congenital long
‘QT’ syndrome—Jervell-Lange-Nielson, AR. Asymptomatic—beta-blocker. Symptomatic
(syncope) -- beta-blocker+a DDD pacemaker.

67
34. Isolated systolic hypertension (ISH) is an important cause of hupertension in elderly pts. It is
created by dereased elastic properties of the arterial wall. Always treat ISH, in spite of the fact
that diastolic blood pressure is not elevated. The drug of choice would be hydrochlorothiazide.
35. The Waterhouse-Friderichsen syndrome (adrenal glands hemorrhage) is a very serious
complication of Meningococcal meningitis caused by Neisseria meningitidis.
36. Acute lymphoblastic leukemia is predominantly a disease of children. Lymphoblasts lack
peroxidase positive granules but often contain cytoplasmic aggragates of PAS positive
material. Immunostaining for TdT is positive in more than 95% of pts. TdT is expressed only
by pre B and pre T lymphoblasts.
37. The serum ascites albumin gradient (SAAG) is used to distinguish between portal
hypertension and other causes of ascites. A SAAG of 1.1 g/dL or greater is consistent with
portal hypertension (increased hydrostatic pressure within the liver’s capillary beds).
38. Exudate: Pleural fluid protein/serum ratio>0.5; LDH >0.6; Pleural fluid LDH more than 2/3 of
the upper limit of normal serum LDH. Normal pleural pH 7.64; pH<7.3 indicates pleural
inflammation. pH 7.35-transudate. Glucose <60 mg/dL parapneumonic effusion, TB, or RA.
39. SIADH is characterized by hypotonic hyponatremia with euvolemia. Low plasma osmolality
(<280mOsm/Kg) with high urine osmolality (>100-150mOsm/Kg) in suspected pts is
diagnostic.
40. Until proven otherwise, suspect lung cancer in smokers who present with Horner’s syndrome
(partial ptosis, miosis, anhidrosis). Simple CXR should be the first test.
41. Cavernous sinus thrombosis typically occurs in pts with infections of the skin adjacent to the
eye or nose, but ethmous and sphenoid sinusitis can cause this condition as well. Pts usually
present with a headache, low-grade fever, periorbital edema, and cranial nerve palsies (III, VI,
IV, V1, V2).
42. Mitral regurgitation classically results in a holosystolic murmur heard best at the apex with
radiation to the axilla. Common clinical features of MR include exertional dyspnea, fatigue,
atrial fibrillation, and signs of heart failure.
43. Blastomycosis is a pulmonary fungal infection endemic to the Great Lakes, and Mississipi and
Ohio River basins. Systemic blastomycosis may cause skin and bone lesions in addition to
pulmonary manifestations. Broad-based budding yeast grown from the sputum confirm the
diagnosis. Itraconazole or amphotericin B may be used to trat symptomatic disease.
44. The ophthalmic branch (V1) of the trigeminal nerve controls corneal sensation. When it is
damaged, pts can suffer from corneal injury without awareness.

12/27/09
1. Pts with cerebellar hemorrhage present with atxia, vomiting, occipital headache, gaze palsy,
and facial weakness. There is no hemiparesis. It is crucial to make an early and correct
diagnosis, because emergent surgical decompression may be life-saving in such cases.
Pontine hemorrhage accounts for 5-12% of cases of hypertensive intraparenchymal hemorrhage.
Pts present with a deep coma and paraplegia that developed with a few minutes. The pupils are
pinpoint and reactive to light. There is decerebrate rigidity. There are no horizontal eye movement.
2. Acute iron intoxication occurs in 3 phases, beginning with disturbance of the GI mucosa
(30min to 6 hours, nausea, vomiting, hematemesis, melena, abdominal pain) and potentially
progressing to hepatic failure (12-96 hours, shock, metabolic acidosis) and death. The dx is

68
made when serum iron concentrations are greater than 350mcg/dL. Latent phase occurs 6-24
hours.
3. Low back pain in pts with history of malignancy should always raise suspicion for bone
metastasis. Tumors metastatic to bone typically cause constant progressive pain worsened at
rest.
4. Two-sample t test—compare the means of two groups of subjects. Two sample Z test—two
means, but population (not sample) variances are employed in the calculations. Because
population variances are not usually known, the test has limited applicability. ANOVA—
compare 3 or more means. Chi-square—categorical data and proportions. Meat-analysis—an
epidemiologic method of pooling the data from several studies to do an analysis having a
relatively big statistical power.
5. The basic pathophysiologic mechanism of Paget disease is impaired bone remodeling.
6. Chest pain that is reproducible with palpation suggests a musculoskeletal etiology.
7. Observer bias occurs when the investigator’s decision is adversely affected by knowledge of
the exposure status.
8. Chest pain in a young person with cardiovascular risk factors warrants a thorough cardiac
work-up. Aspirin should be the first drug administered when suspicion of a coronary artery
event is high due to its ability to prevent platelet aggregation and coronary spasm.
9. Optic glioma occurs in 15% of pts with neurofibromatosis, type 1. Retinal hamartoma is
typical for tuberous slcerosis. Optic neuritis is frequently the early manifestation of multiple
sclerosis. Visual loss develops typically faster, and no exophthalmos is present. Pituitary
adenoma results in bitemporal hemianopsia.
10. Most thyroid nodules are benign colloid nodules.
11. Porphyria cutanea tarda is a condition that is characterized by painless blisters, hypertrichosis,
and hyperpigmentation. It si often associated with HCV infection, and can be triggered by the
ingestion of certain substance (eg. ethanol, estrogens), which should be discontinued once
suspected.
12. Fecal occult blood testing is the most commonly used screening test for colon cancer. A
positive FOBT test warrants further evaluation using colonoscopy (better than sigmoidoscopy
and double contrast barium enema).
13. treatment initiation thresholds and goals based upon LDL level
14. Alzheimer’s disease is the most common cause of dementia in the US. It affects up to 50% of
people over age 85 and is the 6th leading cause of death among the elderly. Early signs include
visuospatial deficits (e.g getting lost in one’s own neighborhood) and problems with
anterograde memory formation (old memories tend to be preserved). Personality and
behavioral changes (eg. hupersexualit, agitation) may occur as the disease progresses.
Hallucination, changes in alertness, gait impairment, and urinary incontinence are late
findings.
Unlike Alzheimer’s disease, demential with Lewy bodies presents with alterations in alertness,
visual hallucinations, and extrapyramidal symptoms. Memory deficits occur later in the course of
disease as compared to Alzheimer’s.
15. Hypoparathyroidism (PTH deficiency) is characterized by low calcium and elevated
phosphorus level in the presence of normal renal function. Causes of hypoparathyroidism
include post-surgical, autoimmune parathyroid destruction and defective calcium-sensing

69
receptor (pseudohupoparathyroidism).
16. The adult and pediatric pneumococcal vaccines contain capsular polysaccharides and induce a
T-cell-indipendent B-cell response.
Oral polio vaccine—anti-polioviris IgA antibodies in the GI tract.
17. Phencyclidine is a hallucinogenic drug, which characteristically causes vertical nystagmus. It
can also cause dissociative feelings, phychotic and violent behavior, severe hypertension, and
hyperthermia. Tx includes urine acidification, haloperidol in the case of psychotic behavior,
and close monitoring until the condition resolves.
18. Suspect aortic dissection in a pt with acute retrosternal pain and a normal EKG. Check blood
pressure in both arms and auscultate for diastolic murmur of aoric regurgitation.
Transesophageal echocardiography is the preferred diagnosic tool. Before performing the
TEE, HTN shouldn be controlled.
19. CK-MB is the most useful lab test in assessing for coronary re-occlusion after an MI because
it typically returns to normal levels within 1-2 days. While troponin T is generally a more
sensitive marker for cardiac injury, it takes up to 10 days to return to normal after an MI,
making elevations difficult to interpret in this sort of scenario.
20. Chronic inflammatory diarrhea is typically associated with inflammatory changes in the blood
(anemia, elevated ESR, acut phase reactants. Reactive thrombocytosis). Blood/leukocyte-
positive stool is another important finding.
21. The 3 MCC of aortic stenosis in the general polulation are senile calcific aortic stenosis (>70
years old), bicuspid aortic valve, and rheumatic heart disease (less common than the other 2).
A bicuspid aortic valve is the cause of aortic senosis in the majority of pts under 70 years old.
22. sensitivity, specificity
23. The strongest influence on long-term prognosis following an ST-elevation myocardial
infarction is the duration of time that passes before coronary blood flow is restored (PTCA
–‘door-to-balloon time’ less than 90 min or fibrinolysis—‘door-to-needle time’ 30min).
24. Lactose intolerance is characterized by a positive hydrogen breath test, a positive Clinitest of
stool for reducing substances (undigested lactose), and an increased stool osmotic gap.
25. Risk factors for non-alcoholic steatohepatitis include obesity, DM, huperlipidemia, total
parenteral nutrition, and the usage of certain medications (corticosteroids, amiodarone,
diltiazem, tamoxifen, HAART, endocrinopathies (eg. cushing’s syndrome). Labs show mild
elevation in aminotransferases and alkaline phosphatase, with AST/ALT <1.
26. Excisional biopsy with narrow margins is the preferred study for the dx of melanoma. If the
depth of the lesion is less than 1mm, the melanoma can be excised with a 1mm tumor free
margin and they have a 99% 5-year survival. Tumors greater than 1mm in depth should have a
sentinel lymph node study.
27. Reentrant ventricular arrhythmia (ventricular fibrillation) is the MCC of death in pts with
acute MI. Electromechanical dissociation is typical for pulmonary thromboembolism and
pericardial tamponade.
28. Influenza vaccine—annually, pneumococcal not yearly
29. Primary hyperaldosteronism is characterized by perissitently elevated blood pressure,
hypokalemia, metabolic alkalosis, mile huypernatremia, very low plasma rennin activity and
absent edema. (metabolic alkalosis—K+ move into extracellular to correct hypokalemia, H+
move into cells)

70
30. Consider Clostridium difficile diarrhea in all pts who have received antibiotics and develop
nausea, vomiting, abdominal pain, diarrhea, and elevated while count. Cytotoxin assay in the
stool is a highly sensitive test to diagnosis this condition. *Extremely high yield!!
31. The most common renal vascular lesions seen in hypertension are arteriosclerotic lesions of
afferent and efferent renal arterioles and glomerular capillary tufts. DM nephroparhy is
characterized by increasd extracellular matrix, basement membrane thickening, mesangial
expansion, and fibrosis.
32. Cataract is a vision-impairing disease characterized by blurred vision and a glare. It is due to
progressive thickening of the lens. Definitive tx is lens extraction.
Macular degeneration affects central vision. The two forms are atrophic and exudative. The
atrophic type si characterized by drusen and patchy depigmentation in the macular region. The
exudative form is characterized by new blood vessels which can leak, bleed, and scar the retina.
Dry type ARMD usually presents a progressive, slow loss of vision in one or both eyes, while wet
type ARMD presents as an acute distortion in vision, with wavy lines or loss of central vision,
usually in one eye.
33. Trachoma presents with follicular conjunctivitis and pannus (neovascularizatio) formation in
the cornea.
34. Ankylosing spondylitis is a seronegative spondyloarthropathy that typically affects men under
40 years of age. The classic finding is low back pain that is worse in the morning and
gradually improve during the day. Apophyseal joint arthritis
35. In HIV pts, both HSV and VZV can cause severe, acute retinal necrosis associated with pain,
keratitis, uveitis, and fundoscopic findings of peripheral pale lesions and central retinal
necrosis. In contrast, CMV retinitis is painless, not usually associated with keratitis or
conjunctivitis, and characterized by funduscopic findings of hemorrhages and fluffy or
granular lesions around the retinal vessels.
36. Tartrate-resistant acid phosphatase (TRAP) staining is important in the dx of hairy cell
leukemia.
37. The main mechanism responsible for pain relief in pts with anginal pain treated with
nitroglycerin is dilation of veins and decrease in ventricular preload. (dilation of capacitance
vessels).
38. Any elderly pt with bone pain, renal failure, and hypercalcemia has multiple myeloma until
proven otherwise. Approximately 50% of multiple myeloma pts develop some degree of renal
insufficiency, this is most likely due to obstruction of the distal and collecting tubules by large
laminated casts contining paraproteins (mainly Bence Jones protein).
39. Cauda equine is an acute compression syndrome presenting with acute motor and sendory
loss, loss of rectal tone and urinary retention. This is a surgical emergency.
40. Diuretics are a necessary adjunct to the tx of MI if pulmonary edema is present. Beta-blockers
are a standard therapy in MI, but should be avoided in this scenario as they can worsen acute
heart failure.
41. Rotator cuff tears result from chronic rotator cuff tendonitis and shoulder trauma. Shoulder
pain and weakness when lifting the arm above the head is suggestive of rotator suff pathology.
A lidocaine injection ameliorates the pain and weakness of rotator cuff tendonitis, while it
does not improve symptoms of a rotator cuff tear. MRI is excellent at visualizing soft tissue
structures and is the study of choice for dx rotator cuff tear.

71
42. Thiamine deficiency can cause Wernicke’s encephalopathy, the triad of encephalopathy,
oculomotor dysfunction, and gait ataxia. This condition may be induced iatrogenically in
susceptible pts by administration of glucose without thiamine. Chronic thiamine deficiency
can also cause Korsakoff’s syndrome, characterized by irreversible amnesia, confabulation
and apathy.
43. Fanconi’s anemia (FA) is an autosomal recessive disorder marked by progressive bone
marrow failure (macrocytic anemia), areas of skin hypopigmentation, congenital
abnormalities (microcephaly, abnormal thumbs and hypogonadism), and a predisposition to
cancer. Most pts with FA are diagnosed by age 16. Numerous genes have been implicated, all
believed to involved DNA repair. The definitive tx for aplastic anemia is hematopoietic stem
cell transplantation.
44. In pts with suspected MENIIa syndrome, genetic testing had replaced biochemical
measurement of serum calcitonin as the recommended screening test. If genetic analysis is
positive for RET protooncogene mutation, total thyroidectomy is indicated.

12/28/09
1. CT scan is the best test for the diagnosis of diverticulitis in acute setting.
2. CHF exacerbation can cause tychypnea as left ventricular dysfunction allows fluid to pool in
the lungs, causing a pleura effusion and hypoxemia due to reduced ventilation. Tachypnea
causes hypocapnia and respiratory alkalosis. Exam typically shows signs of fluid overload, S3
and S4 gallops, cardiomegaly, and bibasilar crackles in the lungs.
Pulmonary embolism presents with acute-onset dyspnea, tachypnea, and pleural chest pain. Pts
have hypoxemia and respiratory alkalosis with a widen A-a oxygen gradient.
3. Choledochal cysts are congenital abnormalities of the biliary tree characterized by dilatation
of the intra and /or exta hepatic biliary ducts. The clinical presentation varies with the age. In
infants it may present with jaundice and the passage of acholic stools. In children it causes
abdominal pain, jaundice, and attacks of recurrent pancreatitis, which may be eveident by
increase in the amylase and lipase levels. Adults with choledochal cysts commonly present
with vague epigastric or right upper quadrant abdominal pain or cholangitis. Choledochal cyst
can degenerate into cholangiocarcinoma. The initial investigation of choice is an
ultrasounogram followed by CT scan or magnetic resonance imaging (MRI) as needed.
Caroli’s syndrome is a congenital disorder characterized by intrahepatic dilatation of bile ducts.
Biliary atresia usually presents early in the infancy with marked obstructive jaundice and acholic
stool.
4. The typical clinical scenario for vasovagal syncopy (common faint, neurocardiodiogenic
syncopy) including prodrome (lightheadedness, weakness, and blurred vision), provocation by
an emotional situateion, and repid recovery of consciousness. Vasovagal syncope is frequently
recurrent. Upright tilt table testing with or without pharmacologic provocation (isoproterenol)
may be indicated to confirm the diagnosis.
5. Wenchbech or Mobitz type I heart block—a narrow QRS, pregressive increase in PR interval
until a ventricular beat is dropped, then the sequence is repeated. It is a benign arrhythmia and
it transient. Unless the pts is symptomatic, it requires no tx; symtomatic-atropine.
6. Asbestosis is a form of pneumoconiosis resulting from inhalation of particles involved in
many industrial processes. Progressive dyspnea, clubbing and end-inspiratory crackles are

72
typical signs and symptoms. Pulmonary function tests reveal a restrictive lung disease pattern,
with decreased lung volume, decreased DLCO (diffusion lung capacity), and a normal
FEV1/FVC.
7. Opiod intoxication presents with miosis, depressed mental status, decreased respiratory rate,
decreased bowel sounds, hypotension and bradycardia. Of these, decreased respiratory rate is
the best predictor of intoxication and is also a frequent cause of mortality.
8. Herpex simplex keratitis –corneal vesicles and dendritic ulcers. Herpes zoster ophthalmicus is
an infection caused by varicella-zoster virus. Most episodes occur in the elderly. It presents
with fever malaise and a burning, itching sensation in the periorbital region. Examination
reveals a vescular rash in the distribution of the cutaneous branch of the first division of the
trigeminal nerve. Conjunctivitis and dedriform corneal ulcers characterize the eye
involvement.
9. G6PD deficiency is an X-linked recessive disorder commonly seen in African-American men.
It is characterzied by episodic hemolysis in response to oxidants drugs, or fava beans.
Peripheral blood smear reveals bite cells and red blood cells inclusions (Heiz bodies) seen
after crystal violet staining.
10. Conjugared hyperbilirubinemia is mainly because of intrahepatic or extrahepatic obstruction
or congenital impaired hepatic excretion of bilirubin. It is important to rule out an extra
hepatic obstruction with an ultrosound or a CT scan of abdomen. If these imaging techniques
fail to show an extra hepatic biliary dilatatio then the next step would be to go for an ERCP or
PTC (percutaneous tranhepatic cholangiogram).
11. Suspect medullary cystic disease in adults with recurrent UTI or renal stones and typical
radial arrangement contrast filled cysts demonstrated by IVP. It is autosomal dominant
whereas the juvenile form, also known as nephronophthisis is autosomal recessive. There is
no therapy to prevent the disease progression but in most pts the cysts are small and have no
harmful effects. These pts are usually asymptomatic initially; later, they develop UTI and
renal stones. Renal stone are managed in the same manner as pt without medullary cystic
kidney. These pts must take adequate salt and water to replenish the renal losses. Periodic
screening is required for renal stones, hematuria, and UTI. Pts with medullary cystic disease
and hemi-hypertrophy should be screen for cancer.
12. COPD exacerbation is characterized by an acute worsening of symptoms in a pt with chronic
obstructive lung disease. It is classically caused by an upper respiratory tract infection and
may be preceded by cough and fever. Examination typically shows bilateral wheezes, and
ABG finding include respiratory acidosis and hypoxia.
13. Goodpasture’s disease affects the lungs (causing cough, dyspnea, and hemoptysis) and kidney
(nephritic range proteinuria <1.5g/day, acute renal failure, and dysmorphic red cells/red cell
casts on urinalysis). Systemic symptoms are uncommon. Diagnosis is made by renal biopsy
showing linear IgG antibodies along the glomerular basement membrane.
14. Multiple sclerosis is best diagnosed by MRI. The MRI shows white matter disease, which is
very characteristic. Tonometry is done to measure the pressure in the ocular chambers to
assess the presence of glaucoma.
15. Mild manifestation of a drug allergy (urticaria and pruritis without systemic symptoms) are
usually treated with antihistamines and discontinuation of the offending drug. Tx with
adrenalin and steroids are reserved for more severe reactions accompanied by systemic

73
symptoms (anaphylaxis).
16. Short-acting beta-adrenergic agonists administered 20 min before exercise are the first-line
treatment for isolated exercise-induced asthma. The inciting trigger in exercise-induced
asthma is rapid ventilation of cold, dry air which stimulates mast cell degranulation and
airway constriction. Long acting beta-adrenergic agonists may be used in children or athletes
who engage in athletic activities throughout the day.
17. Acetaminophen should be the initial tx for pts with mild to moderate pain from osteoarthritis.
It has proven efficacy and a favorable side effect profile.
18. A copious amount of purulent drainage in newborns who are 2-5 days old is most consistent
with gonococcal conjunctivitis. Conjunctivitis casued by Chlamydia trachomatis is usually
milder and presents more than 5 days after birth. Chemical conjuctivitis can result from
prophylactic silver nitrate eye drops, but it is usually fairly mild.
19. All pts with suspected bacterial pneumonia should have a CXR done as the first step, and
antibiotics should be administrated as soon as possible without waiting for sputum gram stain
or cultures.
20. First-degree heart block is completely benign arrhythmia and requires no tx.
21. CT scan is the best test for diagnosisng and evaluating the abdomen of pts during an acute
episode of diverticulitis.
22. Non-selective beta-blockers (propranolol) are used for the primary and secondary prevention
of variceal bleeding in cirrhotic pts who have portal hypertension with esophegeal varices.
23. ACE inhibitors are the drug of choice for lowering blood pressure to target levels (<130/80
mmHg) in DM.
24. Gastric outlet obstruction typically presents with persistent vomiting, which leads to
hypokalemic, hypochloremic metabolic alkalosis. Emergent tx includes nasogastric suctioning
to deccompress the stomach, hydration with sodium chloride, and potassium replacement.
25. case control study
26. Post cholecystectomy pain most commonly occurs due to one of three reasons: common bile
duct stone, sphincter of Oddi dysfunction or functional causes (no obvious organic basis and
should treated symptomatically with analgesics and reassurance).
27. Aortic regurgitation produces an early diastolic murmur and can be associated with several
physical signs caused by a hyperdynamic pulse, including bounding or “water hammer”
peripheral pulses.
28. A confounder is an extraneous factor which had properties linking it with the exposure and
outcome of interest.
29. Decreased esophageal body peristalsis and poor relaxation of the LES on manometry are
typical for achalasia. Achalasia presents with progressive dysphagia, chest pain, food
regurgitation and aspiration. Barium swallow shows a dilated esophagus and a “bird beak”
deformity of the LES.
30. Indications for surgery in pts with AAA (abdominal aortic aneurysms) include: diameter
greater than about 5cm, presence of symptoms, or rapid rate of growth. For pts being
medically managed, smoking cessation is the intervention with the greatest likelihood of
slowing AAA progression.
31. Condyloma acuminata are verrucous, papilliform lesions located in the anogenital region.
Podophyllin is one of the available tx options.

74
32. Atheroembolism results from dislodgement of cholesterol plaques from the aortic root. “Blue
toe syndrome”, in which emboli to the pedal circulation cause cyanotic and painful toes with
intact pulses, is one common presentation of cather-induced atheroembolism.
33. Histopathologically, demonstration of invasion of the capsule and blood vessels is required for
differentiating follicular cancers from follicular adenomas. Follicular thyroid cancers have the
propensity to invade blood vessels and metastasize to distal organs.
34. Suspect hemochromatosis in a patient with new-onset diabetes, arthropathy, and
hepatomegaly.
35. Suspect malignant otitis extrena in any diabetic pt with severe ear pain, otorrhea, and evidence
of granulation tissue in the ear canal. Pseudomonas aeruginosa is the most frequent cause of
malignant otitix externa.
36. Inflammatory bowel disease may occur in association with an inflammatory arthritis.
Ankylosing spondylitis and IBD are both associated with HLA-B27 and may occur in
association with one another. Both conditions may also be associated with a positive p-ANCA
despite the absence of vasculitis in both conditions.
37. Polymyalgia rheumatica (PMR) is characterized by pain and stiffness in the neck, shoulders
and pelvic girdle in a patient over 50 years old with an elevated ESR and morning stiffness
lasting over one hour. The treatment of choice for PMR when it is not associated with giant
cell arteritis is low-dose prednisone. In PMR, the physical examinaiton is frequently
unremarkable with patients denying tenderness with active or passive range of motion. Signs
of inflammation in the joints are absent. When asked to identify the location of their pain, pts
typically indicate that it is in the soft tissues and not the joints.
38. Suspect ehrlichiosis in any pt from an endemic region with a history of tick bite, systemic
symptoms, leukopenia and/or thrombocytopenia, and elevated aminotransferses. The drug of
choice is doxycycline.
39. Always consider malaria in patients from endemic areas with high-grade periodic fever and
chills. Anemia and splenomegaly are the clinical clues.
40. The most likely diagnosis of an asymptomatic (ie. Lesion-free), immunocompetent adult pt
with a non-healing, isolated ulcer in the vermilion zone of the lower lip and a significant
history of sun exposure is squamous cell carcinoma.
41. Raising the cut-off point of a diagnostic test decreases its sensitivity, but increases its
specificity. Consequently, TN is increased and FP is decreased.
42. Heparin-induced thrombocytopenia (HIT) is an immune-mediated process that causes
thrombocytopenia several days after heparin therapy is started. Somewhat paradoxically, HIT
is highly thrombogenic and pts must be monitored closely for arterial and venous clots.
43. Plain film of an osteoarthritic knee reveal narrowed joint space, osteophytes, and subchondral
sclerosis or cysts. Obesity is the most readily modifiable risk factor of OA, and weight loss
decreases joint pain, increases function, and slows progression of the disease. Pharmacologic
therapy and exercise also play integral roles in OA therapy.
44. Pharygoesophageal (Zenker’s) diverticulum develops immediately above the upper
esophageal sphincter by herniating posteriorly between the fibers of cricopharyngeal muscle.
Motor dysfunction and incoordination are responsible for the preblem. * The surgical tx of the
disorder includes excision and frequently cricopharyngeal myotomy. * Extremely high yield!!

75
12/29/09
1. Tinea corporis is charecterized by ring-shaped scaly patches with central clearing and dintinct
borders. Topical tx with 2% antifungal lotions and creams (eg. terbinafine), or systemic tx
with griseofulvin (for extensive dieasea) offer good relief.
2. The incidence of a disease is not changed by any kind of treatment. On the other hand, the
prevalence may be affected by tx of the diease. In this case, tx of an acute and rapid fatal
diease (eg. lung cancer) prolonged the suevival of pts; however, such tx did not cure the
disease. This will result in more people having the said disease at one point in time; hence, the
prevalence will increase.
3. Pts with Hashimot’s thyroiditis (postive anti-thyroperoxidase) are at increased risk for
developing thyroid lymphoma.
4. Choose the appropriate initial antihypertensive therapy in a pt with asthma.
Hydrochlorothiazide is the initial drug of choice the general population, and beta-blockers are
relatively C/I with asthma.
5. Neurofibromatosis type 1 is an autosomal dominant disorder.
6. ACEIs have been shown in multiple studies to improve mortality in pts with CHF. Other
commonly used medications with a mortality benefit in CHF include beta-blockers,
angiotensin II receptors blockers, and spironolactone.
7. Raising cut-off point (eg. increasing the inclusion criteria) of a screening test results in an
increase in speicificity and decease in sensitivity.
8. Ethylene glycol is associated with hypocalcemia and calcium oxalate deposition in the
kidneys. The leads to flank pain, hematuria, acute renal failure and anion gap metaboic
acidosis. Tx involves administration fomepizile or ethanol to achieve ADH dehydrogenase
inhibition, sodium bicarbonate to allevate acidosis and hemodialysis in the case of acidosis
and /or end organ damage.
9. Viral infection may present with symmetric arthritis. Viral arthritis is distinguished form RA
and other causes of symmetric arthritis by its acute onset, lack of elevated inflammation
markers, and resolution with two months.
10. Achalasia—the recent onset of hysphagia to both solids and liquids is characteristic. Reflux of
food particles from the esophagus at night may lead to aspiration symptoms, as in this case.
The typical rediologic finding shows a dilated esophagus with an air fliud level. The dx is
made by manometry; however, endoscopy is required to ensure that there is no malignancy.
11. Strawberry hemaniomas (capillary hemaniomas) are benign vascular tumor of children. They
appear during the first weeks of life; initially grow rapidly, and typically regress by 5-8 years
old. These tumors are composed of capillaries separated by connective tissue.
12. PEA Mx--Any pt in cardiac arrest with a non-shockable ryhthm (anything other than VF/VT)
should immediately receive CPR including manual chest compressions, establishment of a
definitive airway and ventilation with 100% oxygen. IV access should also be established so
ACLS medications such as epinephrine, vasopressin and stropine can be adminstered. It is
important to make the dx of PEA as there are a variety of potentially treatable cauese (6Hs
and 6Ts): Hs: hypotension, hypoxia, hydrogen ions (acidosis), hypothemia, hypoglycemia,
hyper/hypokalemia. Ts: tamponade (cardiac), tension pneumothorax, thrombosis (MI, PE),
trauma (hypovolemia), tablets (drugs) and toxins.
13. Crohn’s disease can involve any component of the GI tract from the mouth to the anus, and

76
charateristically has skip areas of involvement. Although nonspecific, aphthous ulcers in the
mouth can be seen in Crohn’s disease. Granulomas are identified pathologically in up to 30%
of pts.
14. FFP contains all clotting factors and is the first line tx for bleeding pts with a coagulopathy.
15. Chronic GI blood loss is the MCC of iron-deficiency anemia in an adult male or a post-
menopausal woman.
16. Post-splenectomy pts are at increased risk for sepsis from encapsulated organisms due to
impaired antibody-mediated opsonization in phagocytosis.
17. Immediate anticoagulation with heparin and surgical intervention (embolectomy) are crucial
to prevent tissue death in a pt with onging ischemia of the limb.
18. Gout can result from overprodiction or underexcretion of uric aric. Induction chemotherapy
results in rapid tumor cell lysis and release uric acid into the circulation. It is important to
distinguish between the prevention and tx of gout. For prevention, allpurinol and probenecid
ar used, while colchicine, NSAIDs, and steroids are used during acute attacks.
19. Transjugular intrahepatic portosystemic shunt precedure is used in cirrhotic pt for refractory
ascites, refractory hydrothoras, and surgical mx of acute recurrent variceal bleeding.
Pleurodesis is a choice for refractory pleural effusion secondary to maligancy but not for a
hydrothorax secondary to cirrhosis.
20. NPV is the probability of being free of a disease if the test result in negative. The NPV will
vary with the pretest probability of a disease. A pt with a high probability of having a disease
will have a low NPV, and a pt with a low probability of having a disease will have a high
NPV.
21. Staph. Aureus causes toxin-induced gastroenteritis. Because the illness is due to preformed
toxin, emesis is a predominant feature and symptoms start in less the 6 hours. Mayonnaise-
containing salads are often implicated.
22. Lyme arthritis is a late manifestation of Lyme infection; suspect in pts with history of travel to
endemic areas (Connecticut, Rhode Island, New York, Pennsylvania, New Jersey, Delaware,
Maryland and Wisconsin). Intermittent inflammatory arthritis is a typical presentation.
23. Poison ivy and nickel jewelry can cause allgergic contact dermatitis, which is a type IV
hypersensitivity reaction.
24. Hepatic encephalopathy is an alteration in CNS function due to poor hepatic clearance of
toxin. There is typically a clear precipitant and symptoms may range from mild cofusion to
coma. Tx includes lactulose, neomycin or rifaximin, and laxatives.
25. Digoxin is a cardiac glycoside whose adverse effects include nause, vomiting, diarrhea, vision
changes, and arrhythmias. Pt’s digoxin levels must be followed closely.
26. CMV retinitis occurs when the CD4 count is less than 50/ul. Pts may be asymtpmatic. The
characterisic fundoscopic findings of yellow-white patches of retinal opacification and
hemorrhages are diagnostic.
27. Vigorous hydration with IV normal saline is the first step in managing acute hypercalcemic
crisis (vomiting, oliguria, anuria, somnolence and eventually, coma.)
28. Suspect Huntington’s disease in a pt with mood disturbances, dementia, chorea (Whithing
movement) and a family history of similar symptoms.
29. Wilson’s disease is the likely diagnosis is a pt younger than 30 years old with unexplained
chronic hepatitis. The presence of low serum ceruloplasmin and increased urinary copper

77
excretion or Kayser Fleischer rings confirms the dx.
30. Contrast-induced nephropathy presents as a transient spike in creatinine with 24 hours of
contrast administration, with a return to normal renal function with 5-7 days. Pts with diabets
and elevated baseline creatine are at especially high risk. Adequate IV hydration with isotonic
bicarbonate and administration of acetylcysteine help to minimize the risk of contrast-induced
nepropathy.
31. Tinea versicolor (Malassezia furfur) is characterized by pale, velvety pin or whitish,
hypopigmented macules that do not tan and do not appear scaly, but scale on scraping.
Topical tx with selenium sulfide lotion and ketoconazole shampoo is recommeded. *
Extremely important!!
32. Rapid developing huperandrogenism with virilization is highly suggestive of an androgen-
secreting neoplasm of the ovary or adrenal. Serum testosterone (ovarian source) and DHEAS
level (adrenal source) are very helpful in delineating the site of excess angrogen production.
33. A bronchodilator response test is used to demonstrate reversibility of airway obstruction. It
help to differentiate between COPD and asthma (significantly improvement in FEV1),
although a subset of pts with COPD may also demonstrate airway reactivity.
34. Hypertension and spontaneous hypokalemia are characteristic for both primary
hyperasdosteronism and renovascular hypertension. The plasma renin activity (PRA) can
distinguish between these two diagnoses; pts with primary hyperasdosteronism have low
PRA, while pts with renovascular hypertension have high PRA.
Bartter’s syndrome
35. Chornic lymphocytic leukemia is almost always seen in elderly pts. The dx features are
marked leukocytosis with predominant lymphocytosis and smudge cells.
36. Spinal cord compression is characterized by signs asdn symptoms of upper motor neuron
dysfunction distal to the site of compression. These include weakness, huperreflexia, and an
extensor plantar response. Cord compression is a medical emergency requiring prompt
diagnosis by a spinal MRI.
37. Diabetic ketoacidosis causes an anion gap metabolic acidosis. Typical symptoms of DKA
include polyuria, polydipsia, nausea, vomiting and abdominal pain that may mimic a surgical
abdomen.
38. The most serious side effect of hydroxychloroquine is retinopathy. Eye examiniations every 6
months are the only form of routine screening required with hydroxychloroquine therapy.
39. Cushing’s syndrome due to extopic ACTH production is characrized by the rapid development
of symptoms, hypokalemic alkalosis, pigmentation and hypertension. Serum cortisol and 24
hour urine cortisol levels are not supressed following the administration of high dose
dexamethasone in most pts with ectopic ACTH syndrome.
40. Anasarca 全 身 水 肿 typically results from organ failure or hypoalcuminemia. In pts with
glomerulonephritis, edema results from the decreased glomerular filtration rate as well as
proteinuria in some cases.
41. Macrovascular hemolysis can cause microcytic anemia in pts with artificial heart valves or
severely calcified aortic valves.
42. Normal pressure hydrocephalus is characterized by the triad of gait disturbance, demantia and
urinary incontinence. Lumbar puncture reveals the normal CSF pressures, and MRI shows the
enlarged ventricles.

78
43. Chronic GERD and Barrett’s esophagus are risk factors for adenocarcinoma of the esophagus.
The major risk factors for squamous cell cancer of the esophagus are smoking and alcohol.
44. Paget’s disease of the bone is characterized by increased bone remodeling and abnormal
osteoid formation. It may be asymptomatic or accompanied by pain and deafness. Alkaline
phosphatase will be elevated but serum calcium, phosphorus, and other liver function tests
will be normal.

12/30/09
1. Pericardial effusions will appear as an enlarged, ‘water bottle’ shaped cardiac silhouette on
CXR. Physical exam findings with pericardial effusions without cardiac tamponade include
deminished heart sounds on auscultion and a difficult to palpate point of the maximal impulse.
Pts has a pericardial effusion may not have cardica tamponade.
Pulsus bisferiens, or biphasis pulse, refers to two strong systolic impulses with a midsystoli dip. It
can be seen in aortic regurgitation with or without aortic stenosis and in hypertension obstructive
cardiomyopathy.
2. Age-dependent idiopathic sclerocalcific changes are the most frequent cause of isolated aortic
stenosis in elderly pts. These changes are common and usually have minimal hemodynamic
significance, but sometimes may be severe.
3. Microangiopathic hemolytic anemias are disorders in which intravascular fragmentation of
red blood cells occurs. One cause is shearing of RBCs on artificial heart valve. Schistocytes
are the classic finding on peripheral blood smear.
4. Systolic heart failure is characterized by depressed cardiac index (CI) accompanied by
inceased total peripheral resistance (TPR) and left ventricular end-diastolic volume (LVEDV).
5. Constrictive pericarditis results in pericardial fibrosis and diastolic dysfunction. Pts presents
with sings of decreased cardiac output (fatigue, dyspnea on exertion, muscle wasting) and
venous overload (elevated JVP, positive Kussmaul’s sign, pedia edema). Sharp ‘X’ and ‘Y’
descents on central venous tracing are characteristic of constrictive pericarditis as is the
preence of a pericardial knock (early heart sound after S2).
6. Massive pulmonary embolism can result in pulmonary hypertension, evidence by elevated
right atrial and pulmonary artery pressures. While the right heart is strained by massive PE,
the left heart is not. Massive PE may also result in shock. A right atrial pressure >10 and
pulmonary artery systolic pressure of 40 are the criteria used to diagnose massive embolism.
7. Rhadomyolysis should be suspected in the following situations: 1) Presence of risk factors
such us alcoholism, cocaine use, and electrolyte abnormalities (eg. hypokalemia,
hypophosphatemia) 2) Disproportionate elevation of creatinine as compared with BUN. 3)
Urine dipstick positive for blood but no RBC on microscopic examination. The underlying
pathology is acute tubular necrosis. Serun CK should be measured in suspected patients. The
tx is aggressive iv hydration and alkalinization of urine, In some cases, forced diuresis with
mannitol may be required.
8. ARDS is a form of noncardiogenic pulmonary edema caused by inflammatory damage to
alveoli. Sepsis, severe bleeding, trauma, and toxic injury may result in ARDS. Criteria for
diagnosing ARDS include: acute dyspnea, an inciting condition, bilateral infiltrates on CXR,
PCWP<18, and PaO2/FiO2 ratio<200.
9. The murmur of aortic regurgitation is typically decribed as a diastolic decrescendo murmur at

79
the left third intercostal apace. Medical therapy consists of afterload reduction with calcium
channel blockers or ACE inhitors.
10. Toxic epidemal necrolysis is a severe mucocutaneous exfoliative disease. It is characterized
by an erythematous morbilliform eruption that rapidly evolves into exfoliation of the skin.
Positive Nikolsky’s sign
11. Folate and cobalamin deficiency both result in megaloblastic, macrocytic anemia. Folate
supplementation can improve the anemia of either disorder but will not improve the
neurologic changes of cobalamin deficiency. It is important to distinguish between the two
prior to initiating therapy.
12. Lung consolidation causes dullness to percussion, increased fremitus, bronchophony,
egophony, and whispered pectoriloquy on physical exam.
13. Elderly pts with poor oral intake living in nursing homes and taking medications such
NSAIDs, ACEIs and diuretics (blunt the renal response to low intravascular volume.) should
be suspected of having prerenal azotemia due to intravascular volume depletion and poor
renal perfusion.
14. Low serum iron, ferritin, and increase TIBC indicates iron deficiency anemia.
15. Aspirin sensitivity syndrome is believed to be a pseudo-allergic reaction. It results from
aspirin-induced prostaglandin/leukotriene misbalance in susceptible individuals. Tx includes
avoidance of NSAIDs and the use of leukotriene receptor antagonists (drug of choice).
Extremely high yield!!
16. Mucormycosis requires aggrassive surgical debridement plus early systemic chemotherapy
with amphotericin B.
17. Pulmonary TB presents with productive cough, hemoptysis, and recurrent fevers. Weight loss
may also be present. Imaging most commonly reveals upper lobe cavity lesions (CT picture).
TB must be suspected in pts from endemic regions with subacute or chornic pulmonary
complaints.
18. Acute venous thromboemboli should be treated with a combination of heparin and warfarin
initially. After 5 days the heparin may be discontinued if the pt’s INR is therapeutic (2-3).
Warfarin should be continued for at least 6 months in most pts with a first-time clot. In
patients with a second episode of clot, lifetime anticoagulation is necessary.
19. GVHD is caused by recognition of host major and minor HLA-antigens by donor T-cell and
consequent cell-mediated immune response. The organs that are typically affected include the
skin, intesine and liver.
20. Pulmonary emboli classically present with sudden-onset pleuritic chest pain, cough, dyspnea,
and hemoptysis. Chest CT showing wedge-shaped infarction is virtually pathognomonic.
21. Unacceptability bias occurs when participants purposely give desirable response which lead to
the underestimation of risk factors.
Admission rate bias refers to a distortion in risk ratio due to different hospital’s admission of
certain cases. For instance, pts with cardiac diseases may prefer to be admitted to a particular
hospital.
22. Asbestos exposure increases the risk of malignancy, pulmonary fibrosis and pleural plaques.
In terms of malignancies, both bronchogenic carcinoma and mesothelioma are linked to
asbestos exposure, with bronchogenic cacinoma being the MCC.
23. Diastolic dysfunction refers to impaired ventricular filling due to poor myocardial relaxation

80
or diminished ventricular compliance. It is usually due to hypertension and is treated with
diuretics and antihypertension. In severe cases, diastolic dysfunction can cause
decompensated heart failure.
High-output heart failure refers to supranormal vantricular function that still cannot meet the
body’s metabolic demands. Causes include anemia, hyperthyroidism, beriberi, Paget’s disease, and
arteriovenous fistulas.
24. Factor V leiden is the most common inherited disorder causing hypercoagulability and
predisposition to thromboses, especially DVT of lower extremities. Factor V Leiden is the
result of point mutation in a gene coding for the coagulation factor V. As a result of this
mutation, Factor V becomes resistant to inactivation by protein C, an important
counterbalance factor in hemostatic cascade. Other disorders causing inherited predisposition
to thrombosis-like protein C deficiency, protein S dificiency, antithrombin III deficiency, and
plasminogen disorders are quite rare.
25. Intravenous nafcillin or cefazolin is the tx of choice for cellulitis with systemic signs. Many
cases of cellulitis are due to Staphylococcus aureus. For this reason, IV aqueous crystalline
penicillin G (almost 99% resistance) is not an effective tx for cellulitis.
26. Oligoclonal bands are present in 85-90% of cases of multiple sclerosis, CSF pressure, protein
and cell count are grossly normal. Elevated immunoglobulin levels may be found in other
diseases such as neuropathies, chronic central nerivous system infections or viral syndromes.
The presence of oligoclonal bands is not considered dx for MS.
27. Post TURP (transurethral resection of the prostate) hyponatremia is caused by absorption of
free water from the nonconductive irrigation solutions (containing glycine, mannitol, or
sorbitol to flush) used during the procedure. The decrese in sodium concentration is rapid and
should be corrected quickly if the patient is symptomatic.
28. TSH-secreting pituitary adenoma leads to excessive TSH levels, which stimulate the thyroid
gland and thereby lead to a hyperthyroid state. It is biochemically characterized by elevated
circulating thyroid hormone levels with inappropriately normal or elevated TSH levels. Most
of the TSH-secresting pituitary adenomas are macroadenomas. Pts with this condition
typically have goiter due to the effect of TSH on the growth of the thyroid follicles. Pts don’t
have extrathyroidal manifestation of Grave’s disease, such as infiltrative ophthalmopathy and
pretibial myxedema.
29. Creutzfeldt-Jakob disease is characterized by repidly progressive dementia, myoclonus and
sharp, triphasis, synchromous discharges on EEG. This spongiform encephalopathy is caused
by a slow virus (prion).
30. Polycythemia Vera (PCV) is distinguished from the other kind of myeloproliferative disease
(MPD) by the remarkable increase in RBC mass and total blood volume. A typical pts is an
old plethoric 多血症的 male who may complain of prutitus after bathing. Symptoms such as
headache, dizziness, and parethesias are frequently related to hyperviscosity. Both thrombosis
and bleeding can occur due to elevated platelet count and impaired platelet function.
Reversible moderate hypertension frequently occurs as a result of expanded blood volume.
They also have granulocytosis and thrombocytosis on peripheral blood smear, and
aplenomegaly. Bone marrow is virtually always hypercullular. There is an elevated leukocyte
alkaline phosphatase, normal oxygen saturation, and low erythropoetin level. A urinary
erythropoetin assay reveals the absence of measurable erythropoetin in the urine.

81
31. Amyloidosis can result in a restrictive cardiomyopathy with thickened ventricular walls and
preserved ventricular dimensions, as well as involvement of the liver (inhibit the synthesis of
coagulation factors—increased bruisability) and kidneys (proteinuria). Alcohol causes a
dilated cardiomyopathy. Hemochromatosis, Sarcoidosis—restrictive cardiomyopathy.
32. Wegener’s granulomatosis is defined clinically as the triad of systemic vasculitis, upper and
lower airway granulomatous inflammtion, and glomerulonephritis. Nasal cartilage destruction
and vasculitic cutaneous lesions (tender nodules, palpable purpura, ulcerations) are common
external manifestation.
33. Asymptomatic gallstones should not be treated. Laparoscopic cholecystectomy is the tx of
choice of symptomatic gallstone disease.
34. Invasive aspergillosis occurs in immunocompromised pts, who may present with fever, cough,
dyspnea or hemoptysis. CXR may show cavity lesions, and CT scan shows pulmonary
nodules with a halo sign or lesions with an air crescent.
35. CSF findings of elevated white count, elevated protein, and decreased glucose levels are
suggestive of bacterial meningitis. Typical skin lesions during meningococcal infection are
petechiae and purpura.
36. Imatinib has changed the diagnosis of pts with chronic myelogenous leukemia. It is a tyrosine
kinase inhibitor and works by blocking signals within cancer cells and preventing a series of
chemical reactions that cause the cancer cells to grow and divide.
37. 3 tx options for myasthenia gravis: orall anticholinesterase is usually the initial tx of choice
(pyridostigmine or neostigmine, not edrophonium—for dx). Atropine can be used to prevent
muscarinic side effects of anticholinesterase therapy in pts with myasthenia gravis.
Anticholinesterases provide symptomatic benefit, but do not induce remission.
Immunouppressive agents and thymectomy may induce remission. All pts should be evaluate
for a thymectomy. Immunouppressive agents that are used in the tx of mysthenia gravis
include prednisolone, zazthioprine and cyclosporine. Corticosteroids are used in (1) pts who
are over 60 years of age, and (2) pts who show poor response to anticholinesterase and have
already undergone thymectomy. Plasmapheresis removes acetylcholine receptor antibodies
from the circulation. It is used in seriously ill pts when other treatments are not effective or
C/I. Its effect is transient and cannot be used on a long-ter basis. Other indications of
plasmapheresis include (1) stabilizaion of the pt before thymectomy, and (2) myasthenia
crisis. IV immunoglobulins act by an unknown mechanism, and are used in the same setting
as plasmapheresis.
38. Adrenal insufficincy and adrenal failure are characterized by nonspecific symptoms and signs
including anorexia, fatigue, GI complaints, weight loss, and hypotension. Hyponatremia is the
most common associated electrolyts abnormality. Hyperkalemia is also common.
39. Pica refers to an appetite for nonnutritive substances, such as ice, clay, dirt, and paper
products. Pica, esp.for ice (Pagophagia), can be indicative of iron deficiency (in old person,
chronic bleeding). Pica may also be present as a manifestation of psychiatric diseaseas and
people may eat very odd items, such as light bulbs or hair.
40. Agents used to shift potassium intracellularly include insulin and glucose, sodium
bicarbonate, and beta-2 (albuterol) agonists.
41. Primary polydipsia is a problem of excessive water ingestion. The kidneys continue to work
appropriately, excreting dilute urine. The dx of primary polydipsia is confirmed by

82
withholding water from the pt and monitoring the urine. Increases in urine osmolality and
specific gravity confirm the dx. Primary polydipsia is relatively common among pts with
schizophrenia. Simutaneous administration of phenothiazines to schizophrenic pts with
primary polydipsia can worsed the hyponatremia because dry mouth is a side effect that
further stimulates thirst. Carbamazepine intensifies the kidney’s response to ADH, causing
water retention, hyponatremia, and inappropriatedly concentrated urine, as in SIADH.
42. S. aureus pneumonia is a serious complication of influenze pneumonia. It can occur in both
children and adults. Pts must be treated with anti-staph antibiotics when post-influenza
pneumonia is suspected.
43. The median is the value that is located in the middle of a datasheet. It divides the right half of
the data from the left half. If the number of observation is even, find the middle two values,
add them together, and divide by two.
44. Steven Johnson syndrome (erythema multiforme major) is characterized by the sudden onset
of target-shaped, mucocutaneous lesions (oral and conjuctival) and systemic signs of toxicity.
The pathology involves immune complex mediated hypersensitivity. The most commonly
involved drugs are sulfonamides, NSAIDs, and phentoin. Tx is similar to burns—primarily
supportive, with regular assessment of hemodynamic stability, fluid status, and pain control.
Erythema multifome minor (under a spectrum of disease with ‘major’) is characterized by
sudden onset erythematous rash and target lesions, and usually occurs after a herpes simplex
infection. Mucosal involvement is rare, and systemic symptoms are not so severe as in
‘major’.

1/1/10
1. Suspect pseudodementia in elderly pts with dementia in the setting of severe depression.
Antidepressants are the tx of choice (reversible with tx). Donepezil is an acetyle-
cholinesterase inhibitor used in the tx of Alzheimer’s dementia. A good distinguishing factor
to remember is that pts with pseudodementia seem significantly concerned about their
impaired memory, whereas, pts with Alzheimer’s dementia seem relatively unconcerned.
2. The 2 initial tests, which are recommeded in all the pts with possible benign prostatic
hyperplasia, are serum creatinine and urinalysis. Such symptoms are not always due to BPH,
other diseases: bladder cancer, prostate cancer, urethral stricture, neurogenic bladder, and
urinary tract infection. Urinalysis helps to rule out urinary tract infection and serum creatinine
detects renal insufficiency that might due to bladder outlet obstruction. Serum creatinine is a
better index of renal function than blood urea nitrogen and therefore is the preferred test.
3. Paget’s disease presents as a persistent dermatitis of the nipple. The lesion is red, oozing,
crusted, and often unresponsive to topical steroids and anbiotics. Biopsy shows malignant
cells that invade the epidermis and have abundant pale-staining cytoplasm surrounding a
hyperchromatic nucleus with prominetn nucleoli.
4. The 3 cardinal signs of Parkinson disease (accumulation of alpha-synuclein within the
neurons of the substantia nigra pars compacta) are rest tremor, rigidity, and bradykinesia. The
presence of at least two of these signs on physical examination is gounds for a clinica
diagnosis. To date, there are no imaging or lab tests that can be used to confirm the dx with
any greater accuracy than physical exam.
5. Women who were treated for cervical intraepithelial neoplasia II/III should have Pap smear

83
with or without colposcopy and endocervical curettage every 6 months until 3 negative results
are obtained. Pts may resume standard screening after that milestone is met (annual Pap smear
testing).
6. The most common renal stones are calcium stones. CT scan of the abdomen without contrast
is the dianostic precedure of choice because it can detect redioopaque (eg. calcium) as well as
radiolucent (eg. uric acid) stones. (KUB or abdominal radiograph is not the best test!). The
dietary recommendations for pts with renal culculi are: 1) ↓ dietray protein and oxalate. 2) ↓
sodium intake (↑sodium intake enhances calcium excretion). 3) ↑fluid intake. 4)↑dietary
calcium. Mega doses of VitC lead to increased formation of oxalate stones.
7. A nail puncture would in an adult resulting in osteomyelitis in most likely due to
Pseudomonas aeruginosa.
8. The best screening test for pts with suspected primary adrenal insufficiency is the cosyntropin
(analogue of ACTH) stimulation test. An increase in serum cortisol levels above 20 mcg/dl
30-60 min after the administration of 250mcg of cosyntropin virtually rules out primary
adrenocortical insufficiency (addison’s disease). Measurement of plasma ACTH is performed
to distinguish between primary and secondary adrenocortical deficiency once the dx of
Addison’s disease has been made. Plasma ACTH levels greater than 50pg/ml indicate primary
adrenocortical deficiency, whereas levels less than 50pg/ml indicate secondary adrenocortical
deficiency.
24hr urinary free cortisol measurment and low-dose dexamethasone suppression test are screeing
tests ofr Cushing syndrome.
9. TCA overdose is characterized by CNS depression, hypotension, and other anticholinergic
effects including dilated pupils, hyperthermia and intesitnal ileus. It can also cause QRS
prolongation on EKG, leaving the pt susceptible to ventricular arrhythmias. Pts suspected of
TCA overdose should first undergo the ABC’s and sodium bicarbonate should then be
administrated to improve blood pressure, shorten the QRS interval, and prevent arrhythmia.
10. Amoxicillin-clavulanate is the antibiotic of choice for prophylaxis/treatment of infections
caused by a human bite (polymicrobial—G+, G-, anaerobes). This is also the drug of choice
of dog bites. Clindamycin is effective against G+ and anaerobes, typically used for treating
lung abscesses and infections of the female genital tract. Ciproflaxacin does not have good
coverage against anaerobes. If the pt were allergic to penicillin, a combination of
ciprofloxacin and clindamycin would be a possible regimen.
11. If a pt presents with priapism, always check his medications first. This symptom is often drug-
induced. The most common drug that causes Priapism is prazosin, although for the boards, it
is important to remember the association of priapism with trazodone. The common causes of
priapism: 1) sickle cell disease and leukemia—usually in children or adolescents. 2) Perineal
or genital trauma—results in laceration of the cavernous artery. 3) Neurogenic lesions—such
as spinal cord injury, cauda equina compression, etc. 4) medications.
12. Proctosigmoidoscopy with biopsy establishes the dx of ulcerative colitis. One of its
advantages is that this dx test can be performed in acute settings. Fulminant colitis is a serious
complication of ulcerative colitis; plain radiographs can usually reveal the distended colon.
13. Brainstem lesons typically involve the cranial nerves and sensory loss of one half of the face
and the contralateral half of the body. Lesions of the thalamus or cortex are associated with
sensory loss of one half of the face and the same half of the body. Cortical lesions are

84
associated with findings specific to the cerebral cortex (eg. aphasia, neglect, abnormal
graphesthesia or stereognosis).
Medial medullary syndrome is typically caused by an occlusion of the vertebral artery or one of its
branches. As a result, the pt will demonstrate contralateral paralysis of the arm, leg, contralateral
loss of tactile, vibratory, and position sense, and tongue deviation to the injured side.
Lateral medulla—Wallenberg syndrome. Ipilateral Horner syndrome, loss of pain and temperature
sensation of the face, weakness of the palate, pharynx, and vocal cords; and cerebellar ataxia.
There is also loss of pain and temperature sensation on the contralateral side of the body.
Medial pons—medial midpontine syndrome. Common characteristics include ipsilateral limb
ataxia and contralateral eye deviation and paralysis of the face, arm, and leg; impairment of touch
and position sense is variable.
Lateral pons—lateral midpontine syndrome, which results in impaired sensory and motor function
of CNV with accompanying limb ataxia.
Central midbrain—a number of different syndromes, including Weber’s syndrome, Benedikt’s
syndrome, Claude’s syndrome, Nothnagel’s syndrome, and Parinaud’s syndrome. Oculomotor
paresis and othe abnormalities of CNIII function are common to all of thers syndromes, and
cerebellar ataxia and contralateral hemiplegia amy be noted as well.
14. Pts with acute pyelonephtitis susally present with fever, chills, nausea, vomiting and flank or
suprapubic pain. Physical examination shows costovertebral angle tenderness. Hospitalization
and iv antiboitics are indicated in complicated acute phylonephritis.
15. Pneumococcal vaccine is recommeded for all HIV-infected pts whose CD4 count is above 200
cells/ml. Annual influenza vaccination is also recommoded for all HIV-infected pts.
16. In any pt, the PH and PaCO2 are the 2 lab values that provide the test picture of acid-base
status; the HCO3- can be calculated form these values using the Hendeson-Hasselbalch
equation. PH=6.1+log {(0.03xPaCO2)/[HCO3-]}
17. Acute pancreatitis classically causes nausea, vomiting and epigastric pain that radiates to the
back. The pt’s clinical history, physical exam findings, serum lipase and amylase levels all
help to make the diagnosis. Alcohol abuse is the MCC of acute pancreatitis in men and
gallstones are the MCC in women. Potential complications of acute pancreatitis include
exudative left-sided pleural effusions when the amylase concentration is high, abdominal
compartment syndrome, intraabdominal hemorrhage, shock, diabetes, pancreatic pseudocyst
formation, and abdominal pseudoaneurysm.
18. Dementia with lewy bodies causes alterations in alterness, visual hallucintions, and
extrapyramidal symptoms. At autopsy, ‘lewy bodies’, or eosinophilic nuclear inclusions
representing accumulations of alpha-synuclein protein, may be seen in neurons of the
substantia nigra, locus ceruleus, dorsal raphe, and substantia innominata. Lewy bodies are a
pathologic finding also present in Parkinson’s disease. The key distinction between these two
conditions is the early appearance of dementia in Lewy body disease and of motor symptoms
in Parkinson.
19. Proteus species produce urease, which makes the urine alkaline (PH 8.5). This infection is
particularly common in pts who live in long-term care facilities and have chronic indwelling
catheters. Candida, Psudomonas, and Klebsiella infections are also common in pts with
chronic indwelling catheters but they do not produce alkaline urine. E. Coli ist he MCC of
UTIs, but it does not produce urease and thus does not alter the normal acidic pH of urine.

85
20. Suspect trichinellosis in a pt who present with GI complaints followed by the characteristic
triad of periorbital edema, myositis, and eosinophilia. Other clues include subungual splinter
hemorrhage and conjunctival or retinal hemorrhages.
21. A non-productive cough is observed in 5-20% of pts receiving ACE inhibitors. It is caused by
an accumulation of kinins, and possibly by the activation of the arachidonic acid pathway.
22. Alveolar hypoventilation, as in a COPD exacerbation, can cause CO2 retention and a resultant
lethargy, somnolence, seizures, coma, and even death.
23. Cholesterol screening in men above age 35 and in women >45 when there is average risk for
coronary heart disease. Screening should begin at younger ages (men 20-35, women 20-45)
for individuals at increased risk.
24. Recurrent bacterial infections in an adult pt may indicate a humoral immunity defect.
Quantitative measurement of serum Ig levels helps to establish the diagnosis.
25. The overall incidence of vertical transmission of HCV is approximately 2-5%. All pts
including pregnant pts, with chronic hepatitis C should receive vaccinations againt HAV, HBV
if not already immune. * Extremely important!
26. PE cause dyspnea, tachypnea, pleuritic chest pain, and tachycardia. While there are suggestive
CXR is normal in most cases. The most common ECG finding is sinus tachycardia.
27. Most pts who are initially controlled with one antidiabetic medication eventually require the
addition of more antidiabetic drugs to achieve optimal glycemic control. Combining
antidiabetic agents with different mechanisms of action is typically done to achieve better
glycemic control. Metformin is the only antidiabetic drug that causes some weight loss.
28. Atrial myxoma is the most common primary intracardiac tumor, and is usually located in the
left atrium. It can casue systemic symtoms such as fever and weight loss, neurologic
symptoms sue to tumor embolization, and presents as a mass on echocariography.
Myxomatous valve degeneration –mitral valve prolapse.
29. Dipyridamole can be used during myocardial perfusion scanning to reveal the areas of
restricted myocardial perfusion. The redictribtution of the coronary blood flow to ‘non-
diseased’ segments induced by this drug is called coronary steal phenomenon. Dipyridamol
and adenosine are coronary vasodilators. Infusion of these substances in pts without CAD,
increases coronary blood flow 3-5 times above the baseline levels. However, in pts with CAD,
the diseased vessels distal to the obstruction are already maximally dilated, and their ability to
increase myocardial perfusion is limited, therefore, redistribution of coranary blood flow to
‘non-diseased’ areas occurs, and the perfusion of ‘diseased’ segments diminishes.
30. Newborns of mothers with active HBV infection should be passively immunized at birth with
HBIG followed by active immunization with recombinant HBV vaccine.
31. Postictal lactic acidosis commonly occurs following a tonic-clonic seizure, It is a transient
anion gap metabolic acidosis that resolves without tx with 60-90 mins following resolution of
seizure activity.
32. Platelet dysfunction is the MCC of abnormal hemostasis in pts with CRF. PT, PTT and
platelet count are normal. BT is prolonged. DDAVP is usually the tx of choice, if needed.
DDAVP increases the release of factor VIII:VWF multimers from endothelial storage sites.
Platelet transfusion in not indicated because the transfused platelets quickly become inactive.
* Extremely high yield!
33. Chloroquine-resistant plasmodium falciparum is particularly common in Sub-Saharan Africa

86
and the India subcontinent (eg. India, Pakisan, and Bangldesh). Mefloquine is the drug of
chioce for chemoprophylaxis against chloroquine-resistant malaria. The use of primaquine
(both for prophylaxis and tx) is indicated in settings where malaria is due to Plasmodium
vavax or plasmodium ovale; there organisms cause persistent infection in the liver.
34. Acute confusion, extreme huperthermia >40.5 (105F), tachycardia, and coagulopathic
bleeding after heavy work under direct sunlight are most likely due to exertional heat stroke.
Malignant hyperthemia affects genetically susceptible individuals during anesthesia involving
agents like halothane and succinylcholine.
A hypothalamic stroke may disrupt normal neural thermoregulatory mechanisms and result in
neurogenic fever (NF). In NF the hypothalamic set-point becomes pathologically elevated.
35. Cystoscopy can cause an enterococcal bacteremia in pts with chronic genitourinary infections.
Pts with underlying valvular lesions are at risk for endocarditis in this setting.
36. Antiphospholipid antibody syndrome (VDRL positive FTA-ABS Negative) places a pregnant
pt at increased risk of first and second trimester abortin. The use of heparin and aspirin
reduces this risk. Warfarin is not used due to its teratogenicity. Type I: false-positive syphilis
serology; Type II is lupus anticoagulant, and it falsely elevates the APTT level; Type III is
anticardiolipin antibody.
37. The initial tx of both acute and chronic anal fissures includes dietary modification (eg. high-
fiber diet and large amounts of fluids), a stool sofener, and a local anesthetic. Colonoscopy in
not needed in the pt as the bright red blood on defecation is clearly due to the fissure. His
stool caliber is also normal. However, if the fissue heals and the pt still has blood in the stool,
then colonoscopy is warranted. * Extremely high yield!!
38. Degenerative joint disease (osteoarthritis) presents in adults over age-40 with indolent
progressive anterior hip pain worsened by activity and relieved by rest. The hip is not tender
and systmic symptoms are absent. Short-lived morning stiffness may occur.
39. Macular degeneration is the most common cause of blindness in industrialized nations.
Activities that require fine visual acuity are usually the first affected, and pt with this
condition may report that straight grid lines appear curved.
40. The first line of medical therapy for hypertrophic cardiomyopathy is either beta-blockers or a
cardiac acting CCB such diltiazem since they promote diastolic relaxation. Amlodipine is a
peripherally acting CCB, so it does not have the diastole-prolong effects of a drug such as
diltizem. It acts primarily as an arterial vasodilatior.
41. Emtamoeba histolytica is a protozoan, which can cause amebic liver abscess. Remember the
Mexico trip.
42. Childre with cystic fibrosis should adhere to routine vaccination schedules with a few
additions. They should receive yearly influenza vaccinations and may require pneumococcal
booster shots. In pts with egg allergies, the influenza and yellow fever vaccines should not be
given, and MMP should be given with caution.
43. Acyclovir can cause crystalline nephropathy if adequate hydration is not also provided.
44. Norcadiosis is dx by the presence of crooked, branching, beaded, G+, partially acid-fast
filaments on microscopy. Trimethoprim-sulfamethoxazole is the tx of chioce. Picture!

1/1/10
1. A mobile cavitary mass in the lung, which presents with intermittent hemoptysis, is usually

87
indicative of asperggilloma. The CXR may demonstrate a crescent radiolucency next to a
rounded mass. Cavitary lesions may form because of destruction of the underlying pulmonary
parenchema, and debris and hyphae may coalesce and form a fungus ball, which lies free in
the cavity and moves around with position change.
2. Intermittent catheterization is an effctive measure to reduce the risk of UTI in pts with
neurogenic bladder.
3. In heat stroke the temperature is usually above 40.5C (105F). Heat stroke results from an
insufficient evaporative cooling machanism. Therefore, tx involves induction of evaporative
cooling to reverse hyperthermia.
4. When isolated proteinuria occurs, the evaluation of the pt should begin by testing the urine on
at least two other occasions.
5. Orthostatic hypotension is characterized by a drop in blood pressure after standing and is
typically preceded by a lighthearded or presyncopal sensation.
6. Atypical pneumonia presents with a more indolent course and higher incidence of
extrapulmonary manifestations than pyogenic pneumonia. CXR findings may be out of
proportion to findings on physical exam. Erythema multiforme is a characteristic
extrapulmonary manifestations of Mycoplasma pneumonia, the MCC of atypical pneumonia.
7. ERCP is the investigation of choice for pts with recurrent pancreatitis with no obvious cause
—ductal abnormalities including pancreatic divisum, choledochal cyst, and common bile duct
stones. It would also allow for gall bladder aspiration of bile to look for biliary crystals or
microlithiasis.
8. Chronic liver disease or cirrhosis from almost any cause is a risk factor for hepatocellular
cancer (hepatoma). Hepatocellular cancer is responsible for 30% of deaths in pts with
hemochromatosis.
9. Ischemic cardiac pain can sometimes be mistaken for epigastric pain, but should remain high
on the differential, especially in the setting of symptoms worsened with exertion. An exercise
strss test without imaging is the most reasonable first step if the baseline resting EKG in
normal (pt’s heart burn has exertional nature).
10. Pts presenting to the ER with confusion or coma should receive empiric tx with dextrose,
oxygen, naloxone, and thiamine. Thiamine will help to prevent or reverse Wernick’s
encephalopathy in alcoholics, and should be administered bofore dextrose.
11. Aortic dissections are life-threatening emergencies that classically present with tearing chest
pain radiating to the back. Sequelae include cardiac tamponade, acute aortic regurgitation,
stroke, and renal failure.
12. After 48-72 hours of parenteral therapy for uncomplicated pyelonephritis, the pt can be
usually switched to an oral agent. Oral therapy (TMP/SMX) is more convenient and less
expensive; if the results of antibiotic susceptibilit testing are known, the appropriate antibiotic
can be easily chosen.
13. The MCC of pulmonary complications in pts with systemic sclerosis is interstitial fibrosis.
14. Herpes zoster is a painful reactivation of the varicella zonster virus that causes a vesicular
rash in a dermatomal distribution. Acyclovir and other antivirals may be used to decrease both
the duration of disease and the incidence of post-herpetic neuralgia.
Topical 5-FU—actinic keratoses, superficial basal cell carcinoma, Bowen’s disease, and solar
keratosis.

88
Topical cortisosteroids—numerous inflammatory skin conditions.
Mupirocin—superficial skin infection and to eradicate methicillin-resistant Staph aureus
colonization.
Fluconazole—superficial and systemic fungal infections.
15. An aortic aneurysm of the descending aorta may appear on a CXR as a well-circumscribed
lesion. These aneurysms are due to atherosclerosis.
16. Cocaine intoxication is a leading cause of myocardial infarctions in young people, and
myocardial ischemia associated with cocaine use should be treated initially with
benzodiazepine, aspirin, and nitrates. Elevated blood pressure may be controlled with alpha-
blockers, but beta-blockers are C/I.
17. Obesity hypoventilation syndrome is a long-term consequence of severe obesity and untreated
obstructive sleep apnea. It causes chronic hypercapnic/hypoxic respiratory failure, secondary
erythrocytosis, pulmonary hypertension, and cor pulmonale. In a compensatory effort to
maintain a normal PH, the kidney increases bicarbonate retention and decrease chloride
reabsorption.
18. ARDS is a form of noncardiogic pulmonary edema caused by inflammatory damage to the
alvioli. Criteria for the dx include: acute onset of respiratory distress in the setting of a
predisposing condition (sepsis, pneumonia, etc.), PaO2/FiO2 ratio<200, bilateral infiltrates on
CXR, and a normal PCWP.
19. All COPD pt with PaO2<55mmHg or SaO2 <88% are candidates for long-term home oxygen
tx. Pt with signs of pulmonary hypertension or hemotocrit >55% should be start on home
oxygen when the PaO2 < 60mmHg.
20. SIADH is characterized by euvolemic hypoatremia, decreased serum osmolarity, elevated
urine osmolarity (often greater than serum osmolarity), increase urine sodium concentration,
and failure to correct with normal saline infusion.
Serum osmolarity Extracellular Urine findings Causes of hupomatremia
volume
Normal - - Hyperproteinemia
hyperlipidemia
High(>295 mOsm/kg) Hyperglycemia
Exogenous solutes
(radiocontrast, mannitol)
Low(<280 mOsm/kg) Hypovolemic UNa < 10 mEq/L Nonrenal salt loss
Dehydration
Vomiting
Diarrhrea
UNa >20 mEq/L Renal salt loss
Diuretics
ACEIs
Mineralocoricoid deficiency
Euvolemic UNa >20 mEq/L Psychogeic polydipsia
Uosm < 300 Beer potomania
UNa >20 mEq/L SIADH
Uosm >300
Hypervolemic CHF

89
Hepatic failure
Nephrotic syndrome

21. Autoimmune platelet destruction is a common cause of thrombocytopenia and should be


suspected in patients with ecchymoses, petechiae and mucosal bleeding without signs or
symptoms of TTP/HUS, pancytopenia, marrow failure or splenomegaly.
22. Severe symptomatic hyponatremia requires ts with hypertonic (3%) saline with or without
furosemide. Rapid correction of serum sodium levels can lead to central pontine myelinosis.
Mx of hyponatremia due to SIADH:
Mild (asymptomatic with sodium 120-130 meq/L) = Fluid restriction
Moderate (asymptomatic with sodium 110-120 meq/L) = Loop diuretic + normal saline
Severe (symptomatic) = Hypertonic saline
23. Pick’s disease is a fronto-temporal dementia that is characterized by personality changes,
compulsive behavior, and impaired memory.
24. Whipple’s disease is a multi-system illness characterized by arthralgias, weight loss, fever,
diarrhea and abdominal pain. PAS-positive material in the lamina propria of the small
intestine is a classical biopsy finding.
25. Aldosterone deficiency, which occurs in Addison disease, causes a non-anion gap,
hyperkalemic, hyponatremic metabolic acidosis. TB is a common cause of primary adrenal
failure in geographic regions where this organism is endemic (CXR shows a right upper lobe
cavity).
26. Injection drug users are prone to get tricuspid endocarditis caused by S. aureus. Fragments of
the vegetation can embolize to the lungs, causing the characteristic nodules infiltrate with
cavitation.
27. Measles (paramyxovirus) is characterized by a prodrome of cough, coryza, conjunctivitis,
followed by Koplik’s spots and maculopapular rash initially appearing on the face.
28. Always suspect sickle cell trait in a young black male who presents with painless hematuria—
papillary ischemia-necrosis.
29. Lumbosacral stain is the MCC of acute back pain. The typical clinical scenario includes acute
onset of back pain after physical exertion absence of radiation, presence of paravertebral
tenderness, negative straight-leg raising test, and normal neurologic exam.
30. Inhale albuterol and systemic steroids are appropriate tx for acute asthma attacks. Pts on high
doses of beta-2 agonists may develop hypokalemia (by driving potassium into cells), which
may present with muscle weakness, arrhythmias and EKG abnormalities. Other common side
effects of beta-2 agonists include tremor, palpitations and headache.
31. Impairment of daily functioning is essential in distinguishign between demantia and normal
changes of aging. Pts with dementia have functional impairments.
Normal aging Dementia
Independence in daily activities perserved Person becomes dependent on others for ADL
Person complains of memory loss but can May complain of memory problems only if
provide details about incidents of forgetfulness asked, unable to remember specific instances
where memory loss was noticed (by others)
Patient is more concerned about memory loss Close family members are more concerned
about memory loss
Recent memory for important events and Notable decline in memory for recent important
conversationd intact events and conversations

90
Occasional word-finding difficulties Frequent word-finding difficulty and
(expressive aphasia) substitutions, also some receptive aphasia
Does not get lost in familiar territory, may have Can get lost for hours in familiar territory while
to pause briefly to reorient walking or driving
Able to operate common appliances Becomes unable to operate common appliances
Maintain previous interpersonal social skills Shows loss of interest in social activities and
inappropriate behavior
Normal performance on mental status Abnormal mental status exam
examination

32. Unrecognized bowel ischemia is one of the common causes of lactic acidosis (results from the
production of lactate as an end product of anaerobic metabolism in the ischemic tissue) in pts
with severe atherosclerotic disease or atrial fibrillation.
33. Wilson’s disease is diagnosed by decreased serum ceruloplasmin, increased urinary copper,
and Kayser-Fleischer rings seen on slit lamp exam of the eye. Liver biopsy may resemble
alcoholic—mallory body
34. The persence of antibodies to ds-DNA is highly specific for and confirms the dx of SLE.
35. The indications for aortic valve replacement are: 1) all aymptomatic pts with AS. 2) pts with
severe AS undergoing CABG or other valvular surgery. 3) asymptomatic pts with severe AS
and either poor LV systolic function, LV hypertrophy>15mm, valve area <0.6 cm2 or
abnormal response to exercise. Aortic valvotomy is not nearly as effective as aortic valva
replacement. It should only be considered as a bridge to surgery in the hemodynamically
unstable pt or for poor surgical candidates.
36. Pts with carcinoid syndrome are at risk of developing niacin deficiency, owing to the
increased formation of serotomin from tryptophan. The classic triad of carcinoid syndrome
includes flushing, valvular heart disease, and diarrhea. Carcinoid tumors with hepatic
metastasis may be associated with this syndrome. Isolated tumors without metastasis do not
produce carcinoid syndrome. Niacin deficiency—3 D’s diarrhea, dermatitis and dementia
37. The leukocyte alkaline phosphatase score is high in leukemoid reactions, and usually low in
chornic myeloid leukemia.
38. Echocardiogram is necessary for the definitive dx of AS. Symptomatic AS requires valve
replacement.
39. Long-term cyclophosphamide use is associated with the increased incidence of acute
hemorrhageic cyctitis and bladder carcinoma.
40. Pts with non-inflammatory chronic prostatitis are afebrile and have irritative voiding
symptoms. Expressed prostatic secretions show a normal number of leukocytes and culture of
these secrestions is negative for bacteria.
41. up to 70% of pts with mitral stenosis will develop atrial fibrillation because of the significant
left atrial dilatation.
42. Of the vacterial pathogens know to cause secondary pneumonia complicating a viral upper
respiratory infection, only Staph aureus is associated with necrotizing bronchopneumonia
resulting in pneumatocoeles.
43. Clinical scenario describing a woman with chronic headache who presents with painless
hematuria is typical for anagesic nephropathy. Papillary necrosis is the cause of hematuria.
44. PPV increases with an increase in prevalence of disease in the study population. For NPV, the
inverse is true.

91
1/2/10
1. Nocardiosis in the setting of chronic corticosteroid use for RA. Nocardia asteroides is a G+
partially acid –fast, filamentous aerobe that is found in soil. Nocardia may initially be
confused with TB or Actinomyces infection. The most common symptoms include weight
loss, fever, and night sweats. Pulmonary involvement is characterized by cough productive of
purulent sputum. CXR findings include alveolar infiltrated and nodules, often with cavitation.
Chest wall invasion may be seen. Disseminated disease may involve any organ, but is most
likely to manifest as subcutaneous or brain abscesses. Nocardiosis usually occurs in
immunocompromised pts, including those on chronic corticosteroid therapy. Tx of choice—
trimethoprim-sulfamethoxazole. Penicillin G is the tx of choice for Actinomyces, another
filamentous G+ bacterium. 2 diseases may present similarly but there are several key
difference: 1) Actinomyces is anaerobic, not aerobic; 2) Actinomyces is more likely to cause
cervicofacial disease and sinus tracts than Nocardia; and 3) sulfur granules may be found with
Actinomyces.
2. Unexplained hemolytic anemia and thrombocytopenia in a pt with renal failure and neurologic
symptoms should raise strong suspicious for thrombotic thrombocytopenic purpura-hemolytic
uremic syndrome (TTP-HUS). The presence of fragmentated cells in the peripheral smear
suggested macioangiopathic hemolytic anemia (MAHA), a characteristic finding in TTP-
HUS. Idiopathic TTP-HUS is thought to be due to a deficiency of or autoantibody against a
specific von Willebrand factor-cleaving protease (ADAMTS-13). This causes the
accumulation of large von-Willebrand factor multimers and platelet aggregation. TTP-HUS is
fatal in 80% of cases if appropriate therapy is not promptly insituted, so a high degree of
clinical suspicion is necessary. Plasmapheresis (plasma exchange) is the tx of choice and
should be started as soon as possible. Plasmapheresis removes the offending autoantibodies
and repletes the deficient enzyme. Renal function impairment and peripheral blood smear
schistocytes may persist for several weeks following clinical recovery.
3. Elderly pts with depression often present with memory loss (pseudodementia). This can
mimic other causes of dementia, including Alzheimer’s dementia. CT findings are usually
normal and non-contributory for the differential diagnosis, whereas DST may be of value.
4. Lowering the cut-off point will consequently increase the sensitivity of a test. The true
positives will increase. However, the false positives will have a relatively large increase. This
results in a decrease in the PPV, as well as the FN.
5. In low risk pts (ie. <40 years and non smokers) a solitary pulmonary nodule is not a sign of
immediate alarm. The best approach is to ask for an old x-ray.
6. Carpal tunnel syndrome is the most common mononeuropathy of the upper extremity,
paresthesias of the first three-and-a-half digits and occasionally thenar eminence atrophy are
typical.
7. Pts with RA are at increased risk of developing osteopenia and osteoporosis due to the
combined effects of the disease process itself, glucocorticoid therapy, female sex and
decreased ability to perform weight-bearing exercises that prevent osteoporosis. Osteitis
fibrosis cystica (Von Recklinghausen disease of bone) is a condition in which osteoclastic
resorption of bone leads to replacement with fibrous tissue (brown tumors).
Hyperparathyriodism is the cause. Avascular necrosis is most commonly seen with the use of

92
systemic glucocortcoids and alcohol.
8. BAL is most useful in the evaluation of suspected malignancy and opportunistic infection.
BAL is >90% sensitive and specific for PCP. BAL is of less diagnostic utility in evaluating
other forms of interstitial lung disease, such as interstitial pulmonary fibrosis, sarcoidosis, and
connective tissue diseases.
9. Respiratory alkalosis is characterized by an increased pH and a primary decrease in the
PaCO2. Typically causes include huperventilation due to pneumonia, high altitude or
salicylate intoxication.
10. Hemorrhage is the most common complication of peptic ulcer disease.
11. Systemic sclerosis is a form of scleroderma with widespread organ involvement. GERD, right
heart failure and hypertension result from involvement of the esophagus, pulmonary atreries
and kidneys, respectively. Antinuclear autoantibodies and anti-topoisomerase-I antibodies
would most likely be present.
12. Any pt with an acute, severe disease may have an abnormal thyroid function test. This
condition is called sick euthyroid syndrome, and the most common thyroid hormone pattern in
such pts is a fall in total and free T3 levels with normal T4 and TSH levels. (Low T3
syndrome).
13. Pupillary dilation and blood at the external nares 鼻孔 in a pt with chest pain can be a clue to
cocaine-induced vasospasm. ST elevation myocardial infarctions can occur as a consequence
of cocaine use. Cocaine-induced ST elevation myocardial infarction (STEMIs) are treated the
same as classic STEMIs, with PTCA or thrombolysis. Aspirin and nitrates are also
appropriate, but beta-blockers should be avoided. Beta-blocker therapy administered to
cocaine-using individuals allows unopposed alpha agonist activiy that can worsen vasospasm.
CCBs and alpha blockers like phentolamine can also help reduce vasospasm.
14. Alzheimer’s disease is the most common cause of dementia in the western world. It is initally
characterized by memory loss, language difficulties and apraxia, followed by impaired
judgement and personality changes. CT scan shows generalized cortical atrophy.
15. Inactive of pancreatic enzymes by increased production of stomach acid may lead to
malabsorption in pts with Zollinger-Ellison syndrome (multiple duodenal ulcers are typical,
and a jejunal ulcer is almost pathognomic for this condition.).
16. Neutropenia is defined as an absoute neutrophil count (ANC) <1500/ml; this pt’s ANC is 280.
Susceptibility to infection increases when ANC falls below 1000/ml; ability to control
endogenous flora is lost and risk of death is markedly increased when the ANC falls below
500/ml. Fever in a neutrophenic pt is defined as a single temperature reading of greater than
38.3 (100.9F) ir a sustained temperature of greater than 38C (100.4F) over one hour. Bacteria,
fungi and viruses can all cause infection in neutropenia pts. Bacteria infections are the most
common and are frequently caused by endogenous skin or colon flora. Over the past decade,
there has been a shift from G- to G+ bacteria being the most frequent cause of neutropenic
infection. Febrile neutropenia is considered a medical emergency, thus, empiric antibiotics
should be started immediately. Empiric therapy should be broad-spectrum and should cover
Psudomonas aeruginosa. Either monotherapy or combination therapy can be employed.
Monotherapy consists of ceftazidime, imipenem, cefepime or meropenem. Combination
therapy is equally effective, and consists of an aminoglycoside plus an anti-pseudomonal beta-
lactam. Vancomycin is added to the empiric regimen when the pt is hypotensive, has severe

93
mucositis, had evidence of skin or line infection, has a history of colonization with resistant
strains of S.aureus or pneumococcus, or has had recent prophylaxis with fluoroquolones.
Fungal infections occur quite commonly in pts with prolonged neutropenia. When neutropenic
fever persists despite empiric antibacterial therapy, antifungal agents such as amphotericin B
are added to the empiric regimen.
17. The first step in the mx of a pt with heparin-induced thrombocytopenia (HIT) is immediate
cessation of all exposure to heparin, including low-molecular weight heparin. Subsequently,
an alternative means of anticoagulation is usually given, since most pts remain at risk of
thrombosis and require ongoing anticoagulation (eg. those with mechanical heart valves).
Currently, the 2 recommended alternative tx are danaparoid and a direct thrombin inhibitor
(lepirudin, argatroban). The use of warfarin without other anticoagulants should be avoid in
pts with HIT until the platelet count rises above 100,000/ml. Without heparin, warfarin
therapy alone may increase the risk of venous limb gangrene in pts with deep vein thrombosis.
18. Anemia can be due to decreased RBC production, increased RBC destruction, and frank blood
loss. The anemia of lymphoproliferative disorders is due to bone marrow infiltration with
cancerous cells.
19. Disseminated gonococcemia causes high fever, chills, tenosynovitis, migratory polyarthralgias
and a small number of hemorrhagic pustular lesions on the extremities. Routine blood and
pustule cultures are typically negative due to the growth requirements of the organism.Toxic
shock syndrome doesn’t have pustules,
20. Outflow obstruction in hypertrophic cardiomyopathy results from both septal hypertrophy and
systolic anterior motion (SAM) of the mitral valve.
21. If a pt has mild acne (non-inflammatory comedones), topical retinoids are usually tried first.
Oral antiboitics are usually used if papular and inflammatory acne (moderate-to-severe
inflammation) is present. Oral isotretinioin is reseved for pts with nodulocystic and scarring
acne.
22. Chronic HCV is associated with number of extra hepatic complications like 1)
cryoglobulinemia, 2) B cell lymphomas. 3) Plasmacytomas, 4) Autoimmune diseases like
Sjogren’s syndrome and thyroiditis, 5) Lichen planus, 6) Porphyria cutanea tarda, 7)
Idiopathic thrombocytopenic purpura. Some of these complications are immune complex
mediated, however immune complex mediated extrahepatic complications are more common
with HBV than with HCV except for cryoglobulinemia.
23. Chronic corticosteroid use and chronic excessive ingestion of alcohol account for over 90% of
cases of avascular necrosis of bone (osteonecrosis). In the hip, pts present with slowly
progressive anterior hip pain with limitation of range of motion.
24. Always suspect multiple sclerosis (MS) in a female with multiple neurologic presentations
that are interspaced between time periods. Optic neuritis can result in blurring of vision, and is
often associated with retrobulbar pain.
25. Abdominal CT scan is the next diagnostic test when abdominal ultrasound does not explain
cholestatic jaundice.
26. The two most common causes of painless GI bleeding in elderly pt over 65 years of age are
diveriticulosis and angiodysplasia (vascular ectasia, Painless bleeding). There is a well-
defined association between aortic stenosis and angiodysplasia. Another well-recognized
association is renal failure. Ectasia can present thoughout the colon; however, the most often

94
site of bleeding is the cecum or ascending colon. The colonoscopic finding of a cherry red
fern-like pattern of blood vessels that appear to radiate from a central feeding vessel is
diagnostic. Pts with diverticulosis will have colonoscopic findings of multiple diverticuli with
a diverticular bleed. Furthermore, most diverticula are located in the sigmoid colon; therefore,
sigmoidscopy would have shown these. *Extremely high Yield!!
27. Pts with rheumatoid arthritis and those who have joint damage are an increased risk of
developing septic arthritis, particularly with Staph aureus. Inflammatory monoarthritis
presents acutely with a red, swollen joint and motion limited by pain; the differential
diagnosis includes septic arthritis, crystal-induced arthritis or trauma.
28. Damage to the lateral spinothalamic tracts causes contralateral loss of pain and temperature
sensation beginning two levels below the level of the lesion.
29. Hypocalcemia with concordant changes of serum calcium and phosphate levels are usually
caused by vit D deficiency (eg. due to malabsorption in alcoholic). Hypoalbuminemia is a
common cause of decreased serum calcium level, but ionized calcium level remains
unchanged; therefore, no symptoms of hypocalcemia are usually present. Primary
hypoparathyroidism and pseudohypoparathyroidism cause discordant changes in serum
calcium (low) and phosphate levels (high).
30. Loop or thiazide diuretic use is one of the MCC of metabolic alkalosis. Such pts are typically
volume depleted and hypokalemic. Other causes of metabolic alkalosis include
hyperaldosteronism and vomiting.
31. Primary syphilic (chancre) is best diagnosed with dark field microscopy. Serological tests
(RPR & VDRL) are usually negative in primary syphilis but are positive in almost all cases of
secondary syphilis.
32. Seborrheic dermatitis is characterized by waxy scales with underlying erythema on the scalp,
central face, presternal region, interscapular areas, umbilicus and body folds. It may be seen in
association with Parkinsonism or HIV. Lichen simplex chronicus is the result of chronic
scratching, picking or rubbing of the skin. Manifestation include thickened plaques on the
skin with increased skin markings and hyperpigmentation or thickened papules with
excoriated centers.
33. Anterior artery stroke: contralateral motor and /or sensory deficits which are more pronouced
in the lower limb than the upper limb. Other features that may be seen include urinary
incontinence, gait apraxia, primitive reflexes (eg. grasp and sucking), abulia, and paratonic
rigidity.
Middle cerebral artery stroke: contralateral motor and /or sensory deficit (more pronounced in the
upper limb than lower limb) and homonymous hemianopia. If the dominant lobe (left) is involved,
the pt may have aphasia; whereas, if the non-dominant lobe (right) is involved, the pt may have
neglect and/or anosognosia.
Posterior cerebral artery stroke: homonymous hemianopia, alexia without agraphia (dominat
hemisphere), visual hallucinations (calcarine cotex), sensory symptoms (thalamus), third nerve
palsy with pareses of vertical eye movement, and motor deficits (cerebral peduncle, middle brain).
Internal carotid artery generally results in signs and symptoms of middle cerebral artery occlusion,
along with visual symptoms, such as smaurosis fugax (temporary monocular blindness).
Lacunar infarcts are small noncortical infarcts cuased by occlusion of a single penetrating branch
of a large cerebral artery. The associated risk factors are hypertension, DM, or polycythemia.

95
Lacunar infarcts usually do not produce both sensory and motor deficits, and instead have well
recognized syndromes (eg. pure motor hemiparesis, pure sensory stroke, dysarthria-clumsy hand,
ataxic hemiparesis).
34. Glomerular hyperfiltration is the earliest renal abnormality seen in diabetic nephropathy. It is
also the major pathophysiologic mechanism of glomerular injury in these pts. Thickening of
the glomerular basement membrane is the first change that can be quantitated.
35. An increase in serum levels of both insulin and C-peptide and hypoglycemia is diagnostic of
insulinoma. The whipple triad of insulinoma clinically: 1) episodic hypoglycemia, 2) central
nervous system dysfunction temporally related to hypoglycemia (confusion, anxiety, stupor,
convulsions, coma). 3) dramatic reversal of CNS abnormalities by glucose administration.
36. Necrotizing fasciitis is characterized by purpish discoloration of the skin with gangrenous
changes and systemic signs of toxicity. Thorough surgical debridement of all the necrotic
tissues is the most important therapy. Empiric antibiotic therapy should cover aerobic and
anaerobic organisms (ampicillin dn sulbactam plus clindamycin). CT scan is useful in
identifying the involved site, which reveals necrosis, asymmetrical fascial thickening and gas
in the tissues. Thrombophlebitis is characterized by palpable, indurated, cord-like, tender,
subcutaneous venous segments.
37. Progressively decreasing baroreceptor sensitivity and defects in the myocardial response to
this reflex are the main reasons for the increased incidence of orthostatic hypotension in the
elderly.
38. Pregnancy causes a physiological chronic compensated respiratory alkalosis due to the
stimularoty effect of progesterone on the medullary respiratory center.
39. Mitral stenosis classically presents during pregnancy. It is most often due to rheumatic fever
and occurs much more often in countries with limited access to antibiotics.
40. Diphenhydramine overdose leads to anti-histamine effects including drowsiness, confusion
and anticholinergic effects including dry mouth, dilated pupils, blurred vision, reduced bowel
sounds, and urinary retention. Physostigmine, a cholinesterase inhibitor can counteract these
anti-cholinergic effects. Serotonin syndrome tends to occur as a result of intentional overdose,
or as a consequence of drug interactions (classically when MAOI’s are mixed with other
serotonergic agonists). Presentation includes tachycardia, diaphoresis, dilated pupils,
hypertension, and hyperthemia. The other anticholinergic symptoms/signs are uncommon in
this syndrome.
41. Cervical cancer screening with a Pap smear should begin at age 21 or 3 years after the
initiation of sexual intercourse, whichever comes first. The interval of screening can increase
in women with 2-3 consecutive negative smears and the same monogamous partners. Women
without a cervix and any history of gynecologic malignancy do not need screening.
42. All pts who are start on anti-TB therapy should also be started on vitamin supplements,
especially pyridoxin (10ng/day), to avoid peripheral neuropathy and other neurological
complications. Hepatitis is another known side effect of isoniazide.
43. Acute bronchitis is a common cause of blood-tinged spurum and is usually viral in etiology. In
an afebrile pt with new-onset blood-tinged sputum without significant signs of more serious
disease, observation and close clinical follow-up is the best tx strategy.
44. 1) pts with central retinal vein occlusion usually present with subacute monocular visual loss.
The diagnossi is usually made by finding a “blood and thunder” appearance on fundoscopic

96
exam consisting of optic disk swelling, retinal hemorrhage, dilated veins, and cotton wool
spots. 2) Central retinal artery occlusion is characterized by sudden, painless loss of vision in
one eye, and the funduscopic exam reveals pallor of the optic disk, a cherry red fovea, and
boxcar segmentation of blood in the retinal veins.

1/3/09
1. Malignant otitis externa (MOE) is a serious infection of the ear seen in elderly pts with poorly
controlled diabetes, and is most commonly caused by Peudomonas aeruginosa. The
characteristic presentation consists of ear pain and ear drainage, and granulation tissue may be
seen within the ear canal on examination. Progression of the infection can lead to
osteomyelitis of the skull base and destruction of the facial nerve. Pts with poorly controlled
diabetes are susceptible to Rhizopus infections, but these infections typically begin in the
paranasal sinuses and extend into the orbit and brain.
2. Suspect the following conditions whenever a pt presents with hypokalemia, alkalosis and
normotension: 1) Surreptitious vomiting, 2) Diuretics abuse, 3) Bartter syndrome, 4)
Gitelman’s syndrome. Physical findings that are characteristic of surreptitious vomiting are
scars/calluses on the dorsum of the hands, and dental erosions. The dorsal scars result form
repeated chemical/mechanical injury as the pt uses his/her hands to induce vomiting. Dental
erosions result deu to increased exposure to gastric acid. Surreptitious vomiting may also
result in hypovolemia and hypochloremia, which in turn lead to a low urine chloride
concentration.
Pts with diuretic abuse and Bartter/Gitelman’s Syndrome may also present with hypokalemia,
alkalosis and normotension, but their urine chloride concentrations are high. Pts with diuretic
abuse may give a history of diuretic abuse and have a positive urine assay for diuretics.
Pts with chornic diarrhea have metabolic acidosis and hypokalemia (loss of bicarbonate in the
stool).
Pts with primary hyperaldosteronism have hypokalemia, metabolic alkalosis and hypertension.
3. TIA can be due to intrinsic blood vessel abnormalities (eg. atherosclerosis, inflammation,
dissection, etc.), embolic disease or decreased perfusion pressure. In elderly pts, the MCC of
TIAs are atherosclerosis and emboli. In younger pts, atherosclerosis is less likely, and one
should consider other causes such as emboli, vasculitis, dissection, malformations and
hypercoagulable states—most likely investigations- transthoracic echocardiogram
4. Histoplasma capsulatum is a common and usually asymptomatic infection in endemic areas
like Mississippi and Ohio River velleys and Central America. It is usually found in soil with a
high concentration of bird or bat guano droppings.
5. Anserine bursitis presents with sharply localized pain over the anteromedial part of the tibial
plateau just below the joint line of the knee. Valgus stress test fails to reproduce the pain,
thereby ruling out damage to the medial collateral ligament, and radiographs are classically
normal.
Prepatellar bursitis presents with pain and swelling directly over the parella. Examination shows
cystic swelling over the patella with variable signs of inflammation.
Patellofemoral syndrome is a common overuse pain syndrome of the knee. Pts present with
peripatellar pain worsened by activity or prolonged sitting (due to sustained flexion) and may also
complain of crepitus with motion of the patella.

97
6. If a pt presents with 3 hrs after the onset of an ischemic stroke, thrombolytic therapy with tPA
(after CT scan) should be started. Many trials have demonstrated improved neurological
outcomes with this approach. In the first 24-hrs the tole of CT scan is essentially to rule out
any hemorrhage in which case aspirin and tPA are C/I.
Nimodipien is used in the mx of subarachnoid hemorrhage (SAH). It is a selective cerebrovascular
dilator and reduces the vasospasms that are induced by the SAH.
7. Ewing’s sarcoma is often confused with osteomyelitis; however, characteristic x-ray findings
can point to the diagnosis. The lesion is usually lytic, central, and accompanied by endosteal
scalloping. The ‘onion -skin’ appearance is often followed with a “moth-eaten” or mottle
appearance and extension into soft tissue. The tx includes surgery, radiation, and multi-drug
chemotherapy. Radiation or chemotherapy is used preoperatively.
8. Pulmonary embolism is a common cause of pleuritic chest pain. It should be high in the
differential of a young person presenting with pleuritic chest pain, tachycardia, and dyspnea,
particularly if the pt is taking an OCP. One common cause of pleuritic chest pain is a
pulmonary embolism with subsequent minor or massive pulmonary infarction.
9. Acute acalculous cholecystitis is an acute inflammation of the gallbladder in the absence of
gallstones, most commonly seen in hospitalized and severely ill pts. 1) extensive burns, 2)
Severe trauma, 3) Prolonged TPN, 4) Prolonged fasting, 5) Mechanical ventilation. Involved
pathophysiology is localized or generalized ischemia, biliary stasis, infection, or external
compression of the cytic duct. It generally presents with right upper quadrant abdominal pain
with fever, leukocytosis, and abnormal live function tests. It can lead to complications like
gangrene, perforation, and emphysematous cholecystitis. The initial investigation of choice is
an USG, which shows signs of acute cholecystitis and no gallstones; however, CT scan and
HIDA are more sensitive and specific for the dx. It is important to have a high degree of
suspicion in high-risk pts in order to correctly identify this problem.
10. Emphysematous cholecystitis is a common form of acute cholecystitis in elderly diabetic
males. It arised due to infection of the gallbladder wall with gas-forming bacteria (eg.
Clostridium, Escherichia, Staph, Strep, Pseudomonoas, and Klebsiella). Predisposing factors
include vascular compromise (by obstruction or stenosis of the cystic artery, which is the sole
arterial supply to the gallbladder), immunosuppression (as seen with DM), gallstones, and
infection with gas-forming bacteria. Clinical manifestations of emphysematous cholecystitis
include right upper quadrant pain, nausea, vomiting, and low-grade fever. Crepitus in the
abdominal wall adjacent to the gallbladder is occasionally detectable. But peritoneal signs are
absent. Complications include gangrene and perforation. If perforation occurs, the abdominal
pain is relieved but peritonitis results. Dx is confirmed with an abdominal radiograph
demonstrating air fluid levels in the gallbladder or an ultrasound showing curvilinea gas
shadowing in the gallbladder. CT scan may be of help in confirming the dx if other imaging
modalities are unclear. Laboratory findings may include a mild to moderate unconjugated
huperbilirubinemia or a small elevation in aminotransferases. Tx includes immediate fluid and
electrolyte resuscitation, early cholecystectomy, and parenteral antibiotics therapy that is
effective against G+ anaerobic Clostridium sp. (ampicillin-sulbactam, piperacillin-
tazobactam, or the combination of an aminoglycoside or quinolone with clindamycin or
metronidazole).
11. Hypercalcemia typically causes nonspecific symptoms such as abdominal pain and

98
constipation, polyuria and neuropsychiatric disturbances. The acute tx of hypercalcemia is iv
0.9% saline hydration followed by a loop diuretic. Bisphosphonates (Pamidronate) decrease
plasma calcium concentrations by decreasing osteoclast activity. They are more effective than
calcitonin and are the 2nd-line agent used for pts with moderate to severe hypercalcemia.
12. The power of a study is its ability to detect a difference between two groups (treated versus
non-treated, exposed versus non-exposed). Increasing the sample size increases the power of a
study, and consequently makes the confidence interval of the point estimate (eg. relative risk)
tighter.
13. Surgical resection followed by whole brain radiation is the standard of practice in the mx of
solitary brain metastasis with stable extracranial disease. Multiple brain metastases are best
treated with palliative whole brain radiation. * Extremely high-yield!!
14. Chronic HBV pts with persistently elevated ALT levels (one of the most important predictors
of response to both interferon and lamivudine), detectable serum HbsAg, HbeAg, and HBV
DNA should be treated with interferon or lamivudine.
15. Brain death refers to a total loss of brain function and is a legally acceptable definition of
death. Criteria include: absent cranial nerve reflexes, fixed and dilated pupils, no spontaneous
breaths, and agreement of two physicians. People who die within 24 hours of presnetation, die
from an unknow cause, die from a medical complication or due to suspected illegal activities
must be reported to the coroner or medical examiner.
16. Infective endocarditis is especially common in IV drug abusers. It can be fatal if initial
empirical tx (eg. vancomycin+gentamicin, covers Staph, Enterococci, and Strep) is delayed
17. Amidarone is a class III antiarrhythmic agent; well know for causing pulmonary fibrosis.
Thyroid dysfunction (hypo and hyper), hepatotoxicity corneal deposits and skin discoloration
(blue-grey) are other potential side effects.
18. Acute exacerbation of COPD is treated with a combination of inhaled/neubulized
bronchodilators and systemic steroids. N-acetylcysteine is a mucolytic agent, which is no
linger used in exacerbation of COPD, as they are even implicated in worsening
bronchospasm. Aminophyllin is proven to inferior to a combination of bronchodilator and
corticosteroids for the tx of acute exacerbation of COPD.
19. Trimethoprim/Sulfamethoxazole is used in HIV-positive pts to prevent opportunistic
infections caused by pneumocystis jiroveci and toxoplasma gondii.
20. A detailed medical history and physical examination is the most effective way to screen a low
risk population for the presence of underlying cardiac disease (eg. hypertrophic
cardiomyopathy).
21. Squamous cell carcinoma is the second most common form of non-melanoma skin cancer.
The single most important risk factor for the development of squamous cell carcinoma is
exposure to sunlight.
22. Mammograms should be performed every 1-2 years starting at age 40 in women at average
risk for breast cancer. There is no clear stop time established but many experts recommended
discontinuing mammograms no sooner than age 70.
23. Suspect absence seizures in a 4- to 8-year-old child with frequent daydreaming episodes, brief
staring spells, decline in school performance, and no post-ictal phase. The preferred tx is
ethosuximide or valproate.
24. When evaluating a pt suspicious for pseudotumor cerebri, do a lumbar puncture only after

99
completely ruling out a space-occupying brain lesion with neuroimaging (CT/MRI). Most pts
with pseudotumor cerebri have an empty sella seen on neuroimaging, this is probably caused
by the downward herniation of arachnocele due to the high CSF pressure. The following
criteria is used for the dx (exclusion) of idiopathic intracranial hypertension: 1) presence of
features of increased intracranial pressure in an alert pt. 2) absence of focal neurological signs
except sixth nerve palsy. 3) Normal CSF examination except increased CSF pressure. 4)
Absence of any ventricular abnormality other than enlargement on neuroimaging and absence
of other causes of increasd intracranial pressure.
25. Seminoma—PLAP, Embryonal Carcinoma—AFP+ beta-hCG (50%), Choriocarcinoma: beta-
hCG
26. Screening for bladder cancer is not recommeded, even in pts who are at risk of developing the
disease.
27. Empyema most commonly occurs in the setting of untreated pneumonia due to bacteria
seeding of a pleural effusion. Inflammation and collection of pus in the pleural cavity leads to
the formation of a loculated, abnormally contoured effusion with adjacnet pulmonary
consolidation. A bronchopleural fistula would most likely have radiographic evidence of air in
the pleural space.
28. Polycystic kidney disease is a relatively common autosomal dominant disease. It typically
presents with hematuria and hypertension but may also cause abdominal masses and pain. It is
associated with intracerebral aneurysms.
29. Enterotoxigenic E. Coli is the most frequent cause of traveler’s diarrhea. Travelers with
abdominal cramps, diarrhea, and malaise are likely to be infected with this organism.
30. Diagnosis of bullous pemphigoid is made by skin biopsy which shows deposits of IgG
(autoantibodies against the basement membrane glycoproteins BP230, BP180), and
complement in a linear pattern at the dermal-epidermal junction. It is characterized by tense
blisters in the flexural areas. This condition usually occurs in elderly pts (>60) with an
antecedent history of months of urticaria, and no oral lesions. Pts usually present due to the
pruritis. The identified precipitating factors are ultra-violet rays, NSAIDs, and antibiotics.
Pemphigus vulgaris is characterized by IgG deposits intercellularly in the epidermis. This
condition is an intra-epidemal blistering diease with autoantibodies to adhesion molecules
expressed in skin and mucous membranes. The bullae are usually flaccid.
Cicatricial 瘢 痕 的 pemphigoid is characterized by IgG deposits in a linear band at the dermal-
epidermal junction. This is an autoimmune blistering disease that affects the mucous membranes,
including the mouth, oropharynx, conjunctiva, nares, and genitalia.
Herpes gestationis is characterized by C3 the basement membrane zone. This condition usually
presents as sub-epidermal blisters in the second trimester of pregnancy.
Intradermal edema with leukocyte infiltraion is a feature of urticaria.
31. Brain death is a clinical diagnosis. The characteristic findings are absent cortical and brain
stem functions. The spinal cord may still be functioning; therefore, deep tendon reflexes may
be present.
32. Anabolic steroids are commonly abused by athletes in an effort or improve performance. Side
effects of these drugs include suppressed endogenous testicular function (resulting in reduced
fertility), gynecomastia, erythrocytosis, hepatotoxicity, psychological disturbance, cardiac
disease, dyslipidemia (lowered HDL and elevated LDL), increased coagulation, and

100
premature peiphyseal function (which stunts growth). Virilization is often seen in females who
abuse steroids.
33. Staph aureus is the most frequent cause of prosthetic joint septic arthritis.
34. Treat acute glaucoma emergently with mannitol, acetazolamide, pilocarpine or timolol. Avoid
mydriatic agent such as atropine.
35. The dx of pernicious anemia is confirmed by achlorhydria, decreased serum vit B12, positive
IF antibodies, and extremely elevated LDH. Elevated bilirubin and serum iron are also seen
but they are not specific.
36. Beta blocker overdose causes AV block, bradycardia, hypotension, wheezing (bronchospasm,
beta2 blockade), and potential cardiogenic shock. Atropine and IV fliuds are the first line of
therapy, and if they do not completely reverse the cardiac symptoms, glucogon should be
administered. Intoxication of CCBs, digoxin, and cholinergic agents would cause some of the
above symptoms, but wheezing is indicative of beta-blocker toxicity.
37. Respiratory acidosis is characterized by a decreased pH and a primary increase in the PaCO2.
Alveolar hypoventilaton is an important cause of respiratory acidosis. Acute pulmonary
embolism, atelectasis, pulmonary edema and pleural effusion can cause a decrease in PaO2
(hypoxemia), but these conditions typically also cause a decrease in PaCO2 (respiratory
alkalosis) due to concomittant tachypnea.
38. CPK levels should be checked in any pt on a statin who presents with myalgias. If highly
elevated, the first step is to discontinue the statin.
39. Cardiovascular disease is the MCC of death in dialyssi pts. It accounts for approximately 50%
of all deaths in this population. Cardiovascular disease is also the MCC of death in renal
transplant pts.
40. Pseudomonas is commonly responsible for nosocomial pneumonia in intubated pts. Fourth
generation cephalosporins (cefepime, ceftazidime) have been used successfully for tx. Other
effective medications include aztreonam, ciprofloxacin, imipenem/cilastin, tobramycin,
gentamicin, and amikacin. Piperacillin-tazobactam plus tobramycin is also highly effective.
41. Bilateral lower extremity pitting edema may be a sign of hypoalbuminenia secondary to
malnutrition. Decreased oncotic pressure causes the edema.
42. Severe recurrent respiratory infections and positive sweat chloride test are characteristic for
cystic fibrosis. Fat malabsorption is common in pts with cystic fibrosis and is due to exocrine
deficiency of the pancreas; therefore, deficiency of fat-soluble vitamins, including vitamin K,
is frequently present if no vitamin supplementation is provided. Vitmin K is an important
cofactor in posttranslational modification of several coagulation factors: Factors II, VII, IX,
and X as well as protein S and C.
43. Neisseria gonorrhea is the MCC of arthritis in young, sexually active pts. Gonococcal septic
arthritis may present in one of two ways: as an asymmetric polyarthritis (often associated with
tenosynovitis and skin rash) or as an isolated purulent arthritis affecting one or a few joints.
Diagnosis may be confirmed by gram stain of the synovial fluid, blood cultures, urethral
cultures or skin cultures. Although skin rash and tenosynovitis are not seen in this pt, purulent
arthritis in as sexually active individual is gonococcal arthritis until proven otherwise.
Reactive arthritis may be caused by genitourinary infection with Chlamydia trachomatis or by
certain GI infections. It presents with a triad of arthritis, conjunctivitis, and urethritis.
44. CT scan without contrast should be done in all pts with a stroke to distinguish between

101
hemorrhagic or ischemic stroke. If there is evidence of ischemic stroke, carotid Doppler and
TEE are performed to evaluate the possible source of embolism. Stroke can be subdivided
into subarachnoid hemorrhage, ischemic stroke and intracerebral hemorrhage. Subarachnoid
hemorrhage is usually accompanied by a sudden, dramatic onset of a severe headache.
Ischemic strokes are usually accompanied by a history of previous transient ischemic attacks.
Pts with ischemic stroke usually don’t have headache and impaired consciousness.
Hemorrhagic stroke is characterized by focal neurological signs that develop suddenly and
gradually worsen over minutes or hours. The onset of symptoms is not abrupt as in
subarachnoid hemorrhage or embolic stroke. Symptoms usually start during normal activity or
may be precipitated by sex or strenuous activity. As the hemorrhage expands, headache,
vomiting and altered mental status develop. Hypertension is the most important risk factor for
hemorrhagic stroke.

1/4/10
1. Vasospasm is the major cause of morbidity and mortality in pts with subarachnoid
hemorrhage (SAH). CCBs (nimodipine) are used to prevent vasospasm in pts with SAH.
2. Renal transplant dysfunction in the early post-operative period can be explained by a variety
of causes, including ureteral obstruction, acute rejection (biopsy shows heavy lymphocyte
infiltration and vascular involvement with swelling of the intima), cyclosporine toxicity,
vascular obstruction, and acute tubular necrosis. Radioidotpy scanning, renal ultrasound,
MRI, and renal biopsy can be employed in conducting a differential diagnosis. Acute rejection
is best treated with iv steroids. Cyclosporine toxicity does not present with graft tenderness.
The transplant’s function is usually restored when the dose of cyclosporin is decreased. In this
case, the pt’s clinical features (ie. Normal serum cyclosporine level, renal biopsy findings) are
more consistent with acute rejection. Administration of IV diuretics may be employded in
acute tubular necrosis. * Extremely high yield!!
3. Pts with Huntington’s chorea usually present in their forties or fifties with chorea and/or
behavioral disturbance. Atrophy of the caudate nucleus is a characteristix feature. Diffuse
atrophy of the cerebral cortex is a feature of Alzheimer’s disease. Atrophy of the lenticular
nucleus is most marked in pts with Wilson’s disease. Atrophy of the frontal and/or temporal
lobes is a feature of Pick’s disease, which presents with dementia and personality changes.
4. The p value is the probability that the result of a study was obtained by chance alone. A study
is considered statistically significant when the ‘p’ value used is less than 0.05. Know how to
interpret the ‘p’value and its relationship with the confidence interval.
5. High serum alpha-fetoprotein (>500 ng/ml) in an adult with liver disease and no obvious GI
malignancy is strongly suggestive of hepatocellular carcinoma.
6. Cholangiocarcinoma can complicate primary sclerosing cholangitis, especially in pts who
smoke and have ulcerative bolitis. Next best step in Mx—order biopsy of the prominent
stricture.
7. Macrocytic or megaloblastic anemia is charaterized by the following RBC indices: elevated
MCV, elevated MCH (in 90% of cases), and normal MCHC. The peripheral blood smear
typically shows macroovalocyte RBCs and hypersegmented neutrophils with a mean lobe
count greater than 4. Anisocytosis, poikilobytosis, and basophilic stippling (not only seen in
lead poisoning—microcytic anemia, MCV<80) are also seen.

102
Lab tests Pernicious Anemia (B12 Folic acid deficiency
deficiency)
Serum B12 level Decreased Normal
Serum folic acid level Normal Decreased
Serum LDH level Increased Normal
Achlorhydria Present Absent
Schilling test Positive Negative
Methyl malonyl CoA in urine Present Absent
Neurological signs Present Absent

8. The tx of Clostridium difficile diarrhea includes discontinuation of the offending antibiotic


(usually clindamycin) and prompt administration of metronidazole.
9. Pts with nephrotic syndrome are at incrased risk for developing hypercholesterolemia
(accelerated atherogenesis), hypertriglyceridemia, and hypercoagulable state.
10. Lumbar spinal stenosis is a common cause of back pain in pts over 60 years of age. It is
characterized by back pain radiating to the buttocks and thighs that interfere with walking and
lumbar extension. Lumbar flexion alleviated the symptoms. The prdal pulses should remain
normal, as arterial perfusion is not affected. The dx is confirmed with MRI. Lumbar disk
herniation typically presents with the acute onset of back pain with or without radiation down
one leg. Pts usually recall an inciting event. As with lumbar spinal stenosis, the pain may be
radiating and be associated with neurologic symptoms. However, in disc herniation, lumbar
flexion and sitting will make the pain worse.
11. Ankylosis spndylitis occurs most often in adults aged 20-30 years. Pts suffer from limited
spine mobility and progressive back pain of greater than 3 months duration. The pain and
stiffness are worse in the morning and improve with exercise. Bilateral ascroiliitis on plain
film of a pt with the above symptoms is diagnostic. The most prominent extraarticular
manifestation of AS is anterior uveitis.
12. HSV most frequently affects the temporal lobes of the brain. As a result, features such as
bizarre behavior and hallucinations may be present. The disease is usually abrupt in onset,
with fever and impaired mental status. Maningeal signs are frequently absent. Cerebral fluid
(CSF) findings are nonspecific, with low glucose levels and pleocytosis. The diagnostic test of
choice is CSF PCR for HSV DNA. However, IV acyclovir should be started without delay if
there is a suspicion of this dx.
13. Angle closure glaucoma occurs predominanty in people aged 55-70 years. It presents with an
acute onset of severe eye pain and blurred vision associated with nausea and vomiting.
Examination reveals a red eye with steamy cornea and moderately dilated pupil that is non
reactive to light.
14. Effect modification occurs when the effect of a main exposure on an outcome is modified by
another variable. In this scenario, the effect of oral contraceptives on the incidence of breast
cancer is modified by the family history: women with a positive family history have an
increased risk, while women without a positive family history do not have an increased risk.
There are other well-known examples of effect modification, such as: studying the effect of
estrogens on the risk of venous thrombosis (modified by smoking), and studying the risk of
lung cancer in people exposed to asbestos (greatly depends on /modified by smoking status).
Effect modification is NOT a bias!

103
15. Jaundice in the 3rd trimester of pregnancy should be evaluated specifically for those hepatic
disorders associated with pregnancy. Marked pruritus and elevated of total bile acids are
suggestive of intrahepatic cholestasis of pregnancy. Tx is based on symptom releif and
preventing complications in the mother and fetus. Ursodeoxycholic acid is most promising, as
it increases bile flow and can releive prutitus. Hydroxyzine and cholestyramine are alternative
therapies, though they appear to be less effective and can have concerning side effects.
The maternal prognosis of those with ICP is good, as the condition resolves shortly after delivery.
There are no hepatic sequelae. ICP may recur in subsequent pregnancies, and affected women are
also at increase risk of developing gallstones.
ICP poses more danger to the fetus than the mother, as fetal prematurity, meconium-stained
amniotic fluid, and intrauterine demise are all known comolications. Therefore, the preferred
approach in managing the pregnancies of women with ICP includes an early delivery once fetal
lung maturity is established.
Normal physiological changes of pregnancy include a mild elevation of alkaline phosphatase (due
to placental secretion of the enzyme) but normal bilirubin, aminotransferases, and GGT.
Acute fatty liver of pregnancy is a complication of the 3 rd trimester of pregnancy. Pts are
sometimes asymptomatic, but may also complain of sever nausea, malaise, headache, and
abdominal pain. Over half of pts also have preeclampsia, and 60% will develop acute renal failure.
This condition can also cuase a prolonged prothrombin time and moderate to severe elevation of
transaminas levels. Histologically, acute fatty liver of pregnancy is characterized byh
microvesicular fat deposition in the hepatocytes without evidence of inflammation or necrosis.
16. Metformin should not be given to pts with renal failure, hepatic failure or sepsis, as these
conditions all worsen a pt’s lactic acidosis.
17. Vertebral osteomyelitis is usually hematogenous, and the lumbar area is most commonly
affected. Back pain accompanied by low-grade fever and elevated ESR is the typical
presentation. High-grade fever and chills are uncommon. Physical examination may
demonstrate local tenderness on percussion over the affected vertebrae and paravertebral
muscular spasm. Early diagnosis is very important in pts with this condition, because epidural
abscess and spinal cord compression may develop if the tx is delayed. MRI of the spine is the
most sensitive test for identification of vertebral osteomyelitis. Since MRI is excellent for
evaluation of soft tissues, it will also reveal whether there is any abscess or cord compression.
Blood cultures are also important in dx, and allow directed antibiotic therapy.
18. This pt most likely has hereditary telangiectasia (Osler-Weber-Rendu syndrome), an
autosomal dominant disorder characterized by diffuse telangiectasias, recurrent epistaxis, and
widespread AV malformations (AVMs). In hereditary telangiectasia, AVMs tend to occur in
the mucous membranes, skin and GI track, but may also be present in the liver, brain, and
lung. AVMs in the lungs can shunt blood from the right to the left side of the heart, causing
chronic hypoxemia and a reactive polycythemia. Pulmonary AVMs can also present as
massive, sometimes fatal, hemoptysis. (On physical exam, there are several ruby-colored
papules on his lips that blanch partially with pressure).
19. Diabetic foot infection can be classified as follows: 1) Non-limb-threatening infections are
superficial, lack systemic toxicity and have minimal cellulitis extending less than 2cm from
the portal of entry. Ulceration, if present, does not fully extend through the skin, lacking
significant ischemia. S. aureus is the major pathogen followed by facultative streptococci;

104
however, facultative G- bacilli and anaerobes are not uncommon. 2) Limb-threatening
infections have more extensive cellulitis, lymphangitis, ulcers penetrating through the skin
into subcutaneous tissue, and prominent ischemia. Such infections are commonly
polymicrobial with S. aureus, Group B streptococci, Enterococcus and facultative G- bacilli
being major pathogens along with anaerobic G+ cocci and Bacteroides species.
IV cefotetan, ampicillin/sulbactam, or the combination of clindamycin and a fluroquinolone is the
appropriate empirical tx for limb-threatening infections; whereas mild, or non-limb threatening,
infections can be treated with oral antibiotics like cephalosprin, clindamycin,
amoxicillin/clavulanate and fluoroquinolones.
20. Restless leg syndrome (RLS) had 4 cardinal symptoms: 1) an uncomfortable sensation or urge
to move the legs. Discomfort--2) which worsens in the evening or during sleep, 3) worsens at
rest, and 4) alleviated by movement of the affected limb(s). The lower extremity discomfort
may also get better after massage or warming. RLS is more common in middle aged and older
pts, as well as in those with chronic kidney disease and/or iron deficiency anemia. It can have
a profoundly negative impact on the sleep of pts and their sleeping partners. The mechanism
of primary RLS is unclear but it may involve abnormalities of dopaminergic transmission in
the central nervous system. Dopaminergic agonists (pramipexole and ropinerole) or levodopa
can be effective in reducing symptoms and improving sleep quality.
21. Essential tremor is characterized by a tremor that is suppressed at rest and exacerbated toward
the end of a goal-directed movement. Affected pts typically have difficulty holding a
newspaper, writing legibly, drinking a cup of coffee, of feeding themselves.
22. First line tx for essential tremor if propranolol, esp if the pt is also hypertension. Alternative
medications include primidone or topiramate
23. Mild intermittent asthma, the least severe form, is defined as symptom <= 2x/week, <=e
nighttiem awakenings/month, with a normal FEV1 and no limitations on activity. For this
form of asthma, only a PRN albuterol inhaler is required. Daily controller corticosteroids are
reserved for persistent asthma.
24. Rubella is characterized by maculopapular rash, posterior cervical and posterior auricular
lymphadenopathies, and polyarthralgia.
25. Obstructive uropathy causes flank pain, low volume voids with or without occasional high
volume voids, and renal dysfunction. Renal artery stenosis can cause hypertension with or
without renal dysfunction. It does not cause flank pain or urinary symptoms, however.
26. In the tx of cocaine-related cardiac ischemia, the first-line drugs are benzodiazepines, nitrates,
and aspirin.
27. The anemia of chronic kidney disease is due to erythropoietin deficiency. One must be careful
to ensure adequate iron stores prior to erythropoietin replacement because the erythropoietin-
induced surge in RBC production can precipitate an iron-deficient state.
28. Autosomal dominant polycystic kidney disease is a potential cause of hypertension.
Intracranial berry aneurysm is a common complication (5-10% cases). The other major extra-
renal complications are: 1) hepatic cysts-most common; 2) valvular heart disease-most often
mitral valve prolapse and aortic regurgitation; 3) colonic diverticula; 4) abdominal wall and
inguinal hernia. Although such aneurysms are common and dangerous when coupled with
hypertension, routine screening for intracranial aneurysms is not recommended.
29. Primary pulmonary hypertension (>25 at rest or >30 mmHg with exercise) can be seen in

105
middle-aged pts, and it presents with exertional breathlessness. Lungs will be clear to
auscultation. CXR would show enlargement of the pulmonary arteries with rapid tapering of
the distal vessels (pruning) and enlargement of the right ventricle.
30. Early morning hyperglycemia is sometimes, but not always, due to inadequate insulin dosage.
Another cause is nocturnal hypoglycemia, which leads to morning hyperglycemia (Somogyi
effect). Measurement of the 3AM blood glucose level is very helpful for determing the
particular etiology of early morning hyperglycemia.
In this case, the pt’s 3:00am blood glucose level shows that her morning 7:00an hyperglycemia is
due to the Somogyi effect, most probably due to a high NPH dosage at night. The effect of NPH
peaks between 4-10 hours after injection; with this in mind, it becomes evident that the time of
peak effect of this pt’s suppertime NPH coincides with (and most probably causes) the occurrence
of her hypoglycemia at 3am. This low blood glucose level results in an increase in the levels of
counterregulatory hormones (ie epinephrine, norepinephrine, glucagon) in the circulation, which
consequently increase the rate of glycogenolysis and gluconeogenesis, thereby eventually causing
the evelation of the morning blood glucose level.
Dawn phonomenon is responsible for morning hyperglycemia due to decrease insulin sensitivity
between 3:00-8:00am. Pts typically have elevated blood glucose levels (hyperglycemia) at 3am
and 7am. The underlying pathology involves spikes of growth hormone release, which occurs
shortly after falling sleep.
Waning of circulating insulin levels can also result in morning hyperglycemia; however, the
recorded blood glucose level at 3am is usually normal. Dawn phenomenon and waning of insulin
action often coexist.
31. Cholecystectomy is indicated in all pts with symptomatic gallstones who are medically stable
enough to undergo surgery.
32. Excessive alcohol intake can lead to aspiration pneumonia. Risk factors for aspiratory
pneumonia are: 1) altered consciousness-seizure, alcoholism, drug overdose, and CVA; 2)
dysphagia-esophageal reflux, diverticula, obstruction; 3) Neurologic disorder-advanced
dementia, Parkinsonism, myasthenia; 4) Sedation to precedures, such as bronchoscopy,
intubation, endoscopy.
33. Td should be given at least every 10 years. Recently, CDC have been recommending a one-
time Tdap booster for adults aged 19-64 years.
Influenza vaccination is recommended for adults who are within the ages of 19-49 years, and have
close contact with children aged 0-59 months. Either intranasal or injected influenza vaccines may
be used. All adults over 50 years of age and those with certain comorbid conditions should receive
the intramuscular influenza vaccine.
The HPV vaccine decreases the incidence of genital warts, abnormal Pap smears, and cervical
cancer. It may be given to females aged 9-26 years, and is most effective if given before coitarche.
34. Schistocytes (helmet cells) are fragmented erythrocytes. They occur in microangiopathic
hemolytic anemias (DIC, HUS, TTP) and due to RBC destruction by prosthetic cardiac
valves. Hemolytic anemias are characterized by a decreased serum haptoglobin level as well
as in an increased LDH (erythrocyte destruction) and bilirubin. Haptoglobin is a serum protein
that binds free hemoglobin and promotes its excretion by the reticuloendothelial system. In
intravascular hemolysis, the amount of free hemoglobin in the serum exceeds the binding
capacity of haptoglobin thereby decreasing the level of haptoglobin.

106
35. Febrile reaction is a common transfusion reaction that is caused by antibodies in the pt’s
plasma reacting with the donor’s leukocytes. It is characterized by fever and chills that usually
respond to NSAIDs and acetaminophen. Leukocyte depletion techniques, like cell washing,
decrease the probability of febrile, nonhemolytic transfunction reaction. Calcium gluconate
infusion is employed in rare cases of severe hypocalcaemia following massive blood
transfusion. Warming the blood is recommended only during rapid massive transfusion to
prevent hypothemia.
36. Saline-responsive metabolic alkalosis has a urine chloride concentration less than 20mEq/L
and is typically due to conditions that cause hypovolemia or GI proton loss. It is treated with
infusion of isotonic saline.
37. Cardiac tamponade is one of the most deadly consequences of aortic dissection. It is
characteried by hypotension, tachycardia, and heart failure.
38. Age-related macular degeneration is usually seen in pts above 50 years of age. It presents with
progressive and bilateral loss of central vision. Navigational vision is preserved. Open angle
glaucoma usually presents as a gradual loss of peripheral vision (over a period of years) and
consequent tunnel vision. Central vision is spared.
39. Influenza presents as the acute onset of fever, chills, cough, malaise, and myalgias. Influenza
pheumonia CXR-intersitial or alveolar pattern; physical exam--wheezes, crackles, and course
breath sounds. Antiviral therapy must be started withi 48 hours to signigicantly decrease the
disease duration and severity. The newest therapies are the neuraminidase inhibitors,
oseltamivir and zanamivir. Rimantadine and amantadine are other options, but only effective
against influenza A.
40. HIV pts with dysphagia should receive one to two weeks of empiric oral fluconazole therapy,
since candidal esophagitis is the most likely dx. If symptoms persist, endoscopy with biopsy
should be performed. The ulcers of HSV esophagitis are usually multiple, well circumscribed,
and have a “volcano-like” (small and deep) appearance (aciclovir), where the ulcers seen in
CMV infection tend to be large, shallow, and superficial (tx is ganciclovir).
41. SLE is an autoimmune disease that is most commonly diagnosed in young females. Systemic
manifestations include fever, fatigue, weight loss, proteinuria, rash, arthralgias and anemia.
Greater than 90% of pts have joint involvement most commonly affecting the hands. Joint
pain is common, but the arthritis induced by SLE is considered non-deforming. Psteophytes
and cartilage degradation—OA. Subluxation of the cervical vertebrae and tendon damage—
RA. Distal phalangeal resorption is prominent in the arthritis mutilans variant of psoriatic
arthritis leading to classic “pencil-in-cup” deformities.
42. Agranulocytosis is a serious side effect of antithyroid drug (ATD) therapy. It occurs in
approximately 0.3% of pts taking ATDs.
43. Chronic alcoholics typically present with multiple electrolyte abnormalities such as
hypokalemia, hypomagnesemia and hypophosphatemia. Hypomagnesemia, causes refractory
hypokalemia; therefore it is important to correct the magnesium along with the potassium
levels to be albe to correct electrolyte abnormalities of such pts.
44. Pts with a T-score of less than -1.5 plus risk factors for osteoporosis or a T-score of less than
-2.0 should receive preventative medications. The preferred agents are oral bisphosphonates
or raloxifene. Estrogen is also an effective medication in preventing osteoporosis; however, its
vascular and embolic risks make it a much less desirable option at this time. Calcium and vit

107
D should be given to all pts who osteopenia or osteoporosis but they are not nearly as
effective in preventing or treating osteoporosis as bisphosphonates or raloxifene.

1/5/10
1. Autoimmune hemolytic disease and hereditary spherocytosis are both extravascular hemolytic
anemias; however, the former is acquired and the latter has an autosomal dominant
transmission. A negative family history and positive Coomb’s test are thus suggestive of
autoimmune hemolytic disease, whereas a positive family history is more suggestive of
hereditary spheocytosis. The peripheral blood smear in both conditions may show
spherocytosis.
2. Syphilis consists of 3 stages. Primary syphilis is characterized by a painless ulcer on the
genitalia, perianal area, rectum, lip, tongue, or elsewhere, 2-6 weeks after exposure. Reginal
LN will be enlarged but non-tender. Serology for syphilis (RPR, VDRL) may or may not
positive at this time. Secondary syphilis is marked by a generalized maculopapular skin rash
that spread to the palms, soles, and mucous membranes, and is accompanied by generalized
lymphadenopathy. Pts may also suffer from fever, arthritis, iritis, hepatitis, meningitis, and
ostitis. Serological tests are always positive by this point. Tertiary syphilis is characterized by
tumors (gummas) that infiltrate that bones, liver, and skin. Aortitis, aneurysms, and aortic
regurgitiation may be seen. CNS disorders (neurosyphilis) may occur.
3. Tetracyclines are an important cuase of phototoxic drug eruptions. These eruptions manifest
as exaggerated sunburn reactions with erythema, edema and vesicles over sun-exposed areas.
Photoallergic contact dermatitis can occur, and certain sunscreens are ironically well-known
causes of this eruption. The skin manifestations are typically more eczematous in appearance, and
prior sensitization is required.
Stevens-Johnson syndrome is an epidermal necrolysis syndrome typically caused by drugs (ie
TMP-SMX, NSAIDs, anticonvulsants). The mucosal surfaces are almost always involved.
The common side effects of benzoyl peroxide are irritation, contact dermatitis, dryness, erythema,
peeling and stinging.
4. Digoxin is medication with a narrow therapeutic window. Several drugs, including verapamil,
increase digoxin concentrations and predispose to toxicity. The most common side effects of
digoxin toxicity are GI (anorexia, nausea, vomiting).
5. Be overly suspicious for an intraocular foreign body in pts with high-velocity injuries
(drilling, grinding, etc). If the initial pen light examination does not reveal any conjunctival
and corneal abrasions or foreign bodies, proceed with fluorescein examination.
6. Malignancy is the most common cause of SVC syndrome (Superior vena cave syndrome: a
condition where obstruction of the SVC impedes venous return from the head, neck and arms
to the heart. Signs and symptoms include dyspnea, venous congestion, and swelling of the
head, neck and arms.). Lung cancer (particularly small cell lung cancer) and NHL are often
implicated. Other possible causes include fibrosing mediastinitis (secondary to histoplasmosis
or Tb infection) or thrombosis secondary to indwelling central venous devices. When the
history and physical examination are suggestive, CXR is warranted.
7. Autosomal dominant polycystic kidney disease is a heritable form of renal disease
characterized by multiple renal cysts and intermittent flank pain, hematuria, urinary tract
infections, and nephrolithiasis. (Palpable mass doesn’t have to be at both sides of flank,

108
maybe only one side). RCC alone would not give renal failure, but ADPKD does.
8. An enlarged left atrium in mitral stenosis can cause a persistent cough and elevation of the left
main stem bronchus.
9. Amoxicillin is the preferred tx for pregnant or lactating pts with early localized lyme disease.
Doxycycline is the first-line tx for early localized lyme disease in pts without C/I to its use.
Doxycycline has the advantage of simultaneously treating possible coexistent human
ehrlichiosis, since this condition has the same vector as lyme disease. Doxycycline is avoided
in children and pregant/lactating women because it can cause skeletal and dental peoblems in
exposed children and fetuses. IV 3rd generation cephalosporins (ceftriaxone and cefotaxime)
are indicated for early disseminated and late lyme disease.
10. The typical CT/MRI findings in high-grade astrocytoma are heterogenous and serpiginous
contrast enhancement. Recognize the classic butterfly appearance of glioblastoma multiforme
(GBM).
11. Several values related to pulmonary mechanics can be calculated in mechanically ventilated
pts. Lung compliance is calculated by performing the end-inspiratory hold maneuver. Airway
resistance is calculated by examinating the peak airway pressures. Positive end-expiratory
pressure (PEEP) is calculated with the end-expiratroy hold maneuver.
12. Features of atrial fibrillation on EKG include an irregular irregular R-R interval with absent P
waves and narrow QRS complexes. In unstable pts immediate DC cardioversion should be
performed. It is a common complication of CABG, occurring in up to 40% of pts. In CABG
with aortic valve replacement, the incidence jumps to 50%.
13. Alcohol withdrawl should be suspected in any hospitalized pt with a history of significant
alcohol abuse. Earl symptoms include anxiety, insomnia, tremors and diaphoresis (complains
of bugs crawling on skin). Delirium tremens, which generally occurs 48-96 hours after the last
drink, involves hupertension, hyperthermia, agitation, hallucinations, and possible death. All
pts suspected of alcohol withdrawal should be placed in a protetive environment, and treated
with benzodiazepines (Chlordiazepoxide-librium).
Haloperidol is often used ot control agitation and aggression in psychiatric emergencies, including
those caused by certain drugs of abuse. However, it is not used in pts undergoing alcohol
withdrawal because it does not exhibit cross-tolerance with alcohol (as benzodiazepines do), and it
also lowers the seizure threshold.
14. Erythema Nodosum (EN) is a condition of painful, subcutaneous, pretibial nodules. It can be a
symptom of more serious disease processes including sarcoidosis, TB, histoplasmosis, recent
streptococcal infection, and inflammatory bowel disease. The association of EN with
sarcoidosis is particularly strong in young, African-American women. Cough, arthritis, uveitis
and hilar adenopathy on CXR are also associated with sarcoidosis.
15. Molluscum contagiosum is characterized by multiple dome-shaped lesions with central
umbilication. It is caused by poxvirus, and occurs as an opportunistic skin infection in HIV-
infected pts though it also commonly affects otherwise healthy individuals as well.
16. Carbamazepine is used for the tx of atypical bipolar depression and trigeminal neuralgia. It is
the most effective tx for trigeminal neuralgia, being effective in approximately 80% of such
individuals. Aplastic anemia can occur with prolonged use; therefore, routine CBC is included
in the follow-up management of such pts. When medication fails to control the pain, surgical
gangliolysis or suboccipital craniectomy for decompression of the trigeminal nerve are

109
options.
17. “Cerebral salt-wasting syndrome” may occur in pts with SAH. The pathology involves: 1) an
inappropriate secretion of vasopressin, which causes water retension, and 2) an increased
secretion of atrial/brain natriuretic peptide, which causes cerebral salt-wasting. These changes
result in hyponatremia, which usually resolves within 1-2 weeks. SIADH is also commonly
seen in pts with intracranial hemorrhage. SIADH also results in hyponatremia, for which
water restriction is the initial treatment of choice.
18. Metabolic alkalosis can be classified into two broad categories, chloride-sensitive and
chloride-resistant based on urinary chloride levels and ECF volume status. Chloride sensitive
metabolic alkalosis is associated with low urinary chloride excretion and volume contraction.
19. The presence of diffuse musculoskeletal pain in the absence of joint swelling, muscle
weakness or laboratory abnormalities is suggestive of fibromyalgia. Fibromyalgia is the most
common cause of generalized musculoskeletal pain in women ages 20-55. The pathogenesis is
made based on the presence of widespread musculoskeletal pain and exerssive tenderness on
palpation of at least 11 of 18 predefined soft tissue locations during physical examination.
These sites include the upper quadrants of the buttocks and medial aspect of the knees, as well
as the sternocleidomastoid and trapezius muscles. The absence of joint swelling or muscle
weakness is also characteristic of fibromyalgia, as is the worsening of symptoms with
exercise.
20. A pt with known CHD or a CHD risk equivalent should be treated with lifestyle modification
and statins to reach a goal LDL of less than 100mg/L.
21. Newer antipneumococcal quinolones, like levofloxacin or gatifloxacin, are the drugs of choice
for in-pt tx of community-acquired pneumonia (caused by a host of bacteria and viruses, tx is
empirical. Most common pneucoccus, others-hemophilus, moraxella). For out-pt therapy,
either azithromycin or doxycycline can be used.
22. Aortic dissection is the most dangerous complication of Marfan’s syndrome. It typically
presents as tearing chest pain that radiates to the back and neck, and it must be identified
immediately to decrease the risk of death. One common exam finding is an early diastolic
murmur as the dissection causes aortic regurgitation.
Fixed splitting of the second heart sound (S2) is associated with atrial septal defects.
Kussmaul’s sign is an increase in jugular venous pressure that occurs in response to deep
inspiration. It occurs in situations that cause right ventricular failure like constrictive pericarditis
and right ventricular infarction.
Pulsus parvus et tardus refers to a carotid pulse that is slow (parvus) and late (tardus). It is
classically observed in aortic stenosis, not aortic regurgitation or aortic dissection.
23. Intracranial hypertension is diagnosed when the intracranial pressure equals or exceeds
20mmHg. It may be due to a number of different causes and typically presents with
headaches, vision changes, nausea and vomiting, changes in awareness, and /or focal
neurologic deficits.
24. Asymptomatic pts with carotid artery stenoses of 60-99% are considered to have a proven
indication for carotid endarterectomy. Complete occlusion (100% stenosis) of the carotid
artery is a C/I to surgery.
25. Chronic HCV classically presents with waxing and waning transaminase levels but few
symptoms. Pts may complain of arthralgia or myalgias. Extrahepatic sequelae include

110
cryoglobulinemia, porphyria cutanea tarda, and glomerulonephritis.
26. Subacromial bursitis is the result of repetitive overhead motions. Pts complain of pain with
active range of motion of the shoulder, and passive internal rotation and forward flexion at the
shoulder also elicits tenderness.
27. The most likely explanation for the isolated urinalysis finding of mild to moderate proteinuria
in a middle-aged adult with renal insufficiency and a history of longstanding diabetes is
diabetic glomerulosclerosis secondary to diabetic microangiopathy.
28. ‘Smudge cell’—the pathologist reports the presence of “leukocytes that have undergone
breakdown during preparation of a stained smear or tissue section, because of their greater
fragility”—CLL. The staging system is directly related to the prognosis.
Stage Clinical feature Prognosis
0 Lymphocytosis only Good
I Lymphocytosis + adenopathy Fair
II Splenomegaly Fair
III Anemia Intermediate
IV Thrombocytopenia Poor

29. Hemi-neglect syndrome is characterized by ignoring the left side of a space, and involes the
right (non-dominant) parietal lobe.
30. A generalized tonic clonic seizure is initially characterized by an aura, followed by stiffness
and loss of consciousness. The next phase involves tonic clonic jerky movements of the whole
body. Biting the tongue during the seizure and subsequent confusion with incontinence are
typical.
31. Back pain is one of the most common compaints in adult medicine. It is important to identify
signs that suggest a serious etiology, such as pain not relieved by rest, night pain, constant or
dull pain, fevers, lack of exacerbation with movement or palpation, and presence of
neurologic changes. Cancer pain is often no tender to palpation.
32. Normal saline is the initial fluid of choice in hypotensive, dehydration pts with DI. Hypotonic
fluids (0.45% saline or D5%W) can be started when the iv volume improves. Water
deprivation should not be performed when a pt is hypovolemic.
33. The most important steps in the mx of lactic acidosis from septic shock are IV normal saline
with or without vasopressor therapy to maintain the intravascular pressure and antibiotics to
correct the underlying infection.
34. Nitrates cause venodilation, which improves cardiac chest pain by reducing cardiac preload
and thus decreasing myocardial oxygen demand.
35. Ostiomalacia is characterized by defective mineralization of the bone. Rickets is characterzied
by defective mineralization of both bone and growth plate cartilage. Disordered skeletla
remodeling in focal areas of the bone is the underlyng pathophysiology of Paget’s disease of
the bone. Osteoporosis is characterized by low bone mass, but the bone that is present is
normally mineralized per unit volume.
36. Erythema chronicum migrans in a pt with a tick bite history warrants immediate tx with
doxycycline. Serological testign is not necessary to confirm a Lyme disease dx if the pt
presents with classic EM.
37. Ethosuximide is used most exclusively for children absence seizure.
38. Loop diuretics cause hypokalemia and hypomagnesemia. These eletrolyte abnormalities can

111
cause ventricular tachycardia, and also potentiate the side effects of digoxin.
39. Factors that improve the prognosis in a pt with chronic renal failure are protein restriction and
the use of ACE inhibitors. ACE inhibitors are likely to worsen renal failure when serum
creatinine levels are greater than 3-3.5 mg/dl.
40. Charcot’s joint, is a precess of developing neurogenic arthropathy starts with decreased pain,
proprioception, and temperature perception, which can occur due to diabetes, peripheral nerve
damage, syringomyelia, spinal cord injury, B12 deficiency, or tabes dorsalis. As normal
neurologic input is lost, pts unknowingly traumatize their weight bearing joints. This causes
secondary degeneration joint disease, joint deformation, and functional limitations. X-rays
will reveal loss of cartilage, osteophyte development, and loose bodies. Associated pain is
typically mild. Mx of a Charcot’s joint involves treating the underlying disease and providing
mechanical devices (eg. special shoes) to assist in weight bearing and to decrease further
trauma.
41. Warfar-induced skin necrosis presents with pain, followed by bullae formation and skin
necrosis. The breasts, buttocks, thighs, and abdomen are commonly involved.
42. The resolution of muscular weakness with rest is a hallmark feature of myasthenia gravis.
43. Pellagra, a niacin deficiency syndrome is characterized by a triad of 3 Ds of Diarrhea,
Dermatitis, and Dementia and if untreated eventually leading to Death (4th D). Causes: dietary
deficiency, alcoholics, carcinoid syndrome and Harnup’s disease. The skin rash is present in
sun-exposed areas (similar to SLE). Acute intermittent porphyria is a subtype of the
porphyrias, it has an acute presentation, occurs more commonly in females and is episodic
(episodes triggered by certain drugs). The episodes are characterized by abdominal pain,
nausea, vomiting, diarrhea, and sweating. It also causes agitiation, anxiety, paraesthesia, and
confusion. Unlike the other porphyrias, there is an absence of photosensitivity in this subtype.
44. CREST syndrome refers to a constellation of findings including Calcinosis cutis, Raynaud
phenomenon, Esophageal dysmotility, Sclerodactyly and Telangiectasias.

1/6/09
1. Testing for both HbsAg and anti-HBc offers the best screening for acute HBV infection, as it
won’t miss the window perios when HbsAg has disappeared but anti-HBS has not yet appeard
in the serum.
2. In pts with suspected stroke, non-contrast head CT is the initial diagnosis test of chioce,
critical for guiding further mx.
3. Pts with rotator cuff tendonitis complain of shoulder pain aggravated by activities such as
reaching or lifting arm over the head. The condition results from repetitive activity above
shoulder height and is most common in middle-aged and older individuals. For this reason,
painters are particularly prone to developing rotator cuff tendonitis. Impingement of preent in
all pts with rotator cuff tendonitis. It is comfirmed on physical exam by performing the Neer
test (passive motion of the arm above the head). Pain and guarding during the Neer test
confirmed impingement. To distinguish rotator cuff tendinitis from other forms of rotator cuff
pathology, lidocaine is injected into the joint. Improved range of motion and pain relief after
the injection corroborates the diagnosis of rotator cuff tendonitis. MRI is used for definitive
dx.
Rotator cuff tear may result from trauma (eg. falling on an outstretched arm) or as the end result of

112
chronic impingement and tendonitis. Similar to pts with rotator cuff tendonitis, pts often complain
of pain upon reaching and lifting the arm over the head. However, weakness of the shoulder is
more common in rotator cuff tears, and symptoms do not improve with lidocaine injection.
Adhesive capsulitis, or frozen shoulder, is an idiopathic condition characterized by pain and
contracture. This condition presents with an inability to lift the arm above the head. Even after
injection of lidocaine, the arm still cannot be lifted above the head due to fibrosis of the shoulder
capsule.
Vascular compression may occur in thoracic outlet syndrome. This most commonly presents with
a combination of numbness, weakness and swelling due to compression of the subclavian vessels
and lower trunk of the branchial plexus. A weakened radial pulse and reproduction of symptoms
with specific arm movements supports the dx.
4. HAV vaccine or serum immune globin should be given to all non-immunized travelers to
endemic countries. If travel will occur in less the 4 weeks, serum immune globulin should be
given. If travel will occur in greater the 4 weeks, HAV vaccine should be given instead as it
offers long-term protection.
5. Hepato-jugular reflex is a useful tool that can be used to differentiate between heart and liver
disease-related causes of lower extremity edema. Positive reflex indicated that the venous
pressure is elevated and suggests that the heart disease-related edema is present. Negative is in
pt with the liver disease-related edema.
6. Caustic poisoning does not cause alteration in consciousness. It presents with dysphagia,
severe pain, heavy salivation and mouth burns. The damage is the result of necrosis of the
tissue that lines the GI tract. In severe cases, perforation of the stomach or esophagus can
occur, causing peritonis or mediastinitis.
7. For frostbite 冻 伤 injuries, the best tx is rapid re-warming with warm water. Whenever
frostbite or cold injuries are diagnosed, no attempts should be made to debride any tissue
initially. Rapid re-warming with dry heat (like a fan) is not effective for frostbite.
8. Acute pancreatitis can cause an acute abdomen. It should be managed conservatively with
anagesics, intravenous fliuds, and nothing by mouth.
9. In bronchiectasis, bronchial dilation causes impaired clearance of secretions and resultant
airway obstruction. Acquired bronchiectasis is ofter secondary to prior Tb or recurrent
pneumonia, while inherited bronchiectasis is most often due to cystic fibrosis. Symptoms
include cough, mucopurulent sputum, and atelectasis. HRCT is used for definitive diagnosis.
Bronchoscopy and alveolar lavage (BAL) if often used to establish diagnosis like
pneumocystis pneumonia and certain lung cancers. Rigid bronchoscopy may occasionally be
used in bronchiectasis, but non-invasive HRCT is preferable.
10. Tumor burden is the single most important prognostic consideration in the tx of pts with
breast cancer. It is based on TNM staging. ER+ and PR+ are good prognostic features.
Overexpression of the Her-2/neu is related to a worse prognosis. The histological grade also
tends to reflect the outcome. Poorly differentiated tumors have the worst prognosis.
11. An outlier is defined as an extreme and unusual observed in a datasheet. The mean is very
sensitive to outliers and easily shifts toward them. The median and mode are more resistant to
outliers.
12. The dx of cholelithiasis is best confirmed with abdominal ultrasound. Once diagnosed, the tx
of choice for symptomatic cholelithiasis is laparoscopic cholecystectomy. Those pts who

113
prefer not to undergo cholecystectomy may be given a bile salt such as ursodeoxycholic acid,
which decreases the cholesterol content of the bile by reducing the hepatic secretion and
intestinal reabsorption of cheolesterol. However, this medication is very costly and associated
with a high risk of relapse when therapy is halted.
13. Breaking bad news is a very sensitive issue to both pts and physicians. Step-wise approach: 1.
Make sure the pt is in a quiet, private, and comfortable enviroment. 2. Ask the pt how much
he knows, or what he thinks he might have. This will give you an idea of what his
expectations are. “What do you think of your symptoms?” 3. Ask the pt how much he wants
to know. “How much would you like to know about your condition?” 4. Give him a warning
shot. “Unfortunately, the situation is more serious than what I earlier thought.” 5. Break the
news if he wants you to. “The results show that you have advanced lung cancer.” 6. Give his
prognosis, but always keep him aware of all the options available to make his life as
comfortable as possible. 7. Try to explain everything as clearly and simple as possible. *
High-yield!!
14. Impetigo initially prestns as an erythematous macule, which repidly evovlves into vesicles
and pustules that rupture and leave honey-colored, crusted exudates.
15. Postexposure prophylaxis for HBV infection includes administration of the immune globulin
(HBIG) and 3 shots of HBV vaccine given at set intervals.
16. Amitriptyline and cyclobenzaprine have been shown to be effective in the tx of fibromyalgia
as they are able to increase the amount of restorative phase 4 sleep a pt gets. This is helpful
b/c one possible etiology of fibromyalgia is fatigue due to lack of phase 4 sleep.
17. Cough can be a presenting symptom of GERD. A 24-hour pH recording is the most specific
test available for acidic gastroesophageal reflux. It is usually employed to diagnose the cause
of chest pain or supra esophageal complications of GERD in pts with negative esiphagoscopy.
18. Parvovirus causes arthritis of the MCP, PIP, wrist and ankle joints. Anti-B19 IgM is the
dignostic study of choice when Parvovirus infection is clinially suspected.
19. Mitral regurgitation (mitral valve prolapse—most common in US) is the most common
valvular abnormality observed in pts with infective endocarditis not related to IV drug abuse
(tricuspid regurgitation).
20. Temporomandibular joint (TMJ) dysfunction can result in referred pain the ear that is
worsened with chewing. Pts typically report a history of nocturnal grinding.
Ramsay Hunter syndrome is a form of herpes zoster infection that causes Bell’s palsy. In this
condition, vesicles are typically seen on the outer ear.
21. Lumbar spinal stensis is most commonly casued by degenerative disk disease and presents
with low back and leg pain. The leg pain of spinal stenosis (“neurogenic claudication”) can be
confused with that of PVD (peripheral vascular disease) -induced claudication. In both cases,
leg pain worsens with walking; however, the leg pain of neurogenic claudication is position-
dependent and persists while standing still. PVD-induced claudication is exertion dependent
and resolve with standing still. On exam, normal ankle-brachial index and arterial pulses are
seen in spinal stenosis.
22. The most important factor when diagnosis carbon monoxide poisoning is the hsitory, b/c
physical symptoms can be vague. Suspect CO poisoning in cases of smoke inhalation and
when multiple people from the same confined quarters present with headache, nausea, and
abdominal discomfort. Pinkish-red skin hue is noted on examination and diagnosis is

114
confirmed by carboxyhemoglobin level.
23. A reliable test gives similar results on repeat measurements. Reliability is maximal when
random error is minimal. Validity or accuracy is defined as the test’s ability to measure what it
is supposed to measure. In order to determine the validity of a test, the results are compared to
those obtained from the gold standard test.
24. Unexplained hemolytix anemia and thrombocytopenia in a pt with renal failure and
neurologic symptoms should raise strong suspicions for TTP-HUS. HIV increases the risk for
TTP. A key clue to the dx of TTP-HUS in this pt is his reticulocyosis. A peripheral smear
with>1% schistocytes would be virtually diagnostic of microangiopathic hemolytic anemia
(MAHA), a component of TTP-HUS. However, MAHA can also occur in DIC and malignant
hypertension. Coagulation test would help to rule out DIC. (DIC pts tend to bleed and have
abnormal coagulation studies. TTP-HUS pts do not bleed despite their low platelet count).
Malignant hypertension should be considered if there is a history of hypertension and
evidence of hypertensive retinopathy on fundoscopic exam.
25. IV adenosine is the drug of choice for paroxysmal SVT (HR>140/min, regular, loss of ‘P’
waves and narrow QRS complex). Know ECG. If the pt is hemodynamically unstable,
electrical cardioversion should be performed immediately. However, if the pt is stable, vagal
manuvers should be attempt initially. If these fail to convert him to normal sunus rhythm, IV
adenosine push is the drugh of choice.
26. Open angle glaucoma is more common in African-Americans. It is generally asymptomatic in
the initial stages, followed by a gradual loss of peripheral vision over a period of years, and
eventual tunnel vision. On exam, the intraocular pressure is high. There may be cupping of the
optic disc with loss of peripheral vision. Annual exam in high-risk populations is of great
benefit in prevention of the condition. Beta-blockers such as Timolol eye drops are effective
in the initial mx of the pt. Laser trabeculoplasty is used as an ajunctive measure. If there is a
continuous increase in intraocular pressure, surgical traceculectomy is done.
27. Suspect syringomyelia in a pt with areflexia weakness in the upper extremities and dissociated
anesthesia in a “cap” distribution. The presence of a cord cavity is the most characteristic
feature. Caudal displacement of the fourth ventricle or cerebellar tonsils, and focal cord
enlargement may occur. * Just know how to diagnose.
28. Normal age-related cognitive changes include tiredness, occational forgetfulness, occasional
word finding difficulty, and trouble falling sleep. Dementia cannot be diagnosed unless there
are functional impairments.
29. Hereditary hemochromatosis can cause cirrhosis, pancreatic fibrosis (diabetes) and increased
skin pigmentation due to melanin and hemosiderin deposion (bronze diabetes), as well as
abnormalities of cardiac conduction (conduction block).
30. Ankylosing spondylitis presents with low back pain and stiffness typically in young HLA-B27
positive males. These pts may experience many systemic effects including fatigue, uveitis and
pulmonary disease. The pulmonary symptoms result from of the costovertebral jounts
resulting in chest wall motion restriction and a restrictive pattern on pulmonary function
testing.
31. Always suspext malignant melanoma in a pt who presents with a changing mole. The
ABCDEs of melanoma (Asymmetry, Border irregularities, Color variegations, Diameter
greater than 6mm and Enlargement) help in screening and early detection.

115
Melanocytic nevi are common, benign lesions found in the integument 包皮 of most individuals.
They typically obey the ABCDE rules.
32. Bartonella henselae and Bartonell quintana cause bacillary angiomatosis in
immunocompromised individuals. Pts present with cutaneous and visceral angioma-like blood
vessel growths. Antibiotic tx causes lesion regression.
33. Bacteria overgrowth is a malabsorption syndrome which can be associated with a history of
abdominal surgery. Symptoms may be nonspecific and include abdominal pain, watery
diarrhea, dyspepsia, and weight loss. In severe and advanced cases, pts may present with
tetany (hypocalcemia sue to vit D deficiency), night blindness (Vit A deficiency), neutopathy
(B12 deficiency), dermatitis, arthritis, and hepatic injury. Physical exam may reveal
abdominal distention with identifiable succussion 振 荡 splash (due to palpable soft, fluid-
filled loops of bowel). Macrocytic anmia (vitB12 malabsorption) can be seen on alb exam.
34. Membraneoproliferative glomerulonephritis, type 2, is a unique glomerulopathy that is caused
by persistent activation of the alternative complement pathway. Dense intramembranous
deposits that stain for C3 is a characteristic microscopic finding.
35. Barium swallow followed by endoscopy is usually done when a pt is suspected of having an
esophageal cancer (heartburn, significant weight loss, fatigue, smoking history).
36. Spontaneous bacterial peritonitis (SBP) should be suspected immediately in cirrhotic pts with
ascitis who develop fever or abdominal pain. The dx of SBP is confirmed by a positive ascitis
fluid bacterial culture and an elevated ascitic fluid absolute neutrophil (PML) count of more
than 250 cells/mm3. Emipirical tx: cefotaxime (or ceftriaxone), a third-generation
cephalosporin, ampicillin may be added to cover enterococcus. Antibiotix therapy is usually
successful, but because SBP ofhter recurs, liver transplantation is the most effective tx.
37. Drug-induced pancreatitis is mild and usually resolves with supportive care. CT scan is
diagnostic for pancreatitis. Scenarios for drug-induced pancreatitis: pt on 1) diuretics—
furosemide, thiazides. 2) inflammtory bowel disease—sulphasalazine, 5-ASA. 3)
immunosuppressive agents—azathioprine, L-asparaginase. 4) history of seizure or bipolar
disorder—valproic acid. 5) AIDs—didanosine, perntamidine. 6) antibiotics—metronidazole,
tetracycline.
38. Spontaneous subconjunctival hemorrhage is a benign finding, and does not require any tx. It
may be due to simple trauma from rubbing the eyes vigorously, violent coughing spells,
hypertensive episodes or coagulopathy.
39. AV fistula (history of trauma) causes high output cardiac failure by shunting the blood form
the arterial to venous side, thereby increaseing cardiac preload. The pt develops heart failure
despite maintaining a normal or high cardiac output b/c the circulation is unable to meet the
oxygen demand of the peripheral tissues. Clinical signs include widened pulse pressue, strong
peripheral arterial pulsation (brisk carotid upstroke), and tachycardia. The extremities are
typically flushed. The left ventricle hypertrophies, and the point of maximal impulse is
displaced to the left. EKG shows left ventricular hypertrophy.
40. Mitral stenosis in left atrial dilation and a risk of AF and cardiac emboli. The pressue is also
transmitted to the pulmonary vasculature, which can result in dyspnea, cough and hemoptysis.
41. Waldenstrom’s macroglobulinemia is characterized by hyperviscosity of the blood owing to
the excess production of IgM. 2 important diagnostic clues for this disorder are: 1) an IgM
spike on electrophoresis, and 2) hyperviscosity. Signs and symptoms: 1) increased size of the

116
spleen, liver and some lymph nodes. 2) Tiredness, usually due to anemia (too few RBC). 3)
Tendency to bleed and bruise easily. 4) Night sweats. 5) Headache and dizziness. 6) Various
visual problems. 7) Pain and numbness in the extremities due to a predominantly
demyelinating sensorimotor neurophathy.
Multiple myeloma—present with similar symptoms, but the involved Igs are usually either IgG or
IgA. Furthermore, such pts rarely have hyperviscosity syndromes such as retinal vein
engorgement.
Monoclonal gammopathy of undetermined dignificance (MGUS) are mostly asymptomatic or
have mild symptoms.
Heavy chain disease present like those with abdominal lymphoma; however, IgA is the main Ig
involved.
42. Presbyopia is common age-related decrease in lens elasticity that leads to difficulty with near
vision. A history of a middle-aged individual who has to hold books at an arms length to read
is classic.
43. Pseudotumor cerebri is a condition that presents with headache, blurry vision, papilledema,
and/or cranial nerve palsies. CSF examination reveals increased opening pressure. Young,
obese women are msot commonly affected. Vit A and isotretinoin can also predispose.
44. All pts with atherothrombotic TIA should receive an antiplatelet agent if there is no C/I to its
use. Aspirin is the initial agent of choice for this purpose.

1/7/10
1. Recommended therapy ofr an acute bacterial exacerbation of COPD includes supplement
oxygen, inhaled bronchodilators (beta-2 agonists and anticholinergics), broad-spectrum
antibiotics, a two-week corticosteroid taper and smoking cessation.
2. Latex allergy can manifest as an anaphylactic reaction during exposure to latex-containing
products such as surgical and condoms.
3. Autonomic neuropathy may lead to a denervated bladder, resulting in overflow incontinence
(due to detrusor weakness). This condition is characterzied by a high post-void residual
volume. Aside from strict glycemic control, tx opitons include intermittent catheterization and
cholingergic medications such as bethanechol. Heavy alcohol intake may contribute to the
effects of autonomic neuropathy; therefore, advising this pt to stop drinking alcohol beverages
may be helpful.
The pt has a normal sized prostate; therefore, overflow incontinence due to bladder outlet
obstruction is unlikely. Overflow incontinence due to medication is usually seen with the use of
anticholinergics, antipsychotics, tricyclic antidepressants, and sedative-hypnotics. Pelvic floor
weakness is a common cause of stress incontinence in women, although it may occur in older men
after a radical or transurethral prostatectomy.
4. Pancytopenia is common in pts with SLE. It occurs due to the formation of autoantibodies
against blood cells, a form of type II hypersensitivity.
5. Sudden onset right upper quadrant abdominal pain, fever, vomiting, and leukocytosis are
highly suggestive of acute cholecustitis. The symptoms are caused by impaction of gallstone
in cystic duct. Ingestion of fatty foods then stimulates the contraction of the gallbladder
against the obstructed cystic duct, causing severe colichy pain. The tissue behind the duct
obstruction becomes inflammed, typically as a result of bacterial overgrowth secondary to

117
stasis. The subsequent ischemic changes can cause gangrene and perforation, with generalized
peritonitis or a well-circumscribed abscess the most common outcomes. Other potential
complications include cholangitis and chronic cholecystitis. Tx for acute cholecystitis includes
supportive care (nothing by mouth, with iv antibiotics, alimentation, and analgestics).
Laparoscopic cholecystectomy is recommended shortly after hospitalization and should be
performed immediately in cases of perforation or gangrene.
6. Opacification of a single lung with shifting of the mediastinum toward the affected lung
indicates a collapsed lung due to atelectatic volume loss. Urgent bronchoscopy is required to
identify and possibly treat the obstructing bronchial lesion (mucus plugging, tumor, foreign
body or external compression).
7. Graves ophthalmopathy is the most common cause of exophthalmos in adults. This proptosis
occurs secondary to autoimmune lymphocytic infiltration of the extraocular muscles resulting
in fibroblast proliferation, hyaluronic acid deposition, edema and fibrosis.
8. Total parenteral nutrition (TPN) causes gall bladder stasis (decrease CCK) and predisposes to
both gall stone formation and bile sludging, both of which may lead to cholecystitis.
9. Zenker’s diverticulum is defined as a herniation of mucosa through the fibers of the
cricopharyngeal muscle. Barium esophagograph is the confirmatory test of choice.
Esophagoscopy should be avoided due to the associated risk of perforation.
10. A pelvic fracture with urethral injury is commonly accompanied by erectile dysfunction (ED).
The causes of ED in this case are nerve injury and latered arterial supply.
11. Many pts with influenza are treated with bed rest and simple analgesia (eg. acetaminophen).
Antiviral medications can reduce the duration of influenza symptoms by 2-3 days; however,
these drugs are only effective if administered within 48 hours of the onset of illness.
Amantadine and rimentadine are only active against Influenza A. The neuraminidase
inhibitors (ie. Zanamivir and oseltamivir) are active against both influenza A and B. *
extremely important!!
12. The tx of disseminated histoplasmosis in HIV pts is IV amphotericin B, followed by lifelong
tx with itraconazole. Initial tx with IV amphotericin B decreases the fungal load, and lifelong
tx with itraconazole prevents relapse.
13. In transient ischemis attack (TIA), the associated focal symptoms resolve in less than 24
hours. In reversible ischemic neurologic deficit (RIND), the associated focal symptoms
resolve in 24 hours to one week.
14. Craniopharyngiomas are benign suprasellar tumors which usually present with signs of
hypopituitarism, headaches and bitemporal blindness. Although they are infrequently seen
adults, craniopharyngiomas actually have a bimodal age distribution—children and 55-65
years age group. * Extemely high yield!!
15. Primary CNS lymphoma—the presence of EBV DNA in the CSF is specific for this condition.
MRI reveals a weakly ring-enhancing mass that is usually solitary and periventricular.
Toxoplasmosis is a good differential diagnosis; however, it is unlikely in this pt since he is
receiving trimethoprim-sulfamethoxazole. MRI reveals multiple, ring-enhancing, spherical
lesions in the basal ganglia. A postive Toxoplasma serology is quite common in normal
subjects in the US, and is therefore not specific.
16. Osteoarthritis predominantly involves the DIP joints. The major radiographic features include:
joint space narrowing, subchondral sclerosis, osteophytes, and subchondral cysts. Heberden’s

118
nodules are seen at the DIP joints, Boucher’s nodules are seen at the PIP joints.
RA involves the MCP and PIP joints. Bony erosions are classically seen.
Hemochromatosis-associated arthropathy include: squared-off bone ends and hook-like
osteophytes in the second and third MCP joints.
17. Thymectomy may induce remission in pts with myasthenia gravis, and should be considered
in all pts who are between puberty and 60, and in those with disease that is not confined to the
extraocular muscles.
18. Diffuse esophageal spasm is usually seen in young females and can present with intermittent
episodes of chest pain and dysphagia. Barium swallow may reveal a ‘corkscrew’ esophagus.
The tx is supportive. * Extremely high yield!!
19. Hypertrophic cardiomayopathy—Carotid pulse with dual upstroke, strong apical impulse and
a systolic ejection-type murmur along the left sternal border. The mormur is worsened by
maneuvers that decrease preload (eg. Valsava, standing); while decrease by increase preload
(squatting, recumbency, leg raising). Hand grip (increase afterload) is typically used to
differentiate between the murmurs of aortic stenosis and mitral regurgitation where it causes a
decrease and increase, respectively, in the severity of the murmur.
20. When a motility disorder of the esophagus is suggested by contrast studies, the next step is
usually esophagoscopy to exclude mechanical causes of dysphagia, such as stricture or
esophageal cancer; then followed by manometry, which confirms the dx.
21. Anemia of chronic disease is a disorder of iron utilization that most commonly occurs in the
setting of chornic inflammation. It is typically a normocytic anemia with a decreased serum
rion, decreased TIBC, and decreased iron saturation, as well as a normal or elevated ferritin
level.
22. Blood in the chest, if it is evacuated, can get infected. The majority of pts will present with a
low-grade fever, dyspnea, and chest pain. Surgery is required to remove the clotted blood and
fibrinous peel (when the empyema is localized, complex, and has a thick rim).
23. Abdominal CT scan is a very sensitive and specific tool used in the diagnosis of pancreatic
carcinoma.
24. Measurement of glycosylated hemoglobin is an excellent way to monitor chronic gylcemic
control. It is reflective of the pt’s average glucose levels over the preceding 100-120 days
(which correlates with RBC survival time). Generally, every 1% increase in HbA1c
corresponds with a 35mg/dl increase in the mean plasma glucose level.
25. VIPomas (pancreatic cholera) are cancerous tumors that affect cells in the pancreas that
produce vasoactive intestinal peptide (VIP). VIPoma cause diarrhea, hypokalemia resulting in
leg cramps, and a decrease in the amount of acid in the stomach. The cause is not known.
VIPomas are diagnosed most commonly at age 50 or so. Women ar more likely to be affected
than men. Other symptoms include dehydration, abdominal pain and cramping, weight loss,
facial flushing and redness. High level of VIP in the blood si diagnostic. A CT scan or MRI is
ordered to determine the location of the tumor.
26. Aortic dissection presents as sudden, tearing chest pain that radiates to the back. Mediastinal
widening is ofter, though not always, seen on CXR. Leg weakness can occur if the dissection
progresses to involve the arteries feeding the spinal cord.
27. Always suspect hypophosphatemic rickets in pts of rackets who had normal serum clacium,
normal serun alkaline phosphatase and normal 25-OH vitamin D.

119
28. A lesion in the upper thoracic spinal cord results in paraplegia, bladder and rectal
incontinency, and absent sensation from the nipple downwards. A lesion in the lower thoracic
spinal cord causes absent sensation from the umbilicus downwards.
29. Acut pericarditis typically occurs in the first several days after myocardial infarction. It is
characterized by sharp, pleuritic pain that is worse in the supine position and improved by
sitting up and leaning forwards. Diffuse ST elevations, especilly with PR depressions, are
typically ECG findings.
30. A consolidation (or fluid-filled area) persents with dullness to percussion, bronchial breath
sounds, and egophony (asking the pt to say the letter E and listening with a stethoscope; it
sounds like “A” with a nasal or bleating quality).
31. Vitamin B6, folate, and Vitamin B12 are involved in the metabolism of homocysteine.
Vitamin B6 lowers homocysteine levels by acting as a cofactor for the enzyme cystathionine
beta-synthase, which metabolizes homocysteine into cystathionine.
32. Hemolytic anemia in a pt with a maligant lymphoproliferative disorders likely to be of the
warm autoimmune type, caused by anti-RBC IgG antibodies. If prednisone therapy is
ineffective, splenectomy is usually indicated.
33. The ophthalmic artery (central retinal A. and ciliary branches) is the first intracranial branch
of the internal carotid artery. Central retinal artery occlusion is emergently treated with an
ocular massage and high-flow oxygen administration.
34. Suspect G6PD deficiency in a pt who develops acute hemolysis after ingesting primaquine or
sulfa drugs, G6PD levels are often normal during the hemolytic episode. Pyruvate kinase
deficiency can also lead to a similar clinical picure of hemolytic anemia, however, the
hemolysis in such cases is not precipitated by sulfa drugs. Furthermore, the typical peripheral
smear does not include bite cells.
35. Firm, flesh-colored, dome-shaped, umbilical papules are typical of molluscum contagiosum.
Cellular immunodeficiency, corticosteroid use, and chemotherapy predispose pts to
molluscum contagiosum. It is particulary common in pts with HIV.
36. SIADH is often seen with small cell carcinoma of the lung.
37. The US Preventive Services Task Force (USPSTF) recommeds screening male active or
former smokers aged 65-75 years with a one-time abdominal ultrasound to evaluate for an
abdominal aortic aneurysm.
38. Hypercalcemia of malignancy is due to several machanisms, including osteolytic metastasis,
increased production of PTHrP, increased production of 1,25-dihydroxyvitamin D, and
increased IL-6 levels. PTH levels are suppressed in most pts with hupercalcemia of
malignancy. Serum calcium levels are generally higher in pts with hypercalcemia of
malignancy than in those with primary huperparathyroidism. * High yield!!
39. Cessation of alcohol and staying on a low purine diet are important measures in the
prevention of future attacks in pts with acute gouty arthritis.
40. Suspect optic neuritis in a pt with central scotoma, afferent pupillary defect, changes in color
perception and decreased visual acuity. Remner the association between optic neuritis and MS
(USMLE love this topic!!).
41. PCP can occur when a pt’s CD4 count <200/ml. The typical clinical features include fever, dry
cough, and exertional dyspnea. Chest radiograph will show diffuse bilateral interstitial
infiltrates. Prednisone is used in combination with trimethoprim-sulfamethoxazole when the

120
PaO2 is less the 70 mmHg in pts with PCP.
42. Most pts with cat-scratch disease (CSD) have a gradual resolution of symptoms, even without
specific antibiotic therapy; however, tender regional lymphadenopathy and systemic
symptoms may be debilitating. Thereforem a short course of antibiotic therapy (5 days of
azithromycin) is recommended for pts with CSD.
43. Orthotopic liver transplantation remains the only effective mode of tx of fulminant hepatic
failure and should be considered in any pt presenting with this condition, regardless of
etiology.
44. Alpha-glucosidase inhibitors (acarbose, miglitol) block dietary carbohydrate breakdown in the
intestinal tract. The most significant side effects are GI disturbances due to the increased
undigested carbohydrate concentration in the stool.

1/9/10
1. Anorexia is a common problem of terminally ill pts. In palliative care, the rule-of-thumb is
that it is always better to have oral rather than parenteral nutrition. All efforts need to be made
to encourage the pt to eat adequate meals (eg. giving the pt his preferred meal, presenting the
food attactively, giving small and frequent meals). The next step is pharmacologic. The drug
of choice for cancer-associated anorexia is megestrol acetate, which is a synthetic progestin
with progestional effects similar to those of progesterone. It is currently used as an appetite
stimulant in pts with advanced malignancies. Prednisone may be used in addition to this drug.
Dimenhydrinate, ondansetron, and metoclopramide are anti-emetics which are usually given to pts
who are undergoing chemotherapy.
2. Anti-thyroid peroxidase and anti-thyroglobulin antibodies are present in more than 90% of pts
with Hashimoto’s thyroiditis.
3. Cor Pulmonale is a term for right-sided heart failure most commonly due to pulmonary
disease. Signs of right-sided heart failure include: jugular venous distension, right-sided S3,
right ventricular heave, hepatomegaly, ascites, and dependent edema.
4. Risk factors for pancreatic cancer are: 1) Male sex. 2) Increasing age (50 years). 3) Black
race. 4) Cigarette smoking; it is the most consistent risk factor. Pancreatic cancer is 2-3 times
more likely in heavy smokers than in non-smokers. 5) Chronic pancreatitis. 6) Long-standing
diabetes. 7) Obesity. 8) Familial pancreatitis. 9) Pancreatic cancer in a close relative. The
following are not risk factors for pancreatic cancer: 1) alcohol consumption. 2) gall stones. 3)
coffee intake.
5. Anti-mitochondrial antibodies are present in 90% of pts with primary biliary cirrhosis, a
chronic liver disease characterized by autoimmune destruction of the intrahepatic bile ducts
and cholestasis. Anti-smooth muscle antibodies and anti-LKM antibodies are associated with
acute and chronic hepatitis. The highest titers are linked with chronic active hepatitis. P-
ANCA is associated with Churg-Strauss syndrome and microscopic polyangiitis.
6. The most common cause of death in pts with acromegaly is cardiovascular, accounting for
approximately 38-62% of deaths.
7. In pts with hyperthyroidism-related tachysystolic atrial fibrillation, a beta-blocker is the drug
of choice.
8. Hehatic adenomas are uncommon benign epithelial tumors of the liver that usually arise as a
solitary mass in the right upper lobe. They are found predominantly in young and middle-aged

121
women who have a lengthy history of oral contraceptive usage. Hepatic adenomas may also
be seen is association with anabolic androgen use, glycogen storage disease, pregnancy, and
DM. A hormonally mediated pathogenesis is suspected, but the mechanism is not well
understood.
Histologically, hepatic adenomas contain sheets of enlarged adenoma cells that contain glycogen
and lipid. While the nuclei are small and regular, the normal hepatic archetecture is absent (with
septa, portal tracts, and bile ductules missing).
Hepatic adenomas are typically identified when pts complain of abdominal pain in the epigastrium
or right upper quadrant, when imaging is obtained for unrelated issues, or when an individual
suddenly collapses because of rupture and intraabdominal bleeding. The risk of such bleeding is
thougth to approach 40% in symptomatic patients, which is why obtaining biopsy of these masses
is normally C/I.
Common physical finding include a palpable abdominal mass, hepatomegaly, or jaundice. Liver
function tesets are usually normal, but are elevated on occasion. Alkaline phosphatase and gamma
glutamyl transpeptidase (GGT) are most commonly elevated in those pts with intratumoral
bleeding or multiple adenomas. Alpha-fetoprotein is normal unless malignant transformation of
the tumor has occurred, which happens in 8-13% among all pts with hepatic adenomas.
The diagnosis is beased upon the clinical presentation, imaging studies (typically CT scan or
ultrasound), and /or surgical resection. There is some controversy regarding preferred tx of this
tumor, but the general consensus is that all symptomatic hepatic adenoma should be resected.
Those adenomas that are not symptomatic can be initially managed ocnservatively with
discontinuation of oral contraceptives and careful observation with repeated imaging and serial
AFP measurements. Complete regression of the tumor can happen with consevative therapy, but
growth, rupture, and malignant transformation are also possibilities. Because of these risks, some
experts recommended the resection of all hepatic adenomas, regardless of size or symptoms.
Focal nodular hyperplasia (FNH) is a very common non-malignant hepatic tumor that is not of
vascular origin. It arises as a hyperplastic response to hyperperfusion by anomalous arteries that
are persent in the center of the nodule. The biopsy usually shows sinusoids and Kupffer cells,
neither of which is seen with hepatic adenomas. FNH is not associated with the use of oral
contraceptives.
9. In a cross-sectional study, exposure and outcome are measured simultaneously at a particular
point of time (remember: snapshot study). In other study designs, a certain time period
separates the exposure from the outcome.
10. Bonr marrow iron stain is the most definitive way to dignose iron deficiency anemia.
11. In diabetic pts, concurrent vascular and neurological diseases contribute to foot problems,
which are important causes of morbidity. The lifetime risk of foot ulcer in pts with diabetes is
15%. Foot ulcers can be classified as follows:
Grade 0: High-risk foot without an ulcer
Grade1: Superficial ulcer with full skin thickness involvement, but not involvement of underlying
tissue.
Grade 2: Deep ulcer penetrating to ligament or muscle, but not bone involvement or abscess
formation.
Grade 3: Deep ulcer with cellulitis, abscess formation or osteomyelitis
Garde 4: Localized gangrene

122
Grade 5: Extensive gangrene involving that whole foot.
Mx: 1) off-loading, 2) debridement, 3) wound dressings 4) antibiotics, 5) revascularization 6)
amputation. The key steps in the mx of grade 1 and 2 diabetic ulcers are proper wound care and
debridement. Debridement and removal of all the infected and necrotic tissue is essential in such
pts.
12. Erosion joint disease in RA is a clear-cut indication for the use of diseases modifying anti-
rheumatic drug (DMARDs). Methotraxate is the initial drug of choice for this purpose.
Glucocorticoids offer rapid improvement in the clinical and radiographic findings of RA, but
due to their significant adverse effects they are typically not used for long term mx. Celecoxib
is a selective COX2 inhibitor and its anti-inflammatory and analgestic action is comparable to
conventional NSAIDs. In contrast to conventional NSAIDs, celecoxib does not affect
hemostasis and does not produce GI side effect. COX-2 inhibitors are avoided currently due to
their ability to cause an increased risk of heart disease. Azathioprine is a purine analog that is
used as an immunosuppressive and a DMARD. It is not as efficacious as methotrexate;
therefore, it is not used as a first-line agent in RA. Etanercept and infliximab are TNF
inhibitors. They are very effective in pts who have disease refractory to methotrexate. These
agents, however, are very expensive and do not have the long safety record that methotrexate
had because they are relatively new. Reports of increased risk of lymphoma, TB reactivation,
CHF and demyelineating disease have been associated with these agents. They are not yet
indicated as first-line tx.
13. Gastric contents are rich in hydrogen, chloride, and potassium. Therefore vomiting causes
hypochloremic metabolic alkalosis and hypokalemia. Bicarbonate levels rise as a result of
hydrogen loss and activation of the renin-angiotensin system. The administration of isotonic
sodium chloride and potassium is used to reverse these eletrolyte abnormalities.
14. Right ventricular failure will cause elevated jugular venous pressure, hepatosplenomegaly,
ascites, and peripheral edema (increased pulmonary artery systolic pressue), but is not a
common cause of pulmonary edema.
15. CMV pneumonitis should be considered in the differential diagnosis of any bone marrow
transplant (BMT) recipient with both lung and intestinal involvement. Risk factors include
certain types of immunosuppressive therapy, older age, and seropositively before
transplantation. The median time of development of CMV pneumonitis after BMT is about 45
days (range of 2-4 weeks). Typical chest x-ray findings include multifocal diffuse patchy
infiltrates. High resolution CT scan show parenchymal opacification or multiple small
nodules. Bronchoalveolar lavage is diagnosis in most cases. Othe than pneumonitis, CMV
infection in post-BMT pts also manifests as upper and lower GI ulcers, bone marrow
suppression, arthralgias, myalgias, and esophagitis.
16. Unfortunately, no serologic or radiographic test has proven effective in screening for
pancreatic cancer in asymptomatic adults.
17. The most probable diagnosis in this pt is acquired sideroblastic anemia. This condition results
from defective heme synthesis, most commonly due to pyridoxine-dependent impairment in
early steps of protoporphyrin synthesis. This pt is taking isoniazid, a well-known pyridoxine
antagonist that can be responsible for this condition. Acquired sideroblastic anemia frequently
manifests as microcytic hypochromic anemia simulating iron-deficiency anemia. Usually two
groups of RBC can be demonstrated on microscopy-hypochromic and normochromic

123
(“dimorphic” RBC population). Besides that, iron studies typically reveal increased serum
iron concentration and decreased total iron binding capacity (TIBC), which helps to
differentiate sideroblestic anemia from iron-deficiency anemia. In this pt, with an easily
identifiable cause of Vit B6 deficiency, administration of pyridoxine is the most reasonable
next step. Bone marrow sampling can confirm the dx by demonstrating ‘ringed sideroblasts,’
(a specific finding in this type of anemia), but it si not necessary in this pt.
18. Pt with acute, severe pain should reveive the same standard of pain management regardless of
drug history. IV morphine is appropriate tx for acute, severe pain. Physicians should never
underteat pain, even if there is a risk of abuse. In the cases of concern for abuse, frequent
reassessment, putpatient follow-up, and referral to a pain specialist is appropriate.
19. Pts with Trousseau’s syndrome (migratory thrombophlebitis) usually have an occult tumor,
which may not always be detectable at the time of presentation. The most common tumor is
an adenocarcinoma. Based on the reviews, 24% -pancreatic carcinoma, 20% lung carcinoma,
13% prostate cancer, 12% stomach cancer, 9% acute leukemia, and 5% colon cancer. The
thrombophlebitis of the atypical sites such as arm and chest is an important clue for the
underlying carcinoma. So, the presence of this along with symptoms of upper GI discomfort
immediately leads to the possibility of pancreatic carcinoma that needs to be thoroughly
investigated. Abdominal CT scan is the test of chioce.
20. Cluster headache is characterized by intense unilateral retroorbital pain which starts suddenly
(usually at night, wakes the pt from sleep), peaks rapidly, and lasts for approximately 2 hours.
It is more common in men. It may be accompanied by redness of the ipsilateral eye, tearing,
stuff or runny nose, and ipsilateral Horner’s syndrome. The attacks occur in clusters, daily, for
6-8 weeks, followed by remission lasting for up to a year. Since the attacks are short but
severe, prophylaxis is the key to mx. The recommended prophylactic medications include
verapamil, lithium and ergotamine. The tx for an acute attack is inhalation of 100% oxygen
and sbcutaneous sumatriptan. * Extremely high yield!!
21. Albendazole or mebendazole is the first-line tx ofr enterobius vermicularis infection (not in
pregnant pts). Pyrantel pamoate is an alternative (second line).
22. Recurrent peptic ulcers with hypercalcemia are best explained by MEN type I. Serum
parathyroid levels and gastrin levels should be performed first.
23. The most common form of congenital adrenal hyperplasia (CAH) is 21-hydroxylase
deficiency, accounting for more than 90% of cases. The metabolite that accumulates us 17-
alpha-hydroxyprogesterone. Elevated levels of 17-alpha-hydroxyprogesterone serve as
precursors of adrenal androgens, thereby leading to hyperandrogenism.
Complete deficiency of 21-hydroxylase presents at birth with virilization of the female child and
excessive androgenization of the male child, there is also a decreased production of
mineralocorticoids and glucocorticoids, which causes salt wasting. Partial deficiency is also know
as non-classical CAH pts usually present during puberty or adulthood with hyperandrogenism,
which is not usually associated with salt wasting. The dx is suggested by increased levels of 17-
alpha-hydroxyprogesterone, and confirmed with an ACTH stimulation test.
17-hydroxylase deficiency leads to delayed puberty and mineralocorticoid excess.
11-alpha-hydroxylase deficiency results in androgen and mineralocorticoid excess.
3-beta-hydroxysteroid dehydrogenase deficiency results in DHEA-S excess and decreased
testosterone and mineralocorticoids.

124
Cystathionine synthase deficiency or homocystinuria is characterized by Marfan body habitus,
dislocation of the lenses, fair skin and vascular thrombosis.
24. TB test:
1. indutation >5mm
HIV-positive persons
Individuals with recent contact with a TB-positive person
Individuals with signs of TB on CXR
Organ transplant pts, pts on immunosuppressive therapy
2. >10mm
Individuals who have recently emigrated from a location where TB is endemic
Injection drug users
Residents/employees of high-risk settings (eg. prisons, homeless shelters)
Pts with diabetes, chronic kidney disease, hematologic malignancies, or fibrotic lung disease
Children less than 4 years of age, teens exposed to high-risk adults
3. >15mm
Healty individuals with no risk factors for TB infection

25. Hypertrophic cardiomyopathy can lead to ventricular arrhythmias, and should be suspected in
a sudden death of an otherwise healthy young individual.
26. Pharmacologic mx of variant angina (transient ST elevation on ECG at night) involves
calcium channel blockers and/or nitrates to prevent coronary vasoconstriction. Nonselective
beta-blockers and aspirin should be avoided b/c they can promote vasoconstriction.
27. The classic symptom triad of normal pressure hydrocephalus (NPH) is abnormal gait,
incontinence, and dementia. NPH is treated with large volume lumbar punctures and, if
successful, ventriculoperitoneal shunting.
28. Anticoagulation with heparin should be initiated immediately in any pt with a high likelihood
of pulmonary embolism and respiratory distress before diagnosis tests are done to confirm the
dx.
29. Cerebellar tumors usually produce ipsilateral ataxia, nystagmus, intention tremors, and loss of
coordination.
30. Arterial pH or anion gap is the most reliable indicator of metabolic recovery in pts with
diabetic ketoacidosis. Serum ketones level correct later than arterial blood pH or anion gap do.
31. Venous insufficiency results from incompetence of the valves within the lower extremity
veins. Complications include chronic edema, stasis dermatitis, and ulceration. Therapies
include leg elevation, compression, stockings, and wound care.
32. The Chi-square test is used to compare proportions. A 2x2 table may be used to compare the
observed values with the expected values.
33. Glucocorticoids cause neutrophilia by increasing the bone marrow release and mobilizing the
marginated neutrophil pool. Eosinophils and lymphocytes are decreased. * Extremely high
yield!!
34. Serous otitis media is defined as the presenct of a middle ear effusion without signs of active
infection. Examniation commonly reveals a dull tympanic membrane that is hypomobile on
pneumatic otoscopy. It is the most common middle ear pathology in pts with AIDS.
35. Agents available for Parkinsonism are L-dopa, dopamine agonist, anticholinergics and

125
amentadine. The most effective symptomastic therapy of Parkinsonism is L-dopa. L-dopa is
associated with a greater risk of dyskinesia than dopamine agonist; while dopamine agonists
have slightly less efficacy than L-dopa. Younger pts are usually initially treated with
dopamine agonist, and older pts are given L-dopa. Amantadine has relatively lower efficacy
and is used for pts with mild disease who have no disability. Selegiline is a selective
monoamine oxidase B inhibitor and a neuroprotective agent that may slow down the
progression of disease. It has a mild symptomatic action, and may be used as an adjunctive tx
in pts receiveing L-dopa.
36. Postoperative endophthalmitis is the most common form of endophthalmitis. It usually occurs
within 6 weeks of surgery. Pts usually present with pain and decreased visual acuity.
Examination reveals swollen eyelids and conjunctiva, hypopyon, corneal edema and infection
(esp vitreous). Based on the severity, intravitreal antibiotic injection or vitrectomy is done.
37. Pulmonary-renal syndromes incude a variety of disorders with simutaneous involvement of
the lung and kidney. Quick differential dx is important b/c the mx differs per disease.
Emergency plasmapheresis is required in pts with Goodpasture’s syndrome. Wegener’s
granulomatosis is treated with a combination of cyclophosphamide and steroids.
38. Legionella pneumophilia is an intracellular G- organism commonly spread by cooling towers
and water supplies. Travel-associated infections is well-documented. Symptoms suggestive of
Legionnaire’s disease include cough, fever >39C, GI symptoms, and confusion. Dx is
confirmed by urine antigen testing or culture on charcoal agar. Tx with azithromycin or
levofloxacin is appropriate.
39. IV amphotericin plus flucytosine is the antibiotics regimen of choice for central nervous
system cryptococcal infection in AIDS pts.
40. Rifampin causes red to orange discoloration of body fluids. “Red urine” in a pt taking
rifampin is usually a benign effect.
41. Suspect neurofibromatosis type II in a young pt with acoustic neuroma and multiple café-au-
lait spots. MRI with gadolinium enhancement is the best method to diagnose acoustic
neuromas.
42. Type 4 renal tubular acidosis should be suspected in a diabetic pt with a non-anion gap
metabolic acidosis, persistent hyperkalemia and renal insufficiency.
43. Melena is usually a manifestation of upper GI bleeding. Pain from duodenal ulcers usually
improves with food, whereas pain from gastric ulcers tends to worsen with food.
44. The MCC of asymptomatic elevation of alkaline phosphatase in an elderly pts is Paget’s
disease of bone (osteitis deformans). Pts with this condition are typically asymptomatic at the
time of dx; the dx is made by incidentally finding an isolated elevated alkaline phosphatase on
routine lab testing.

1/9/10
1. Cat bites are of significant concern because these often result in deep puncture wounds.
Furthermore, infection of such wounds with Pasteurella multicida tends to develop quickly,
and is associated with considerable pain, erythema, and swelling. Localized cellulitis can
develop subacutely, and in some cases, systemic effects (eg. fever and lymphadenopathy) may
arise. Prophylactic tx is thus recommended for such cases. For tx of minor cat bite wounds,
amoxicillin/clavulanate for 5 days is recommended.

126
2. Tricyclic antidepressants (amitryptiline, desipramine, nortriptyline) are the drugs of choice for
diabetic neuropathy. TCAs can worsen urinary symptoms (due to cystopathy) and orthostatic
hypotension (due to cardiovascular autonomic neuropathy). Gabapentin is an alternative for
these pts.
3. The main substances of gluconeogenesis are: alanine, lactate and glycerol 3-phosphate.
Pyruvate is an intermediate of alanine during the process of gluconeogenesis.
4. Mixed acid-base disorders may cause inappropriately normal lab values. Calculate the
expected change in the PaCO2 or HCO3 and compare that to the pt’s measured values to
determine if a mixed disorder exists or if normal physiology compensation has occurred.
PaCO2 =1.5 ( HCO3)+8
5. A massive pulmonary embolism can cause right ventricular dilatation and failure that results
in hypotension, tachycardia, syncope, and shortness of breath. Hemodynamic instability is an
absolute indication, and right ventricular strain a relative indication, for thrombolytics in the
setting of PE.
6. Wegener’s granulomatosis is a vasculitis affecting small and medium-sized arteries. Pts
present with a combination of glomerulonephritis and upper and lower respiratory tract
disease. Diagnosis is made by C-ANCA positivity and tissue biopsy. Tx involves high-dose
corticosteroids and cytotoxic agents.
7. Pericardial cysts are usually found in the middle mediastinum. Thymoma is usually found in
the anterior mediastinum. All neurogenic tumors are located in the posterior mediastinum.
8. Internuclear ophthalmoplegia is a pathognomonic finding of multiple sclerosis, and is due to
demyelination of the medial longitudinal fasciculus.
9. Premature atrial complex (PACs) maybe a manifestation of underlying heart disease or may
be idiopathic. Tobacco and alcohol are reversible risk factors for PACs. Beta-blockers are
often helpful in pts who are symptomatic.
10. Pts suffering from ankylosing spondylitis for two decades or longer are at an increased risk of
vertebral fracture due to decreased bone mineral density. Vertebral fractures in these pts may
occur with minimal trauma; therefore, the clinical suspicion for vertebral fracture in pts with
longstanding ankylosing spondylitis must be high.
11. Lifestyle modification should always be a part of the mx of hypertension. All pts should be
encouraged to lose weight, reduce salt in the diet, avoid excessive alcohol intake, and stop
smoking.
12. Muddy brown granular cast—Acute tubular necrosis
RBC casts—Glomerulonephritis
WBC casts—interstitial nephritis and pyelonephritis
Fatty casts—nephrotic syndrome
Broad and waxy casts—Chronic renal failure * Extremely high yield!!
13. how to calculate predictive values of a test
14. Pts with hemochromatosis are vulnerable to Listeria monocytogenes infections. Iron overload
is also a risk factor for infection with Yersinia enterocolitica and septicemia form Vibrio
vulnificus, both of which are iron-loving bacteria.
15. Adenocarcinoma of lung has the least association with smoking. It is usually located
peripheraly and consists of columnar cells growing along the septa. It presents as a solitary
nodule and may be detected incidentally.

127
16. Lumbar spinal stenosis is most commonly caused by degenerative joint disease (DJD, disc
herniation and facet osteophytes impine upon the spinal cord). The term “neuropathic
claudication” is often used to describe lumbar stenosis. This refers to the exacerbation of leg
symptoms with walking (similar to peripheral vascular disease-PVD). Howeve, unlike PVD,
the symptoms are positional and remain while standing still. Pain is relieved by flexion of the
spine (widening of the spinal canal). Dx is made based on clinical history and classic findings
on spinal MRI.
17. In developed countries, more than 80% of pts have primary adrenal insulficiency due to
autoimmune adrenalitis. Pts with autoimmune adrenalitis have autoimmune involvement of
other endocrine glands, such as the thyroid, papathyroid, and ovaries.
18. Neurofibromatosis is a neurocutaneous syndrome which has a tendency to form tumors in the
central nervous system, peripheral nervous system, skin and viscera. Other neurocutaneous
symptoms include: 1) Sturge-Weber syndrome (port wine stains in the V1 cranial nerve
distribution, angiomatous malformations of the brain, seizures, hemiparesis), and 2) Tuberous
slcerosis (ash leaf hypopigentation, cardiac rhabdomyomas, kidney angioleiomyomas, mental
retardation, seizure).
19. Uremic pericarditis is an absolute indication for dialysis in pts with chronic renal failure.
Know the indications and C/I of dialysis in uremic pts. Remember that the decision to start
dialysis is not strictly based on serum creatinine or BUN levels. Absolute indications: 1) Fluid
overload not responsive to medical tx. 2) hyperkalemia not response to medical mx. 3)
Uremic pericarditis. 4) refractory metabolic acidosis. Relative indictions: 1) Glomerular
filtration rate <10 ml/min (<15 ml/min in diabetics). 2) serum creatinine >8mg/dl (> 6mg/dl in
diabetics). 3) severe uremic symptoms (Uremic seizure and coma is an indication of
immediate dialysis). C/I: 1) debilitating chronic disease. 2) severe irreversible dementia.
20. Moderate to severe asthma exacerbations are treated with a combination of beta2-agonists,
inhaled anticholinergics, and intraveous steroids. Pts who develop respiratory failure, marked
by hypercapnia, or severe hypoxia, require intubation and mechanical ventilation.
21. A negative D-dimer is very helpful in excluding the dx of pulmonary venous
thromboembolism. A positive D-dimer, however, is far from diagnostic and must be followed
by more specific studies.
22. Bladder tumors are the MCC of painless hematuria in adults. More the renal cell carcinoma.
23. Cushing’s syndrome is caused by corticosteroid excess (eg asthma pt). Hypokalemia and
hypernatremia are the electrolyte abnormalities most common observed (have some
mineralocorticoid activity and will bind to aldosterone receptors in the kidney).
24. Cystinuria is an inherited disease causing recurrent renal stone formation. Look for a personal
history of recurrent kidney stones for childhood and a positive family history. The
characteristic stones are hard and radioopaque. Urinalysis shows typical hexagonal crystals.
The urinary cyanide nitroprussid test is widely used as a qualitative screening procedure. The
problem lies with the defective transport of dibasic amino acids (cystine, lysine, arginine, and
ornithine) by the brush borders of renal tubular and intestinal epithelial cells. Cystine is
poorly soluble in water; this leads to the formation of hard, redioopaque renal stones.
25. Pts with a retropharyngeal abscess often complain of a sore throat, fever, difficulty
swallowing (dysphagia), pain with swallowing (odynophagia), and pain with certain neck
movements, particularly neck extension. Tx must be initiated rapidly in order to prevent the

128
spread of infection into the mediastinum.
26. Cutaneous larva migrans, or creeping eruption, is a helminthic disease caused by the
infective-stage larvae of Ancylostoma brazilliense, the dog and cat hookworm. Infection
occurs after skin contact with soil contaminated with dog or cat feces containing the infective
larvae. It is common in travelers in tropical regions, and is characterized by pruritic, elevated,
serpiginous 匐行的 lesions on the skin. Infection is often acquired through contact with sand.
27. SLE is an autoimmune disease that most commonly affects African-American women aged
20-40 years. Systemic manifestations include fatigue, fever, weight loss, non-deforming
arthritis, oral ulcers, serositis, hematologic abnormalities, proteinuria, and rash. Greater than
90% of pts have arthritis, most commonly affecting the MCP and PIP joints of the hands.
28. Cyclosporine and tacrolimus have the same mechanism of action (calcineurin-inhibitors,
inhibiting the transcription of IL-2 and several other cytokines). The major side effects of
cyclosporine include nephrotoxicity, hyperkalemia, hypertension, gum hypertrophy,
hirsutism, and tremor. Tacrolimus has similar toxicities, except for hirsutism and gum
hypertrophy. Major toxicity of azathioprine (purine analog) is dose-related diarrhea,
leukopenia, and hepatoxicity. Mycophenolate is a reversible inhibitor of inosine
monophosphate dehydrogenase (IMPDH), which is that rate-limiting enzyme in de novo
purine synthesis. The major toxcity of mycophenolate is bone marrow suppressio.
29. Consider 3 possibilities when a flat film of the abdomen and pelvis does not show a stone in a
pt with typical renal colic: 1) radiolucent stone disease (uric acid stones). 2) calcium stones
less than 1-3m in diameter. 3) Non-stone causes (eg. obstruction by blood clot or tumor). Uric
acid stones accounts for approximately 10-15% of cases of total nephrolithiasis. These are
most commonly seen in pts with unsusally low urine PH levels (which may be due to a defect
in renal ammonia secretion) and hyperuricosuria. These are radiolucent, but can be seen on
USG and CT scan. Tx includes hydration, alkalinization of urine, and a low-purine diet
with/without allopurinol, depending on the presence of hyperuricosuria. Since uric acid
stones are highly soluble in alkaline urine, alkalinization of urine to pH>6.5 with oral sodium
bicarbonate or sodium citrate is indicated.
30. In general, immunosuppressed pts and those with immunodeficiencies should receive an
extensive panel of vaccinations, but should not receive live vaccines: BCG, varicella
(chickpox), varicella zoster (shingles), anthrax, oral typhoid, intranasal influenza, oral polio,
and yellow fever vaccines. The only exception is the MMR, which may be used in pts
without evidence of immunity if their CD4+ count is >200/mm3 and they have no history or
evidence of an AIDS-defining illness.
31. Acute inflammatory arthritis caused by CPPD (calcium pyrophosphate dehydrate) crystals is
called pseudogout. It often occurs in the setting of recent surgery or medical illness.
Pseudogout is diagnosis by the presence of rhomboid-shaped, positively birefringent crystals
on joint aspiration microscopy and radiographic evidence of chondrocalcinosis (calcified
articular cartilage).
32. receiver-operating characteristic curve (ROC)
33. Suspect acute glaucoma in a pt with a sudden onset of eye pain, photophobia, and a mid-
dilated pupil. Tonometry is the best diagnostic test.
34. The concept of latent period is an important issue in chronic disease epidemiology. Exposure
must be continuously present for a centain peroid to time (called latent period) to influence

129
the outcome.
35. Exercise EKG testing is recommended for pts with an intermediate pre-test probability of
angina based on clinical features and risk factors. Medications that should be withheld prior
to testing include anti-ischemic medications, digoxin and medications that slow the heart (eg.
beta-blockers).
36. Cholesterol embolization usually follows surgical or interventional manipulation of the
arterial tree. Renal failure, livedo reticularis, systemic eosinophilia, and low complement
levels should make you think of cholesterol embolism. Renal failure in the presenct of
systemic eosinophilia should make you think of cholesterol embolism, allegic interstitial
nephritis, or polyarteritis nodosa. Acute allergic interstitial nephropathy is a drug-induced
hypersensitivity reaction characterized by rash, renal failure, eosinophilia, and eosinophiluria
(Hansel stain). The common medications that you should remember of the exam are: 1)
antibiotics (most common is methicillin group). 2) NSAIDs (often cause heavy proteinuria).
3) Thiazides 4) phenytoin 5) allopurinol * Extremely high yield!!
37. Acute otitis media should be considered in any pt with symptoms of ear drainage and
difficulty hearing. Ear pain is also common, but may be absent in young pts. Nonspecific
systemic symptoms including fever, irritability, and diarrhea can sometimes be present as
well. Cholesteatoma can present with hearing loss and ear drainage, but fever and systemic
symptoms are not usually present.
38. Succinylcholine is a depolarizing neuromuscular blocker that can cause life-threatening
hyperkalemia. It should not be used in pts with or at high risk for hyperkalemia, such as burn
and cruch injury pts and pts with prolonged demyelination.
39. The classic findings in pts with amyloidosis are renal amyloid deposits that show apple-green
birefrigence under polarized light after staining with Congo red. Clues—RA pt with enlarged
kidney and hepatomegaly.
40. Many expert group including the US Preventative Services Task Force recommended a one-
time screening of all women who are 65 years and older with a DEXA scan—ostepporosis.
41. “Soap bubble appearance in the epiphyseal end of long bone”—giant cell tumor of bone. The
classical pt is 20-40 yr old female with knee pain and some mass. The tumor cells are oval or
spindle shaped intermingled with numerous multinuclear giant cells in fibrous stroma. It is
benign but locally aggressive. It frequently recurs even after local currettage. The best way to
handle this tumor is to recognize it and leave it for experts.
42. The most feared complication of a retropharyngeal abscess is spread of infection into the
mediastinum, which can lead to acute necrotizing mediastinitis. The major complication of an
infection in the parapharyngeal space is involvement of the carotid sheathm which may lead
to erosion of the carotid artery and jugular thrombophlebitis.
43. Crystal-induced nephropathy is a well-know side effect of indinavir (a protease inhitibor)
therapy. Remember: 1) didanosine-induced pancreatitis. 2) abacavir—related hypersensitivity
synrome. 3) Lactic acidosis sencondary to the use of any of the NRTIs. 4) Stevens-Johnson
syndrome secondary to the use of any of the NNRTIs. 5) nevirapine—associated liver failure.
44. Alcoholism is the most common cause of cirrhosis in the US. Infection with HCV is the
second MCC of cirrhosis in the US.

1/10/10

130
1. IgA nephropathy is the most common cause of glomerulonephritis in adults. Pts have
recurrent episodes of gross hematuria, beginning 1-3 days after an upper respiratory infection.
Serum complement levels are normal. The latent peroid from infection to hematuria in post-
streptococcal glomerulonepharitis averages 10 days for pharyngitis and 21 days for impetigo,
while it is less than 5 days for IgA nephropathy. Serum complement levels are low in cases in
post-streptococcal glomerulonepharitis.
2. In chronic tophaceous gout, urate crystals can be deposited in the skin resulting in the
formation of tumor with a chalky white appearance.
3. Viridans group strepcocci are the most likely cause of endocarditis in native valves following
dental procedure. 4 members of the viridans group cause infective endocarditis: Streptococcus
mitis, S. sanguis, S. Mutans and S. salivarius. S. mutans also causes dental caries.
S. bovis is a normal inhabitant of the GI tract, and S. bovis bacteria is associated with colon
cancer. Colonscopy should be perfomed when this organism is isolated from blood culture.
S.epidermidis is an important cause of prosthetic valve endocarditis. It is also seen in infants with
IE sencondary to umbilical venous catheter infection in neonatal intensive care units.
Enterococci are normal inhabitants of the GI tract and also occasionally colonize the anterior
urethra. Enterococcal endocarditis generally affects older men after genitourinary manipulation or
young women after obstetric procedures.
4. Guillain-Barre syndrome is an acute or subacute ascending papalysis, CSF analysis reveals an
elevated protein level with normal glucose levels, WBC and RBC counts.
5. Gallbladder carcinoma (GBC) is a rare malignancy that most often arises in Hispanic and
Southwestern Native American females who have a history of gallstones. It is typically
diagnosed during or after cholecystectomy. Because the symptoms of gallbladder carcinoma
are vague and nonspecific (eg. pain, anorexia, nausea, vomiting, malaise) and the gallbladder
is nestled among other organs, GBC tends to be late-presenting and to have spread by the time
of diagnosis. Radical surgical interventio for pts with advanced disease includes
cholecystectomy and removal of the gallbladder bed, pancreatic head, LN, and sometimes
portions of the liver or biliary tree. If the GBC is discovered while still confined to the lamina
propria of the gallbladder, however, simple cholecystectomy results in a cure rate of 73%-
100% and is considered sufficient tx. Chronic choledochoithiasis is a risk factor for GBC
because if causes inflammation and irritation of the gallbladder epithelium, thereby inviting
malignant change.
6. Hawthorne effect is the tendency of the study population to affect the outcome since they are
aware they are being studied.
Sample distotion is seen when the estimate of exposure and outcome association is biased b/c the
study sample is not representative of the target populationn with respect to the joint distribution of
exposure and outcome.
Information bias occurs due to the imperfect assessment of association between the exposure and
outcome as a result of errors in the measurement of exposure and outcome status. It can be
minimized by using standardized techniques for surveillance and measurement of outcomes, as
well as trained observers to measure the exposure and outcome.
Confounding bias occurs due to the presence of one or more variables associated independently
with both the exposure and the outcome. For example, cigarette smoking can be a confounding
factor in studying the association between maternal alcohol drinking and low birth weight babies,

131
as cigarette smoking is independently associated with both alcohol consumption and low birth
weight babies.
7. The most common site of ulnar nerve entrapment is the elbow where the ulnar nerve lies at
the medial epicondylar groove. Prolonged, inadvertent compression of the nerve by leaning on
the lebows while working at a desk or table is the typical scenario. * Extremely high yield!!
8. 70% of cases with interstitial nephritis are caused by drugs such as cephalosporines,
penicillins, sulfonamide containing diuretics, NSAIDs, rifampin, phenytoin, and allopurinol.
Discontinuing the offending agent is the tx of drug-induced interstitial nephritis. Steroids may
hasten recovery in cases of drug-induced interstitial nephritis, but they may aggrevate the
underlying infection.
9. Otimally, the first step in treating a pt with a witnessed cardiac arrest is defibrillation. When a
cardiac arrest is unwitnessed or when there is a greater than 4-5 minute lapse between arrest
and arrival of the defibrillator, a trial of cardiopulmonary resuscitation (CPR) should precede
defibrillation.
10. Hodgkin’s disease is a curable lymphoma that tends to affect young pts. Unfortunately, pts can
develop secondary malignancies form chemotherapy and/or radiation, especially those treated
before the age of 30. The risk of secondary malignancy is highest when chemotherapy is
combined with radiation, reaching up to 3.2% within 20 years of tx. Lung and breast cancers
are among the most common secondary malignancies affecting pts previously treated for
Hodgkin’s disease.
11. Wafarin is a vit K antagonist used for anticoagulation in numerous clinical settings. Foods rich
in vits K (eg. dark green vegetables: brussels sprouts, green tea, spinach) will decrease its
efficacy whereas numerous other foods, supplements, and medications will increase its
activity. Examples of agents that increase warfarin activity include alcohol, vit E, garlic,
ginkgo biloba, ginseng, St. John’s wort, and several types of antibioticss.
12. IgA deficiency significantly increases the risk of developing an anaphylactic reaction to
transfused blooc products. The risk of anaphylaxia may be reduced in susceptible pts by
providing IgA-deficient blood products or by preforming additional washes on RBC or
platelet products.
13. Clear lung fields, hypotension, and jugular venous distension in the setting of an inferior wall
MI are suggestive of an RV infarct. Fluid resuscitation is the appropriate mx, and nitrates
should be avoided. Nitrates are not indicated in the setting of aortic stenosis, recent
phosphodiesterase inhibitor use, or right ventricular infarction.
14. It is important to distiguish between the different patterns of muscle weakness. The hallmark
of myasthenia gravis is decreasing muscular strength with continued contraction. While this
can affect any skeletal muscle, extraocular symptoms (eg. diplopia) are most common.
15. Normally, more than 95% of circulating bilirubin is the unconjugated fraction, which is highly
insoluble in water and is tightly bound to albumin. When bound, this unconjugated bilirubin
cannot be filtered by the glomerulus and is therefore not excreted in the urine. In the contrast,
the conjugated bilirubin is water soluble, loosely bound to albumin, and excreted in urine
when presnt in excess. Therefore, the presence of bilirubin in urine is indicative of a
conjugated hyperbilirubinemia (urine dipstick)—eg. Rotor syndrome.
16. Leukoplakia presents as hard to remove white patches in the oral mucosa and may lead to
squamous cell carcinoma. It is usually caused by chronic irritation to the oral mucosa due to

132
smoking, alcohol, or ill-fitting dentures.
17. First generation H1-antihistamines (diphenhydramine, chlorpheniramine, doxepine and
hydroxyzine) have potent anticholinergic effects and may cause eye and oropharyngeal
dryness as well as urinary retention, especially in older males who may have some underlying
BPH. Urine retention caused by anticholinergic agents results from failure of detrusor
contraction.
18. Pts at average risk of developing colon cancer should begin screening at age 50 with fecal
occult blood testing (FOBT), flexible sigmoudoscopy, a combination of the two, colonoscopy,
or double-contrast barium enema. Pts with an affected first-degree relative should begin
screening 10 years before the age of relative was diagnosed.
19. Facial nerve palsy and classical erythema migrans (EM) indicate Lyme disease, which is a
tick-bite disease.
20. The first step in the evaluation of a pt with a thyroid nodule is measurement of the TSH level.
If the TSH level is elevated, the levels of thyroid antibodies and thyroxine (T4) are measured b/c
pts with Hashimoto’s thyroiditis can have thyroid nodule formation. Since thyroid cancer can
occur with Hashimoto’s thyroiditis, FNAB is usually done if the nodule is larger than 1-1.5cm.
There is no real pathognomic sign that can conclusively differentiate benign form malignant
thyroid nodules; however, a rapid decrease in nodule size with levothyroxine tx is very reassuring
for a bening process.
If the TSH level is suppressed or below normal, radioiodine uptake and scan are typically
performed. If the scan shows a hot nodule (increased uptake of the tracer in the nodule with
decreased uptake in rest of the tryroid gland), FNAB is not performed b/c the chances of
malignancy in a hot nodule are extremely low.
If the TSH level is normal, FNAB is the next step.
21. Muscle weakness in paraneoplastic syndrome can be due to a variety of processes at different
levels. A good history and physical exam can help localize the process in most cases.
Myopathy association with lung cancer--The muscle involvement may be due to
inflammation, necrosis, fiber atrophy, and antibody-mendiated damage. Proximal muscles are
typically affected, and muscle strength is diminished symmetrically. Reflexes are normal, and
no sensation abnormality is present. Elevated CK level and myopathic electromyograph help
to confirm the dx. The pathologic precess in localized in muscle membrane.
22. The dx of cystic fibrosis is made on the basis of compatible clinical findings with laboratory
comfirmation. Sweat chloride test ( >60 mEq/L) is the gold standard test for the diagnosis of
cystic fibrosis and preferred over direct mutation analysis as more than 1,250 mutations in
CFTR can lead to cystic fibrosis.
23. There is no evidence that regular abdominal ultrasounds can help decrease mortality from
ovarian cancer.
24. The body compensates for chronic hypercarpnia by increasing renal bicarbonate retention.
Typically 0.1 mEq/L of bicarbonate is retained for every 1 mmHg increase in pCO2.
25. Senile purpura is characterized by ecchymoses that occur on elderly pts’ extensor surfaces due
to perivascular connective tissue atrophy. The lesions develop rapidly and resolve over several
days. Typically leaving a brownish discoloration from hemosiderin deposition. Senile purpura
is not a dangerous condition and requires no further investigation.
26. Psudogout cannot be reliably distinguished from gout and septic arthritis based on history and

133
physical alone. It is diagnosed by the presence of rhomboid, positively birefringent crystals on
synovial fluid analysis, and radiographic evidence of chondrocalcinosis.
27. It is important to recognize that oral contraceptives can be a potential cause of hypertension
(an estrogen-mediated increase in the synthesis of angiotensinogen in the liver), and simply
discontinuing its use can correct the problem.
28. Chronic pancreatitis is a serious disease and often leads to severe disability. Surgical
interventions in the case of chronic pancreatitis are indicated in case of chronic pseudocyst,
debilitating pain or to treat other complications. This pt has severe debilitating pain affecting
her work and daily life. She is an ideal candidate for ERCP and stone removal with stent
insertion, which would relieve the pain in 80% of such pts.
29. Turcot’s syndrome refers to an association between brain tumor (primarily medulloblastomas
and gliomas) and FAP (Familiar Adenomatous polypsis) or HNPCC (Hereditary nonpolyposis
colorectal cancer). The majority of FAP-associated brain tumors are medulloblastomas, but
gliomas have also been described. Pts with HNPCC are prone to high-grade gliomas. It is
autosomal recessive and mainly occurs in teens.
Gardner’s syndrome is autosomal dominant where colonic polyps are seen with prominent
extraintestinal lesions. These include desmoid tumors, sebaceous or epidermoid cysts, lipomas,
osteomas (especially of the mandible), supernumerary teeth, gastric polyps, and juvenile
nasopharyngeal angiofibromas.
Peutz-Jeghers syndrome is an autosomal dominant inherited disorder characterized by intestinal
hamatomatous polyps in association with mucocutaneous melanocytic macules.
Multiple hamartoma syndrome (Cowden syndrome) is associated of GI tract hamartomas with
breast Ca, thyroid Ca, and nodular gingival hyperplasia.
Cronkhite-Canada syndrome is association of juvenile-type polyps and ectodermal abnormalities
like alopecia, hyperpigmentation, and nail loss (onycholysis).
30. Malignant otitis externa (MOE) is a potentially serious infection of the external ear that is
usually caused by Pseudomonas aeruginosa. It is typically seen in elderly pts with poorly
controlled diabetes and presents with ear pain, drainage, and granulation tissue within the ear
canal on otoscopic exam. IV ciprofloxacin is the most effective tx.
31. Opioid intoxication does not always present with miosis. Coingestions can lead to normal
pupil size or even mydriasis and certain opioids (meperidine, propoxyphene) do not reliably
cause miosis even when taken alone. As a result, pupil examination is not as reliable as the
recognition of bradypnea in opioid intoxication. Tx—naloxone. Buprenorphine is an
alternative opioid agonist used in long-term mx of opioid addiction. However, it is not helpful
in cases of acute intoxication.
32. In a normal (bell-shaped) distribution:
68% --within 1 SD from the mean
95% --within 2 SD from the mean
99.7% --within 3 SD from the mean
33. Cryptosporidium parvum is a major cause of chronic diarrhea in HIV-infected pts with CD4
counts less than 180 cells/mm3. A modified acid-fast stain showing oocysts in the stool is very
suggestive.
34. Parkinsonism is caused by overactivity of cholinergic neurons and underactivity of
dopaminergic neurons in the substanta nigra. A shuffling gait (ie. The pt appears as if he was

134
chasing his center of gravity) is characteristic of the disease. –hypokinetic gait * Extremely
high yield!! Other syndroms—multiple system atrophy, spinocerebellar atrophies, and
multiple-infarct gait disorder. Wide-based gait is also seen in sensory ataxia, some cerebellar
disorders and muscular dystrophies.
Cerebellar ataxia is usually ipsilateral (ie. The pt tends to fall towards the side of the lesion). Other
features of cerebellar disorders include nystagmus, hypotonia, dysarthria, loss of coordination, and
the inability to perform rapid, alternating movements.
Muscular dystrophy results in a waddling gait 蹒 跚 而 行 owing to weakness of the gluteal
muscles.
Spastic gait is seen with lesion of the upper motor neuron (eg. spinal cord injury or cerebral
palsy). The movements of the affected extremities are slow, stiff, and effortful.
Gait disequilibrium results from disorders of the frontal lobe and multiple sensory systems.
Sensory ataxia is seen with lesions involving the peripheral nerves, dorsal roots or posterior
columns. Loss of proprioception results in a wide-based, high-stepping gait. Romberg’s sign may
be positive (the pt sway on standing with his feet together and eyes closed).
Vestibular ataxia results in an en-bloc gait, with minimal movements of the head during walking.
It is typically a staggering gait 蹒跚 and accompanied by vertigo and nystagmus.
35. Petit mal (absence) seizures are characterized by a sudden cessation of mental activity. An
episode is very short, but may occur repeatedly throughout the day. There are no associated
complex automatisms or tonic-clonic activity. The diagnosis is best confirmed by EEG studies
(activation procedures—hyperventilation, photic stimulation, sleep).
36. ARP (arributable risk percent) represents the excess risk in the exposed population that can be
attributed to the risk factor. ARP= (risk in exposed –risk in unexposed)/risk in exposed. It can
be easily derived from the relative risk: ARP = (RR-1)/RR
37. Fluid replacement is the most important step in the mx of non-ketotic hyperglycemic coma.
5% dextrose is given once blood glucose level has been lowered to 250 mg/dl by insulin
therapy. It prevents the development of cerebral edema. Regular insulin should be
administrated in all cases of nonketonic hyperglycemic coma, but fluid replacement alone can
reduce hyperglycemia significantly.
38. Abdominal ultrasound is the best tool for the initial investigation of gallbladder pathology.
While asymptomatic pts typically do not require tx, laparoscopic cholecystectomy (not open
cholecystectomy) is the tx of choice of those with symptomatic gallbladder disease.
39. Renal artery stenosis is a common cause of resistant hypertension in a pt with advanced
atherosclerosis. Carefully auscultate the periumbilical area of such a pt to reveal continuous
(or systolic with diastolic component) murmur characteristic of renal artery stenosis.
40. Syncope in a young pt with a crescendo-decrescendo murmur at the lower left sternal border
without carotid radiation is most likely due to hypertropic cardiomyopathy. Syncope in
hypertrophic cardiomyopathy is often multifactorial and inappropriately causes vasodilation.
41. Cholesteatomas in children can either be congenital or acquired secondary to chronic middle
ear disease. New-onset hearing loss or chronic ear drainage despite antibiotic therapy are
typically presenting symptoms of cholesteatomas, and granulation tissue and skin debris may
be seen within retraction pockets of the tympanic membrane on otoscopy.
42. Volume resuscitation with normal saline will correct contraction alkalosis (loss of gastric fluid
and activation of the renin-angiotensin-aldosterone system). Hypokalemia shound be treated

135
well.
43. Opioid withdrawal presents with symptoms of nausea, vomiting, abdominal pain, diarrhea,
restlessness, arthralgias and myalgias. Signs of opioid withdrawal on exam can include
increased bowl sounds, mydriasis and piloerection. Unlike ethanol and sedative withdrawal,
opioid withdrawal does not cause seizures. Methadone is proven effective for tx of opioid
withdrawal (not iv morphine).
44. Sjogren syndrome is diagnosed when subjective and objective evidence of dry eyes
(xerophthalmia) and mouth (xerostomia) exists in the presence of either histologic evidence of
lymphocytic infiltration of the salivary glands or serum autoantibodies against SSA (Ro)
and /or SSB (La).

1/10/10
1. Neurocardiogenic, or vasovagal, syncope occurs due to excessive vagal tone. Episodes are
preceded by nausea, diaphoresis, tachycardia, and pallor. Pain, stress, and situations like
medical needles and urination can all precipitate vasovagal syncope.
2. All pt with chronic liver disease should be immunized against HAV and HBV.
3. The correlation coefficient shows the strength and direction (positive, negative) of linear
association between two variables. It does not necessarily imply causality.
4. Lateral epicondylitis (tennis elbow) manifests as pain with supination or extension of the wrist
and point tenderness just distal to the lateral epicondyle.
5. Pts with infectious mononucleosis are at risk for splenic rupture. All pts with splenomegaly
should avoid excessive physical activity, particularly contact sports, until their spleen
regresses in size and is no longer palpable (usually after one to three months).
6. Viridans group streptococci are a frequent cause of subacute bacterial endocarditis (SBE) in
pts with preexisting valvular disease. Staphylococcus aureus is the major cause of acute
infective endocarditis and if often seen in IV drug abusers.
7. Allergic conjunctivitis is an acute hypersensitivity reaction that is caused by enviromental
exposure to allergens. It is characterized by intense itching, hyperemia, tearing, conjunctival
edema and eyelid edema.
8. In a pt with an MI who develops a cold leg, one has to get an ECHO to rule out a thrombus in
the left ventricle. Venous duplex study is an error. The leg is cold and without pulses which
indicates an arterial occlusion. Venous obstruction will present with a swollen leg and the
pulses will be present. In the setting of an MI and administration of heparin, the chances of a
DVT are low.
9. PCP is an important cause of progressive dyspnea and hypoxia in pts with HIV/AIDS.
Hypoxia results from an increased alveolar-arterial oxygen gradient.
10. 100% oxygen (tx of chioce) is an effective and rapid method used to abort an acute attack of
cluster headache. Verapamil is used to prophylaxis of cluster headache. * Extremely high
yield!!
11. The extent of a malignancy determined the most appropriate, timely, and individualized pt
care. CT is a standard diagnostic tool employed in pts with newly diagnosed gastric cancer to
evaluate the extent of the disease. Surgical removal of the affected tissues remains as the
mainstay of therapy. * Extremely high yield!!
12. Always suspect infectious mononucleosis (IM) in a young pt with a sore throat and

136
nonspecific symptoms. Pts with IM can develop autoimmune hemolytic anemia and
thrombocytopenia.
13. The characteristic lab findings of iron deficiency anemia are decreased serum iron level,
decreased percent saturation (serum iron/TIBC), and increased TIBC. Sideroblastic anemia is
characterized by increased serum iron levels and normal TIBC.
14. A thiazide diuretic such as hydrochlorothiazide is the first-line pharmacologic agent for mx of
hypertension in a pt without significant comorbidities. ACEIs are the first-line agent for mx of
hypertension in pts with DM, chronic kidneu disease and CHF. Beta-blockers such as
altenolol are indicated as a first-line antihypertension in pts with angina, low ejection fraction,
or status post-MI.
15. Lacunar strokes are due to microatheroma and lipohyalinosis in small penetrating arteries of
the brain. They often affect the internal capsule and result in pure motor dysfunction. Lacunes
comprise 25% of ischemic strokes. Hypertension and diabetes are two major risk factors.
Because of their small size, lacunes are often not appreciated on non-contrast CT scans
obtained during or shortly after the event.
Todd’s palsy is transient paralysis that occurs during the post-ictal state.
16. Warfarin doding should be adjusted to maintain a goal INR appropriate for the condition
being treated. For pts with idiopathic VTE (venous thromboembolism) or AF, a target INR of
2.0-3.0 provides adequate anticoagulation without an excessive risk of bleeding.
17. Hyperthyroidism is characterized by numerous symptoms including weight change, tremor,
fatigue, anxiety, and weakness. Hyperthyriod myopathy presents as progressive proximal
muscle weakness. It is slowly progressive but reverses quickly when the underlying
hyperthyroidism is treated. Inflammatory muscle diseases include polymyositis,
dermatomyositis, and inclusion body myositis. Tremor is not seen in these conditions.
18. CHF causes decreased cardiac output, which in turn lead to increased hydrostatic pressue in
the pulmonary vasculature. This results in transudation of fluid into the air spaces of the lungs,
which can lead to pulmonary edema and pleural effusion. A pleural effusion presents with
absence of lung sounds and dullness to percussion at the involved lung bases.
19. Most pts will require pharmacotherapy for blood pressure control, but life style changes can
make a difference. Weight control is the most important intervention, followed by physical
activity, salt restriction, and moderation of alcohol consumption. Every 10 kg reduction in
weight can decrease the systolic blood pressure by 5-20 mmHg. For all pts, the goal BMI is
18.5-24.9 kg/m2.
20. COPD, in general, is characterized by reduced FEV1 and repid decline in FEV1. FEV1/FVC
is decreased in COPD as opposed to normal in restrictive lung disease. DLCO is decreased in
emphysema and normal in chronic bronchitis.
21. Paroxysmal, lightning-like pain on the face is usually due to trigeminal neuralgia.
Carbamazepine is the drug of choice.
Carotidynia is a neurological condition caused by inflammation of the carotids and the vagus. The
pain is sharp and localized to the carotid artery distribution in the neck. Usually, the intensity of
pain is much less. The dx is clinical, although an MRI is sometimes required.
Burning mouth syndrome is a rare cause of facial pain. It is caused by a virus and the individual
has reddened mucosa and has significant pain. The condition is aggravated by dryness. It is treated
by supportive care.

137
22. Suspect hepato-renal syndrome in a pt with severe liver disease, hypotension, hyponatremia,
azotemia, and oliguria with normal urinalysis. Take measures to withhold all possible
precipitating factors. Initial mx includes careful volume loading and discontinuation of
furosemide and spironolactone.
23. Acute, symptomatic hyponatremia is a medical emergency requiring a prompt increase in the
serum sodium concentration (3% saline). In contrast, chronic hyponatremia requires slow
correction of the serum sodium concentration (<0.5 mEq/L/h) b/c the cells of the brain have
adjusted to the hyponatremia milieu and rapid increases in the serum sodium can cause
osmotic demyelination syndrome.
24. Staphylococcus aureus is the most common cause of osteomyelitis in both infants and
children.
Staphylococcus epidermidis—prosthetic devices
Salmonella—sickle cell anemia
G- rods (Klebsiella and psudomonas)—history of UTI or urinary tract instrumentation
25. Calcium gluconate is the most appropriate initial tx for hyperkalemic pts with significant
EKG abnormalities.
26. Firm, solitary LN are highly suspicious for LN metastases, particularly in older pts with a
significant smoking history. Non-tender, solitary nodes in the head and neck are concerning
for squamous cell carcinoma.
27. A normal or increased FEV1/FVC ratio with a low VC is seen in restrictive lung disease. The
Dlco is decreased with interstitial lung disease. Neuromuscular disorders are not associated
with a decreased Dlco.
28. Studies have shown that the risk for sepsis is present up to 30 years and probably longer after
spenectomy, Current recommendations state that pts should receive anti-pneumococcal,
harmophilus, and meningococcal vaccines several weeks before the operation, and daily oral
penicillin prophylaxis for 3 -5 years following splenectomy. * Extremely important.
29. Gallstones and alcoholism are the two most common causes of acute pancreatitis in the US.
Abdominal ultrasound should be used to search for gallstones in all pts experiencing a first
attack of acute pancreatitis. Abdominal CT scan is used to confirm the dx of acute
pancreatitis.
30. Prophylaxis against Mycobacterium avium complex with azithromycin is given to HIV-
infected pts when their CD4 count is less than 50/mm3. Dapsone is an alternative agent for
PCP prophylaxis. Clarithromycin, in combination with ethambutol, is used as tx rather than
prophylaxis for Mycobacterium avium complex infection.
31. Amebic abcesses within the liver contain debris charaterized as “anchovy paste” in
appearance, though cyst aspiration is not typically recommended b/c of the associated risk. Tx
is with oral metronidazole.
32. Rule out bladder cancer (urinary cytology and cystoscopy) in all elderly pts with irritative
voiding symptoms and have negative urine culture.
33. Surgical tx of hyperparathyroidism with parathyroidectomy should be reserved for pts who
are less than 50 years old, those with significantly worsening renal function, those with
osteoporosis, and those with serum calcium greater the 1mg/dl over the upper limit of normal.
34. Psoriatic arthritis occurs in 5-30% of psoriasis pts. The classic presentation involves the distal
interphalangeal joints. Morning stiffness, deformity, dactylitis (sausage digit) and nail

138
involvement are common. NSAIDs, anti-TNF agents and methotrexate are used for tx of
psoriatic arthritis; systemic corticosteroids are relatively C/I.
35. Spontaneous rupture of the esophagus (Boerhaave’s syndrome) is a rare and dangerous
condition. It typically occurs after severe retching and presents as chest or epigastric pain,
nausea, vomiting, fever, and unilateral pleural effusion (have a high amylase concentration,
low pH, and may contain particles of food).
36. Colicky flank pain with radiation to the grooin indicates renal colic. 75% -90% of the kidney
stones are composed by calcium oxalate. Small bowel disease, surgical resection or chronic
diarrhea can lead to malabsorption of fatty acids and bile salts; this in turn predisposes to the
formation calcium oxalate stone. Calcium phosphate stones are common in primary
hyperparathyroidism and renal tubular acidosis.
37. Sarcoid can present in a black female with bilateral hilar adenopathy. Recognize the
association with erythema nodosum. Biopsy typically shows non-caseating granulomas
(inflammatory). Serum Ca and ACE levels are elevated. Steroids are the tx of choice in
symptomatic pts.
38. Warfarin is an anticoagulant agent that functions by inhibiting the synthesis of vitK-dependent
factors II, VII, IX, and X (prothrombin), protein C and protein S.
39. New clubbing in pts with COPD often indicates the development of lung cancer.
40. Tamoxifen has a mixed agonist and antagonist activity on estrogen receptors. Estrogenenic
effects of tamoxifen increase the risk of endometrial cancer and venous thrombosis.
41. Thiazide diuretics have some unfavorable metabolic side effects including hyperglycemia,
increased LDL cholesterol, and plasma triglycerides. Electrolyte abnormalities that can be
induced by thiazide diuretisc include hyponatremia, hypokalemia, and hypercalcemia. *
Extremely high yield!!
42. In diabetic pts, the pathogenic mechanism of osteomyelitis adjacent to a foot ulcer is
continguous spread of infection.
43. GERD predisposes to Barret’s esophatus, erosive seophagitis and peptic stricture formation.
Peptic strictures cause symmetric and circumferential narrowing of the involved esophagus
and dysphagia to solids, but not weight loss. Other causes of peptic strictures include
radiation, scleroderma, and caustic ingestions.
44. Germ cell tumors typically affect young pts and display aggressive biologic behavior.
Nonsemnomatous germ cell tumors typically produce both AFP and beta-HCG as tumor
markers.

1/11/10
1. In a post-MI hypertensive pt, beta-blockers and ACEIs are perferred over diuretics and CCBs.
Beta blockers (esp cardio selective, matoprolol, better than propranolol) decrease myocardial
oxygen demand by reducing heart rate and contractility. ACEIs are indecated when EF is
decreased. right ventricle infarction, leading to right sided heat failure. Hypotension, jugular
venous distention and clear lung fields should raise suspicion for right ventriclar infarction.
Tx—IV fluids and avoidance of preload reducing medications such as nitrates and diuretics.
2. In high-risk pts presenting with fevers, chills, and evidence of septic emboli or abscesses (CT
scan of the abdomen reveals a fliud collection within the spleen), infections endocarditis must
be high on the differentia.

139
3. Gonorrhea is a common organism which causes sexually tranmitted disease. It is also a cause
of pharyngitis, which is acquired during oral sex.
4. Lactic acidosis resulting from poor oxygen delivery to the tissues is known as type A lactic
acidosis. Causes include CO poisoning (binds hemoglobin and a left shift of the oxygen
disscociation curve) and circulating failure (shock).
5. Lowering the cut-off point of a diagnostic test will increase its sensitivity.
6. Suspect primary hyperaldosteronism in a young pt with hypertension, muscle weakness and
numbness; the most specific lab value for pts with primary hyperaldosteronism is high
aldosterone/renin ratio, indicating autonomic aldosterone secretion.
7. PAO2 = FiO2 (760-47)-PaCO2/0.8 PAO2 = (0.21* 713)-PaCO2/0.8 = 150- PaCO2/0.8
PaCO2 normal range 33-45mmHg PAO2 93.75-108.75
A normal A-a gradient is <15 in a young person. Values increase with age, but an A-a gradient
>30 is considered elevated regardless of age.
Hypoxia types PaCO2 A-a Correct with
gradient 100% oxygen
Hypoventilation ↑ N Excessive anesthesia
Low inspirated oxygen N N
Shunting N ↑ No Pulmonary edema, pneumonia
and vascular shunt
V/Q mismatch N ↑ Yes PE, excerbation of bronchial
asthma

8. The classic triad of carcinoid syndrome includes flushing, diarrhea, and wheezing. They are
exhibited by pts with carcinoid tumors. Such carcinoid tumors may occur in the small
intestine, colon, bronchial tubes, or appendix. Surgery with complete removal of the tumor
tissue is usually the first-line tx. The mainstay of tx for advanced carcinoid tumors that cannot
be removed surgically is octreotide injection. Pathognomonic plaque-like deposits of fibrous
tissue occur most commonly on the endocardium on the right side of the heart.
9. A pt with asthma-like symptoms that occur only at night and not during the day may actually
be suffering from GERD. Such pts should adopt anti-GERD lifestyle modifications and
reveive a trial of a PPI (eg. pantoprazole).
10. Bright red, firm, friable, exophytic nodules in an HIV infected pt are most likely bacillary
angiomatosis (picture). It is caused by Bartonella, a G- bacillus. Oral erythormycin is the
antibiotic of chioce.
11. It is important to be able to identify the signs and symptoms of anticholinergic toxicity and to
know common possible causes. The classic findings are dry skin, dry mouth, constipation,
urinary retention, flusing, vision change, and confusion. Medications for Parkinson’s disease –
trihexyphenidyl, benzathopine (excess) can cause anticholingeric symptoms.
12. AF fibrillation presents on EKG as an irregularly irregular, narrow QRS complex techycardia
that lacks P waves. The causes of acute AF are numerous and include cardiac, pulmonary,
metabolic (hyperthyroidism), and drug-related (alcohol, cocaine, amphetamines, theophylline)
etiologies.
13. Suspect vit B12 deficiency in strict vegetarians with anemia and neurologic complications. In
such pts, folic acid supplementation may correct the anemia, but not the neurologic
complications.

140
14. Pleural effusions are classified as transudative or exudative based on Light’s criteria. Pts with
tuberculosis may develop exudative pleural effisions due to increased capillary permeablility.
This fluid characteristically has a high adenosine deaminase concentration.
15. Calcium oxalate crystals (rectangular, envelope-shaped crystals) are seen in pts with ethylene
glycol (anti-freeze) poisoning. Ethylene glycol, methanol and ethanol intoxication cause
metabolic acidosis with both an anion gap and an osmolar gap.
16. Several randomized controlled clinical trials have demonstrated the beneficil effect of ACE
inhitibors on slowing the progression of diabetic nephropathy. The target limit of HBA1c is
7%. * the use of ACEIs in pts with DM is a very high-yield topic in the USMLE.
17. Ischemic damage in the setting of MI may lead to diastrolic dysfunction and a stiffened left
ventricle, resulting in an atrial gallop (S4).
18. Severe pain in a pt with a mild urinary obstruction, such as BPH, may cause urinary retention
due to inability to Valsalva. Classic symptoms of disk prolapse include unilateral redicular
pain in a dermatomal distribution. Back tenderness due to spasm of the paraspinous muscles is
common, and symptoms are usually worsened with straight leg testing.
19. Barium swallow is the initial test of choice for all pts with dysphagia. Subsequent endoscopy
is dependent on the barium swallow findings (ie. The outline of the pts current esophageal
anatomy). Motility studies are indicated once organic causes have been rule out by upper
endoscopy. If Zenker’s diverticulum is found on esophagography, endoscopy should be
avoided.
20. Diastolic and continous murmur as well as loud systolic murmurs revealed on cardiac
auscultation should always be investigated using transthoracic Doppler echocardiography.
Midsystolic soft murmurs (grade I-II/IV) in an asymptomatic young pt are usually benign and
need no further work-up.
21. Controlling the rhythm or rate in pts with prolonged tachysystolic AF usually improves the
LV function significantly, sometimes even dramatically.
22. Pts who abuse IV drugs are at increased risk for subacute bacterial endocarditis (SBE) of both
right and left heart. If the tricuspid or pulmonary valve is affected, SBE can present with
embolic phenomena to the lung. SBE must be in the defferential for all IV drug abusers
presenting with fever and malaise. Tricuspid regurgitation is common in individuala with
endocarditis of the tricuspid valve, and cause a systolic murmur that increase on inspiration.
Aortic regurgitation produces a diastolic murmur that is best heard when the pts sits up.
Paradoxical splitting of the second heart sound (S2) occurs in pts with left bundle branch
block (LBBB) due to delayed closure of the aortic valve.
23. An isolated liver mass is much more likely to be the result of metastatic disease than a
primary malignancy of the liver. Primary tumors of the GI tract, lung and breast are the most
common diseases causing liver metastases. Liver metastases are generally asymptomatic; pts
tend to present instead with symptoms related to their primary malignancy.
24. Tearing pain with radiation to the back and a difference in BP of greater than 30mmHg
between the arms are important clinical clue for aortic dissection. Transesophageal
echocardiography or CT with contrast are the two diagnostic studies of choice for suspected
aortic dissection.
25. When an acute pulmonary embolus occurs with hemodynamic compromise, the best tx is
fibrinolytic therapy. But if the pt has C/I (a fresh postoperative pt) to the use of fibrinolytic

141
therapy, an embolectomy is the tx of choice.
26. Type 2 diabetics are prone to the development of a hyperosmolar hyperglycemic state without
ketoacidosis. Decreased consciousness is the most common symptom. Other reversible
neurologic abnormalities, such as blurred vision, can also be caused be hyperglycemic
hyperosmolarity (a myopic increase in lens thickness and intraocular hypotension sencondary
to hyperosmolarity).
27. Inhaled anti-muscarinic agents such ipratropium are the mainstay of symptom mx in COPD.
These anti-cholinergic medications may be combined with short-acting beta-adrenergic
agonists for greater symptom relief.
28. The Rumack-Matthew nomogram provides the likelihood of hepatoxic effects from
acetaminophen overdose and the need for N-acetylcysteine. The first data point on the curve is
at 4 hours. The decision of whether or not to administer the antidote can be made after 4
hours, based on the actaminophen level. As long as it is administered within 8 hours of
ingestion, the outcome will not be adversely affected.
29. Cusing’s syndrome is an important cause of secondary hypertension. In addition to blood
pressure elevation, high systemic cortisol concentrations cause hyperglycemia, hypokalemia,
proximal muscle weakness, central adiposity, thinning of the skin, weight gain, and
psychiatric problems (eg. sleep disturbances, depression, and psychosis).
30. Ondensetron is the drug of choice for chemotherapy-induced emesis. Methoclopramide and
prochlorperazine are classic anti-emetic agents that can be given either alone or in
combination with ondansetron; however, these are not the best drugs for chemotherapy-
induced emesis.
31. Ultrasound of the kidney, ureter and bladder should be done in pts of BPH whose serum
creatinine is elevated. All pts with irritative or obstructive voiding symptoms should have
their urinalysis and serum creatinine done; as such symptoms are not always due to benign
prostatic hyperplasia. Such symptoms may be produced by multiple causes like BPH, bladder
cancer, prostate cancer, urethral stricture, neurogenic bladder and urinary tract infections.
Urinalysis helps to rule out urinary tract infection and serum creatinine detects renal
insufficiency that might be due to bladder outlet obstruction.
32. Hypoxemia-induced increases in erythropoietin production are responsible for the
polycythemia observed in obstructive sleep apnea. Tx of the OSA cause causes the
polycythemia to improve.
33. Suspect progressive multifocal leukoencephalopathy (PML) in an HIV-infected pt with focal
neurological signs and multiple non-enhancing lesions with no mass effect on the CT scan. It
is caused by JC virus, a human polyomavirus. PML predominantly involves the cortical white
matter, but the brain stem and cerebellum may also be involved. The most common presenting
symptoms are hemiparesis and disturbances in speech, vision and gait. Cranial nerve deficits
may occasionally develop. Classical MRI findings in PML consist of multiple demyelinating,
non-enhancing lesions with no mass effects. There is no effective tx for PML, and the mean
duration of survival from the time of dx six 6 months.
In pts with AIDS dementia complex imaging studies reveal cortical and subcortical atrophy and
secondary ventricular enlargement.
34. PCP is likely if an HIV pt has a non-productive cough, exertional dyspnea, fever, severe
hypoxia, bilateral interstitial infiltrates on CXR, and a normal white count. PCP usually

142
affects HIV pts when wheir CD4 counts falls below 200/ml. TMP-SMX is the drug of choice.
Steroids have been shown to decrease mortality in pts with severe PCP infection. Indications
for steroid use in PCP include PaO2 <70mmHg or A-a gradient >35mmHg.
35. The most common site of hypertensive hemorrhages in the putamen (35%). The internal
capsule that lies adjacent to the putamen is almost always involved, thereby leading to
hemiparesis.
36. Dry and rough skin with horny plates over the extensor surfaces of the limbs is a hallmark of
ichthyosis (Lizard skin).
37. A case-control study is used to compare the exposure of people with the disease (cases) to the
exposure of the people without the disease (controls). The main measure of association is the
exposure odd ratio. Prevalence odds ratio is calculated in cross-sectional studies to compare
the prevalence of a disease between different populations.
38. Always consider ACEI therapy as a potential cause of chronic cough. Simple discontinuation
of the drug should precede any diagnostic testing in pts with chronic dry cough who are taking
an ACEI. * Extremely high yield!!
39. Delirium is an acute confusional state due to organic illness which is commonly superimposed
upon dementia in the elderly. Urinary tract sepsis and metabolic disturbance are common
precipitants in older pts. The initial work-up of delirium should include urinalysis and
measurement of serum electrolytes.
40. Asymptomatic diverticulosis needs only dietary modifications in the form of high fiber intake.
41. Acanthosis nigricans presents with thickening and hyperpigmentation of skin of the flexural
areas. It has a classic ‘velvety’ texture. It is commonly associated with insulin resistant states
(DM, acromegaly, abesity and others) as well as GI malignancies (middle-aged or elderly).
42. Methotrexate is a DMARD. It works by inhibiting dihydrofolate reductase. Macrocytic
anemia is a common side effect. Other side effect include: nausea, stomatitis, rash,
hepatotoxicity, interstitial lung disease, alopecia and fever.
43. The most likely culprit 犯 人 lesion for acute inferior wall myocardial infarction is right
coronary artery occlusion, especially if it is complicated by right ventricular infarction
(hypotension) and bradycardia (SA node). * Extremely high yield!!
44. The tx of acute cholangitis includes provision of supportive care and broad-spectrum
antibiotics. Pts who do not respond to this tx regimen should undergo biliary drainage with
ERCP.

1/12/10
1. This pt has postsperative cholestasis, a benign condition that often develops after a major
surgery characterized by hypotension, extensive blood loss into tissue, and massive blood
replacement. The jaundice is thought to develop secondary to 3 factors: increased pigment
load (caused by the transfusion), decreased liver functionality (caused by hypotension), and
decreased renal bilirubin excretion (caused by tubular necrosis). Generally, jaundice is evident
by the second or third day after a prolonged surgery, with the bilirubin levels peaking at 10-40
mg/dL by the 10th postoperative day. Although alkaline phosphatase levels can also be
markedly elevated, the AST and ALT levels are typically normal or only mildly elevated.
2. The renin-angiotensin-aldosterone system is integrally involved in the pathophysiolocy and tx
of hypertension. Numerous antihypertensives perturb this system and understaning how

143
different agents affect it important.
3. Streptococcus bovis endocarditis is associated with colorectal cancer. Colonoscopy should be
pursued for futher evaluation.
4. Atherothrombotic strokes occur at rest (unable to wake up in the morning) and have a gradual
onset. Often, the pt experience successive strokes with increasing frequency. Atherosclerotic
risk factors are frequently present in such pts.
5. Epidemiological difference (eg. incubation peroid, route of transmission, risk factors) offer
some of the best means of identifying the virus responsible for a pt’s hepatitis. Confirmation
can be established with serological testing.
6. Nonketotic hyperosmolar syndrome (NKHS) occurs in type 2 DM because the level of insulin
in these pts is sufficient to prevent ketosis, but not hyperglycemia. In most cases, severe
hyperglycemia develops, thereby resulting in glycosuria and severe dehydration. The severe
hyperosmolality is also responsible for lethargy, weakness, altered mental status, focal
neurological deficits and eventual coma. Left arm weakness may make you think of a
cerebrovascular event, but keep in your mind that focal neurological deficits are common in
NKHS. A head CT or MRI may be considered afterwards to rule out a cerebrovascular
process, especially if the blood glucose turns out to be normal.
7. This pt’s presentation and radiologic feature (ie. Symmetrical ‘loose zones’ or
pseudofractures, blurring of the spine) are classic for osteomalacia. Low or low-normal serum
calcium, low serum phosphate, increased serum parathyroid hormone, and low plasma 25-OH
vit D levels are characteristic. Vit D deficiency leads to decreased intestinal calcium and
phosphorus absorption, thereby resulting to hypocalcemia and hypophosphatemia.
Hypocalcemia then stimulates the parathyroid glands. The resultant secondary
hyperthyroidism brings the serum calcium levels to normal or near-normal (esp. in the early
stage), and increases urinary phosphate excretion, thereby worsening the hypophosphatemia.
Vit D deficiency thus causes more marked hypophosphatemia than hypocalcemia, esp in the
early stage. Normal serum calcium, phosphate and parathyroid hormone levels are seen in pts
with osteoporosis and Paget’s disease. The serum alkaline phosphate level is markedly
elevated in Paget’s disease. Hypercalcemia can occur in pts with moderately severe Paget’s
disease following immobilization.
8. Whenever an infective endocarditis is suspected empiric antibiotics should be administered
after drawing blood culture. TEE should be performed after.
9. Symtomatic sinus bradycardia should be treated with iv atropine, followed by transcutaneous
pacing. Medications that slow the HR should be replaced with alternative therapeutic
regimens. If the bradycardia does not resolve, a permanent pacemaker may be necessary.
10. Second degree heart block is identified by prolongation of the PR interval with occasional
dropped beats. In Mobitz type I, the PR interval gradually lengthens until a beat is dropped. In
type II, the PR intervals are of constant length before the dropped beat. The prognosis is better
for Mobitz type I, which is the result of AV node dysfunction (impaired conduction).
Ventricular preexcitation usually results in premature ventricular complexes (PVCs). Impaired
AS node automaticity (SSS) most often results from fibrosis of the sinus node or disease of
the SA nodal artery. Pts may present with bradycardia, lightheadedness, or syncope. On EKG,
this often appears as tachycardia-bradycardia syndrome (bursts of atrial tachyarrhythmia
followed by bradycardia). Atrial reentry leads to atrial tachycardia of abrupt onset and

144
termination. Bundle branch block occurs below the AV node and impedes ventricular
depolarization. The result is a prolonged QRS complex.
11. Hepatic encephalopathy is a central nervous system complication of liver failure secondary to
accumulation of ammonia in blood because of inability of liver to detoxify ammonia into
urea. It is characterized by reversal of sleep cycle, asterixis, progressive coma, and
characteristic delta waves on EEG.
12. Neuroleptic Malignant Syndrome (NMS) typically presents with fever, muscle rigidity,
autonomic instability and mental status change. Serum CK is often elevated. Based on clinical
evidence, dantrolene, a muscle relaxant, is the most comon drug used to the condition,
followed by bromocriptine (a dopamine) agonist and amentadine (an antiviral drug with
dopaminergic properties). Cessation of all dopaminergic antagonists.
13. Sympathetic ophthalmia is also known as “spared eye injury”. It is characterized by an
immune-mediated inflammation of one eye (the sympathetic eye) after a penetrating injury to
the other eye. The typical manifestation is anterior uveitis, but panuveitis, paplillary edema,
and blindness may develop. The pathophysiological mechanism is believed to be the
uncovering of “hidden” antigens. Some antigens contained within the eye are protected from
immunologic recognition by natural barriers. Breaking these barriers results in the uncovering
of “hidden” antigens. An immune response against these antigens can involve autoantibodies
as well as a cell-meidated reaction.
14. Septic arthritis of the hand is a rapidly destructive infection that causes intense pain, swelling,
erythema, and tenderness with active and passive movement. It is diagnosis via joint
aspiration. Tx is with drainage and antibiotics.
Osteomyelitis may complicated puncture wounds, but would be unlikely to cause this pattern of
joint pain and swelling.
Flexor tenosynovitis is inflammation of the flexor tendon sheath on the hand. Pts present with a
finger held in slight flexsion, fusiform swelling, tenderness along the flexor sheath, and pain with
passive extension. In contrast, this pt had pain with flexion.
A midpalmar space infeciton would affect the potential space between the middle, ring and small
finger flexor tendons and the volar interosseous muscles. This condition typically results from
direct trauma.
Paronychia is characterized by pain, swelling, and erythema at the nail folds. It is the most
common kind of hand infection.
15. CT of the chest should be done to look for a thymoma in all newly-dignosed myasthemia
gravis pts.
16. ACEIs at the most common cause of acquired angioedema, which typically begins within one
week of starting the drug. Other adverse effects of ACEIs are cough, hyperkalemia, and
precipitation of acute renal failure in pts with bilateral renal artery stenosis. Pts present with
edema in the face, mouth, lips, tongue, glottis and larynx. Laryngeal edema can cause airway
obstruction and be life threatening. Angioedema occurs due to the pro-inflammatory action of
bradykinin, which promotes edema, inflammation and the sensation of pain.Angiotensin
converting enzyme (ACE) is also known as kininase; it functions to degrade bradykinin. The
first step in mx of angioedema is to check for airway compromise and vasomotor instability,
which require subcutaneous epinephrine administration if present. If airway obstruction fails
to respond to epinephrine, an emergency tracheostomy is done. The ACEIs should be stopped

145
immediately.
17. The lupus anticoagulant, or anti-phospholipid antibody, is a prothrombotic immunoglobulin
that causes a spuriously prolonged PTT. It is an IgM or IgG immunoglobulin that prolongs
PTT by binding the phospholipids used in this assay. It needs to be considered in pts with
clotting disorders and prolonged PTT. Hence, it is strictly a lab artifact and does not actually
promote bleeding. The Russel viper venom test is designed specifically to test for the lupus
anticoagulant and will be prolonged in the disease. With the lupus anticoagulant: 1) The PT
will be normal or slightly prolonged. 2) VWF, bleeding time, platelet count will be normal. 3)
D-dimer will be normal or high. D-dimer is typically elevated in the presence of blood clots.
18. Any HIV-positive pt with bloody diarrhea and a normal stool examination should have a
coloscopy with biopsy to look for CMV colitis. CMV colitis is characterized by bloody
diarrhea with abdominal pain. Colonoscopy shows multiple ulcers and mucosal erosions.
Biopsy demonstrates the presence of large cells with eosinophilic intranuclear and basophilic
intracytoplasmic inclusions (“owl” eye “effect”). The tx of choice is ganciclovir. Foscarnet is
used in case of ganciclovir failure or intolerance.
19. HIV often presents with non-specific symptoms such as weight loss, malaise, and neurologic
changes. Physicians must maintain a high index of suspicion for HIV infection, especially in
pts with known risk factors (drug use).
20. Most cases of active TB in US occur in people who have immigrated to his country from
endemic geographic regions.
21. Pts with Wolf-Parkonson-White syndrome who develop atrial fibrillation with a rapid
ventricular rate should be treated with cardioversion or antiarrhythmics like procainamide. AV
nodal blockers like beta-blockers, CCBs, digoxin and adenosine should be avoided b/c they
can cause increased conductance through the accessory pathway (conducts depolarization
directly from the atria to the ventricles without traversing the AV node.).
22. Neurocysticercosis is the most common parasitic infection of the brain. It is mose prevalent in
rural areas with poorer sanitary conditions and where pigs are raised. Autopsy shows multiple
fluid-filled cysts in the brain parechyma. Creutzfeldt-Jacob disease and Kuru are prion
diseases that cause spongiform encephalopathy, in which intracytoplasmic vacuoles are
evident diffusely throughout the gray matter on microscopy.
23. Campylobacter is the MCC of bloody diarrhea in the US (uncooked poultry-- chicken burger).
The presence of simutaneous severe abdominal pain is a helpful clue. Shigella is very
common cause of diarrhea in the US, and is the second MCC of food-borne illness. Diarrhea
due to Shigella is often bloody and usually occurs in daycare centers or other institutional
settings. E coli—uncooked hamburger. Yersinia enterocolitica—uncooked pork
24. Atopic dermatitis in infancy presents with pruritus and skin lesions typically distributed
symmetrically over the face, scalp, chest and extensor surfaces of the extremities. The diaper
region is typically spared. Lesions usually begin with prutitus alone and evolve to
erythematous excoriated 表 皮 脱 落 papules and plaques that may weep and become
secondarily imperiginized. Atopic dermatitis is the result of decreased skin barrier function
due to improper synthesis of components of the epidermal cornified cell envelop. This allows
allergens ready access to the deeper levels of the epidemis where they may generate the
immune response characteristic of atopic dermatitis. Tx is with improvement of skin barrier
function through the use of mild cleansers and thick, bland emollients and in addition to mild

146
topical anti-inflammatory ointments.
25. In evaluation the asymptomatic evaluation of aminotransferases, the first step is to take a
thorough history to rule out the more common hepatitis risk factors (eg. alcohol or drug use,
travel outside of the country, blood transfusions, high-risk sexual practices).
26. Dapsone is an effective tx for dermatitis herpetiformis (commly occurs in association with
celiac sprue). 1% lindane solution is used as a topical agent in the tx of scabies. Azithioprine
—blistering autoimmune diseases, such as pemphigus vulgaris.
27. The differential dx for chest pain is broad and varies dignificantly depending upon a pt’s age
and risk factors. In young women, anxiety and other psychosocial factors are s common cause
of chest pain (next step mx--reassurance).
28. Lynch syndrome II is distinctly associated with a high risk of extracolonic tumors, the most
common of which is endometrial carcinoma, which develops in up to 43% of females in
affected families. HNPCC subgroups: 1) hereditary site specific colon cancer (Lynch
syndrome I); 2) cancer family syndrome or (Lynch syndrome II).
29. Guillain-Barre syndrome is characterized by ascending paralysis, antecedent history of
infection, and CSF finding of albumino-cytologic dissociation. The tx includes IV
immunoglobulin and plasmapheresis (not corticosteroids).
30. Aortic regurgitation causes widening of the pulse pressure, which can be felt as a “water
hammer” pulse. Lying down and turning to the left brings the heart close to the chest wall and
can make the pt more aware of the forceful heartbeat.
31. Condyloma lata and bilatery symmetrical maculopapular rash involving the entire trunk and
extremities are characteristic of second stage of syphilis. Remember the rash is present on
palms and soles. Serological tests are positive in secondary syphilis.
32. Retinal detachment usually presents with a sudden onset of photopsia and floaters. The most
classic description is that of “a curtain down over my eyes”.
33. Hemodynamically stable supraventricular tachycardia should be treated with vagal maneuvers
followed by adenosine and AV nodal blockers. Unstable pts with supraventricular tachycardia
should undergo DC cardioversion.
34. Constrictive pericarditis and restrictive cardiomyopathy can be difficult to distinguish. A low
voltage EKG can occur in either condition. The pt’s other issue are key to making the dx.
New-onset diabetes mellitus and liver test abnormalities along with congestive heart failure
suggest hemochromatosis, which can cause restrictive cardiomyopathy. Iron studies and an
echocardiogram would help confirm this dx.
35. Laxative abuse is characterized by very frequent, watery, nocturnal diarrhea. The diagnosis
can be confirmed with the characteristic biopsy finding of dark brown discoloration of the
colon with lymph follicles shining through as pale patches (melanosis coli).
36. Always suspect lacunar stroke if a pt with a limited neurologic deficit. The typical lacunar
stroke scenarios are pure motor stroke, pure sensory stroke (thalamus), ataxia-hemiparesis,
and dysarthria clumsy hand syndrome. The principle cause of lacunar stroke is hypertension.
Memorize 4 common lacunar syndromes:
Syndrome Pathology Presentation
Pure motor Lacunar infarction in the Unilateral motor deficit (face, arm, and, to a
hemiparesis posterior lime of the lesser extent, leg); mild dysarthria; NO
internal capsule sensory, visual or higher cortical dysfunction
Pure sensory Stroke in the Unilateral numbness, paresthesias, and

147
stroke ventroposterolateral hemisensory deficit involving the face, arm,
nucleus of the thalamus trunk, and leg
Ataxic-hemiparesis Lacunar infarction in the Weakness that is more prominent in the lower
posterior limb of the extremity, along with ipsilateral arm and leg
internal capsule incoorfination
Dysarthria-clumsy Lacunar stroke at the base Hand weakness, mild motor aphasia, NO
hand syndrome of pontis sensory abnormalities

37. UTI is common in diabetes, the elderly, and postoperative pts. Elderly pts with UTI may
present with confusion and an absent of geintourinary complaints. The finding on UA, taken
in the context of the history and physical exam, help to confirm UTI dx.
Trimethoprim/sulfamethoxazile is appropriate antibiotic therapy for UTI.
38. Chronic pancreatitis is an inflammatory condition characterized by chronic abdominal pain
and pancreatic insufficiency. A fecal elastase study is the most sensitive and specific test to
diagnose pancreatic exocrine failure.
39. The S4 is a low frequency sound heard at the end of diastole just before S1 that is commonly
associated with left ventricular hypertrophy from prolonged hypertension.
40. Migratory thrombophlebitis and atypical venous thromboses are suggestive for chronic DIC,
most likely due to some visceral malignancy (lung, pancreas, stomach and prostate).
41. Zollinger-Ellison syndrome (gastrinoma) is associated with MEN I (primary
hyperparathyroidism, pituitary tumors and enteropancreatic tumors).
42. Paracentesis can be used for both dx and therapeutic purpose in pts with ascites.
43. Corticosteroid-induced avascular necrosis of the femoral head usually presents as progressive
hip or groin pain without restriction of motion range and normal radiograph on early stages.
MRI is gold standard for the dx of avascular necrosis of hip.
44. In elderly pts with multiple medications, it is very important ot regularly assess their general
well-being and current medication list, as well as to promote pt adherence to the appropriate
regimen. Hemorrhages are seen as hyperdense areas on CT scan, while infarcts
charateristically have hypodense parenchymal areas on CT scan.

1/13/10
1. Diabetic cystopathy manifests as overflow incontinience. The pathology involves weak
detrusor contraction secondary to autonomic neutopathy. The initial treatment usually
involves strict voluntary urinary voiding schedule and bethanechol; if there is no response,
intermittent catheterization is recommended. Oxybutynin is an antispasmodic agent used for
urge bladder incontinence. Tamsulosin is an alpha-1 blocker that is used for BPH.
2. Suspect squamous cell carcinoma of the lungs in a pt with a significant smoking history (ie.
More than 45 years), hypercalcemia and a hilar mass. Small cell carcinoma of the lung usually
causes papaneoplastic syndrome such as ACTH production and SIADH. Adenocarcinoma is
usually a peripheral lesion that is usually not associated with hypercalcemia. It is typically
associated with hypertrophic pulmonary osteoarthropathy.
3. Lifestyle modification should be the first-line intervention for newly diagnosted stage 1
hypertension. Reduction of alcohol and sodium intake, weight loss, and aerobic exercise can
all decrease BP. Quitting smoking will decrease a pt’s risk of cardiovascular disease, but does
not improve BP itself.

148
4. The differential dx of normotensive pts with hypokalemia and metabolic alkalosis include: 1.
Diuretic use; 2) Surreptitious 秘 密 的 vomiting; 3) Bartter’s syndrome; 4) Gitelman’s
syndrome. Classic Bartter’s syndrome ussually presents early in life as polyuria, polydipsia,
and growth and mental retardation; howwver, such presentation can occur much later. The
underlying pathology is defective sodium and chloride reabsorption in the thick ascending
limb of the loop of Henle, thereby resultig in hypovolemia and consequent activation of the
renin-angiotensin aldosterone system (RAAS). Activated RAAS then causes an increase in
potassium and hydrogen ion secretion, which eventually leads to hypokalemia and metabolic
alkalosis. Pt with Batter’s syndrome have hypokalemia, urinary chloride level above 20mEq/L
(most often above 40 mEq/L), metabolic alkalosis, and normal BP.
Primary hyperaldosteronism and renin-secreting tumors are characterized by hypertension,
metabolic alkalossi and hypokalemia. Measurement of the plasma renin activity (PRA) and
plasma aldosterone levels (PA) may be used to distinguish between these two diagnoses. In
primary hyperadosteronism, PRA is supressed and PA is elevated; whereas, in renin-secreting
tumors, both PRA and PA are elevated.
The MCC of hypokalemia seen in clinical practice is the use of diuretics. It may be very difficult
to differentiate this diagnosis from Batter’s syndrome, especially if there is no documented
diuretic use in pt’s medical records. If diuretic abuse if strongly suspected, measurement of the
urine diuretic level can be performed.
5. Drug induced intersitital nephritis is usually cuased by cephalosporins, penicillins,
sulfonamides, NSAIDs, rifampin, phenytoin and allopurinol. Pts present with althralgias, rash,
renal failure and the urinalysis will show eosinophiluria.
6. Pemphigus vulgaris is a mucocutaneous blistering disease that is characterized by flaccid
bullae and intercellular IgG deposits in the epidermis. Autoantibodies are formed against
desomoglein, an adhesion moelcule. Bullous impetigo is a contagious infection of the skin
caused by Staph, Strep, or both. The bullous type is associated with Staphy aureus infection.
The lesion consist os macules, vesicles, bullae, and honey colored crusts, which leave red
denuded areas when removed.
7. Until proven otherwise, suspect squamous cell carcinoa in any pt with a chronic scar that
deveops into a non-healing, painless, bleeding ulcer. Punch biopsy is used to confirm the
diagnosis of squamous cell carcinoma of skin.
8. Nasal polyps are often associated with chronic rhinosinusitis, asthma, and aspirin- or
NSAIDs-induced bronchospasm in a condition known as aspirin-excerbated respiratory
disease (AERD). The frequently cause symptoms of bilateral nasal obstruction, nasal
discharge, and anosmia.
9. Shingles is characterized by a vesicular eruption that occurs in a dermatomal distrubution and
is often preceded by pain. It is caused by the varicella zoster virus, which also causes chicken
pox.
10. Riluzole is a glutamate inhibitor that is currently apporved for use in pts with amyotrophic
lateral sclerosis (a neurodegerative disease). Although it cannot arrest the underlying
pathological process, it may prolong surviving and the time to tracheostomy. Its side effects
are dizziness, nausea, weight loss, elevated liver enzyme and skeletal weakness.
Corticosteroids, iv IgG and cyclophosphamide have no role in its mx.
11. Acute deep vein thromboses should be treated with a combination of warfarin and heparin for

149
the first 5 days. After 5 days, the heparin may be discontinued if the pt’s INR is therapeutic.
Warfarin should be continued for at least 6 months in most pts with a first-time clot. A
progressing clot in a pt with a sub-therapeutic INR requires bridging heparin until the INR is
therapeutic. Thrombolytics are indicated for pts with hemodynamically significnat pulmonary
emboli. They are not routinely used for pt with DVTs. Inferior vena cava filters may be used
in pts with lower ectremity DVT where anticoagulation is C/I. This pt has no C/I to
anticoagulation.
12. The absence of peristaltis waves is the lower two-thirds of the esophagus and significant
decrease in lower esophageal sphincter (LES) tone are characteristic for esophageal
dysmotility associated with scleroderma.
13. The clues for dx of syncope caused by arrhthmia include sudden onset of syncope without
warning signs, presence of structural heart disease (post-infarction scar and probable mitral
regurgitation because of the characteristic murmur), and frequent ectopic beats. Another
disturbances predisposing to ventricular arrhthmia,
14. It appears that this pt developed parkinsonian features from the illicit agent MPTP.
Amantadine was first in India to treat influenza epidemics. It was noticed that the drug has a
beneficial effect in some Parkinsonism pts. The drug then changed its indication from treating
influenza to treating Parkinson’s disease as well. The problem is that its effects are not long
lasting. It can also cause depression and hypotension.
15. An isolated, round, smooth-borded, ring-enhancing intracranial lesion on contrast CT scan of
the brain in an immunocompetent pt with a known extracranial bacterial infection is most
likely a brani abscess. Aerobic and anaerobc streptococci and Bacteroides species (anaerobes)
are the MCC organisms (more than strap).
16. Iron studies are indicated in pts who present with microyctic/hypochromic anemia. These are
useful in confirming the dx of iron-deficiency anemia, which is the usual cause of
microcytic/hypochromic anemia, and in ruling out other causes.
microcytic/hypochromic anemias Findings in Iron studies
1. Iron-deficiency anemia Typically depressed serum iron level, increaed TIBC, and
decreased serum ferritin level
2. Thalassemia Normal to high serum iron and ferritin levels
3. Anemia of chornic disease Below normal TIBC, Normal or increased serum ferritin
level
4. Sideroblastic anemia Normal to high serum iron and ferritin values

17. Simple renal cysts are almost benign and do not require further evaluation. Know how to
recognize these on a CT scan. For dx, make sure does not have the following findings: 1) A
multilocular mass; 2) Thickened, irregular walls; 3) Thickened septae within the mass; 4)
contrast enhancement.
18. The most common organisms responsible for the development of epiglottitis, esp in the adult
population, are Haemophilus influenzae and Streptococcus pyogenes. Epiglottitis is a medical
emergency, and rapid tx must be initiated in order to prevent obstruction of the airway.
19. Paget disease of bone (osteitis deformans) is associated with normal serum calcium and
phosphate levels and increased alkaline phosphatase and urinary hydroxyproline levels.
20. Suspect HUS in a child who has recently recovered from a diarrheal illness and presents with
acute renal failure, microangiopathic hemolytic anemia, fever, thrombocytopenia and

150
characteristic peripheral smear finding of schistocytes. ITP is characterzied by an isolated
thrombocytopenia. On peripheral smear, the morphology of red cells is normal. * Extremely
high yield!!
21. Acute epididymitis in younger pts is usually caused by sexually transmitted organisms such as
C trachomatis or N. gonorrhea. In older men it is usually non-sexually transmitted and is
caused by G- rods (E.coli).
22. Beta-thalassemia minor occurs in people heterozygous for the beta-hemoglobin gene. It is
common in people in Mediterranean (Greek) decent and manifests as a hypochromic
microcytic anemia of moderate severity.
23. GERD is characterized by a retrosternal burning sensation after eating and with lying down. It
may also be accompanied by hoarseness and chronic cough, esp while recumbent. The initial
tx is and H2-receptor antagonist or a proton pump inhibitor.
24. Learn the clinical presentation, pattern of organ involvement, and complications of mumps.
Orchitis is one of the most frequent complications of mumps in adults.
25. The major cause of anemia in pts with end stage renal disease is deficiency of erythropoietin.
The anemia is normocytic and normochromic. The tx of choice is recombinant erythropoietin,
which is started if the Hb<10g/dL. The most common side effects are worsening of
hypertension (30% of pt), headaches (15% of pts) and flu-like syndrome (5%); Red cell
aplasia (a rare, but potential side effect).
26. All chronic hepatitis C pts with elevated ALT, detectable HCV RNA and histologic evidence
of chronic hepatitis of at least moderate grade are candicates for antiviral therapy with
interferon and ribavirin. Combined therapy with interferon and lamivudine is used in tx of
HBV.
27. Primary hyperparathyroidism is the MCC of hypercalcemia in ambulatory pts. Hypercalcemia
due to primary hyperparathyroidism is associated with elevated or inappropriately normal
serum PTH levels (normal 8-24 pg/ml).
28. In acute appendicitis, the initial peri-umbilical pain is referred pain and visceral in nature;
however, pain shifts to the right lower quadrant with involvement of the parietal pritoneum
and becomes somatic in nature.
29. Third degree, or complete, heart block is marked by complete independence of the P waves
and QRS complexes, both of which contract at completely separate rates. Tx for 3rd degree
heart block is expeditious pacemaker placement, as the rhythm can degenerate into ventricular
tychycardia or ventricular fibrillation.
30. Primidone is an anticonvulsant agent which can be used to treat benign essential tremors. Its
administration can precipitate acute intermittent porphyria (colichy abdominal pain,
confusion, headaches, hallucination and dizziness), which can be diagnosed by checking for
urine porphobilinogen.
31. Hypertension, hypokalemia and suppressed plasma renin activity are highly suggestive of
primary hyperaldosteronism. The MCC of primary hyperaldosteronism is aldosterone-
producing adenoma.
32. Prolactin-producing lactotroph adenoma is the most common primary pituitary tumor.
33. Cellulitis is an infection of the skin and subcutaneous tissue. Predisposing risk factors for
cellulitis include obesity, venous insufficiency, disruption of cutaneous barrier, and previous
cellulitis. Red, edematous skin that is hot to the touch is consistent with cellulitis. Fever,

151
lymphagitis and regional lymphadenopathy are often seen. Group A Streptococcus and
Staphylococcus aureus are the most common infectious agents.
34. Suspect pseudotumor cerebri in a young obese female with a headache that is suggestive of a
brain tumor, but with normal neuroimaging and elevated CSF pressure. The tx includes
weight reduction and acetazolamide. Shutting or optic nerve sheath fenestration may be
performed to prevent blindness (the most significant complication if left untreated).
35. Free air underneath the diaphragm signifies bowel perforation and requires urgent laparptomy.
36. Amidarone can cause pulmonary toxicity and should be avoided in pts with preexisting lung
disease since they have less pulmonary reserve.
37. Insulin resistance plays a central role in the pathophysiology of non-alcoholic fatty liver
(biopsy shows macrovascular fat deposition with displacement of the nucleus to the
peripheral) disease by increasing the rate lipolysis and elevating the circulating insulin levels.
38. Fibromyalgia is a chronic widespread pain disorder associated with fatigue, poor sleep, and
depression. Pts have multiple trigger points of tenderness.
39. Sarcoidosis is most common in African-American woman during the third and fourth decades
of life. The lungs are the most commonly affected organ system. When symptomatic, cough,
erythema nodosum, anterior uveitis, and arthritis may be seen. Hilar adenopathy and reticular
opacities on CXR are classic findings.
40. Currently, quantitative estimation of stool fat (72-hour fecal fat collection) is the gold standard
for the diagnosis of steatorrhea.
41. Long-term supplement oxygen therapy has been proven to prolong survival in pts with COPD
and hypoxemia. Criteria for therapy include PaO2 <55, SaO2 <88%, erythrocytosis
(hematocrit >55), or evidence of cor pulmonale.
42. Pts with multiple myeloma are at increased risk for infection due to both a total decrease in
functional antibodies and leukopenia that develops as the bone marrow is filled with
malignant plasma cells. Multiple myeloms mnemonic—CRAB: clacium (hypercalcemia),
renal impairment, anemia, and bones (bone pain, lytic lesions, fractures).
43. HIV pts with CD4 cell counts less than 50/mL require MAC prophylaxis with azithromycin or
clarithromycin. Oral ganciclovir may be effective for prophylaxis against CMV infection in
HIV pts with CD4 cell counts below 50/mL, however, it is not currently recommended for this
purpose.
44. Molluscum contagiosum is characterized by single or multiple rounded, dome-shaped papules
with central umbilication. The causative agent is poxvirus. It is ussally a self-limited infection
and resolves spontaneously in immunocompetent individuals. Genital lesions should be
treated to prevent sexual transmission. The best form of tx is curettage or application of liquid
nitrogen.

1/16/10
1. A combination of HBV immune globulin and lamivudine is the most effective measure to
prevent recurrent HBV infection after liver transplantation.
2. Pts with severe aortic stenosis can have anginal chest pain due to increased myocardial
oxygen demand.
3. The dx of chronic mesenteric ischemia is suspected in pts with unexplained chronic
abdominal pain, weight loss, and food aversion. Evidence of associated atherosclerosis

152
disease is usually present. Physical findings are usually nonspecific. Abdominal exam may
reveal a bruit (50% of pts).
4. Tamoxifen reduces the risk of breast cancer in pts who have an increased risk of developing
breast cancer.
5. Noninvasive test for H.pylori should be the first step in the mx of pts with dyspepsia under 45
years of age who do not have alarming symptoms. GI referral is required in pts older than 45
years of age or those with alarming symptoms at any age, due to the small risk of gastric
cancer. Such pts should be considered for endoscopy and H.pylori testing.
6. Osteromyelitis most often presents as focal pain, fever, chills, and a grossly elevated ESR. It
is particularly common in iv drug abusers and, in that setting, often involves the spine.
7. Appropriate pt education about photo-protection, esp in the high-risk group, is important. Sun
avoidance remains as the best method of photo-protection. Know this: Sunscreens should be
applied 15-60 min prior to sun exposure to allow enough time fro protective film
development. * Extremely high yield!!
8. Collapsing focal and segmental glomerulosclerosis is the most common form of
glomerulopathy associated with HIV. Typical presentation of focal segmental
glomerulosclerosis includes nephritic range proteinuria, azotemia, and normal sized kidneys.
* Extremely high yield!!
9. A MMSE (mini-mental state examination) score of less than 24 points (total 30) points is
suggestive of dementia. CT finding of diffuse cortical and subcortical atrophy are
characteristic for Alzheimer’s dementia.
10. Lithium is a common cause of nephrogenic diabetes insipidus. Lithium-induced nephrogenic
DI is treated with salt restriction and discontinuation of lithium. Other drugs: demeclocycline,
foscarnet, cidofovir, and amphotericin.
11. Consider the natural history of a disease when evaluating the effectiveness of a drug in a trial.
12. Cardiac dysfunction associated with hemochromatosis can be reversed with early
identificantion of the disease and tx (phlebotomy). Symmetric thickening of the ventricle
helps to differentiate restrictive from hypertrophic cardiomyopathy, in which the
interventricular septum is thickest.
13. Lowering LDL cholesterol levels is the primary goal of lipid modification, and should be
prioritized over increasing HDL levels. Fibrates (increase HDL by 10-20%, decrease LDL
modestly, and decrease triglycerides significantly) and nicotinic acid (niacin, elevates HDL
levels by 15-35% and lowers LDL and triglyceride level as well) are the drugs of chice for
raising HDL. Fibrates are often preferred because they have fewer side effects.
14. Ventricular tachycardia in the presence of a stable BP does not warrant cardioversion. The
best tx is loading lidocaine or amidarone (drug of choice).
15. Hairy cell leukemia is characterized by lymphocytes with fine, hair-like irregular projections
and a tartrate-resistant acid phosphatase (TRAP) stain. The bone marrow may become
fibrotic; thus leading to dry taps. Cladribine is the drug of choice for pts with hairy cell
leukemia.

Psychiatry
1. Pts who develop acute dystonia from the use of antipsychotics should be treated with
anticholinergic (benztropine or trihexyphenidyl) or antihistamines (diphenhydramine). It is

153
characterized by muscle spasms or stiffness, tongue protrusions or twisting, opisthotonus and
oculogyric crisis (a forced, sustained elevation of the eyes in an upward position).
Beta-blockers such as propranolol can be used to treat akathisia, which is a condition that can
occur at any time during the course of antipsychotic use. These pts report a subjective feeling of
restlessness and may pace constantly, unable to sit still.
Dantrolene is used to treat neuroleptic malignant syndrome (NMS).
When Parkinsonism develops as a side effect from antipsychotic use (usually within 4 days to 4
months of tx), the drug of choice is an anticholinergic agent such as benztropine.
2. Pts taking monoamine oxidate inhibitors like phenelzine should avoid foods high in tyramine,
as the combination can result in hypertensive crisis (most appropriate to monitor--BP).
3. If a pt’ family disagrees with his living will and demands care that contradicts the pt’s written
wishes, the best initial step is to discuss the matter with the family. If a discussion fails to
resolve the situation, then the hospital’s ethics committee should be consulted.
4. For the dx of major depressive disorder, pts should have at least 5 of 8 depressive symptoms
for at least 2 weeks, with the symptoms causing significant functional impairment. Primary
insomnia is characterized by the isolated symptom of having difficultu in falling or staying
asleep.
5. Rationalization is the offering of a rational, logical reason for an upsetting event rather than
the real reason. Intellectualization is a neurotic defense mechanism that allows the individual
to behave in a detached manner by separating emotional concerns from intellectual curiosity
about an issue that causes anxiety. An example would be a recently laid of woman who then
starts researching the political causes of the country’s recent economic downturn.
6. The antidepressant of choice in depressed pts suffering from sexual dysfunction (whether as a
medication side effect or as a pre-existing condition) is bupropion.
7. When choosing between medications, look for a medication that is effective for more than one
problem. Buproprion is an example of a medication with two uses, as it is an antidepressant
and a smoking cessation aid.
8. Pts with psychiatric dx can give informed consent as long as their judgment and decision-
making abilities are determined to be intact.
9. Propranolo is the drug of choice for treating performance-related anxiety. Buspirone is used to
treat generalized anxiety disorder, but not specific phobias. Clomipramine is a TCA that is
often used in the tx of OCD.
10. Methylphenidate is a CNS stimulant that is frequently used to treat attention deficit
hyperactivity disorder. Common side effects include nervousness, decreased appetite, wight
loss, insomnia, and abdominal pain. Prolonged therapy has been shown to cause mild growth
retardation or weight loss. Methylphenidate should not be used in children younger than 6
years old because safety and efficacy in this age group have not been evaluated.
11. If a medical error occurs, the physician must take responsibility for the situation, inform the pt
of precisely what happened, and offer a sincere apology to the pt and his family.
12. Odd behavior, magical thinking, and a lack of close friends are common features of
schizotypal personality disorder. While individual with schizoid personality disorder also lack
close friends and have a restricted range of emotional expression, they do not have eccentric
behavior or odd thinking. Those with avoidant personality disorder want friends but fear
ridicule.

154
13. Altruism involves minimizing internal fears by serving others. It is an example of a mature
defense mechanism in which individuals provide a helpful, gratifying service to others as a
means of quelling their own anxieties.
Reaction formation is the transformation of an unwanted thought or feeling into its opposite. An
example would be having a bias against a particular race and then embracing that race as ideal and
wonderful.
Sublimation is a mature defense mechanism that allows for unacceptable impulses to be channeled
into more acceptable activities. An example would be a man with a fiery temper who channels shi
anger into athletic pursuits.
14. Benzodiazepines and other sedative hypnotics are contraindicated in pts with breathing-
related sleep disorders.
15. When a woman presents complaining of dyspareunia, the presence of a general medical
condition or underlying psychological stressors should first be ruled out. Medical causes of
dyspareunia include endometriosis, local infections, vulvar or vagianl growthsm and estrogen
deficiency. Psychological causes include somatization disorder, pain disorder, or dyspareunia
without associated symptoms. Tx should target the underlying cause.
16. Bupropion is an antidepressant that produces its effects primarily through the inhibition of the
re-uptake of norepinephrine, dopamine, and serotonin. Contraindications to bupropion use
include seizure disorders, eathing disorders, or other conditions predisposing to seizure (eg.
concurrent alcohol or benzodiazepine use). Individuals with anorexia nervosa or bulimia
nervosa frequently develop electrolytes abnormalities that can precipitate seizures.
17. Pathological gambling is the likely dx in an individual with a chronic history of gambling and
a seeming inability to stop. Significant financial losses or damaged relationships are common
consequences of this behavior.
18. First line therapy for major depression is prescription of a SSRI such as sertraline. Major
depression is diagnosed when at least five of the following symptoms are present in a 2-week
period (including either depressed mood or anhedonia). Depressed mood; Sleep disorder;
Interest deficit (anhedonia); Guilt (worthlessness, hopelessness, regret), Energy deficit;
Concentration deficit; Appetite disorder; Psychomotor retardation or agitation; Suicidality
(SIGECAPS). Because TCAs cause a number of significant side effects—including those
considered particularly undesirable in the elderly (eg. orthostatic hypotension)—they are not
recommended for this pt. Moreover, there is no indication for prescribing bezodiazepines in
the tx of depression.
19. Pts who are acute dangers to themselves should be hospitalized (involuntarily, if necessary)
for tx and stabilization. This principle applies to minors as well, even when parents or
guardians disagree.
20. Reaction formation is the transformation of an unwanted thought or feeling into its opposite.
21. Always have a high index of suspicion for physical/sexual abuse in children (esp females)
with sudden behavioral problems, families with unstable economic backgrounds, or parents
with a history of drug/alcohol abuse.
22. Displacement is a psychological defense machenism in which unacceptable feelings about one
object or person are displaced onto anther, “safer” object or person.
23. The grandiose delusions, racing thoughts, distractibility, and irritablility demonstrated by this
pt are highly suggestive of mania. Lithium is a first-line medication in the tx of bipolar

155
disorder. This pt, however, has impaired renal function as evidence by his elevated BUN and
creatinine levels, and since lithium is excreted though the kidneys and has been shown to
cause nephrogenic diabetes insipidus after prolonged use, impaired renal function is a C/I for
lithium therapy. Suitable alternatives to lithium include valporic acid or carbarmazepine, both
of which are metabolized by the liver.
24. Pts have the right to know their diagnosis. If a family member requests that the dx not be
revealed to the pt, the underlying reasions should be ecplored before deciding how to preceed.
25. All atypical antipsychotics, including olanzapine, can cause weight gain, hyperglycemia,
dyslipidemia, and hypertension. Routine monitoring for the development of these side effects
is recommended in pts taking these medications.
26. Indecisiveness, a fear of being alone, and clingy and submissive behavior are all suggestive of
dependent personality disorder.
27. Confidential pt information should only be disclosed to fellow health care workers who are
directly involved in the pt’s care. Additionally, care should be taken to avoid discussing a pt’s
medical condition in public areas where comments might be overheard. Inappropriate
inquiries form colleagues curious about a pt’s medical condition should be politely but firmely
rebuffed.
28. One of the most common side effects of ECT is amnesia, both retrograde and antegrade.
29. Antipsychotics cause hyperprolactinemia by blocking dopamine activity along the
tuberoinfundibular pathway.
The nigrostriatal pathway extends form the substania nigra to the basal ganglia, and is involved in
the coordination of movement. Increase dopamine—chorea and tics; decrease—Prakinson.
The mesolimbic pathway extends from the ventral tegmental area to the limbic system. Increased
dopamine—euphoria accompanying drug use, as well as delusion and hallucinations experienced
by pts with schizophrenia; decrease—therapeutic effects of antipsychotics.
30. Lithium exposure in the 1st trimester of pregnancy causes a twenty-fold increase in the risk of
Ebstein’s anomaly, a cardiac malformation (a malformed and inferiorly attached tricuspid
valve that causes atrialization of the upper right ventricle and a decrease in the size of the
functional right ventricle). In the later trimester, goiter and transient neonatal neuromuscular
dysfunction are of concern.
31. Depot antipsychotics such as fluphenazine and haloperidol are the tx of choice for
schizophrenic pts who suffer replases due to tx noncompliance.
32. Cocaine abuse should be suspected in an individual presenting with weight loss, behavioral
changes, and erythrema of the turbinates and nasal septum.
33. Schizophreniform disorder can be differentiated from schizophrenia by the duration of
symptoms. To diagnosis schizophreniform disorder symptoms must last for more than one
month but less than six months, while the dx of schizophrenia requires symptoms to be
present for at least six months.
34. The likelihood a schizophrenic pt will relapse is decreased if conflicts and stressors in the
home environment are kept to a minimum.
35. Pts of all ages should be given the opportunity to speak with the doctor alone.
36. When child abuse is suspected, the following steps should be performed: 1) Complete
physical examination. 2) Radiographic skeletal survery, if needed. 3) Coagulation profile (if
multiple bruises are present). 4) Report to Child Protective Service. 5) Admit to hospital if

156
necessary. 6) Consult psychiatrist and evaluate family dynamics.
37. The disscociative disorders are characterized by forgetfulness and disscociation. Dissociative
fugue is the only condition within this group that is associated with travel.
Dissociative identity disorder, formerly known as multiple personality disorder, is characterized
by the presence of two or more distinct identities that alternatively assume control of the preson’s
behavior. Amnesia regarding important personal information about some of the identities is
observed.
Dissociative amnesia is characterized by the presence of one or more episodes of inability to recall
important personal information. The memory disturbance is usually related to a traumatic or
stressful event and is too extensive to be considered ordinary forgetfulness.
Derealization disorder describes the state of experiencing familiar perosons and surroundings as if
they were strange or unreal.
38. Always rule out hypothyroidism in pts who present with symptoms of depression.
39. Obsessive-compulsive personality disorder is characterized not by obsessions or compulsions,
but more by a need for perfection, over-attention to detail, stubbornness, and a need to strictly
follow rules. Individuals often do not identify their behavior as abnormal. OCD is marked by
obsessions that cause anxiety, resulting in compulsive behaviors to decrease the anxiety.
40. The greatest risk factor for committing suicide is a past history of suicide attempt(s).
41. Adjustment disorder is characterized by the development of emotional or behavioral
symptoms in response to an identifiable stressor that causes a significant impairment in the
pt’s life.
Acute stress disorder is very similar to PTSD, with anxiety symptoms developing after
experiencing or witnessing an event that involved actual or threatened death or serious injury. The
symptoms last not longer than one month.
PTSD is characterized by symptoms of anxiety that develop after experiencing or witnessing an
event that involved actual or threatened death or serious injury. Pts with PTSD often re-live the
event through flashbacks or nightmares. Avoid stimuli realated to the trauma, may hypervigilance.
Symptoms of PTSD last longer than one month.
42. Generalized anxiety disorder is characterized by excessive worry over many aspects of one’s
life that causes significant impairment in functioning. Additional symptoms such fatigue,
restlessness, difficulty concertrating, irritabilitym and sleep disturbance are commonly seen in
pts with GAD.
43. Delusional disorder is an uncommon condition characterized by isolatedm non-bizarre
delusions in an otherwise high-functioning individual. Hallucinations, disorganized though,
and functional deterioration do not manifest in pts with delusional disorder, when present,
they are suggestive of a more expansive psychosis. Antipsychotic medications are often used
to treat pts with delusional disorder, but have not proven very effective.
Brief psychotic disorder is characterized by delusions, hallucinations, and disorganized thought
that last for no more than one month.
44. Eating disorder, not otherwise specified is a DSM-IV category that includes all eating
disorders that do not meet the criteria for any one specific eating disorder.

1/17/10
1. Pts with somatization disorder benefit from regularly scheduled appointments intended to

157
reduce the underlying psychological distress.
2. The DSM-IV criteria for many psychiatric conditions required that symptoms be severe
enough to cause significant functional impairment of a specified duration of time. (His levevel
of functioning at work and at home has not been impaired.)
3. Altered levels of the neurotransmitter serotomin play an important role in the development of
OCD.
4. A pregnant woman has the right to refuse tx, even if it places her unborn child at risk.
5. All depressed pts should be screened for suicidal ideation. Actively suidical pts will often
need to be hospitalized for stabilization and to maintain their safety.
6. A delusion is a fixed, false belief not consistent with culture norms. Individuals with
grandiose delusions typically believe they have special powers, extraordinary
accomplishments, or a special relationship with God.
7. PCP and LSD intoxication present similarly, but agitation and aggression occur more often in
pts using PCP. Visual hallucinations and intensified perceptions are hallmarks of LSD use. Pts
wth PCP intoxication usually present with distinct behavioral changes such as physical
aggression, severe agitation, impulsivity, impaired judgement, psychosis, paranoia, or
hallucinations. These symptoms develop shortly after ingestion of the drug. Physical signs
such as nystagmus, hypertension, tachycardia, ataxia, dysarthria, muscle rigidity, seizure, or
coma may also be evident. LSD is a hallucinogen. Shortly after ingesting this drug,
individuals may experience mood impairment, hallucinations, subjective perceptual
inrensification (ie, colors are richer, tastes are heightened, and sensation is enhanced),
depersonalization, and illusion. In addition, two or more of the following signs are often
present: sweating, tachycardia, papillary dilatation, palpitations, tremors, and poor
coordination.
8. Narcissistic personality disorder is characterized by an exaggerated sense of self-importance,
feelings of entitlement, egocentrism, and a lack of empathy for others.
9. Some of the more serious adverse effects associated with lithium include nephrogenic
diabetes insipidus, hypothyroidism, and Ebstein’s anomaly in the fetus.
10. Childhood disintegrative disorder is a rare pervasive developmental disorder that occurs more
commonly in males. It is characterized by a period of normal development for at least two
years, followed by a loss of previously skills in at least two of the following areas: expressive
or receptive language, social skills, bowel or bladder control, or play and motor skills.
Autism is another type of pervasive developmental disorder that is also more frequently seen in
males. The onset of symptoms always occurs before the age of 3 years. The condition is
characterized by qualitative impairments in communication and social interaction. These pts tend
to indulge in repetitive, stereotyped behavior with stange preoccupations. Although the child in
this scenario does have autistic symptoms, his normal development until the age of 4 supports a dx
of childhood disintegrative disorder.
Occurring almost exclusively in female, Rett syndrome is a rare pervasive developmental disorder.
The condition is characterized by an initial period of normal development (typically until six
months of age) followed by the loss of hand coordination and the development of peculiar
stereotyped hand movements. Other common features include a deceleration of head growth, poor
coordination, seizures, ataxia, mental retardation, and diminished social interaction.
Asperger syndrome is characterized by a qualitative impairment in social interaction and

158
restricted, repetitive, and stereotyped patterns of behavior. Unlike pts with autictic disorder,
children suffering form Asperger syndrome have normal cognitive and language development.
ADHD presents before age seven years and is characterized by inattention, hostile behavior.
Children with this condition are disobedient, irritable, spiteful 恶 意 的 and argumentative. They
often blame others for their failings.
11. If a pt presents with refractory mania despite therapy with a mood stabilizer, a urine
toxicology screen (to ensure had not recently taken cocaine or amphetamine, as usage of
either stimulant can result in a manic presentation) and mood stabilizer drug levels should be
obtained in the initial evaluation.
12. Pregnant women with a current of previous dx of anorexia nervosa are at risk for numerous
complications, including miscarriage, intrauterine growth retardation, hyperemeiss, premature
birth, cesarean delivery, and postpartum depression (not postpartum psychosis). Osteoprosis is
also a common finding in anorexic pt, whether pregnant or not. * Extremely high yield!!
13. The following signs and symptoms are indications that psychiatric hospitalization is
necessary: homicidal ideation, suicidal ideation, grave disability, gross disorganization;
agitated or threatening behavior, or severe symptoms of substance intoxication or withdrawl.
14. Pt confidentiality should not be maintained if it would endanger the health and welfare of
others. In cases of sexually transmitted diseases, public health laws require the reporting of
the pt and any known sexual partners to the local health department authorities.
15. In severely depressed pts with active suicidal thoughts, antidepressants should be started
immediately. This is also true for depressed pts suffering from terminal illnesses.
16. Weight gain is a significant adverse effect associated with olanzapine. * Extremely high
yield!!
17. Adolescent seeking a physician’s care for contraception, prenatal care, sexually transmitted
diseases, or substance abuse may be treated without parental consent and are entitled to strict
patient confidentiality.
18. Children with oppositional defiant disorder are disobedient and argumentative. Although they
may be hostile, they do not seriously violate the rights of others. The presence of four or more
of the following symptoms is required for dx of ODD: short temper, argumentativeness with
adults, noncompliance with commands or rules, blaming others, and vindictiveness.
Conduct disorder is characterized by a persistent pattern of behavior in which the basic rules of
society and the rights of others are violated. Children with conduct disorder frequently lie, steal,
destroy property, set fires, and demonstrate cruelty to animals and people.
Antisocial personality disorder presents in the same fasion as conduct disorder but occurs in adults
ages 18 years or older.
19. Avoid benzodiazepine use in pts with PTSD (b/c PTSD is strongly associated with substance
abuse, the prescribing of addictive medications like benzodiazepine is strongly avoided when
possible). Tx of PTSD is best accomplished with a combination of SSRI and exposure or
cognitive therapy. Cognitive therapy involves separating the recurrent flashbacks from the
anxiety they produce. This form of therapy is particularly helpful for those pts who avoid
stimuli that are reminders of the traumatic event.
20. When a pt refuses potentially life-saving tx, it is important to fully discuss the specific reasons
for his decision before honoring it. If the pt continues to refuse medical intervention despite
clarification and reassurance from the physician, then his decision should be respected and the

159
tx withheld.
21. A rare sleep disorder, narcolepsy is characterized by episodes of irresistible, refreshing sleep
during the daytime and one or more of the following traits: cataplexy (diffuse muscle
weakness leading to collapse), hypnogogic and hypnopompic hallucination (hallucinations
wihile falling asleep and waking), and sleep papalysis. Tx options for narcolepsy include:
scheduled daytime naps, psychostimulants (eg. modafinil or methylphenidate), or a
combination of antidepressants and psychostimulants.
Obesity is a common cause of obstructive sleep apnea syndrome, which is characterized by
excessive daytime sleepiness and a history of snoring and frequent waking at night. There is no
history of cataplexy or sleep paralysis, and complaints of fatigue and early morning headaches are
common. Tx options include weight reduction and nasal continuous positive airway pressure
(CPAP).
22. If no significant harm is likely ot result from withholding therapy, parental wishes regarding
the medical care of a child should be honored and the discussion documented in the chart.
23. The extrapyramidal side effects of antipsychotics can be treated with anticholinergic
medications like benzitropine.
24. When treating a single episode of major depression, the antidepressant should be continued
for a period of six months following the pt’s response. If multiple episodes of depression have
occurred, maintained therapy will likely need to be continued for a long period of time.
Nausea is a side effect of most antidepressants (including bupropion), a dose reduction in
sertraline is not warranted unless the nausea is very severe or incapacitating.
25. Studies of brain CT scans have demonstrated that pts with schizophrenia have enlarged
ventricles and prominent sulci.
26. The mechanism of action of antipsychotic medications primarily consists of dopamine-D2
receptor blockade. The added serotonin receptor binding of atypical antipsychotics reduces
the likelihood of extrapyramidal side effects.
27. Physicians should be non-committal and non-judgemental when treating psychotic pts. It is
important to acknowledge the pt’s distress without endorsing specific delusions or
hallucinations.
28. Alcohol withdrawl is best treated with treated with long-acting benzodiazepines such as
chlordiazapoxide (Librium). Withdrawal symptoms should be correlated with the time of the
last alcoholic drink. It is important to rule out other medical conditions that could be
responsible (electrolyte abnormalities, infection, or hypoxia) before making a dx. Pts with
alcohol withdrawal develop signs and symptoms of withdrawal between 12-48 hours after the
last drink. During the acute stage they develop sweating, hyperreflexia, seizure, and tremors.
This is followed by alcoholic hallucinosis (auditory and visual) in the absence of autonomic
symptoms. The last stage is that of delirium tremens, which usually occurs 2-4 days after the
last drink. Pts in this stage present with altered sensorium, hallucinations and autonomic
instability (tachycardia and fever).
29. Munchausen syndrome by proxy is the likely dx when one person intentionally produces
syndromes in another person, with the motive being to assume the role of concerned onlooker.
30. Alzheimer’s disease is the likely dx in an elderly pt with gradually progressive memory loss
and one or more of the following apraxia, aphasia, agnosia, or disturbed executive
functioning. Pick’s disease has a clinical presentation similar to that of Alzheimer’s, though

160
behavioral abnormalities tend to precede the cognitive decline. Pts may present in their 50s
and are more likely to be female. It is causes atrophy of the frontal lobes and temporal lobes.
31. Disorganized speech is common in schizophrenics. Pts with a circumstantial thought process
deviate from the original subject but eventually return to it, while those a tangential process
drift away without ever returning to the subject.
32. Adjustment disorders is characterized by emotional or behavioral symptoms that develop
within 3 months of exposure to an identifiable stressor and that rarely last more than 6 months
after the stressor has ended.
33. When dealing with an angry pt, the best response is to encourage a discussion about the
source of these feelings. Start the discussion with an open-ended question and let the pt talk.
34. A physician is authorized to provided emergent life-saving tx to the unconscious pt. This
remains true even the pt’s spouse requests that the tx not be given b/c it contradicts a belief
system. The religious beliefs of pts should be honored whenever possible. However, in the
event of a life-threatening situation, the physician should act in the pt’s best interests unless he
is specifically told not to by the the pt himself.
35. Fantasy is an immature defense mechanism that substitutes a less disturbing view of the world
in place of reality as a means of resolving conflict.
36. Anorexia nervosa is most common is adolescent girls from affluent families. The DSM-IV
criteria for the dx of anorexia nervosa include: 1) body weight at least 15% below normal
weight accompanied by a refusal to maintain body weight at normal levels; 2) amenorrhea for
3 months; 3) distortion of body image in which the individual views herself as obese when she
is in fact thin, and 4) fear of gaining weight or becoming fat despite being underweight. To
continue to lose weight, individuals suffering from anorexia nervosa will either: 1) fast and/or
excessively (the restricting subtype), or 2) binge eat followed yb laxative usage or induced
vomiting (the binge and purging subtype). Hospitalization is highly recommended for pts with
anorexia nervosa when there is evidence of dehydration, starvation, electrolyte disturbances,
(ie. Hyponatremia, hypokalemia, or hypophosphatemia). Cardiac arrhythmias, physiologic
instablility, or severe malnutrition (ie. Weight < 75% of average body weight for age, sex and
height). The goals of hospitalization include weight agin as well as prevention and mx of the
medical complications caused by anorexia nervosa.
37. Vaginismus is an involuntary spasm of the perineal musculature that interferes with sexual
intercourse. In general, dx of female sexual disorders requires taking a detailed history from
both sexual partners. Multiple sexual disorders may coexist in the same pt.
Hypoactive sexual desire is persistent deficiency in both sexual fantasy and the desire for sexual
activity activity.
Sexual aversion disorder is a recurrent and persistent aversion to and avoidance of genital sexual
contact with a partner.
Female sexual arousal disorder is defined as an inability to attain or maintain the lubrication and
swelling response associated with sexual arousal.
Female orgasmic disorder is a persistent delay in or absence of orgasm following a normal sexual
excitement phase.
38. Minors normally cannot consent to their own medical tx. Parents or legal guardians must
provide consent on the girls’s behalf before the physician can proceed, although exceptions
are made for emergent situations.

161
39. Acute stress disorder and PTSD present with identical symptoms (recurrent nightmares and
flashbacks, potential memory loss, and exaggerated startle response). Acute stress disorder
can last no more than four weeks, however, while PTSD lasts longer than 4 weeks.
40. Cocaine and amphetamine intoxication present in a similar manner, but psychosis is more
commonly associated wht amphetamine use. Common symptoms of stimulant intoxication
include dilated pupils, hypertension, and tachycardia.
A manic episode (DIGFAST): 1) Distractibility; 2) Insomnia; 3) Grandiosity; 4) Flight of ideas; 5)
Activity increase (goal-oriented); 6) Speech (extreme talkativeness); 7) Thoughtlessness
(excessive involvement in high-risk behavior, such as gambling).
41. Dysthymia is depressed mood lasting most of the days for the majority of days for at least 2
years. The dx also requires at least 2 of the following symptoms: decreased or increased
appetite, poor energy, insomnia or hypersomnia, impaired concentration, low-self esteem, and
feelings of worthlessness. Pts with dysthemia will often say that they have the felt depressed
their entire life.
42. When breaking bad news, physicians should begin with exploratory general statement such as
“how are you feeling today?” to help the pt feel at ease.
43. Atypical antipsychotic medications such as risperidone are considered first-line tx for pts with
psychotic disorders b/c of their more favorable side effect profile.
44. Schizophrenia is divided into 4 subtypes based on the predominant symptoms that the pt
presents with during the active phase of the illness. Disorganized subtype is characterizd by
disorganized behavior, disorganized speech, and flat or inappropriate affect.
Catatonic subtype is characterized by a predominance of physical symptoms, including immobility
or excessive motor activity and the assumption of bizarre postures.
Undifferented subtype presents with mixed symptoms that do not meet the criteria for paranoid,
disorganized, or catatonic subtype.
Paranoid subtype presents with preoccupation with delusions or auditory hallucinations without
prominent disorganized speech or inappropriate affect. These pts are usually less severely disabled
and are most responsive to pharmacotherapy.
Residual subtype occurs in pts with previous dx of schizophrenia who no longer have prominent
psychotic symptoms. The persistent symptoms may include eccentric behavior, emotional
blunting, illogical thinking, or social withdrawal.

1/17/10
1. Displacement is an immature defense mechanism in which the individual displaces negative
feelings associated with an unacceptable object or situation onto a “safer” object or situation.
Acting out is the expression of an unconscious impulse through a physical action. An example
would be this engineer screaming and kicking the floor when he discovered he was fired.
Conversion is a defense mechanism in which emotional conflicts are transformed into
physical (often neurological) symptoms. An example would be this engineer being struck
mute after learning the news of his firing.
2. Physical abuse should be suspected in a woman with multiple bruises and frequent injuries. In
these cases, the following steps should be carried: 1) Confront the pt gently, in a non-
judgemental way. 2) Assure her about confidentiality. 3) Emphasize that she should not allow
abuse to happen to her. 4) Suggest informing the police. 5) Ensure safety of the pt and

162
children, if any. 6) Ask her if she had a plan to escape. 7) Suggest talking to a women’s group
dealing with these problems. 8) Assure her of your continuing support.
3. Passive-aggressive behavior is a psychological defense mechanism in which an individual
expresses his aggression toward another person with repeated, passive failure to meet the
other perosn’s need.
4. When caring for a minor, informed consent from one parent or guardian is considered legally
sufficient to justify proceeding with therapy.
5. Enuresis is defined as the recurrent, involuntary voiding of urine into clothing or bed linens
after age five years. Dx criteria for this condition include the occurrence of symptoms at least
twice per week for 3 consecutive months or the development of marked functional
impairment. The child’s history should be reviewed for medical conditions or medications that
couldn be responsible for the enuresis. When behavior modification is unsuccessful, Low
doses of tricyclic antidepressants such as imipramine or desmopressin can be used.
6. The physician has the moral responsibility to always act in the pt’s best interests. When a
mistake made by another physician is discovered, the facts should be clarified and the truth
told to the pt.
7. Older individuals may frequently awaken from sleep and spend less time sleeping overall.
These changes are considered a normal part of aging.
8. Malingering is always associated with secondary gain, whereas the motivating factor in
factitious disorder is to assume the sick role.
9. Severe symptoms of Tourette symdrome are best treated with typical antipsychotics such as
haloperidol or pimozide 哌迷清,双氟苯丁哌啶苯并咪唑酮.
10. Kleptomania is characterized by an inability to resist the impulse to steal objects that either
are of low monetary value or are not needed for personal use. The condition is more prevalent
in females and is occasionally associated with bulimia nervosa.
11. Individuals with schizoid personality disorder are socially detached and aloof 避开 but do not
have bizarre cognition. Those with schizotypal personality disorder are also socially detached
but typically demonstrate “magical thinking” and a more eccentric thought process.
Individuals with schizophreniform disorder have full-blown schizophrenic symptoms (eg.
hallucination, delusions) that have been present for 1-6 months.
12. Benzodiazepines are used for the acute tx of panic attacks, and an SSRI should be used for
long-term symptom relief.
13. Manic episodes are characterized by the presence of elevated mood for at least one week,
accompanied by three or more DIGFAST symptomes.
14. Pts who are extremely agitated, psychotic, or manic should be initially managed with an
antipsychotic medication such as haloperidol.
15. The following guidelines apply to lithium therapy in pts with bipolar disorder: 1) single manic
episodes require long-term maintainance for a period of at least one year, 2) three or more
relapses require tx with lifelong maintainance therapy. If there are no relapse and pt has
attained good symptomatic control, then the lithium can be gradually tapered off and ultimatel
discontinued. Because abrupt cessation of lithium increases the risk of suicide and relapse,
tapering the medications is always recommended.
16. Neuroleptic malignant syndrome (NMS) is an unusual but potentially lethal side effect from
the use of antipsychotics (neuroleptics). It is treated primarily with dantrolene sodium and

163
supportive care.
17. Pyromania is characterized by intentional, repeated fire setting with no obvious motive.
Although a history of arson 纵火罪 may be documented on individuals with conduct disorder,
other features will be present as well (eg. lying, theft, cruelty).
18. If parents refuse to give consent for the tx for their child in a life-threatening situation, the
physician should proceed with the tx despite the parental wishes.
19. The tx of choice for social phobia is assertiveness 自 信 training, which is a component of
cognitive behavioral psychotherapy (CBT). SSRI drugs are the first-line drugs in the mx of
these pts, either alone or in combination with CBT.
20. Imaginary friends are a normal component of development for many young children, and are
usually outgrown by the early elementary school years.
21. Alcoholic hallucinations typically develop within 12-24 hours of last drink and resolve within
24-48 hours. Unlike delirium tremens sensorium is intact and vital signs are usually normal.
22. Lithium, valproic acid, and carbarmazepine are first-line treatments for pts with bipolar
disorder. (Ref. 14)
23. Avoidant personality disorder is characterized by shyness, feelings of inferiority, and a desire
to make friends that is overridden by an intense fear of embarrassment or rejection.
24. For the general population, the lifetime risk of developing bipolar disorders is 1%. However,
an individual with a first-degree relative (eg. parent, sibling, or dizygotic twin) who suffers
from bipolar disorder has a 5-10% risk of developing the condition in his lifetime.
25. Do not share pt information with a third party without first obtaining written consent form
from the pt. A breach of confidentiality is justified only if a pt’s dx could prove a risk to others
and he refuses to divulge 泄露 this information to those in jeopardy.
26. Body dysmorphic disorder is defined by an excessive preoccupation with animagined bodily
defect. Hypochondriasis results in recurrent thoughts of having a serious, potential fatal
“medical condition”. Somatization disorder presents with multiple medical complaints that
persist despite repeated negative workups.
27. Children who suffer from selective mutism are verbal and talkative at howm but refuse to
speak in select settings (eg. school, public events). To establish the dx, symptoms must be
present for at least one month, cause significant functional impairment, and not be caused by
another communication or learning disorder.
Separation anxiety disorder is characterized by extreme anxiety in children when they are
separated from home or their loved ones.
Stranger anxiety is a normal finding in children until 3 years of age. After that, a persistent fear of
strangers is generally attributable to other causes.
28. Suspect heroin withdrawal in pts with papillary dilatation, rhinorrhea, muscle and joint aches,
abdominal cramping, nausea, and diarrhea. The symptoms are severe and out of proportion ot
physical findings.
29. Borderline personality disorder is characterized by “splitting”, unstable relationships, and
impulsivity. Angry outbursts and suicidal gestures are common.
30. NMS is a potentially life-threatening condition that can occur after administration of
antipsychotic medications. Symptoms include fever, rigidity, altered mental status, and
autonomic instability. It is most likely the result of repeated doses of haloperidol.
31. The symptoms of schizophrenia can be classified as positive and negative. The positive

164
symptoms include hallucinations, delusions, disorganized speech, and disorganized behavior.
Negative symptoms include the “five A’s”: affective flattening (diminished emotional
responsiveness), alogia (poverty of speech), apathy (impaired grooming and hygiene,
unwillingness to perform activities), asociality (few recreational interests, social detachment,
impaired relationship); and attention (inattentiveness and impaired concentration when
interviewed). Atypical antipsychotics such risperidone are particularly effective in the tx of
negative symptoms of schizophrenia.
32. If parents refuse to consent to tx of their child for a non-emergent but potentially fatal medical
condition, the physician should seek a court order mandating tx.
33. Specific or simple phobias can be treated with benzodiazepines. Alprazolam is a short-acting
benzodiazepine, which makes it more suitable than long-acting benzodiazepines such as
clonazepam for decreasing anxiety during a short plane ride. Buspirone is used for the tx of
generalized anxiety disorder.
34. Bereavement is the normal reaction to the loss of a loved one. Symptoms are similar to those
seen with major depression but are less intense and usually significantly taper within 2
months.
35. Most antidepressants must be taken for 4-6 weeks before they provide symptomatic relief.
36. Autism is the likely dx in those children who perform stereotyped behavior, demonstrate an
inability to form emotional bonds with others, and participate in restricted activities before the
age of 3 years.
37. Agranulocytosis is a serious side effect associated with clozapine usage. Pts should have their
white blood cell counts monitored frequently for this complication.
38. Reversible acetylcholinesterase inhibitors such as donepezil, rivastigmine, galantamine, and
tacrine are of benefit in slowing the cognitive decline associated with Alzheimer’s disease.
39. Differentiation of Delirium an Dementia
Delirium Dementia
Onset Acute Gradual
Consciousness impaired Intact
Course Fluctuating symptoms Progressive decline
Prognosis Reversible symptoms Irreversible symptoms
Memory impairment Global Remote memory spared

40. Abrupt cessation of alprozolam, a short-acting benzodiazepine, is associated with significant


withdrawal symptoms such as generalized seizures and confusion.
41. Generalized anxiety disorder is characterized by excessive anxiety about multiple events, in
conjunction with 3 or more of the following symptoms for a period of at least 6 months:
impaired sleep, poor concentration, easy fatigability, irritability, muscle tension, or
restlessness.
42. Physicians should respond to inappropriate pt requests politely but firmly. If the pt persists, it
is acceptable to be more direct and assertive in response. Professionalism should be
maintained at all times.
43. Dysthymia is characterized by depressive symptoms not meeting full criteria for a major
depressive disorder, lasting two years or more. Tx with antidepressants and/or therapy can
improve symptoms and quality of life.
44. Somatization disorder is characterized by multiple recurrent somatic complaints that have

165
persisted for several years and that have been evaluated by healthcare providers to no avail 效
用. The condition presents before the age of 30 and most frequently occurs in females.

1/18/10
1. When dealing with pt (including antagonistic, seductive, or familiar types), the physician must
remain polite and professional while addressing the pt’s medical and psychological needs.
Avoiding the pt, behaving rudely, or calling attention to any inappropriate behavior is not
recommended.
2. The mother and daughter are likely suffering from a rare psychotic disorder known as folie á
deux. In this disorder, a delusion or set of delusions is shared simultaneously by individuals
who share a close relationship. Usually, the dominant individual in the pair becomes
delusional and transfer the delusion onto the second person. Tx includes separating the pair to
break the chain of reinforcing each other’s beliefs. The individual who first had the delusion,
in this case the mother, always requires psychiatric tx, whereas the other individual only
requires tx in some case.
3. The tx of choice for adjustment disorder is cognitive or psychodynamic psychotherapy.
4. Tourette syndrome, occurring more frequently in males, is characterized by multiple motor
and one or more vocal tics that present before the age of 18. Pts with Tourette syndrome have
a significantly increased risk of developing ADHD (60%) or OCD (27%).
5. Pts have the legal right to obtain copies of their medical records. The third party requests for a
pt’s medical records should be refused unless the pt has provided written legal consent
beforehand. Pts don not need to justify requests for medical records. Were the pts to insist
upon obtaining the original chart, however, the physician would be justified in asking why
such an unusual transation is necessary.
6. OCD is best treated with an SSRI. Clomipramine (TCA) is a second-line tx. Alprazolam is a
short-acting benzodiazepine used in the tx of anxiety and panic disorder.
7. Although they are considered first line treatments for depression, SSRI are not without side
effects. One common and troubling potential side effect of the SSRIs is sexual dysfunction
(impotence).
8. Circadian rhythm sleep disorder is a likely diagnosis in apt with insomnia The who often
travels between different time zones.
9. The diagnosis of catatonic schizophrenia include the presence of two or more of the
following: 1) catalepsy, waxy flexibility, or stupor; 2) catatonic excitement (ie. Hyperactivity
or violence); 3) extreme negativism or mutism; 4) posturing, grimacing, prominent
mannerisms; and 5) echopraxia 模仿动作 or echolalia 模仿言语. This pt’s unresponsiveness,
motionless, sitting, and muscle rigidity are all suggestive of catatonia. Catatonic pts benefic
most from benzodiazepines (eg. lorazepam) or electroconvulsive therapy.
10. The features of bulimia nervosa: 1) recurrent episodes of uncontrolled binge eating followed
by feelings of extreme disgust or guilt; 2) repeated compensatory behavior to prevent weight
gain after binging (eg. induced vomiting, laxative abuse, diuretic abuse, fasting, or excessive
exercise); 3) binging episodes that occur at least twice per week over a 3 month period; 4)
normal or slightly above normal BMI; 5) dissatisfaction with body weight and shape.
Precipitating factors for a binge-purge episode in bulimic pts includes high levels of anxiety,
emotional tension, boredom, exhaustion, poor self-esteem, environmental cues about food and

166
eating, alcohol use, substance abuse, and mood disorders. Unlike pts with anorexia nervosa,
pts with bulimia nervosa maintain a normal body weight and are not amenorrheic.
11. pts have a right to confidentiality unless their condition poses an imminent danger to other
individuals or society. However, pts should be strongly encouraged to discuss their health and
medical conditions with loved ones.
12. Anorexia nervosa is always associated with amenorrhea and a body weight that is below
normal. (ref. 1/17 NO.36)
13. A history of seizure disorder (epilepsy) is an absolute C/I to the use of bupropion.
14. Trichotillomania is an impulse-control disorder, characterized by compulsive hair pulling,
leading to hair loss. 1) recurrent pulling out of one’s hair resulting in hair loss; 2) inability to
resist the behavior; 3) relief on pulling out of hair; 4) significant functional impairment as a
result; 5) features inconsistent with any other medical or dermatological condition causing
hair loss. Other associated features of this disorder are nail biting and cratching.
15. Because symptoms of hypochondriasis usually develop during periods of stress, pts suffering
from the condition should be asked about their current emotional stressors and then referred
for brief psychotherapy.
16. Pts have the right to refused tx except when doing so would pose a threat to the health and
welfare of others.
17. Schizoaffective disorder is defined as the presence of symptoms of schizophrenia along with
mood symptoms (major depression, bipolar disorder, or a mixed episode). To make the
diagnosis, there should be at least two weeks when psychotic symptoms are present without
any mood symptoms. DD: Major depression with psychotic features—the pt reports depressed
mood, poor sleep, and diminished appetite in the four months since her dog’s death. However,
because her social withdrawal and eccentric behavior presented several months before her
depression started, schizoaffective disorder is the most likely dx.
18. Antisocal personality disorder is diagnosed in pts aged 18 years older who engage in illegal
activities (eg. theft or assault) and disregard the rights of others. These individuals ofter
display evidence of conduct disorder as minors.
19. Marijuana abuse causes behavioral changes and two or more of the following symptoms: dry
mouth, tachycardia, increased appetite, or conjunctival injection.
20. Conversion disorder is likely in a pt with sudden onset neurological symptoms preceded by an
obvious stressor. Pts with conversion disorder may be hysterical or strangely indifferent (“la
belle indifference”) to their symptoms. These pts may show some improvement with the
administration of sodium amytal 异 戊 巴 比 妥 . Tx options include hypnosis and relaxation
techniques in the acute setting, while psychotherapy offers the best long-term results.
21. Antipsychotic medications can cause hyperprolactinemia secondary to their dopamine
blockade effect. Prolactinomas tend to produce very high levels of prolactin (>200 ng/ml).
22. The extrapyramidal symptoms (EPS) frequently seen with typical antipsychotics include
dystonia, Parkinsonism, tardive dyskinesia, akathisia, and neuroleptic malignant syndrome.
The atypical antipsychotic medication most likely to cause EPS is risperidone. Tardive
dyskinesia is characterized by involuntary periodal movements such as biting, chewing,
grimacing, and tongue protrusions. It is best managed by immediately discontinuing the
offending antipsychotic and replacing it with clozapine. Because clozapine is associated with
agranulocytosis, it is typically considered to be a medication of last resort.

167
Pediatrics
1/18/10
1. Jejunal atresia presents with bilious vomiting, abdominal distention, and abdominal
radiography that shows a “triple bubble” as well as gasless lower abdomen. Intestine atresia
can occur anywhere from the duodenum the colon. In general, the more proximal the
obstruction, the earlier the child will develop symptoms. Radiography can usually confirm the
diagnosis by showing air-fluid levels with a gasless lower abdomen. Duodenal atresia appears
as a “double bubble” on radiograph, and 30-40% of affected infants will have other congenital
anomalies (most common Down syndrome). There is only a 10% incidence of associated
anomalies in pts with jejunal atresia. Tx should initially be focused on resuscitation and
stabilization of the pt, followed by surgical correction. The prognosis following surgery
depends on the length of affected bowel as well as the birth age and weight of the infant.
Pyloric stenosis most commonly presents between 2-4 weeks of life with nonbilious, projectile
vomiting as well as visible gastric peristaltic waves and a palpable mass in the epigastric area.
Hirschsprung disease can present with abdominal distention, but is usually suspected by failure to
pass meconium by 48 hours of age.
Necrotizing enterocolitis usually presents with feeding intolerance and abdominal distention.
Abdominal radiographs show dilated bowel loops and pneumatosis intestinalis (air within the
bowel wall).
Gastroesophageal reflux typically presents with nonbilious vomiting. In addition, abdominal
distention is usually not seen.
2. Tricuspid atresia is a cyanotic congenital heart disease characterized clinically by cyanosis
that appears early in life and left axis devation (left ventricle hypertrophy, since the left
ventricle handles both pulmonary and systemic venous returns). This condition is
characterized by an absent connection between the right heart vavities, and a hypoplastic or
absent right ventricle. Ventrical septal defects (holosystolic murmur on auscultation) occur in
90% of cases. Interestingly, the associated heart defects, such as ASD, VSD or PDA are
necessary for survival. The tx generally includes PGE1 (to keep the dutus arteriosus open) and
balloon arterial septostomy (if the ASD is not large enough to allow an adequate flow from
the right to left atriums).
Both tertralogy of Fallot and tricuspid atresia can present with a normal heart size and decreased
vascular marking on CXR; however, the characteristic EKG findings of the former are right atrium
dilation and right ventricle hypertrophy.
In truncus arteriosus, the CXR demonstrates cardiomegaly, increased vascular markings, and right
aortic arch. The typical EKG finding is biventricular hypertrophy.
Common atrioventricular canal is characterized by an endocardial cushion defect involving both
atrial and ventricular septa. The typical EKG finding is are right and left atrial dilation with left
axis deviation.
Ebstein’s anomaly is a rare congenital heart disease that is characterized by a portion of the right
ventricle that has been incorporated into the right atrium, a tricuspid valve that has been displace
into the right ventricular cavity, extreme cardiomegaly, and marked right atrial enlargement.
3. Upper airway compression by a vascular ring most often due to a double aortic arch, right-
sided aorta, pulmonary sling 悬 带 , or anomalous innominate 无 名 的 or left carotid artery,

168
vascular rings can cause tracheal compression. Pts present with signs of chronic upper airway
obstruction including stridor, wheezing, coughing, and shortness of breath. Symptoms are
often worse while supine and relieved with nech extension. Pts are commonly misdiagnosed
as having reactive airway disease but will not respond to inhaled bronchodilators and
corticosteroids. The tx for severe disease is surgery.
4. Polycythemia is defined as a central venous hematocrit level greater than 65%. The MCC of
polycythemia in term infants is delayed clamping of the umbilical cord resulting in excess
transfer of placental blood. As the hematocrit rises, the viscosity of the blood increases as well
as the arterial oxygen content. This change in blood flow may affect various organs. In
newborns, the hematocrit level peaks within 12 hours of birth and then decreases over the
subsequent 12 hours. Not all infants with polycythemia become symptomatic, however,
symptomatic infants may present with manifestations to almost any organ system. The most
common findings are plethora 多血症 and central nervous system disturbances like lethargy,
irritablility, jitteriness, and seizures. Other common symptoms are respiratory distress,
tachypnea, and cyanosis, caused by the decrease in pulmonary blood flow due to the
hyperviscosity of the blood. Poor feeding, hypoglycemia, and hypocalcemia are also
commonly seen in polycethemic infants. Tx includes hydration and, if symptomatic, partial
exchange transfusion.
5. Duodenal atresia presents with bilious vomiting, and its typical radiographic finding is the
‘double bubble sign’. The physical findings of prominent tongue, flat occiput, slanting eyes,
short hands, wide gaps between the first and second digits, and systolic ejection murmur at the
left sternal border with a widely split and fixed S2 (suggestive of atrial septal defect) point
towards Down’s syndrome, which have a high predilection for duodenal atresia.
6. The characteristic description of a Mongolian spot is a well-demarcated, flat, blue/gray, lesion
over the sacral/presacral area that disappears in the first few years of life. This lesion is seen
more commonly in dark skinned children. It is caused by entrapment of melanin-containing
melanocytes in the dermis.
Cutis Marnorata appears as a lace-like pattern on the skin in response to cold or stress. It can
persist in some disease such as Down’s syndrome and trisomy 18.
A salmon patch is a flat salmon-colored lesion commonly seen over the glabella, eyelids, and
neck. It is a vascular lesion that usually disappeara in early childhood.
7. Hyaline membrane disease should be suspected in preterm infants with respiratory distress
and hypoxia not responding to oxygen therapy. It presents as tachypnea, prominent grunting,
intercostals and subcostal retractions, nasal flaring, and duskiness within a few minutes after
birth. The characteristic CXR finding of HMD demonstrates fine reticular granularity of the
lung parenchyma, and tx includes early mechanical ventilation and surfactant administration.
Transient tachypnea usually follows an uneventful normal term vaginal delivery or cesarean
delivery. It is characterized by the early onset of tachypnea, sometimes with retractions or
expiratory grunting, and typically, cyanosis that is relieved by minimal oxygen. The lungs are
usually clear without rales or rhochi, and the CXR shows prominent pulmonary vascular
markings, fluid lines in the fissures, overaeration, a flat diaphragm, and occasionally, pleural fluid.
Hypoxemia, hypercapnia, and acidosis are uncommon.
Persistent pulmonary hypertension of the newborn (PPHN) should be suspected in all term and
post-term infants with cyanosis with or without fetal distress. Persistence of the fetal circulatory

169
pattern of right-to-left shunting through the PDA and foramen ovale after birth is due to very high
pulmonary vascular resistance. The associated hypoxia is universal and unresponsive to 100%
oxygen. The CXR may be normal or may show parenchymal opacification in the chest, depending
on the etiology. Although the clinical presentation of PPH is indistinguishable form HMD, the x-
ray findings of this case are very typical and therefore very suggestive of the latter.
Meconium aspiration syndrome usually occurs in term or post-term infants. Either in utero or
more often with the first breath, thick and particulate meconium is aspirated into the lungs,
resulting in small airway obstruction and consequent respiratory distress that presents within the
first hour of birth. Partial obstruction of some airways may lead to pneumothorax or
pneumomediatinum. Pathy infiltrates, coarse streaking of both lung fields, increased
anteroposterior diameter, and flattening of the diaphragm characterize the typical CXR.
Intracranial hemorrhage in neonates classically presents as periods of apnea, pallor or cyanosis,
poor suckling, abnormal eye signs, high-pitched, shrill cry, muscular twitching, convulsions,
decreased muscle tone or paralysis, metabolic acidosis, shock and a decreased hematocrit. The
fontanel may be tense and bulging. The condition is rarely present at birth and is diagnosed on the
basis of the history, clinical manifestations, and trans-fontanel cranial ultrasonography or CT.
8. ADHD is diagnosed when inattentive or hyperactivity-impulsive symptoms that cause
impairment in two different settings are present for more than 6 months in a child less than 7
years old.
9. Spondylolisthesis 脊 椎 前 移 is a development disorder characterized by a forward slip of
ventebrae (usually L5 over S1) that usually manifests in preadolescent children. In the typical
clinical scenario, back pain, neurologic dysfunction (eg. urinary incontinence), and a palpable
“step-off” at the lumbosacral area are present if the disease is severe.
Other cause of back pain should be considered in this pt. Mechanical causes like lumbosacral stain
and herniated disk, also very common in adults, are uncommon in children. Ankylosing
spondylitis has slow onset, bu neurologic dysfunction argues against this condition. Epidural
abscess is usually accompanied by fever, and the source of infection is frequently present.
Metastatic or pulmonary tumor is a possibility in this pt, but slow development of the pain and a
palpable “step-off” favors spondylolisthesis as the most probable diagnosis.
10. Vitamin D deficiency rickets presents with craniotabes (ping-pong ball sensation over the the
occiput or posterior parietal bones), rachitic rosary (enlargement of the costochondral
jinctions), Harrison groove (horizontal depression on lower border of chest), large anterior
fontanelle, and thickening of the lower end of the long bones. The typical pts are low-birth
weight infants, unsupplemented dark-skinned infants, infants with inadequate sun exposure,
and breastfed infants. The typical pathology is –defecive mineralization of growing bone or
osteoid tissue. The dx can be confirmed by obtaining the serum calcifediol level (decreased),
alkaline phosphatase level (increased), and observing the characteristic radiologic changes
(cupping and fraying of the distal ends of long bones), and double contour along the lateral
outline of the radius. Tx includes oral vit D administration and adequate sunlight (or artificial
light) exposure. The current recommendation to prevent rickets is oral vitD supplementation,
beginning during the first 2 months of life and coutinuing until early adolescence.
Scurvy is commonly seen in 6 to 24 month-old infants with a characteristic history of vitC
deficiency. Clinical manifestations are general tenderness, pseudoparalysis (frog position), bluish-
purple and spongy/swollen gums over the upper incisors, and depression of the sternum. A rosary

170
similar to rickets may be observed, but the angulation of the beads in pts with scurvy is usually
sharper than in those with rickets. The dx is confirmed radiologically (pencil-point thinners of the
long bones’ cortex), and sharply outlined epiphyseal ends.
Nonrachitic craniotabes is unlikely in this pt because this condition usually presents in the
immediate postnatal period, and disappears before rachitic softening of the skull occurs during the
2nd-4th months of life.
Pt with chondrodystrophy may also manifest with “rosary”, however, radiologic findings will
show irregular, concave outlines of the distal ends of the bones. Chondrodystropy is therefore a
less likely diagnosis in this pt.
11. Arthrocentesis followed by empiric tx with iv nafcillin are the most appropriate measures for
the mx of suspected septic arthritis (occur following skin or upper respiratory tract infections)
in a child.
12. In childhood, cystic fibrosis has a wide range of manifestations, such as anemia, heat
intolerance, steatorrhea, wheezing, recurrent infections, hemoptysis, clubbing and rectal
prolapse. A positive sweat chloride test confirms the dx. It must be repeated twice to avoid
any false positives or negatives. Genetic studies should be done when the sweat test is
equivocal. Prenatal screening should be offered to high-risk families. The tx is a high-calorie
diet, pancreatic enzyme replacements and fat-soluble vitamins. Complications of prolonged
CF include gallstones, cirrhosis with portal hypertension and pancreatic fibrosis.
13. HIV test is standardly offered to all pregnant women in the first trimester to prevent possible
transmission to the infant. Persistent oral thrush, lymphadenopathy, and hepatpsplenomagely
may be the presenting symptoms of AIDS in infants.
14. Friedreich Ataxia (FA) MRI of the brain and spinal cord shows marked atrophy of the cervical
spinal cord and minimal cerebellar atrophy. Never conduction velocity results are within
normal limits. EKG shows T wave inversions in the inferior and lateral chest leads. It is an
autosomal recessive condition characterized by an excessive number of trinucleotide repeat
sequences, resulting in an abnormality of a tocopherol transfer protein. The disorder is
progressive with poor prognosis. Most pts are wheelchair bound by the age of 25, with death
occurring by 30-35 years of age. FA is associated with necrosis and degeneration of cardiac
muscle fibers leading to myocarditis, myocardial fibrosis and cardiomyopathy. Cardiac
arrhythmia and congestive heart failure contritbute to a significant number of deaths. T-wave
inversion in this case is most likely due to myocarditis, and not myocardial infarction.
[remember the differential for T-wave inversion: MI, myocarditis, old pericarditis, myocardial
contusion and digoxin toxicity]. Genetic counseling is recommended for prenatal diagnosis
for pts with one affected child.
15. Pubertal gynecomastia is seen in approximately one-half of adolescent boys, at an average age
of 14 years (increase adrenal androgen at adrenarche, androgen synthesis in Leydig cell↑ -
elevating plasma estradiol). It is often asymmetric or transiently unilateral, and frequently
tender. In prepubertal males the testicular size is normally 2 cm in length and 3 ml in volume.
The initial mx involves reassurance and watchful waiting/observation.Gynecomastia can be
physiological (neonatal, adolescent/pubertal and gynecomastia of aging) or pathological
(secondary to drugs, endocrine disorders, tumor, etc) * Extremely high yield!!
16. Rotavirus is the MCC of acute diarrhea in children, esp in those between 6 months and 2 years
of age. Norwalk virus is the second MCC of non-bacterial, acute diarrhea in children. E.coli is

171
the MCC of traveler’s diarrhea. Campylobacter infection is the MCC of diarrhea in adults in
the US. Shigella is the second most common, documented, food-borne disease in US.
17. The healthy appearance of the neonate, the evanescent nature of the rash, and the distinctive
red halo surrounding the lesions support the dx of erythema toxicum. The presence of
numerous eosinophils in the pustules 小 疙 疸 is diagnostic. Erythema toxicum is a benign,
self-limited condition usually found in newborns after the first 2 days of birth. Tx is not
necessary.
Milia 粟粒疹 are small pearly white cysts, and are distinct from the rash of erythema toxicum.
Sebaceous hyperplasia presents as little yellowish papules and are commonly found on the face.
18. Infants with Edwards’ syndrome (trisomy 18) commonly present with microcephaly,
prominent occiput, micrognathia, closed fists with index finger overlapping the 3rd digit and
the 5th digit overlapping the 4th, and rocker bottom feet.
Wolf-Hischhorn’s syndrome is very low-yield for USMLE. You do not need to remember this
disease. Nevertheless, the cause of this condition is a chromosome 4p deletion. Characteristic
findings include a “Greek helmet” facies with ocular hypertelorism, prominent glabella and frontal
bossing, ptosis, and strabismus. Other accompanying findings are microcephaly, dolichocephaly,
hypoplasia of the eye sockets, bilateral epicanthic folds, cleft lip and palate, beaked nose with
prominent bridge, cardiac malformations and mental retardation.
Patau’s syndrome (trisomy 13) can present with cleft lip, flexed fingers with polydactyl, ocular
hypotelorism 间 距 缩 短 , bulbous nose, low-set malformed ears, small abnormal skull, cerebral
malformation, microphthalmia, cardiac malformations, scalp defects, hypoplastic or absent ribs,
visceral and genital anomalies.
Cri-du-chat syndrome (5p deletion) may also present with microcephaly; however, there is a
characteristic protruding metopic suture in such pts. Furthermore, the pt’s cat-like cry will usually
be the tell-tale sign of this condition. Other accompanying manifestations are hypotonia, short
stature, moonlike face, hypertelorism, bilateral epicanthic folds, high arched palate, wide and flat
nasal bridge, and mental retardation.
19. Kawasaki disease is a vasculitis (medium-sized arteries) characterized by prolonged feve
(>39C, for 5 days), conjunctivitis (bilateral and nonpurulent), extremity changes (erythema,
edema, or desquamation of the hands and feet), cervical lymphadenopathy (must be >1.5cm,
usually unilateral), oral changes (erythema, fissured lips, or a strawberry tongue), and a rash.
Supporting lab evidence include sterile pyuria, a CRP >3, an erythrocyte sedimentation rate
>40, and albumin level <3, and elevated ALT, thrombocytosis, and leukocytosis (>12,000).
Other clinical findings may include urethritis, orchitis, arthritis, or hepatitis.
Scarlet fever has a similar presentation to Kawasaki disease. Symptoms may include fever, a
strawberry tongue, a red sandpaper-like rash over the trunk and extremities, and cervical
lymphadenopathy. However, scarlet fever has a shorter duration than Kawasaki disease. Its fever
usually remits by day 3-5, and the rash begins to fade by day 5-6.
Rocky Mountain spotted fever presents with fever and constitutional symptoms followed by a
centripetal rash (starts on the extremities and spreads inward). Arthralgias, myalgias, and headache
may also be present.
20. Coronary artery aneurysms are the most serious complication of Kawasaki disease.
21. Jaudice within the first 24 hours of life requires immediate attention and may be due to
erythroblastosis fetalis, concealed hemorrhage, sepsis or congenital infections. Jaundice that

172
first appears on the 2nd or 3rd day of life is usually “physiologic”, but sometimes may represent
a more severe disorder. Jaundice appearing after the 3rd day and within the 1st week of life
suggests bacterial sepsis or UTI, and requires prompt and aggressive evaluation. Sepsis in the
neonate rarely causes classic findings such as neck stiffness, buldging fontanel or shock. The
susal clues are history of poor feeding, lethargy, vomiting, or alteration of usual activity. All
pts with a suspected dx of sepsis should be investigated with blood cultures and lumbar
puncture.
22. Suspect Beckwith-Wiedemann syndrome in an infant with macrosomia, macroglossia,
visceromegaly, omphalocele, hypoglycemia and hyperinsulinemia. Additional features include
prominent eyes, prominent occiput, ear creases and hyperplasia of the pancreas. It is usually
sporadic, but there are occasional cases involving familiar inheritance. The exact cause is
unknown, although it is sometimes associated with duplication of chromosome 11p. This
region contains the gene encoding for IGF-2, which may explain the macrosomia, Sometimes,
the hypoglycemia may be severe and intractable, and subtotal pancreatectomy may be needed.
Pts have an increased risk of neoplasms such as Wilm’s tumor, hepatoblastoma, and
gonagoblastoma.
Congenital hypothyroidism also presents with hypotonia and an enlarged tongue, and may easily
be confused with Beckwith-Wiedemann syndrome; however, an umbilical hernia (instead of
omphalocele) is another typical finding of hypothyroidism. Furthermore, the head circumference
of pts with congenital hypothyroidism may be increased, whereas pts with Beckwith-Wiedemann
syndrome have microcephaly. There is no hypoglycemia and hyperinsulinemia in pts with
congenital hypothyroidism.
Macrosomia secondary to maternal diabetes is an important differential diagnosis of Beckwith-
Wiedemann syndrome; however, infants with this condition do not present with dysmorphic
features such as omphalocele, prominent occiput and macroglossia. Moreover, the prenatal and
birth histories of the pt in the case were unremarkable. The common congenital problems that you
should remember in an ‘infant of diabetic mother’ are: 1) caudal regression syndrome; 2)
transposition of great vessels; 3) duodenal atredia and small left colon; 4) anencephaly and neural
tube defects.
WAGR syndrome is characterized by Wilm’s tumor, aniridia, genitourinary anomaly and mental
retardation. It is related to a deletion in chromosome 11 involving the gene WT1 and aniridia gene
PAX6.
Denys-Drash syndrome is also associated with an increased rish of Wilm’s tumor, but its clinical
features are totally different from those of Bechwith syndrome. It includes male
pseudohermaphrodism and early onset renal failure characterized by mesangial sclerosis.
The symptoms of galactosemia manifest a few days or weeks after the infant starts taking formula
or breast milk. These include liver failure (hepatomegaly, direct hyperbilirubinemia, disorders of
coagulation), abnormal renal function, emesis, anorexia, acidosis and glycosuria.
Von-Gierke disease is secondary to a deficiency of glucose-6-phosphatase, It also affects the
kidney and liver, and cause severe hypoglycemia; however, the hypoglycemia characteristically
occurs with fasting, since it occurs secondarily to a failure to release glucose from the liver into
the circulation, rather than to hyperinsulinemia.
23. No investigations are needed in the case of migraine headache, unless the neurological exam
is abnormal and/or fever is present. Migraine is a clinical dx, and the initial tx is

173
acetaminophen or NSAIDs.
24. Laryngomalacia or congenital flaccid larynx is the MCC of chronic inspiratory noise in
infants. Larygoscopy shows flaccidity of the larynx, and collapses during inspiration
(epiglottis rolling in from side to side). It is a self-limiting condition in most cases, and
generally subsides by 18 months of age. The mother should be instructed to hold the child in
an upright position for half an hour after feeding, and to never feed the child when he is lying
down.
25. Neuroblastoma is the third most common cancer in the pediatric population (after leukemia
and CNS tumors). The tumor arise form neural crest cells, which are also the precursor cells
of the sympathetic chains and adrenal medulla. The most common site is the abdomen.
Calcifications and hemorrhages are seen on plain x-ray and CT scan. The levels of serum and
urine catecholamines and their metabolites (ie HVA and VMA) are usually elevated.
26. Risperidone is a dopamine and serotonin antagonist that can cause weight gain or
hyperprolactinemia, the latter of which can lead to amenorrhea and galactorrhea. Lamotrigine
is used in the tx of epilepsy and bipolar disorder. The major side effect of lamotrigine is a
rash, although severe skin reactions can develop including Steven-Johnson syndrome or toxic
epidermal necrolysis.
27. Salicylates are C/I in young children with viral infections. Recognize the clinical presentation
of Reye syndrome.
28. Suspect vit A deficiency in a 2 or 3 year old child with impaied adaption to darkness,
photophobia, dry scaly skin, xerosis conjunctiva, xerosis cornea, keratomalacia, Bitot spots
and follicular hyperkeratosis of the shoulders, buttocks and extensor surfaces.
29. Four clinical criteria have been shown useful in differentiating septic arthritis from transient
synovitis: white blood cell count >12,000/mm3, temperature >39C (102F), ESR>40mm/h,
and refusal to bear weight. If at least 3 of these 4 criteria are met, further work-up is indicated
to rule out septic arthritis. Transient synovitis is treated with rest and NSAIDs.
30. Suspect choanal atresia in an infant who presents cyanosis that is aggravated by feeding and
relieved by crying.
31. Acute, unilateral cervical lymphadenitis in children is usually caused by bacterial infection,
and the most common pathogen is Staph aureus.
32. Vaginal discharge in the newborn is due to the effects of maternal estrogens. In such cases,
reassurance of the mother is all that is required.
33. Early neonatal care in an uncomplicated pregnancy include initial physical assessment,
removal of airway secretions, drying the infant and keeping him/her warm, and early
preventive measures (gonococcal ophthalmia prevention, vit K supplementation).
34. Always suspect pyloric stenosis in a neonate who presents with non-bilious, projectile and
persistent vomiting. In such cases, order an abdominal ultrasound to establish the dx.
35. Medulloblastoma is the second most common (after cerebellar astrocytoma) infratentorial
tumor in children and arised form vermis. Recognize posterior vermis syndrome.
36. A large thymic shadow (sail sign) is a normal finding on chest radiographs in children less
than 2 years old.
37. Painless rectal bleeding in a young child often presents a Meckel’s diverticulum (fecal occult
blood test positive), which is diagnosed by performing a technetium-99m pertechnetate scan.
38. A 2-year-old child can build a tower of 6 blocks, obey two-step commands, and use 2- to 3-

174
words phrases.
Language Gross Motor Fine Motor Social
2 months Social smile Recognizes
parents
3 months Holds head
4 months Rolls back-to-
front and front-
to-back
6 months Babbles Sits well Raking grasp Recognizes
upsupported stangers (stranger
anxiety)
12 months 2 words; obeys Walks alone Throw objects Imitates action
1-step command
15 months Build tower of 2
blocks
18 months Plays with other
children
24 months 2-3 words; obeys Walks up- and Build tower of 6 Parallel play
2-step command downstairs blocks; Turn
without help pages of books

39. Conduct disorder is characterized by disruptive behavior patterns that violate basic social
norms for at least one year in pts less than 18 years old.
40. Chlamydia is the MCC agent of infectious neonatal conjunctivitis. Chlamydial pneumonia can
develop in infected infants. Gonococcal conjunctivitis is extremely purulent and presents
between the 2nd and 5th day after birth.
41. Osteogenesis imperfecta is an autosomal dominant disease characterized by mutations in type
I collagen. Pts with the severe type II form of this disease typically expire in utero due to
multiple ntrauterine and/or perinatal fractures. Clinical findings include limb deformities,
growth retardation, multiple fractures and blue sclerae.
42. Pts with genetic B-cell deficiencies begin to develop recurrent infections after passing 6
months of age. The deficient humoral immune response in these pts impairs the body’s ability
to destroy encapsulated organisms. Hence, recurrent sinopulmonary infections with H.
influenzae and S. pneumoniae are common. Lack of IgA also predisposes to Giardia infection.
Chronic granulomatous disease (CGD) is a condition of impaired oxidative metabolism. In CGD,
a defect in the NADPH-oxidase system of phagocytic cells results in defective intracellular killing.
Pts are therefore prone to abscesses secondary to catalse-producing organisms like Aspergillus and
Staphylococcus.
Gonococcal and meningococcal infections are common in pts with complement deficiency.
S.Pneumoniae and H.influenzae infections do occur in this pt population as well, but infections
with Giadia are not typically seen.
Thymic hypoplasia is one element of DiGeorge syndrome. The resultant T-cell deficiency puts pts
at high risk for viral and fungal infections.
Adenosine deaminase deficiency is commonly the genetic defect underlying severe combined
immunodeficiency (SCID). Pts with SCID have a deficiency of both B- and T- cells. The T-cell
deficiency predisposes to viral and fungal infections and the B-cell deficiency predisposes to
infection by bacteria.
43. A 12-month-old child walks alone, speaks two words, throws objects, and comes when called.

175
44. Apgar score
0 1 2
Color Body and Body is pink and Body and
extremities are extremities are blue extremities are
blue/pale pink
Heart rate No activity <100 beats/min >100 beats/min
Reaction to nasal stimulatoin No response grimace Active cough
Tone/Activity limp 柔软的 Some flexion of active flexion of
extremities extremities
Respirations absent Slow and irregular Good respiratory
effort

1/19/10
1. Most small ventricle septal defects close paontaneously and require no tx, as
long as there are no signs of pulmonary vascular disease. Reassurance, surveillance (via EKG
and echocardiograph), and infective endocarditis prophylaxis are all that are needed for mx.
VSD murmur: harsh and holosystolic heard at the left, lower sternal border. Surgical repair si
indicated in large or symptomatic defects.
2. Primary amenorrhea, Tanner stage 2 development, and aortic coarctation in a
teenager strongly suggest Turner syndrome (45, XO). Turner syndrome is the MCC of
primary amenorrhea. Dx is confirmed by karyotype analysis.
17-hydroxyprogesterone measurement is indicated when congenital adrenal hyperplasia is
suspected as a possible cause of primary amenrrhea. Serum 17-OH progesterone is elevated in 21-
and 11 hydroxylase deficiencies and is decreased in 17-hydroxylase deficiency.
3. Cystic fibrosis is an autosomal recessive disorder. If one of the parents has the
disease (is homozygous), you cannot predict whether a child would be likely to have the
disease without knowing the other parent’s carrier status.
4. Mongolian spots are most commonly found in dark-skinned populations,
present at birth, and usually fade or disappear in several years.
5. The most common causative organism of acute otitis media is Streptococcus
pneumoniae (40% of cases), followed by non-typable Haemophilus influenzae (25-30%), and
Moraxella catarrhalis (10-15%). The first-line tx is a ten-day course of amoxicillin.
6. Hand-foot syndrome or dactylitis is the earliest manifestation of vaso-
occlusion in sickle cell anemia, thereby warranting a complete workup for previously
asymptomatic sickle cell pts.
7. Rubella is characterized by low-grade fever, lymphadenopathy (sub-occipital
and posterior auricular) and rash. The rash is erythematous, maculopapular and classically
begins on the face, spreading subsequently down the body.
Erythema infectiosum or Fifth disease is caused by human parvovirus B19. Childre have different
presentations and develop a rash with a “slapped cheeks” appearance. Fever is not present or very
mild in this disease.
The clinical manifestation of chickenpox in healthy children generally develop within fifteen days
after the exposure and typically include a predrome of fever, malaise, or pharyngitis, followed by
the development of a generalized vesicular rash, usually within 24 hours. The lesions are
commonly prutitic and appear as successive crops of vesicles over a three to four day period. Pts
typically have lesions in different stages of development on the dace, trunk and extremities. New

176
lesions formation generally stops within 4 days, and most lesions are fully crusted by the 6 th day in
normal hosts.
Roseola infantum is caused by herpes virus 6, and is characterized by the abrupt onset of high-
grade fever, which is then followed by a maculopapular rash, appearing on the trunk and then
spreading peripherally. The pt is no longer febrile when the rash develops. There are no positive
physical signs such as sore throat or lymphadenopathy during the febrile stage.
8. Friedreich ataxia is the most common type of spinocerebellar ataxias.
Remember the combination of neurologic (ataxia, hysarthrial), skeletal (scoliosis, feet
deformities), and cardiac (concentric hypertrophy cardiomyopathy) manifestations of the
disease. The MCC of death are cardiomyopathy and respiratory complications. * Very high
yield!!
9. Reactive pericarditis with a pericardial effusion can present after surgery for
congenital heart disease and is called postpericardiotomy syndrome. Symptoms usually occur
1-6 weeks after surgery. Although the cause is no known, postpericardiotomy syndrome is
thought to be an autoimmune response, possibly to a viral infection. Most children develop
mild symptoms which are self limited. In infants, pericardial effusion can present with
abdominal pain, vomiting, and decreased appetite. Older children may complain of chest pain.
Fever is often present. On exam, findings are consistent with pericardial and/or pleural
inflammation, including tachycardia, poor perfusion, pulses paradoxus, distant heart tones,
and jugular venous distention. If the effusion is large enough to affect the function of the
heart, pericardial tamponade can occur.
10. In an infant with meningococcemia, watch out for Waterhouse-Friderichsen
syndrome, which is characterized by a sudden vasomotor collapse and skin rash due to
adrenal hemorrhage.
11. The differential diagnosis of a solitary, painful, lytic long bone with overlying
swelling and hypercalcemia in a child should include Langerhans cell histiocytosis
(histoobtosis X, eosinophilic granuloma—the least severe form) as well as other neoplastic
processes like Ewing sarcoma.
12. Remember the classic triad of congenital rubella syndrome (CRS)-
sensorineural deafness, cardiac malformation (eg. PDA and ASD), and cataracts. CRS occurs
most commonly when transmission occurs in the first 4 weeks of pregnancy.
Struge-Weber syndrome is a neurocutaneous disease characterized by a port-wine stain on
trigeminal nerve distribution, mental retardation, seizures and glaucoma.
13. Microcytic anemia caused by iron deficiency can often be differentiated from
thalassemia by an elevated RDW, which is typically greater than 20% in iron deficiency. The
reticulocyte count is low in pts with iron deficiency due to decreased erythropoiesis.
14. A typically presentation of Tetralogy of Fallot (TOF) individuals is squatting,
which increases the systemic vascular resistance and forces blood into the lungs, thus
improving cyanosis. Other manifestations include “tet” spells, which are hypoxic episodes
that are characterized by paroxysms of deep, rapid breathing, and are caused by an increased
pulmonary vascular resistance. Crying, infection, and exercise can lead to tet spells. Severe
spells may precipitate seizures and loss of consciousness. The immediate tx is asministration
of oxygen and placing the child in a knee-chest position, following by the administration of
fluids, morphine and propranolol.

177
15. The MCC of viral meninigitis are the non-polio enteroviruses, such as
echoviruses and coxsachievirus.
16. Marfan’s syndrome (MFS) is an autosomal dominant disorder that results
from the mutations of the fibrillin-1 (FBN1) gene. It presents with tall stature, long and
emaciated extremities, arachnodactyly, hypermobility of the joints, upward lens dislocation,
and aortic root dilation.
Homocystinuria is an autosomal recessive disorder that results from cystathinine synthase
deficiency, an enzyme involved in the metabolism of methionine. These pts share many features of
Marfan’s syndrome but they usually have a fair complexion 肤 色 , thromboembolic events and
osteoporosis. The other main differentiating feature is the type of lens dislocation. In Marfan’s
syndrome, the lens is typically dislocated upward, whereas in homocystinurea it is dislocated
downward.
Congenital contractural arachnodactyly (CCA) is an autosomal dominant condition that presents
with tall stature, arachnodactyly, and multiple contractures involving large joints. Marfan’s pts do
not have joint contractures. Furthermore, ocular and cardiovascular symptoms are not present in
CCA.
Ehlers-Danlos syndrome is a disorder of collagen structure, which may also have hypermobile
joints; however, it also results in easy bruising, poor wound healing and soft, velvety, and
hyperelastic skin. Severe complications such as organ rupture and severe hemorrhage may occur.
17. Internal carotid artery dissection is a potential cause of stroke in children. The
history of a fall on a pencil or on a stick in a child’s mouth within 24 hours of the onset of
symptoms is typical.
18. In pts with sickle cell disease, acute severe anemia with absent reticulocytes in
the peripheral blood smear is due to aplastic crisis.
19. Recurrent hemarthroses in pts with coagulopathies lead to a joint injury called
‘hemaphilic arthropathy’.
20. Transposition of the great vessels (a normal S1, single and a loud S2, and no
murmur) is the most common cysnotic heart disease which presents with cyanosis in the first
24 hours of life. It is commonly seen in infants of diabetic mothers and in males.
Echocardiography confirms the dx. Maintaining the patency of the ductus arteriosus is
important for survival. Surgical tx is definitive.
21. Deficiency of sphingomyelinasae causes Niemann-Pick’s disease, which is
characterized by cherry red macula, protruding abdomen, hepatosplenomegaly,
lymphagenopathy, and regression of development milestones.
Sphingolipidosis due to a deficiency in hexosaminidase A is known as Tay-Sachs’ disease. It is
characterized by hyperacusis, mental retardation, seizure, cherry red macula, but not
hepatosplenomegaly or cervical lymphadenopathy.
Sphingolipidosis due to a deficiency in glucocerebrosidase is known as Gaucher’s disease. It is
characterized by hepatosplenomegaly, anemia, leucopoenia, and thrombocytopenia, but not cherry
res macule.
Sphingolipidosis due to a deficiency in galactocerebrosidase is known as Krabbe’s disease. It is
characterized by hyperacusis 听觉过敏, irritability and seizure.
Mucopolysaccharidoses are characterized by coarse facial features, hydrocephalus and umbilical
hernia.

178
22. Turner syndrome should be considered in newborns with a webbed neck, high
palate, short fourth metacarpal, and nail dysplasia. Lymphedema frequently occurs due to
abnormal development of the lymphatic system.
23. The most common predisposing factor for acute bacterial sinusitis is a viral
upper respiratory infection.
24. Bedwetting is considered normal until the age 4-5 years. If nocturnal enuresis
perisists, DDAVP (drug of choice) or imipramine may be used.
25. Post-exposure rabies prophylaxis can be lifesaving and should be initiated
after exposure to saliva or neural tissue from bats, wild carnivores, or other animals suspected
of being rabid. People bitten by domestic animals suspected of being rabid or not available for
observation, or by wild carnivores like raccoons, skunks, and foxed should also receive
prophylaxis.
26. Aplastic anemia should be suspected in any pt with pancytopenia following
drug use, exposure to toxins or viral infections. A bone marrow biopsy is essential to make the
dx, it typically shows profound hypocellularity with a decrease in all cell lines and fatty
infiltration of the marrow.
Classically, pts with Fanconi’s anemia have pancyotpenia and characteristic congenital anomalies,
such as hyperpigmentation on the trunk, neck and intertriginous areas and/or café-au-lait spots,
short stature, upper limb abnormalities, hypogonadism, skeletal anomalies, eye or eyelid changes,
and renal malformations. Blood counts start to decrease between 4 and 12 years of age, and the
initial manifestation is usually thrombocytopenia, followed by neutropenia, then anemia.
Diamond-Blackfan anemia (DBA), or congenital pure red cell aplasia, presents in the first 3
months of life with pallor and poor feeding. CBC reveals a normocytic or macrocytic anemia with
reticulocytopenia. WBC and platelet counts are normal.
Transient erythroblastopenia of childhood (TEC) is an acquired red cell aplasia which occurs in
healthy children between 6 months and 5 years old. There is a gradual onset of symptoms such as
pallor and decreased activity. The physical exam is unremarkable except for pallor and
tachycardia. The typical lab findings ar normocytic normochromic anemia, with hemoglobin
levels ranging from 3 to 8 g/dL, and extremely low reticulocyte count.
Bone marrow infiltration due to leukemia results in pancytopenia by crowding out the normal
bone marrow element.
27. Suspect necrotizing enterocolitis in a newborn with abdominal distention,
bloody diarrhea, and leukocytosis. These symptoms usually occur after the introduction of
formula feeding. The radiologic finding of pneumatosis intestinalis (ie intramural air) in
infants is diagnostic.
28. Undetected hearing impairment can earsily be confused with certain pervasive
and behavioral disorders of childhood. Therefore, hearing tests should be conducted in all
such children.
29. Compartement syndrome can be a complication of supracondylar humerus
fractures (X-ray picture). It is characterized by severe pain, pallor, poikilothermia,
paresthesias, and the late finding of pulselessness and paralysis. Initial tx incudes removal of
any bandages, measurement of compartment pressures, and emergent orthopedic evaluation
for possible fasciotomy.
30. Intussusception is the most common cause of intestinal obstruction in the first

179
two years of life. In addition to the signs of interstinal obstruction (eg. severe, sudden-onset
abdominal pain and vomiting), pts present with red currant jelly stool containing blood and
mucus.
31. Fetal alcohol syndrome is characterized by irritability, mild to moderate
mental retardation, hypoplastic maxilla, long philtrum, thin upper lip border, and
microcephaly.
Fetal hydantoin syndrome is characterized by hypoplasic nails, cleft palate, and vitK deficiency,
which may result in bleeding.
32. Nursemaid’s elbow occurs when infants or children are lifted or pulled by the
hand or arm. The mechanism of the injury involves radial head subluxation. The child
typically keeps the hand in a pronated position, and refuses (cries out in pain) attempted
forearm supination.
Panner disease is osteochondrosis of the capitellum 小头. The typical pt is an adolescent who is
actively engaged in sport activities that involve throwing, and the common complaints include
pain, crpitation and loss of motion of the arm (particularly pronation and supination).
The most common elbow dislocation is posterior dislocation, with is caused by falling backward
onto the outstretched arm with the elbow extended. An obvious deformity is noted, with the
olecranon process displaced prominently behind the distal humerus.
33. Colic presents by 3 weeks of age with excessive crying for more than 3 hours
a day, more than 3 days a week, and more than 3 weeks a month, usually resolving by 4
months of age.
34. Anemia of prematurity is the most common anemia in premature and low birth
weight infants. The pathology involves a combination of diminished RBC production,
shortened RBC life span, and blood loss. Iron supplementation does not prevent falling
hemoglobin levels, and iron deficiency is not the cause of anemia of prematurity. The tx
involves iron supplementation, periodic hemoglobin checking and blood transfusion, if
needed. Erythropoietin is not routinely used. Lab studies: 1) Peripheral smear shows
normocytic and normochromic anemia. No other abnormal forms are seen. 2) the reticulocyte
count is low, and RBC precursors in the bone marrow are decreased. 3) Normal WBC and
platelet counts. 4) Normal total bilirubin level.
35. ADHD should be considered as the most probable dx in any pts with
symptoms of short attention span, impulsivity, and hyperactivity for more than 6 months in
more than one setting.
36. Vaginal foreign bodies should be suspected in children with purulent, foul-
smelling vaginal discharge and bleeding. If a foreign body is seen, irrigation with warmed
fluid should be performed in an attempt to flush the foreign body. If irrigation is unsuccessful,
exam and foreign body removal should be done with sedation or general anesthesia.
37. The body derives vit K from the diet and from gut flora synthesis. Deficiency
in newborn babies is the result of poor placenta transfer, absent gut flora, and inadequate
levels in breast milk. To prevent hemorrhagic disease of the newborn, it is recommended that
all newborn babies receive a vit K injection.
38. Racemic epinephrine decreases the need for intubation in pts with croup and
should always be tried before any invasive procedure.
39. Painless melena in a 2 to 3-year-old child is most likely due to Mechel’s

180
diverticulum. It results from the failure of the vitelline duct to obliterate during the fetal
development. The dx of a Meckel’s diverticulum is best made with technetium-99m
pertechnetate scanning (uptake by heterotopic gastric mucosa).
40. The tx of Kawasaki’s disease consists of aspirin and IV immunoglobulins.
41. Suspect Duchenne muscular dystrophy in a child with proximal muscle
weakness, positive Gower’s sign, depressed reflexes, and pseudohypertrophy of the calf
muscles. Dystrophin is usually absent. EMG shows myopathic pattern. Serum CK levels are
very high (4000-5000IU). The dx is confirmed by muscle biopsy.
42. Human milk is the ideal form of nutrition for term infants. The major protein
source is whey 乳清, which is more easily digested than casein 酪蛋白 and helps to improve
gastric emptying.
43. Infectious mononucleosis is an infection caused by the EB virus, and is
sometimes detected only when the pt develops a characteristric polymorphous rash after
taking ampicillin for an apparent upper respiratory tract infection.
44. Severe dehydration in a neonate in the presence of hyponatremia,
hyperkalemia, hypoglycemia, and metabolic acidosis suggests a diagnosis of congenital
hyperplasia or salt wasting 21-hydroxylase deficiency.
There are 2 main abnormalities involving the galactose metabolic pathways. Galactose-1-
phosphate uridyl tranferase deficiency is a sever condition that can lead to shock if untreated.
Clinical features include vomiting, diarrhea, hepatomegaly, catatect, mental retardation, and
hypoglycemia. Galactokinase deficiency is a milder condition which generally presents with
catatact if untreated. Both conditions are inherited as autosomal recessive.

1/20/10

1. Subarachnoid hemorrhage (SAH) can be caused by by an intraventricular hemorrhage, which


is common in premature infants. Accumulation of the blood in the subarachnoid space may
lead to destruction of the arachnoid villi and cisterns, thereby blocking the flow or decreasing
the absorption of CSF, and leading to communicating hydrocephalus (CT scan shows dilation
of the entire ventricular system with distinct enlargement of the subarachnoid space over the
cerebral cotex). SAH is the most common cause of communicating hydrocephalus. Dandy-
Walker anomaly and Chiari malformation will both reveal CT findings consistent with
obstructive or noncommunicating hydrocephalus. Dandy-Walker Anomaly will demonstrate a
cystic expansion of the 4th ventricle, and Chiari malformation will reveal protrusion of the
structures of the posterior fossa through the foramen magnum.
2. Intussusceptin is best diagnosed and treated by an air contrast enema.
3. Suspect meningococcemia in a neonate with signs of meningitis and a petechial rash. 75% of
pts with Meningococcus meningitis present with a petechial rash that is prominent on the
axilla, wrist, flanks and ankles. It appears with 24 hours of the infection, and the pt generally
appears sick.
4. Sturge-Weber syndrome is a neurocutaneous syndrome that is characterized by a congenital
unilateral cavernous hemangioma along the trigeminal nerve distribution and radiographic
evidence of intra-cranial calcifications that resemble a tramline.
5. Increasing head circumference and signs of increased intracranial pressure in children should

181
be evaluated with a CT scan of the brain. Symptoms of hydrocephalus in infants include poor
feeding, irritability, vomiting, and decreased activity. On exam, the fontanelle may be tense
and buldgig, the scalp veins may appear prominent with shiny, tight skin overlying the
vessels, and the cranial sutures may be widely spaced. On review of the growth charts, the
head circumference may be rapidly increasing in size to greater than the 97th percentile. Tx
consists of a shunt that is placed form the ventricle to the peritoneum, pleura, or right atrium,
which allow the excess CSF to drain rather than continue to collect in the ventricles.
6. With suprachondylar fracture, the brachial artery can be compromised, resulting in the loss of
the radial artery pulse; therefore, the radial artery pulse must be assessed when the fracture is
reduced.
7. To prevent SIDS (sudden infant death syndrome), infants should be placed on their backs
(supine position) while sleeping. Home monitors do not decrease the risk of SIDS.
8. A positive Coomb’s test points towards autoimmune hemolytic anemia, and a positive osmotic
fragility test points towards hereditary spherocytosis. Hereditary elliptocytosis is a rare
disorder that is characterized by elongated RBC. Microcytes, spherocytes, and other
poikilocytes may also be seen.
9. Recognize fetal alcohol syndrome and know that it is the most common cause of mental
retardation in children. Remember the midfacial abnormalities (short palpebral fissures,
epicanthal folds, ling philtrum, thin upper lip).
10. Suspect ITP in children who develop isolated thrombocytopenia after a viral infection. ITP is
a relatively benign condition. There is some controversy regarding the tx, but in general,
corticosteroids are the drugs of choice in all age groups for thrombocytopenia less than
30,000/mm3, and/or for sever symptoms. Pts with a platelet count of more than 30,000/mm3
usually have very few symptoms and do not require tx.
11. An immediate anaphylactic reaction, an encephalopathy, or any CNS complication within 7
days of administration of the vaccine is a C/I for further administration of DTaP. In these
instrance, DT should be substitute for DTaP since the adverse reactions are susally attribute to
the pertussis compoment of the vaccine. * Extremely high yield!!
12. RSV infection may increase the risk of asthma later in life.
13. Children with a parental history of elevated total cholesterol levels (>240mg/dL), or risk
factors for coromary artery disease should get a screening test for total cholesterol level. A fast
lipid profile is recommended if the total cholesterol level is greater than 200mg/dl. It is
usually not the first step in evaluating an asymptomatic pt.
14. Henoch-Schonlein purpura (HSP) is characterized by palpable purpura, scrotal swelling,
hematuria and abdominal pain. Children with HSP are susceptible to intussusception.
15. Simple febrile seizures do not typically require an extensive work-up, and these pts can
usually be discharged home from the emergency department.
16. Slipped capital femoral epiphysis is a common hip disorder seen in overnight adolescents. On
exam, affected pts tend to hold the hip in passive exernal rotation and exhibit decrased
internal rotation, abduction, and flexion. Dx is made with plain radiographs of the hip (Ap and
frog leg lateral view), which show the posteriorly and inferiorly displaced femoral head. Tx
consists of emergent orthopedia consultation and surgical fixation at the current degree of
slippage to avoud the risk of avascular necrosis. Avascular necrosis can also occur in Legg-
Calve-Perthes disease, which is most commonly seen in younger children (age 5-7) with

182
idiopathic infarction of the femoral head.
17. Turner’s syndrome with 46 XY karyotype is associated with a higher incidence of
gonadoblastoma, hence, prophylactic bilateral gonadectomy is indicated in the mx of such pts.
18. Renal tubular acidosis (RTA) is a normal anion gap metabolic acidosis caused by a defect in
the ability of the renal tubules to reabsorb bicarbonate or excrete hydrogen. There are 3 types
of RTA:
1) Type 1 RTA or distal RTA occurs due to a defect in hydrogen ion secretion. These pts are
acidotic, hypokalemic, and have an elevated urinary PH. In children, Type 1 RTA is often a
genetic disorder. Pts commonly develop nephrolithiasis.
2) Type 2 RTA is caused by decreased bicarbonate reabsorption in the proximal tubule. Fanconi
syndrome is a common cause in children.
3) Type 4 RTA is caused by a defect in the sodium/potassium exchange in the distal tubule
which results in hyperkalemia, hyperchloremic acidosis. In children, obstructive uropathy,
renal disease, or multicystic dysplastic kidney are common causes.
RTA can present as growth failure and should be considered in the differential dx for failure to
thrive. Screening labs will show a low bicarbonate level with an increase in chloride, producing a
normal anion gap metabolic acidosis.
19. Down syndrome pts are prone to endocardial cushion defects which can rapidly cause
pulmonary hypertension.
20. Mammary gland enlargement and non-purulent vaginal discharge are common findings in
newborn infants. These are transitory physiologic events; therefore, such infants only require
observation and routine care. * Extremely high yield!!
21. Pts with functional asplenia are at risk of infection with capsulated organisms, and the most
common cause of sepsis in such pts is pneumococcus. Pts with sickle cell disease encounter
constant clumping of sickle cells, which leads to repeated microinfarctions in the spleen. By
2-3 years of age, most such children have functional asplenia. Since one of the spleen’s
important immunologic functions is the removal of capsulated organisms such as
pneumococci and H. influenza, pts with functional asplenia are at risk of infection with these
organisms. For this reason, antibiotic prophylaxis and vaccination against pneumococci and
H. influenza are standardly given to pts with functional asplenia.
22. Ventrical septal defects can cause failure to thrive, easy fatigability, and heart failure. On
exam they characteristically cause a pansystolic murmur loudest at the left lower sternal
border, plus a rumbling diastolic flow murmur at the apex.
23. X-linked agammaglobulinemia is an inherited immune deficiency characterized by recurrent
bacterial infections early in life. Suggestive lab data includes a normal amount of T
lymphocytes with a low or absent concentration of B lymphocytes. It is treated with regular
infusions of iv Ig.
24. Painless gross hematuria is the most common presentation of sickle cell trait. The
pathophysiology involves a defect in the renal tubular function, thereby leading to the
inability to properly concentrate urine.
25. The long-term neurologic sequelae associated with bacterial meningitis are: 1) hearing loss, 2)
loss of cognitive functions (due to the neuronal loss in the dentate gyrus of the hippocampus).
3) seizures, 4) mental retardation. 5) spasticity or paresis.
26. 21-hydroxylase defeiciency is the most common form of CAH. It presents with virilism, salt

183
wasting, and increased 17-alpha-hydroxyprogesterone level. Other deficiencies leading to
CAH include (1) 11-alpha-hydroxylase deficiency, which results in androgen and
mineralocorticoid excess, and (2) 3-beta-hydroxysteroid dehydrogenase deficiency, which
results in DHEA-S excess and decreased testosterone and mineralocorticoids. In 21-
hydroxylase defeiciency, all other metabolites are decreased.
27. Chronic pyelonephritis is characterized by focal parenchymal scarring and blunting of calices
on IVP. Hydronephrosis is seen on IVP as dilation of the collecting system including the
calyces, pelvis and ureter, depending on the level of obstruction.
28. All children with recurrent episodes of nocturnal vulvar itching should be examined for
pinworms (Scotch tape test) and treated empirically with mebendazole.
29. Pts with Down’s syndrome are more likely to develop duodenal atresia, Hirschsprung’s
disease, endocardial cushion defects and acute leukemia.
30. Food, esp peanut, allergies are the major cause fo outpatient anaphylaxis in children. Signs
include bronchoconstriction, hypotension and urticaria. Subcutaneous injection of epinephrine
is the first-line tx for anaphylaxis in a pt with a patent airway.
31. In pts with apparent subcutaneous emphysema secondary to severe coughing paroxysms,
CXR must be obtained first ot rule out pneumothorax.
32. Vit A can be of benefit in the tx of measles infection.
33. Pts present with precocious pubarchem, it is very important to differentiate between
precocious puberty that is caused by premature activation of the hypothalamus-pituitary-
gonad (HPG) axis and precocious pseudo-puberty that is casued by a gonagotropin-
independent process, typically an excess of sex steroids. Precocious pubarche with signs of
severe androgen excess is suggestive of precocious pseudo-puberty that is caused by a
gonadotropin-independent process (typically an excess of sex steroids). It can be caused by
late-onset congenital adrenal hyperplasia.
Hypothalamic dysfunction leading to precocious puberty is usually less dramatic in presentation.
Sequential development of testicular enlargement, penis enlargement, pubic hair growth, and then
growth spurt is typically present.
34. Displacement anterior fat pad is a radiographic sign of supracondylar fracture, which may be
complicated by Volkmann’s ischemic contracture.
35. Minimal change disease is a highly steroid-sensitive condition and is the MCC of nephritic
syndrome in children. For these reason, empiric steroid therapy is indicated in any child with
a clinical presentation and findings that are suggestive of nephritic syndrome.
36. Edward’s syndrome is characterized by micrognathia, microcephaly, rocker bottom feet,
overlapping fingers, and absent palmar creases. Ventricular septal defect is common in pt with
this disease.
ASD and endocardial cushion defects –trisomy 21
Supravalvular aortic sternosis—William’s syndrome
Conotruncal abnormalities (Truncus arteriosus, Tetralogy of Fallot, interrupted aortic arch)—
CARCH-22 syndromes, including DiGrorge and velocardiofacial syndromes
Congenital heart block—neonatal lupus
Paten ductus arteriosus—congenital rubella
Coronary artery aneurysm—Kawasaki disease
37. Henoch-Schonlein purpura is an IgA-mediated vasculitis of small vessels, which results in

184
rashes, arthralgias, abdominal pain and renal disease. Immunofluorescence microscopy
reveals IgA deposition in the kidney.
38. Anabolic steroids are used to improve physique and athletic performance but they are
associated with numerous adverse effects, including acne, baldness, gynecomastia, hepatic
dysfunction, altered lipid profiles, virilization, testicular failure, and mood and behavior
changes.
39. Neonatal tetanus is generally seen in infants born to unimmunized mothers, frequently
following umbilical stump infection. Affected infants initially present in the first 2 weeks of
life with poor suckling and fatigue, following by rigidity, spasms and opisthotonus.
40. Rheumatic fever is a complication of streptococcal pharyngitis and is diagnosed clinically
using the Jones criteria. The major criteria include carditis, migratory polyarthritis, Sydenham
chorea, subcutaneous nodules, and erythema marginatum.
Juvenile RA is diagnosed by arthritis that is present for greater than 6 weeks. Systemic symptoms
including a rash can be seen in children with JRA, but the arthritis is usually not migratory as
described in this pt.
41. Stranger anxiety is the normal anxiety experienced by infants when they are exposed to
unfamiliar individuals. It peaks at 12-15 months.
42. Isotonic solutions such as normal saline are the fluid of choice for initial resuscitation in
severe hypovolemic hypernatremia.
43. Although adequate hydration is very important in the mx of diarrhea, replacement of
electrolytes is also essential to prevent complications such as water intoxication and
hypomatremia. Water intoxication, hyponatremia, and seizure may result if diarrhea is treated
with large amounts of hypotonic or low-solute fluids (eg. water).
44. The vaccination schedule for preterm infants should be conformed to the child’s chronologic
age, not the gestational age. The exception is that children should be 2kg prior to receiving the
first HBV vaccine.

1/21/10
1. IVH is most commonly seen in premature and LBW infants. Pts may present with pallor,
cyanosis, hypotension, seizurem focal neuologic signs, bulging or tense fontanel, apnea, and
bradycardia; however, many cases remain asymptomatic, thus mandating transfontanel
ultrasound for all newborns with predisposing risk factors.
2. In the pediatric population, infratentorial tumors are more common than supratentorial
tumors, and benign astrocytomas are the most common histology type in both groups.
Medulloblastoma is the second most common.
3. Bedwetting is a normal phenomenon until the age of 5.
4. Umbilical hernia is due to an imperfect closure or weakness of the umbilical ring. It is
especially common in low birth weight, female, and black infants. It appears as a soft
swelling, covered by skin that protrudes during crying, coughing, or straining, and can be
reduced earsily through the fibrous ring at the umbilicus. The hernia consists of ometum or
portions of the small intestine. Most umbilical hernias disappear spontaneously by 1 year of
age. Surgery is advised if the hernia persists to the age of 3-4 years, exceed 2cm in diameter,
causes symptoms, becomes strangulated, or progressively enlarge after the age of 1-2 years.
An omphalocels is a herniation or protrusion of abdominal contents into the base of the umbilical

185
cord, and it usually diagnosed at birth. In contrast to the more common umbilical hernia, the sac is
covered with peritoneum without overlying skin. Immediated surgical repair before infections take
place is essential for survival.
Gastroschisis is diagnosed at birth by protrusion of bright red intestines to the right side of the
umbilicus. There is no covering of the intestines, and the umbilicus is normal. It is a surgical
emergency.
5. The features of Prader-Willi syndrome (obesity hypogonadism syndrome) include hypotonia,
hypogonadism and obesity. Remember mental retardation, and dysmorphic craniofacial
features (ie., narrow bifrontal diameter, diamond-shaped eyes and a small, down-turned
mouth).
6. Breastfeeding jaundice is an exaggeration of physiologic jaundice commonly seen in
exclusively breastfed infants who are not getting enough breast milk. When effective
breastfeeding is not established in the first few days of life, the inadequate enteral intake
prolongs the intestine transit time, resulting in an increased enterohepatic circulation. In
addition, the infant becomes relatively dehydrated from the limited fluid intake. The increased
enterohepatic circulation and relative dehydration together cause an elevated unconjugated
bilirubin level in the first few days after birth.
7. The initial mx of breastfeeding jaundice is to stimulate milk production by increasing the
nursing frequency. The infant should be monitored closely (repeat bilirubin level 12-24 hours)
to ensure that the feeding problems are resoved and the bilirubin decreases. If the infant’s
bilirubin continues to rise, then interventions such phototherapy (bilirubin levels >17-
25mg/dL), exchange transfusion (bilirubin levels > 25mg/dL), or formula supplementation
may become necessary to prevent the complication known as nernicterus.
Breastfeeding jaundice that occurs in the first few days of life must be distinguished form breast
milk jaundice, which usually starts after the first 3 -5 days of life, peaks 1-2 weeks after birth, and
can lasat several months. An otherwise thriving infant will present with jaundice and elevated
unconjugated hyperbilirubinemia. The etiolgoyc is not completely understood, but it is thought
that a substance in breast milk may inhibit UDPGA or that the increased beta glucuronidase
activity in breast milk may increase the enterohepatic circulation of bilirubin.
ABO incompability is caused by maternal antibodies against fetal erythrocytes. Most commonly,
ABO incompatibility is seen when the mother’s blood type is O and the infant’s blood type is A ,
B, or AB. The jaundice of ABO incompatibility tends to manifest in the first 24 hours of life.
8. Urinalysis is a preliminary inverstigation that should be performed first in all pts with
suspected renal disease.
9. Wiskott-Aldrich syndrome is an X-linked disorder characterized by the triad of
thrombocytopenia, eczema (there are dry, scaly patches on his cheeks and lower extremities),
and recurrent bacterial infections. The thrombocytopenia is caused by decreased platelet
production, and the few platelets that exist are typically quite small.
10. Coarctation of the aorta (COA) is characterized by hypertension in the upper part of the body
and relative hypoperfusion in the lower part of the body. A mild, continuous murmur heard all
over the chest is due to the development of collaterals between the hypertensive and
hypoperfused vessels. Rb notching, caused by the dialatation of the collateral chest wall
vessels, is specific for coarctation.
11. The most common esophageal anomaly is esophageal atresia with a tracheoesophageal fistula.

186
This is charaterzied by an atretic esphoageal pouch that communicates diatally with the
trachea just above the carina. This anatomic arrangement allows air to enter the stomach, and
this leads to gastric distention. The discontinuous esophagus prevents the infant from
completely swallowing, thereby resulting in drooling or regurgitation during feeding. In
addition, gastric fluid ascends into the distal esophagus through the fistula, into the trachea
and lungs, thereby producing aspiration pneumonia. Pneumonitis and atelectasis occur
frequently, and rattles are heard during breathing. The inability to pass a feeding tube into the
stomach is suggestive of esophageal atresia with or without tracheoesophageal fistula.
12. Congenital syphilis presents early on with hepatomegaly, cutaneous lesions, jaundice, anemia,
and rhinorrhea. Metaphyeal dystrophy and periostitis may be seen on radiography.
13. The classic triad of Kartegener’s syndrome is recurrent sinusitis, bronchiectasis, and
dextrocardia. The cause is primary ciliary dyskinesia.
14. Pneumococcal vaccination plus penicillin prophylaxis can prevent almost all cases of
pneumococcal sepsis in pts with sickle cell anemia.
15. Arteriovenous malformation (AVM) is the most common of the subarachnoid hemorrhage in
children. The history of seizure and migraine-like headaches is characteristic.
16. Aseptic necrosis of the femoral head is a common complication of sickle cell disease. It
involves occlusion of end arteries supplying the femoral head, bone necrosis, and eventual
collapse of the periarticular bone and cartilage. * Extremely high yield!!
17. Prostaglandin E1 infusions may be used to keek the ductus arteriosus patent in pts with
ductus-dependent congenital heart disease until definitive therapy can be pursued.
18. Those with sickle cell trait have some protection against malaria.
19. Chronic garnulomatous disease is an inherited immunodeficiency disorder marked by an
inability to oxidize pathogens within phagocytes. Pts experience recurrent infections starting
early in life. Pneumonia and suppurative adenitis are especially common infections in these
pts.
20. Acute bacterial sinusitis for children <6 years of age is a clinical dx, and umcomplicated cases
are treated with oral amoxicillin.
21. Hyper-IgM syndrome (HIM) is characterized by high levels of IgM with deficiency of IgG,
IgA and poor specific antibody response to immunization.
22. Individuals who will be exposed to tick-infested areas should wear light-colored clothing with
long-sleeved shirts and long pants tucked into socks or boot tops.
23. Epiglottitis is a pediatric emergency. Secure the airway as soon as possible with endotracheal
intubation and set-up for possible tracheostomy. Visulization of the epiglottis should not be
attempted. In the pre vaccination era almost all the cases were due to Haemphilus influenzae
type b (Hib). Because of the routine infant vaccination with Hib conjugate vaccines, the
antimicrobial spectrum has changed. H.influenza type A and nontypable strains,
H.parainfluenzea, Streptococcus pneumoniae, Staphylococcus aureus, and beta-hemolytic
streptococci are commonly isolated in Hib vaccinated pts. Nebulized racemic epinephrine is
used in the mx of croup.
24. 24-hour esophageal pH monitoring is the gold standard for the dx of GERD.The position that
the child assumes (ie. Tilted head and arched back) is called “posturing of Sandifer’s
syndrome,” and the mechanism by which the child protects his airways and reacts to the pain
of the acid reflux.

187
25. Breath holding spells are episodes of apnea that are sometimes associated with a loss of
consciousness, and are precipitated by frustration, anger, or pain. These episodes usually
occur in children ages 6-18 months. Classically, the child gets frustrated and holds their
breath, which may lead to loss of ocnsciousness followed by spontaneous resolution. Unlike a
seizure, there is no incontinence or postictal phase associated with breath holding spell. These
spells can be simple (no change in circulation or oxygenation), cyanotic, pallid 苍 白 的 , or
complicated. Electroencephalograms are normal. Breath holding spells are self-limited and
should be treated with education and reassurance. Anemic pts, however, should be treated
with iron supplementation.
26. Tetralogy of Fallot consists of pulmonary stenosis, an overrding aorta, a vertriculoseptal
defect, and right ventricular hypertrophy. “hypercyanotic” spell or “tet” spells can be
improved with a knee to chest position, which increase systemic vascular resisitance and
therefore impoves pulmonary blood flow. Morphine and an iv fluid bolus can also be given to
increase pulmonary flow. Oxygen is of little benefit because the abnormality is decreased
pulmonary blood flow, not sufficienty oxygenation.
27. Post-exposure prophylaxis for chicken pox can be provided with VZIG or acyclovir. Post-
exposure prophylaxis with VZIG is preferred and indicated in susceptible high-risk persons
exposed to varicella within 96 hours (preferred 72 hours) of exposure.
28. Streptococcus pneumoniae is the MCC of acute sinusitis in childhood, followed by
Haehophilus influenza and Moraxella cararrhalis.
29. Spontaneous hemarthrosis raises the suspicions for hemophilia, for which factor VIII assay is
diagonistic. Prolonged PTT, normal prothrombin time, normal bleeding time, normal
fibrinogen level and low serum factor VIII activity are the typical lab findings. The standard
tx for hemophilia is to replace the factor VIII. However, mild hemophilia may be treated with
DDAVP, which causes release of factor VIII from the endothelial cells.
30. When a child present with signs of increased intracranial pressure and meningitis, CT scan of
the head should be performed before performing a limbar puncture to avoid the possiblitiy of
herniation; however, this shoud not delay antibiotic (3rd generation cephalosporin, such as
cefotaxime; penicillin-resistant pneumococcus requires the addition of vancomycin)
administration.
31. Marfan’s features + mental retardation + thromboembolic events + downward dislocation of
the lens = homocystinuria. It is an autosomal recessive disease caused by cystathionine
synthase deficiency. Tx mainly involves administration of high doses of vit B6. * High Yield!!
32. Neonatal jaundice with conjugated hyperbilirubinemia is suggestive of neonatal cholestasis
and warrants further evaluation. Indications of the evaluation of neonatal jaundice:
1)conjugated hyperbilirubinemia (>2mg/dL); 2) jaundice that appears in the first 24-36 hours
of life. 3) serum bilirubin rising at a rate faster tham 5 mg/dl/24 hours . 4) Serum bilirubin
greater than 12mg/dl in full-term (esp n the absence of risk factors) or 10-14 mg/dl in preterm
infants. 5) Jaundice persists after 10-14 days. 6) The presence of signs or symptoms.
33. Tetralogy of Fallot, the most common cyanotic congenital disease in children less than 4 years
of age, presents with cyanotic spells and pansystolic murmur on exam (a harsh pansystolic
murmur is noted at the left sternal border, along with a single S2).
34. Suspect duodenal atresia when the pt has bile-stained vomitus, no abdominal distension,
dehydration, double bubble sign, and Down syndrome. Vomiting usually begins in the first

188
24-48 hours after birth.
35. Hydroxyurea is indicated in sickle cell pts with frequent, acute, painful episodes, as it tends to
increase the HbF levels.
36. Iron poisoning presents as nausea, vomiting, diarrhea, abdominal pain, gastrointestinal
bleeding and metabolic acidosis. Since iron is radio-opaque, the tablets can be seen in the
stomach on abdominal x-ray. The dx is confirmed by measuring serum iron levels. Iv
deferoxamina, an iron chelator, is used in moderate to severe cases of intoxication.
37. Capillary blood specimens (using a finger stick) are widely used in childhood screening for
lead poisoning. Since false positive results are common, the first step after an abnormal
fingerstick lead level is to confirm the dx with a serum (venous blood) lead level. If the serum
lead level is > 10ug/dl, then intervention is needed.
38. Recurrent self-limiting episodes of vomiting and nausea in children, in the absent of any
apparent casue, suggest the dx of cyclical vomiting. The etiology of this condition is unclear,
however, its incidence is high in children whose parents have a history of migraine headaches.
Complications that may arise from the condition are anemia and dehydration. Tx consists of
anti-emetics and reassuranve of the parents.
39. A minor who is a parent can consent to treatments or procedures for his/her child.
40. A retropharyngeal abscess should be suspected in pts who present with pharyngitis, moderate
fever, dysphagia, neck stiffness, a muffled voice, a posterior pharyngeal bulge.
41. Inflammed, fluctuant cervical LN are typically caused by streptococcal or staphylococcal
infection. The antibiotic of choice is dicloxacillin 二氯苯甲异恶唑青霉素钠. Other effective
antibiotics include cephalexin or clindamycin.
42. Night terrors occur during non-REM sleep and are characterized by fear, crying or screaming,
decreased level of consciousness, and amnesia of the event.
43. Bordetella pertussis causes whooping cough, which are severe paroxysms of cough that can
last up to two weeks. The coughing spells are so severe that rectal prolapse and
pneumothoraces can ensue.
44. The TORCH infections (toxoplasmosis, rubella, CMV, HSV and syphilis) cause a syndrome
characterized by microcephaly, hepatosplenomegaly, deafness, chorioretinitis, and
thrombocytopenia.

1/22/10
1. Language development:
12-month: “mama”, “dada”, and one other word. Be able to follow simple commands with a
gesture
15-month: “mama”, “dada”, and 3-5 other word. Understand simple commands even without a
gesture.
18-month: a vocabulary of 5-20 words.
2-year: a vocabulary of 150-300 words and be able to combine words into short sentences. In
addition, half of their speech should be understandable to a stranger.
3-year: vocabulary has increased ti close to 1000 words, and about 75% of the children’s speech
can be understood by a stranger. In addition, they speak in 3-4 word sentences and use pronouns
correctly.
2. Suspect growing bone pain in a child with nocturnal pain and no obvious physical or lab

189
finding. The tx is reassurance. Common in age 2-12 year, is usually poorly localized and
common in the legs (usually below the knees and bilateral). In all cases, it resolves in the
morning. The dx of growing pain is a diagnosis of exclusion.
3. Pts with classic PKU present with fair skin, blue eyes, musty body odor and eczema. Dx
of PKU: 1) plasma phenylalanine level greater than 20mg/dl. 2) normal to low-normal plasma
tyrosine level. 3) increased urinary levels of phenylalanine metabolites (phenylpyruvic and o-
hydroxyphenylacetic acids). 4) normal tetrahydrobiopterin level.
Pts with benign hyperphenylalaninemia also have a deficiency in phenylalanine hydroxylase, but
some residual enzyme activity is preserved; therefore, the serum phenylalanine blood levels are
only slightly elevated (less than 20mg/dl). Pts who undergo PKU screening may be identified
during the neonatal period, but they are otherwise asymptomatic and may develop normally
without tx.
Pts with transient hyperphenylalaninemia are asymptomatic.
Tyrosinemia is caused by a deficiency of fumarylacetoacetate hydrolase. Affected infants may
become symptomatic as early as the 2nd week of life. Common manifestations include fever,
irritability, vomiting, hemorrhage, hepatomegaly, jaundice, elevated levels of serum
transaminases, episodes of acute peripheral neuropathy, Fancon-like syndrome, and
hypoglycemia. Pts may smell like boiled cabbage.
4. Sickle cell anemia can cause childhood atroke.
5. This pt with recurrent bacterial infections and necrotic periodontal infection is most
likely suffering from congenital immunodeficiency cause by a leukocyte adhesion defect.
Delayed separation of the umbilical cord (>3 weeks) is characteristic and presents an
important clue to the correct dx. The leukocytes fail to express some adhesion molecules on
their surface. The leukocyte number is increased, but the interleukocyte communication is
defective; therefore, chemotaxis and cytotoxicity are impaired. Normal lymphocyte count and
gamma globulin concentration help to differentiate this condition from a variety of cell and/or
humoral immune defects.
The classic example of opsonization defect is asplenia. Infections with encapsulated bacteria (like
S. pneumoniae) are typical.
Complement deficiencies manifest as recurrent generalized infections with encapsulated bacteria,
typically Neisseria and autoimmune disorders.
Defective intracellular killing is due to impaired oxidative burst within the phagocytes and a
positive NBT test is characteristic.
6. Positive anti-Smith antibodies and/or anti-double stranded DNA antibodies is specific
and comfirmatory for the diagnosis of systemic lupus erythematosus.
7. The USPSTF recommends screening for atrabismus, amblyopia, and refractive errors in
children aged 0-5 years.
8. Batteries lodged in the esophagus on x-ray should be removed immediately under
endoscopic guidance to prevent mucosal damage and esophageal ulceration. Batteries located
distal to esophagus pass unevenfully in most cases and need only to be observed with stool
examination and/or follow up x-rays to confirm excretion.
9. Scarlet fever is characterized by fever, toxicity, pharyngtis, sandpaper-like rash,
circumoral pallor and strawberry tongue. It is caused by strains of Group A streptococcus that
produce erythrogenic exotoxins. Penicillin V is the drug of choice.

190
10. Vit A has been shown to reduce the morbidity and mortality rates of pts with measles.
11. Thyroid dysgenesis is the MCC of congenital hypothyroidism in US.
12. Infants with congenital hypothyroidism initially appear normal at birth, but gradually
develop apathy, weakness, hypotonia, large tongue, sluggish movement, abdominal bloating,
and an umbilical hernia. For this reason, screening newborns for hypothyroidism, along with
phenylketonuria and galactosemia, is standardly performed in all states.
13. Severe combined immune deficiency is a life-threatening syndrome presenting with
recurrent bacterial, viral and fungal infections. Its diagnostic features are: absent LN and
tonsils, lymphopenia, absent thymic shadow on CXR, and abnormal T, B and natural killer
cell enumeration by flow cytometric analysis.
14. Osteogenesis imperfecta can be associated with blue sclera, hearing loss, joint
hypermobility, and dentinogenesis imperfecta (opalescent teeth).
15. Milk protein intolerance may present with vomiting and bloody diarrhea. Stools may
show RBCs and eosinophils, and there may be a family history of an atopic disorder. It is a
hypersensitivity reaction to cow’s milk proteins, and therefore usually occurs in babies who
are fed with cow’s milk, however, it may occur in breastfed babies, as the mother’s milk may
contain proteins from ingested cow’s milk.
16. Topical mupirocin or oral erythromycin is the tx of choice for local impetigo.
17. Watch out for the infant or newborn with failure to thrive, bilateral cataracts, jaundice
and hypoglycemia. Such an infant most likely has galactosemia, which is a metabolic disorder
caused by galatose-1-phosphate uridyl transferase deficiency, thereby leading to elevated
blood level of galatose. The pt presented in this case has the typical presentation: vomiting,
poor weight gain, jaundice, hepatomegaly, convulsions and cataracts. Other common
manifestations include aminoaciduria, hepatic cirrhosis, hypoglycemia, and mental
retardation. Such pts are at increased risk for E.coli neonatal sepsis. Early dx and tx by
elimination of galactose from the diet are mandatory.
Pts with galactokinase deficiency present with cataracts only, and are otherwise asymptomatic.
Uridyl diphosphate galactose-4-epimerase deficiency is a rare diseses compared to uridyl-
transferase deficiency. In addition to the manifestations of transfrease deficiency, pts with
epimerase deficiency have hypotonia and deafness.
18. Meconium aspiration syndrome occurs most commonly in low birthweight and post-
mature infants. It is characterized by respiratory distress at birth, rales, rhonchi,
hyperinflation, and coarse streaking and patchy opacities on CXR.
19. Septic joint in a child is a true surgical emergency and needs immediate surgical
drainage. A delay of even 4-6 hours can lead to avascular necrosis of the femoral head.
20. Hypocomplementemia (low C3, CH50) in post-streptococcal glomerulonephritis
resolves in 8-12 weeks. Hematuria may persist for up to 6 months. Persistent microscopic
hematuria is suggestive of IgA nephropathy. Proteinuria resolves much slower than hematuria.
Up to 15% of pts may have proteinuria for a period of 3 years. Antibody titers tend to rise
after 7 days, peak after one month, and return to normal in 3-4 months.
21. Respiratory distress syndrome is caused by surfactant deficiency and is more common in
premature infants and infants of diabetic mothers.
22. Parinaud’s syndrome and Collier’s sign (paralysis of vertical gaze that may be
associated with papillary disturbances and eyelid retraction) usually indicate a lesion in the

191
rostral midbrain, most likely pinealoma or germinoma.
23. The 3 most common organisms that cause pneumonia in pts with cystic firbrosis are
Haemophilus, Pseudomonas, and Staphylococcus.
It is essential to memorize:
G+ diplococci – Streptococcus pneumoniae
G+ cocci in clusters – Staphylococcus
G- cocci – Neisseria
G+ rods – Listeria and Bacillus
G- rodes – Pseudomonas, Haemophilus, Klebsiella, Legionella
24. Pts wiht cystic fibrosis may initially present with meconium ileus, which is
characterized by bilious vomiting, failure to pass meconium at birth, and ground glass
appearance on abdominal x-rays.
25. By the first birthday, a child should triple his birth weight and increase his height by
50%. The rate of growth slows towards the end of the first year of life. When compared to
birth weight, a child’s weight should double by 5 months and triple by 1 year. Height
increases by 50% in the first year, bouble by 4 years, and triple by 13 yeas.
26. Painful crises (vasoocclusion in connective tissue and muscle) are the most common
manifestation of sickle cell anemia. Sickle cells and reticulocytes will be seen on the
peripheral smear.
Burr cell are speculated RBC of similar size and with regularly spaced profections. They are seen
most commonly in uremia or as an artifact of preparation.
27. Osteonecrosis is a common complication of sickle cell anemia due to vaso-occlusion of
the bone. It causes significant joint pain and functional limitation. The humerus and femur are
the most frequently affected bones.
28. Physiological jaundice usually presents after 24 hours of birth. Jaundice in the first 24
hours of birth or with direct hyperbilirubinemia requires a detailed evaluation to identify its
cause.
29. Howell-Jolly bodies are nuclear remnants of the RBC which are generally removed by a
functional spleen; therefore, their presence in a peripheral smear suggests functional asplenia
in sickle cell pts.
Heinz body-G6PD
Helmet cells are fragmented RBC—traumatic hemolytic conditions such as DIC, HUS and TTP
Basophilic stippling are ribosomal precipitates whichi appear as blue granules of various sizes
dispersed throughout the cytoplasm of the red cell—thalassemias, hevey metal poisoning
30. Acquired torticollis is a relatively common condition in children. The most common
causes include upper respiratory infections, minor trauma, cervical lymphadenitis, and
retropharyngeal abscess. Cervical spine radiographs should be obtained in children with
acquired torticollis to ensure there is no cervical spine fracture or dislocation.
31. Pts with Turner syndrome have a higher risk of osteoporosis due to lower estrogen
levels and only having one copy of X chromosome genes involves in bone metabolism.
32. A unilateral flank mass in a child >3 years of age is most likely due to Wilm’s tumor,
which arises form the metanephros.
A unilateral flank mass in a child <3 years of age is highly suspicious for neuroblastoma, which is
a malignancy of the neural crest cells. Neuroblastoma displaces the kidneys inferolaterally,

192
thereby making the kidneys non-palpable, whereas in Wilm’s tumor, the kidneys are still palpable.
33. The typical pt with glucose-6-phosphatase deficiency (type I glycogen storage disease,
Von-Gierkes’ disease) is 3-4 months of age with hypoglycemia, lactic acidosis, hyperuricemia,
and hyperlipidemia. The characteristic appearance is a doll-like face (ie. Fat cheeks), thin
extremities, short stature, and a protuberant abdomen (due to enlarged liver and kidneys).
Hypoglycemia seizure may occur.
Acid maltase deficiency is a.k.a. type II glycogen storage disease and Pompes’s disease. Although
pts with this condition also have hepatomegaly, they usually present in the first few weeks of life
as a ‘floppy baby’ with feeding difficulties, macroglossia, and heart failure (due to progressive
hypertrophic cardiomyopathy).
Type III glycogen storage disease is caused by a deficiency of glycogen debranching enzyme
activity. Similar clinical manifestations (eg. hepatomegaly, hepoglycemia, hyperlipidemia, and
growth retardation) may make it initially difficult to distinguish from type I; however, their lab
findings differ. Pts with type III have elevated levels of liver transaminases, fasting ketosis, and
normal blood lactate and uric acid concentrations. Other pertinent findings are splenomegaly and
normal kidneys.
Deficiency of branching enzyme activity results in type IV glycogen storage disease or
amylopectinosis. The typical pt is in his first 18 months of life with hepatoslpenomegaly and
failure to thrive. The most common pathology is progressive cirrhosis of the liver.
Pts with liver phosphorylase deficiency have a benign course, and typically present with
hepatomegaly and growth retardation early in childhood. Mild hypoglycemia, hyperlipidemia, and
hyperketosis may occur. Lactic acid and uric acid levels are normal.
34. Guillai-Barre syndrome involves mainly the peripheral motor nerves, although sensory
and autonomic nerves many also affected.
35. Diamond-Blackfan syndrome is a macrocytic pure red aplasia associated with several
congenital anomalies such as short stature, webbed neck, cleft lip, shielded chest and
triphalangeal thumbs.
Fanconi’s anemia is an autosomal recessive disorder characterized by progressive pancytopenia
and macrocytosis. The average age at dx is 8 years. Associated deformities include café-au-lait
spots, microcephaly, microphthalmia, short statue, horseshoe kidneys and absent thumbs (not
triphalangeal thumbs).
36. Suspect Wilm’s tumor in a young child of 2-5 years age with a localized abdominal
mass and hamaturia. If the child age is less than 1 year, suspect neroblastoma.
37. Posterior urethral valves are the MCC of congenital urethral obstruction. The classic
presentation is a male infant with a distended palpable bladder and abnormally low urine
output.
An omphalomesenteric duct cyst is a focal failure of vitelline duct obliteration. Such a vitelline
cyst may cause a small midline mass deep to the umbilicus. The cyst could be attached to the
umbilicus and the wall of the ileum by vitelline remnants and may cause small intestine volvulus.
38. Atlantoaxial instability should be suspected in any pt with Down syndrome who
presents with upper motor neuron findings. Symptomes include behavior changes, torticollis,
urinary incontinence, and vertebrobaslilar symptoms such as dizziness, vertigo, and diplopia.
On exam, upper motor neurom symptoms such as leg spasticity, hyperreflexia, a positive
Babinski sign, and clonus are often present.

193
39. Small for gestational age infants have a weight under the 10 th percentile for gestational
age at birth and may have complications such hypocia, polycythemia (increased
erythropoietin secretion in response to fetal hypoxia), hypoglycemia, hypothermia, and
hypocalcemia.
40. Neonatal necrotizing enterocolitis should be suspected in any preterm or low birth
weight infant with fever, vomiting, abdominal distension, and pneumatosis intestinalis.
41. The best drug available for infantile spasms in the US is intramuscular ACTH, because
it is safer and more effective.
42. Gaucher’s disease is due to the deficient activity of the lysosomal enzyme, acid beta-
glucosidase. The typical pt is an Ashkenazi Jewish adolescent with chronic fatigue, easy
bruisability, bone pain, and pathological fractures. The dx is confirmed with radiologic
(Erlenmeyer flask deformity of the distal femur) and bone marrow studies (Gaucher cells with
wrinkled paper appearance).
Niemann-Pick disease types A and B result from the deficienct activity of sphigomyelinase.This is
a fetal disorder of infancy. Clinical manifestations include failure to thrive, hepatosplenomegaly,
and a rapidly pregrassive neurodegerative course that eventually leads to death by age 2-3 years.
GM1 gangliosidosis results from the deficient activity of beta-galactosidase. It presents at birth or
early in infancy. Clinical features include hepatosplenomegaly and skeletal abnormalities (eg.
anterior beaking of the vertebrae, enlargement of the sella turcica, and thickening of the
calvarium).
The GM2 gangliosidosis include Tay-Sachs’ disease and Sandhoff’s disease. The underlying
pathology involves the deficieny of hexosaminidase activity and the lysosomal accumulation of
GM2 gangliosides, particularly in the central nervous system. Even though hepatospenomegaly is
a common presentation in these disorders, pts usually present at an earlier age with a cherry-red
spot in the retina and neurologic symptoms.
Farber disease results from deficiency of lysosomal enzyme ceramidase, thereby resulting in the
accumulation of ceramide in various tissues, especially the joints. Pts may also present with bone
pain and joint swelling.

43. Fragile X syndrome is characterized by low to normal IQ with learning disabilities,


generalized language disability, short attention span, autism, large head, prominent jaw, large
low set ears and macroorchidism. Increase number of CGG trinucleotide repeats.
44. The initial evaluation of enuresis includes a urinalysis to help rule out treatable causes
such infections, bleeding, or structural defects.

1/23/10
1. Acute UTI in children in most commonly due to vesicoureteral reflux, which is mostly
congenital and is seen in 1% of newborns.
2. Individual with thalassemia trait present with microcytic anemia and a mildly elevated
reticulocyte count. The RDW, MCHC, TIBC (240-450 mcg/dl) and ferritin level (7-140ng/ml)
are typically normal.
3. The clinical findings of hypotonia, hyperactive deep tendon reflexes, learning disabilities,
along with an intrapartum history of prolonged labor and low APGAR scores at birth, point
towards the diagnosis of cerebral palsy. Cerebral palsy is a non-progressive disorder

194
characterized by impaired motor functioning. The MCC is cerebral anoxia.
Friedreich’s ataxia is characterized by gait disturbances, pes cavus, ataxia and absent ankle jerk.
4. Always think of autism first in any pt who is less than 3 years old and presents with
impairment in social interactions and communication, delayed language development, and
stereotypal behaviors. DD Asperger syndrome: although pts may have feature of stereotypical
movements and self-injurious behavior, their language development is normal.
5. Neonatal abstinence syndrome is caused by infant withdrawal to opiates and usually presents
in the first few days of life. It is characterized by irritability, a high-pitched cry, poor sleeping,
tremors, seizures, sweating, sneezing, tachypnea, poor feeding, vomiting, and diarrhea.
Fetal exposure to valporic acid and significantly increases the risk of neural tuble defects, may
result in cardiac anomalies, and can produce dysmorphic facies, including cleft lip, narrow
bifrontal diameter, midface hypoplasia, broad and depressed nasal bridge, and long philtrum.
Prenatal exposure to phenytoin can result in fetal hydantoin syndrome, which is characterized by
nail and digit hypoplasia, dysmorphic facies, and mental retardation. Growth deficiency can also
be seen.
Prenatal exposure to cocaine can result in jitteriness, excessive sucking, and a hyperactive Moro
reflex. Withdrawal syndromes are usually not as severe as with opiates. Long-term effects on
behavior, attention level, and intelligence may be seen.
6. Acute lymphoblastic leukemia is the MCC in children. Diagnosis is mainly based on more
than 25% lymphoblasts in the bone marrow.
7. Most cardiac murmurs in childhood are benign and require only observation. Know the
following features of a benign murmur: 1) asymptomatic pt. 2) murmur intensity grade 2 or
less. 3) Normal S2. 4) No audible clicks. 5) Normal pulses. 6) No other abnormalities.
Pathological murmurs: 1) symptoms such shortness of breath, chest pain, syncope, and dizziness,
etc. 2) Murmur intensity grade 3 or higher. 3) Abnormal S2. 4) pansystolic murmur. 5) Murmur
loudest at upper left sternal border. 6) Absent or diminished femoral pulses. 7)Murmur’s quality is
unchanged with position.
8. The recommended first line tx for pertussis is a macrolide antibiotic (erythromycin,
azithromycin, or clarithromycin), regardless of the age of the immunization status. Tx is given
whenever pertussis is suspected or confirmed, and regardless of the stage of the disease.
9. Myotonic muscular dystrophy (MMD) is an autosomal dominant disease which is a.k.a.
Steinert disease. It is the second most common muscular dystrophy in the US. The pathology
is distinct in that all types of muscles (ie. Smooth, striated, cardiac) are involved. The above
case presentation is typical: the pt initially appears normal at birth, then slowly develops
muscle weakness and progressive muscle wasting, especially in the distal muscles of the
hands, posterior forearm muscles, and anterior compartment of the lower legs. The
characteristic facial appearance if illustrated above temporal wasting, thin cheeks, and an
upper lip in the shape of an inverted V. Pertinent physical findings include emaciated
extremities, atrophy of the thenar and hypothenar eminences, proximal muscle weakness,
positive Gowers sign, winged scapula and myotonia. Myotonia s defined as delayed muscle
relaxation, and the classic example is the inablility to release the hand after a handshake. In
addition, anormalities of the endocrine, immunologic and nervous systems occur. Endocrine
manifestations include DM, testicular atrophy, frontal baldness and hypothyroidism.
10. The MCC of anemia in sickle cell pts is chronic hemolysis. Iron deficiency, folate deficiency,

195
and anemia of chronic disease occur less frequently in such pts.
11. Hydroxyurea benefit pts with sickle cell disease by increasing fetal hemoglobin, which dilutes
the number of sickled cells in the circulation and reduces veso-occlusive episodes. Hydoxyura
has been shown to decrease pain crises, the need for transfusions, and episodes of acute chest
syndrome. Side effects occur because hydroxyurea suppresses the bone marrow. Leukopeniam
anemia and thrombobytopenia may occur. These effects are generally temporary and
reversible but may predispose the pt to infection.
12. Infants with DiGeorge syndrome may have cyanotic heart disease, craniofacial anomalies,
thymic hypoplasia, cognitive impairment and hypoparathyroidism. It is essential to monitor
the calcium levels of these infants. Prolonged QT intervals may be caused by hypocalcemia.
13. The tx for trycyclic antidepressant intoxication includes sodium bicarbonate. This drug not
only helps to correct the acidosis, but also helps to narrow the QRS complex prolongation.
Benzodiazepine (eg. diazepam) is given when the pt presents with seizures that require tx.
Extremely high yield!!
14. Whenever intraveoux access cannot be obtained in emergent pediatric cases, intraosseous
access should be attempt next.
15. Upper GI endoscopy is the diagnostic study of choice when a pt presents with acute alkali
ingestion. Attempting to neutralize the alkali with vinegar or lavage is dangerous since these
may increase the extent of the injury.
16. Congenital abdominal wall defects
Gastroschisis Omphalocele
Bowel protrudes through a defect on the right Intra-abdominal contents protrude through the
side of the umbilical cord umbilical ring. The herniation usually includes
small bowel and may include large bowel and
liver
Bowel is not covered by a protective membrane Covered by an amnioperitoneal membrane
Bowel looks “angry” and matted
Not typically associated with other Can be associated with other congenital
abnormalities outside the GI tract abnormatilies (eg. heart, kidney)
The first step in the mx of gastroschisis is to immediately wrap the exposed bowel in sterile saline
dressing and plastic wrap in order to minimize heat and fluid losses. Caesarean section is
performed only in the presenct of the usual obstetric indications. The mx of omphalocele is similar
to that of gastroschisis. Immediate care of the newborn with omphalocele also involves sterile
wrapping of the bowel, insertion of an orogastric tube (to decompress the stomach). Small defects
(<2cm) can be repaired with primary closure, but most defects will require a staged procedure
with Silastic silo.
17. Pure riboflavin deficiency is unusual in industrialized nations, but has been documented in
regions of the world with severe food shortages. The condition is typically mild and
nonspecific in presentation, but symptoms may include sore throat, heperemic and edematous
oropharyngeal mucous membranes, cheilitis, stomatitis, glossitits, normocytic-normochromic
anemia, seorrheic dermatitis, and photophobia.
18. Cephalohematoma is a subperiosteal hemorrhage, and presents a few hours after birth as scalp
swelling limited to one cranial bone. It is a subperiosteal hemorrhage; hence, it is always
limited to the surface of one cranial bone. Most cases do not require any tx and resorb
spontaneously within 2 weeks to 3 months, depending on the size. Rarely, phototherapy may
be necessary to improve the hyperbilirubinemia.

196
Caput succedaneum is a diffuse, sometimes ecchymotic, swelling of the scalp. It usually involves
the portion of the head presenting during vertex delivery. It may extend across the midline and
across suture lines.
19. The presence of dermatitis herpetiformis and chronic non-bloody diarrhea in a child of 12-15
months it suggestive of celiac disease. It is due to abnormal hypersensitivity to gluten and
presents characteristically at 12-15 months of age, when gluten-containing foods such as
wheat, rye, or barley are introduced to the child’s diet.
20. Klumpke’s paralysis is a brachial palsy that occurs in newborns following excessive traction
on the arm during delivery. It consists of hand paralysis and ipslateral Horner’s syndrome
(ptosis and miosis), and is secondary to injury to 7th and 8th cervical nerves and 1st thoracic
nerve. Tx depends on the severity of the injury and, generally, it consists of partial
immobilization and appropriate positioning to prevent contractures. Gentle massages and
range-of-motion exercises can be started by 7-10 days of age. If by 3-6 months there is no
improvement, neuroplasty, neurolysis, end-to-end anastomosis, and never grafting may be
attempt.
Erb-Duchenne palsy is another type of brachial palsy in which the injury involves the 5th and 6th
cervical nerves. Clinically, it presents with absent Moro reflex and intact grasp reflex of the
affected arm. Pts present with a characteristic position, which consist of adduction and internal
rotation of the arm with pronation of the forearm.
Cervical fracture presents with irregularity, crepitus and fullness over the fracture site and
decreased movements of the arm.
In neonates, facial nerve palsy results from pressure over the facial nerve in utero, from efforts
during labor, or from forceps delivery. Classic features of facial paralysis are present, most
apparent when the infant cries.
Phrenic never injury involves the 3rd, 4th, and 5th cervical nerve. It results in diaphragmatic
paralysis and upper brachial palsy. It presents with irregular and difficult respiration, and an
abdomen that does not bulge with breathing.
21. Pylotic stenosis is treated surgically, however, surgery should be undertaken only after the
infant’s hydration status and electrolytes levels have been stabilized.
22. The first step in the mx of an infant with suspected congenital diaphragmatic herniation is the
placement of an orogastric tube.
23. Absence seizure is a generalized seizure of childhood that is characterized by sudden and brief
lapses of consciousness without loss of postural control. The typical presentation includes
multiple ‘daydreaming’ episodes (eye blinking asdn gabbling) and a decline in school
performance.
24. Adolescent onset of hirsutism and virilism, with normal menstruations and eleveated 17-
hyoxyprogesterone, are diagnostic of congenital adrenal hyperplasia.
25. Constitutional growth delay is characterized by a delayed growth spurt, delayed puberty, and
delayed bone age. Testosterone supplementation is not necessary with constitutional growth
delay. Hormone supplementation may speed up puberty, but this will shorten the growth spurt
and may result in a shorter final adult height. Hypothyroidism can cause growth delay and
should be considered in the differential dx. However, the expected pattern would be normal
childhood growth followed by an abrupt “falling of the growth curve” when the
hypothyroidism develops. An affected pt would also expected to have other symptoms like

197
hypertension, fatigue, constipation, cold intolerance, and dry hair and skin.
26. Reflux is a risk factor for UTI. Repeated attacks can lead to progressive renal scarring, which
is the major cause of end stage renal disease and hypertension in children. Dx of VUR is best
made with a voiding cystourethrogram or a radionuclide cytogram (RNC). Renal
ultrasonography is less sensitive in detecting reflux nephropathy. The American Academy of
Pediatrics recomm the ends that all children aged 2-24 months with a first UTI should
undergo a VCUG or RNC to detect the presence VUR.
27. Infant botulism is transmitted by the ingestion of food contaminated with C.Botulinum. The
germ grows in the gut and produces a toxin, which is responsible for the clinical signs.
The infant form of botulism is different from the adult form. In the latter, the toxin infested per se
produces the symptoms.
28. The organ system with the highest morbidity and mortality after HUS is the kidneys.
29. Epilgottitis presents with abrupt onset of fever, sort throat, dysphagia, and drooling. Airway
obstruction is the most concerning potential complication of epiglottitis, and often requires
nasotracheal intubation. In unimmunized children, H influenza typr b is a likely cause.
30. If a child shows reluctance to potty training, the best step is to stop training attempts for
several months and allow the child to become interested on his own.
31. Immune thrombocytopenia purpura typically presents in early childhood after a viral infection
with easy bruisability and petechiae. Labs show thrombocytopenia and normal Hb, WBC, PT
and PTT. Peripheral smear shows normal morphology of all cell lines except the platelets,
which are reduced in number and large in size.
32. Todd’s paralysis represents a postictal condition that usually rapidly improves with restoration
of motor function within 24 hours.
33. The pt has mild metatarsus adductus, which is a congenital food deformity. Interestingly, the
deformity is most frequently in first-born infants, this is attributed to the molding effect os the
primigravid uterus. There is no sexual predominance. Approximately 10% of pts also have an
associated acetabular dysplasia; hence, careful hip exam is required. Matatarsus adductus is
subdivided into 3 types. Type I metatarsus adductus is characterized by feet that overcorrected
both passively and actively into abduction. These cases tend to spontaneously correct by
themselves; therefore, tx is not necessary. Type II metatarsus adductus is characterized by feet
that correct to the neutral position with passive and active movements; this is managed with
orthosis or corrective shoes; and sometimes plaster casts if initial tx gives no results. Type III
metatarsus adductus is characterized by rigid feet and do not correct, these are managed with
serial casts.
34. Croup is caused by Parainfluenza virus and commonly presents in children younger than 3
years with hoarseness; a barking coughm and varying degress of respiratory distress.
35. Iron supplementation in the form of oral ferrous salts or iron-fortified milk formula is
warranted in premature infants to prevent anemia of prematurity. Solid food-4 months. Fruit
juices—5-7 months.
36. Preseptal cellulites is an infection of the soft tissue anterior to the orbital septum that is most
commonly caused by contiguous spread of infection from local facial or eyelid trauma.
Preseptal cellulites often presents with eyelid swelling, tenderness, erythema, or discoloration.
The presence of decreased eye movements, decreased visual acuity, proptosis, or double
vision is more suggestive of orbital cellulites.

198
37. The most common mutation that leads to cystic fibrosis is a deletion of a three pair encoding
for phenylalanine (DF508) in the CFTR gene located on chromosome 7. It is believed that this
mutation prevents CFTR protein from trafficking to the corret cellular location.
38. The most effective and potentially life-saving intervention in a pt with known anaphylaxis is
prescription of self-administrable epinephrine for use upon reexposure to the inciting allergen.
39. Mechanical ventilation is indicated in a pt with severe asthma exacerbation that is
unresponsive to medications (silent chest or absent air entry, and condinues to desaturate
despite prednisone tx).
40. Cri-du-chat syndrome is due to 5p deletion and presents as microcephaly, hypotonia, short
stature, and a cat-like cry.
41. In a young child with recurrent upper respiratory tract infections and bilateral polyse, cystic
fibrosis must be ruled out.
42. Turner syndrome is associated with a higher incidence of coarctation and bicuspid aortic
valve.
43. The typical pt with Bruton’s aggamaglobulinemia is a male infant with recurrent pneumonia
and otitis media after 6-9 months of age. The levels of all the serum immunoglobulins and
circulating B-cells are decreased.
44. A pt with an acute onset of headaches and focal neurologic symptoms after an episode of
acute otitis media or sinusitis most likely has a brain abscess. Ring-enhancing lesions are
usually seen on CT or MRI scan.

1/23/10
1. A young boy with symptoms of increased intracranial pressure (eg. headaches, vomiting),
bitemporal hemianopsia, and a calcificated lesion above the sella has a craniopharyngioma
until proven otherwise. Presence of a cystic calcified parasellar lesion on MRI is almost
diagnostic of craniopharyngioma.
2. In a pt with a chronic hemolytic anemia and compensatory reticulocytosis, increased demand
for folic acid can lead to folic acid deficiency. For this reason, pts with sickle cell anemia
should be on folic acid supplementation.
3. Niacin (B3) deficiency causes pellagra, which is characterized by diarrhea, dermatitis,
dementia, and possibly death.
4. Increased gastric residues in a preterm neonate is highly suspicious for necrotizing
enterocolitis.
5. Separation anxiety is a normal behavior in young children. The initial step in the mx of
children with speech delay is an audiology evaluation (esp with a history of otitis media).
6. Osgood-schlatter disease is a traction apophysitis 骨突炎 of the tibial tubercle. Radiographic
findings include anterior soft tissue swelling, lifting of tubercle from the shaft, and irregularity
or fragmentation of the tubercle. On physical exam, there is edema and tenderness over the
tibial tubercle. A firm mass can sometimes be felt due to heterotopic bone formation. Pain can
be reproduced by extending the knee against resistance.
Prepatellar bursitis occurs with chronic irritiation of the anterior knee. Symptoms include pain
with direct pressure and superficial swelling over the patella.
Patellar tendonitis is an overuse syndrome resulting from repetitive jumping or kicking. Pts
present with anterior knee pain after exercise. Unlike Osgood-Schlatter disease, pts have point

199
tenderness at the inferior pole of the patella.
Tibial osteomyelitis is a bone infection, usually bacterial in origin. Symptoms include pain,
swelling, tenderness, and erythema. Pts classically present with refusal to bear weight on the
affected extremity. Systemic symptoms may also be present. The pain from ostermyelitis does not
remit with rest.
Patellofemoral stress syndrome is an overuse injury commonly seen in runners. Pts present with
anterior knee pain that worsen upon descending steps or hills. Pain is localized to the patella and
radiographs do not demonstrate separation at the tibial tubercle.
7. Midgut volvulus usually presents in a child less than one month of age as bilious vomiting,
abdominal distension and passage of bloodstained stools.
8. Suspect clubfoot (talipes equinovarus) in a pt who presents with equines and varus of the
calcaneum and talus, varus of the midfoot, and adduction of the forefoot. Clubfoot is initially
managed with stretching and manipulation of the foot, followed by serial plaster casts,
malleable splints or taping. Surgical treatment is indicated if conservative mx gives
unsatisfactory results, and is preferably performed between 3-6 months of age (should be
started immediately!).
9. By the time a childe starts school, he should have recieced
5 doses of DTaP
4 doses of Heamophilus influenza type b (Hib, complete by 15 months), Pneumococcal conjugate
vaccine (PCV complete by 15 months), inactivated polio vaccine (IPV)
3 doses of HBV, rotavirus
2 doses of MMR, varicella, HAV
At the 4-5 years old visit, children sould receive their DTaP #5, IPV#4, MMR#2 and varicella #2.
Children should receive 3 doses of IPV prior to 18 months and then a 4th booster dose at 4-5 years.
10. Birth injuries such as calvicular fracture are common complications in mothers with GDM
and large babies. Most clavicular fractures are green stick and heal rapidly without
complications.
11. A sudden onset of respiratory distress may be caused by foreign body aspiration, and rigid
bronchoscopy is the procedure of choice for both diagnostic and therapeutic purposes.
12. Although, it is typically a self-limited condition, all pts with suspected Kawasaki disease
should be hospitalized and treated with IVIG and high-does aspirin. If untreated, up to 25% of
these pts may develop coronary artery aneurysms.
13. Suspect rheumatic fever in a child with sore throat, fever, pericarditis, erythema marginatum,
arthritis, chorea, and subcutaneous nodules. This condition is caused by group A streptoccus.
14. A PDA is associated with a continuous flow murmur due to constant movement of blood from
the high-pressure aorta to the low-pressure pulmonary artery. Small PDAs are often
asymptomatic and detect incidentally on routine cardiac auscultation.
15. The tx for neonatal chlamydial conjunctivitis is erythromycin, which has to be administrated
systemically (orally) to decrease the risk of chlamydial pneumonia (not topical).
16. Pyloric stenosis presents with non-bilious vomiting in a 4-8 week old infant, and an
abdominal ultrasound confirms the dx.
17. Premature adrenarche and thelarche (isolated appearance of axillary hair before the age of 6
years) often have no clinical significance. On the other hand, pubarche requires a thorough
evaluation, as it may be due to a CNS disorder in 50% of cases.

200
18. The tx for an acute severe exacerbation of lung disease in a cystic firbrosis pt is iv antibiotics
therapy with coverage against Pseudomonas aeruginosa (usually a combination of two drugs,
such as penicillin/cephalosporin+ aminoglycoside). The addition of an anti-staphylococcal
antibiotic (eg.naficillin or vancomycin) depends on the results of the culture.
19. Guthrie test is a qualitative (coloration) test which can detect the presence of metabolic
products of phenylalanine in the urine.
The screening of amonolevulinic acid and porphobilinogen in the blood is used to diagnose
porphyrias, which is a goup of inherited disorders associated with neurological sings,
photosensitivity and hyperpigmentation.
20. Suspect hemolytic uremic syndrome in a child with sudden-onset abdominal pain, bloody
diarrhea, and triad of anemia, thrombocytopenia, and renal failure.
21. The prodrome of measles (Rubeola) may include Koplik spots, a pathognomonic finding
which appears as bluish white lesions on the erythematous buccal mucosa opposite to the first
and second upper molars, and sometimes on the inner conjunctivae and vaginal mucosa. In the
US, even a single case of suspected measles should be reported to the local health authorities.
Aptical measles occurs in persons who have previously received inactivated measles vaccine (the
measles vaccine available now is a live one). This form is potentially life-threatening, and is
characterized by atypical rash (not maculopapular), the absence of Koplik spots, arthritis, hepatitis
and lung involvement. Edema of the hands and feet may occur.
22. Amoxicillin for 21 days is the recommended tx for erythema migrans or stage 1 Lyme disease
in children <9 years old. Doxycycline (100mg po bid for 21 days) is the recommended tx I
children>9 years old. Alternatives for those who cannot take amoxicillin or doxycycline
include cefuroxime axetil or erythromycin for 21 days. Ceftriaxone or penicillin G for 14-21
days is recommended for Stage 3 Lyme disease, which is a disseminated disease with
neurological and/or cardiac manifestations.
23. The most common cause of amblyopia 弱视 is strabismus. Other causes include (1): errors of
refraction and (2) opacity of media along the visual axis. The standard of tx is occlusion of the
normal eye. * High yield!!
24. Infants who should not be breastfed include those with certain inborn errors of metabolism
such as galactosemia, PKU, and urea cycle defects. Other C/I to breastfeeding include
maternal use of certain medications (radioactive isotopes, antimetabolites, chemotherapeutic
agents), active drug abuse, TB, HIV.
25. ECG in pts with Wolff-Parkinson-White syndrome is characterized by a shortened PR
interval, delta waves, and widening of the QRS complex. The underlying cause is an
accessory pathway conducts antegrade chest from the atria to the ventricles faster than the
conduction through the AV node, which allows part of the ventricle to depolarize early.
26. Chronic granulomatous disease should be considered in any pt with recurrent or unusual
lymphadenitis, hepatic abscesses, osteomyelitis at multiple sites, or unusual infections with
catalase-positive organism (eg. S aureus, Aspergillus, Serratia marcescens, and Burkholderia
cepacia).
27. Transmission of HIV by breastfeeding is well documented; therefore, the present of maternal
HIV infection is an absolute C/I to breastfeeding.
28. Myocarditis in children is usually due to viral infection and presents with fever, lethargy, and
signs of myocardial dysfunction after a viral prodrome. Affecte children should be mornitored

201
in the ICU because of the risk of acute decompensation and fetal arrhythmias.
29. Presence of inflamed tympanic membrane and diminished movement of the membrane with
insufflation are the hallmarks of otitis media. Fever is not required to make the dx of acute
otitis media.
30. For pertussus prevention, all close contacts should be given erythromycin for 14 days,
regardless of age immunization, or symptoms.
31. Pts with Down’s syndrome or trisomy 21 present with flat face, upward and slanted palpebral
fissures, epicanthal folds, simian crease, short and broad hands, hypoplasia of the middle
phalanx of the 5th fingers, and high arched palate.
32. Consider IgA deficiency in pts with mild immunodeficiency consisting of recurrent
sinopulmonary and GI infections, and anaphylactic transfusion reaction. Dx may be
confirmed by measurement of a low serum IgA with normal serum IgM and IgG levels.
33. Clavicle fracture presents with irregularity, crepitus, and fullness over the fracture site, and
decreased movements of the arm. Predisposing factors are shoulder dystonia, traumatic
delivery, and large size of the infant. Generally, no specific tx is required.
34. Group B streptococcus is the MCC of neonatal sepsis. Early sepsis presents in the first week
of life and usually involves several organs. Late-onset GBS infection is usually a focal
infection, and is most frequently meningitis (75%). E.Coli meningitis is the second MCC.
Listeria is the 3rd MCC. It also tends to result in multiple abscesses and pneumonia, in
addition to meningitis.
35. 3 P’s of McCune-Albright syndrome: precocious puberty, pigmentation (café au lait spots)
and polycostotic fibrous dysplasia. This pt’s moonlight facise and bruises indicate Cushing’s
syndrome, which occurs in association with McCune-Albright syndrome. Cushing disease
consists of hypercortisolism secondary to an ACTH-secreting pituitary tumor.
36. Osteogenesis imperfecta is caused by mutations in type I collagen. Its typical features are blue
sclera and recurrent fractures. * Extremely high yield!!
37. Cystic fibrosis can present as failure to thrive due to fat and protein malabsorption form a
pancreatic insufficiency.
38. Minimal change disease is the MCC of nephritic syndrome in pts younger tha 16 years old.
Light microscopy and immunofluorescence does not usually reveal any change in kidney
architecture. Electron microscopy show diffuse effacement of foot processes of podocytes.
Steroids are the tx of choice.
39. Childhood obesity is a risk factor fo the development of type 2 diabetes later in life.

1/24/10
OB & GYN
1. In the presence of decreased fetal movement, fetal compromise should be suspected and
the best next step is mx is a nonstress test (NST). NST is usually performed in high risk
pregnancies starting at 32-34 weeks gestation or when there is a loss of perception of fetal
movements in any pregnancy. NST is carried out by recording the FHR while monitoring for
spontaneous perceived fetal movements. A test is considered reactive (normal) if in 20min 2
accelerations of the FHR of at least 15 beats per minutes above the baseline lasting at least 15
seconds each are noted. If less than 2 accelerations are noted in 20min, the test is considered
nonreactive (abnormal) and further assessment is required. The most common cause of a

202
nonreactive NST is a sleeping baby, not a diseased baby, so vibroacoustic stimulation is used
to wake the baby up and allow a timely test.
A biophysical profile (BPP) is a scoring system designed to evaluate fetal well-being. It is
indicated in high risk pregnancies and in cases of maternal or physical concern, decreased fetal
movements or a non-reactive NST.
In a contraction stress test (oxytocin challenge test), the mother is given an infusion of oxytocin
sufficient to result in 3 contractions per 10 minutes, and the effect these contractions have on fetal
heart activity is recorded. If a late deceleration is noted at each contraction, the test is positive and
delivery is usually recommended. Because this is a more invasive test, it is not uses as an initial
examination.
Ultrosonography is not as sensitive as sensitive as NST or BPP for evaluation of decreased fetal
movements and fetal well-being. It is, however, the first step if fetal demise is suspected as it can
document the presence or absence of fetal heart movement.
2. The MCC of an abnormal maternal serum alpha-fetoprotein (MSAFP) level is
gestational age error. After detecting increased MSAFP levels, the physician should first
perform an ultrasound to detect the presence of any anomaly that may be seen by ultrasound,
assess fetal size to determine if it agrees with dates and to identify possible multiple gestation.
Amniocentesis may also be performed to identify the presence of neutral tube defects. In the
presence of a neural tube defect, amniotic fluid AFP and acetylcholinesterase will both be
elevated.
MSAFP is decreased in Edward syndrome and in Down syndrome. These conditions are screened
for using the quadruple screen. Down syndrome will cause low MSAFP, low estriol, eleveated
bata-HCG and elevated inhibin A level. A typical profile for Edward syndrome shows low
MSAFP, low estriol, vey low beta-HCG and normal inhibin A level.
3. Risk factors for osteoporosis include advanced age, thin body habitus, cigarette
smoking, alcohol consumption, corticosteroid ues, menopause, malnutrition, family history of
osteoporosis, and Asian or Caucasian ethnicity.
4. Screening for syphilis is strongly recommended in all pregnant women, regardless of
their risk factors for sexually transmitted infections. The USPSTF recommends screening for
Chlamydia infection in all women age 24 and younger and those at increased risk (history of
sexual transmitted infections, new or multiple partners).
5. Acquired hypogonadotropic hypogonadism is a cause of amenorrhea seen most
commonly in association with significant stressors, eating disorders and excessive exercise.
Pulsatile GnRH therapy can induce ovulation in these pts.
6. The genital ulcers seen in chancroid and hepes genitalis differ from the ulcer of primary
syphilis in that both are painful. Chancroid is also characterized by ulcers with a deep,
purulent base and painful lymphadenopathy. Genital herpes presents with multiple vesicles
following a prodrome of burning and pruritis. Within days, these vesicles become painful
ulcers. Like syphilis, granuloma inguinale (Donovanosis) presents with painless genital
ulcers. These ulcers have a red, beefy base and there is no associated adenopathy. Unlike
primary syphilis, the ulcer of granuloma inguinale does not resolve without antibiotic tx.
7. Nontreponemal serologic tests (VDRL, RPR) are used as a screening test for syphilis,
and treponemal serologic tests (FTA-ABS) are used for confirmation. Darkfield microscopy is
also an effective method of diagnosing syphilis, but requires proper equipment and clinical

203
expertise. In primary syphilic, there is a high rate of false-negative results to serologic testing,
and therefore darkfield microscopy is necessary.
8. Most menstrual cycles in the first one to two years following menarche are anovulatory.
These cycles are typically irregular and may be complicated by menorrhagia.
9. Endometriosis is a cause of subfertility and infertility. Women may be asymptomatic,
but typically experience pelvic pain, dyspareunia and pain with defecation.
10. In a pt with primary amenorrhea, 1) FSH measurement should be ordered if there is no
breast development. Increased FSH indicates a peripheral cause and decreased FSH indicates
a central cause. 2) GnRH stimulation test is the next step if FSH is decreaed. 3) Karyotyping
is the next step if FSH is increased.
11. Labor should be induced immediately in pts with intrauterine fetal demise who develop
coagulation abnormalities. A fibrinogen level in the low-normal range can indicate developing
DIC. When fibrinogen levels are normal the mx decision will depend on the pt’s preference:
the options are either watchful expectancy or induction of labor. The logic behind watchful
expectancy is that labor occurs spontaneously in 80% of cases within 2-3 weeks of IUFD.
12. Casarean section is used in the mx of placental abruption when there are obstetrical
indications, or when there is a rapid deterioration of the state of either the mother or the fetus
and labor is in an early stage such that vaginal delivery is not emergently possible.
13. Asymptomatic bacteriuria occurs when the urine culture grows >100,000 CFU per ml
of a single organism in an asymptomatic pt. It is important to promptly treat the infection to
prevent progression to pyelonephritis in the pregnant pt, because it may cause septicemia,
preterm labor or low birth weight babies. Amoxicillin, ampicillin, nitrogurantoin and
cephalexin are commonly used to treat the pts.
14. Fetal growth restriction (FGR) can be symmetric and asymmetric. In symmetric growth
restricition, the insult to the fetus begins before 28-weeks gestation and growth of both the
head and the body is deficient. It is usually caused by fetal factors such as chromosomal
abnormalities, congenital infections and congenital anomalies. Asymmetric FGR is a result of
fetal adaptation to non-ideal maternal factors. Asymmetric FGR results from fetal
redistribution of blood flow to vital organs, such as the brain, heart and placenta, as the
expense of less vital organs, such as the abdominal viscera. Maternal factors such
hypertension, hypoxemia, cigarette smoking, vascular disease and toxic exposures can lead to
asymmetric FGR. Asymmetric FGR has a better prognosis than symmetric FGR.
15. In pregnant pts, asymptomatic bacteriuria increases the risk of cystitis, pyelonephritis,
preterm birth, and perinatal mortality. E.coli accounts for more than 70% of cases. Tx in
pregnancy consists of a 7-day course of nitrofurantoin, amoxicillin or a first-generation
cephalosporin.
16. Failure to lactate is the classical initial presentation of Sheehan’s syndrome, or
postpartum ischemic necrosis of the anterior pituitary due to hemorrhagic shock. Failure to
produce milk in this condition results from prolactin deficiency.
17. Pelvic muscle exercises (Kegel exercises) and urethropexy are the recommended tx
options for stress incontinence. Kegel exercises should be attempted before pursuing surgery,
because urethropexy is invasive and carries risk.
18. Gestational trophoblastic disease should be ruled out in pregnant pts with severe
vomiting. Measurement of quantitative beta HCG should be checked first. If the beta HCG

204
levels is markedly elevated an ultrasonogram is then performed.
19. Hyperemesis gravidarum is a severe form of vomiting during pregnancy. It presents
with severe and persistent vomiting, weight loss, and ketonuria. The DD includes
pyelonephritis, gestational trophoblastic disease, gastroenteritis, and hepatobiliary disease.
Mild increase in ALT, AST, bilirubin, amylase, and lipase are seen in up to 50% of
hospitalized pts. Elevated amylase and lipase are from salivary gland due to vomiting.
20. A missed abortion is a form of spontaneous abortion that is characterized by
intrauterine fetal death before 20 weeks gestational age with complete retained products of
conception and a closed cervix. Pts typically present with loss of pregnancy symptoms and
some brown vaginal discharge, and a transvaginal ultrasound is necessary to confirm the dx.
21. The most appropriate tx for a missed abortions is removal of the POC (product of
conception) from the uterus. This can be accomplished surgically with dilation and curettage,
medically with vaginal misoprostol or expectantly with serial imaging to ensure complete
natural expulsion of the POC.
22. In a normal pt, the tone is typically louder when placed next to the ear as opposed to on
the mastoid process, indicating increased air conduction compaired to bone conduction. The
normal finding of air greater than bone conduction can also be found in sensorineural hearing
loss. Bone conduction that is greater than air conduction on the Rinne test is suggestive of
conductive hearing loss. Otoslcerosis is the MCC of conductive hearing loss in adults, and is
most common in pts in their 20’s and 30’s. There is a slight female predominance. In
otosclerosis, the stapes footplate becomes fixed to the oval window, resulting in loss of its
piston action. This disorder is sometimes also referred to as otospongiosis b/c CT may show a
lucent (as opposed to sclerotic) focus in the temporal bone near the oval window. Therefore,
CT could be used to confirm the dx in this case.
23. Radiation therapy is highly effective for squamous cell carcinoma of vagina. It is an
excellent alternative of pts who are poor surgical candidates. Tx of vaginal cancer depends on
staging. Stage I and II tumors (no extension to the pelvic wall and no metastases) which are
less than 2cm in size may be removed surgically, while stage I and II which greater than 2cm
in size are treated with radiation therapy.
24. Non-adolescent women with ASC-US (atypical squamous cells of undetermined
significance) on Pap smear should be followed up with reflex HPV testing to exclude the
presence of HPV types that are considered high-risk for the development of cervical cancer
(types 16, 18, 31,33,35,45 and 58). If low-risk HPV stains are found, then the pt can be
followed with an annual Pap smear.
ASC-US
-------------------------------------------------------------
↓ ↓
Adolescents Non-adolescent woman
↓ ↓
Repeat Pap in 12 months Reflex HPV resting

If ‘positive’ → Colposcopy
25. Pseudohyphae are characteristically seen on wet mount preparation of vaginal discharge
from pts with Candida vulvovaginitis. Symptomatic pts can be treated with an azole

205
antifungal, such as fluconazole. It is not a sexually transmitted infection, so sexual partners do
not require tx.
26. Thichomonas infection causes inflammation, pruritus, and a thin malodorous vaginal
discharge (PH 5.0-6.0). In contrast, bacterial vaginosis does not cause inflammation
(abnormal elevation of the vaginal PH), and candida viginitis causes a discharge that is thick
and white (PH 4.0-4.5).
27. In cases of mild preeclampsia, if the pregnancy is remote from term of the fetal lungs
are not yet mature, the pt is best managed with bed rest and close observation.
28. Genital warts are caused by the HPV and present as clusters of pink lesions on the
genitalia. Small lesions can be treated in the office with trichloroacetic acid or podophyllin.
29. The MCC for decreased fertility in women in their fourth decade who are still
experiencing menstrual cycles is age-related decreased ovarian reserve.
30. Pts with PCOS are that increased risk of developing type II diabetes. Standard 2-hour
oral glucose tolerance test (OGTT) identifies most pts with impaired glucose tolerance and
early type 2 diabetes better than fasting glucose alone.
31. DUB is the MCC of abnormal uterine bleeding. It is a diagnosis of exclusion and is
most often due to anovulation. High dose estrogen is the mx of choice for uncontrolled
bleeding. In women >35 with DUB, endometrial biopsy is indicated.
32. In a complete abortion, the whole conceptus passes through the cervix. The cervix then
closes, an the associated pain and uterine contractions subside.
33. Symptoms of menopause include irregular or absent menses, heat intolerance, flushing,
insomnia, and night sweats. Hyperthyroidism and menopause can have similar presentations,
and serum TSH and FSH levels should be checked in pts with these symptoms.
34. In the immediate postpartum period, a low-grade fever, leukocytosis and vaginal
discharge are normal findings. The vaginal discharge (lochia) is initially bloody, then serous
(3-4 days) and finally white to yellow in color days following delivery. Culture of the vaginal
discharge would be required if there is foul smelling discharge or if the pt’s fever and
leukocytosis persists or worsens.
35. Levonorgestrel (“Plan B”) is the recommended method of emergency contraception.
This progestin-only method is considered effective up to 120 hours after intercourse, although
effectiveness is greater the earlier the medication is administered. There are no
contraindications to the use of levonorgestrel, and no physical exam or lab testing is required.
It has the lowest incidence of side effects amongst emergency contraceptives, although nausea
(20%) and vomiting (5%) may occur. Individuals 18 years of age or older can obtain
levonorgestrel over-the-counter, whereas individuals under 18 must obtain a prescription in
most states. Levonorgestrel prevents pregnancy in approximately 7 out of 8 women who
would have otherwise become pregnant form intercourse.
Intramuscular medroxyprogesterone (Depo-provera) is a primary method of birth control, with
shots administered every 3 months. Prosraglandin E2 suppositories may be used as a second
trimester abortifacient. Neither is recommended as a form of emergency contraception.
36. Endometrial biopsy is indicated in cases of DUB affecting women>35 years old. It is
also indicated in cases of DUB if hypertension, diabetes, or obesity are present.
37. One of the effects of preeclampsia is damage to the liver that can cause right upper
quadrant pain due to stretching of the hepatic (Glisson’s) capsule.

206
38. Magnesium sulfate is given to women with eclampsia and severe preeclampsia in order
to prevent the further development of seizures while delivery of the fetus is accomplished.
Delivery is the most important overall tx for eclampsia.
39. Metformin is indicated in polycystic ovarian syndrome pts with impaired glucose
tolerance. It helps in preventing type 2 diabetes mellitus as well as improving obesity,
hirsutism, menstrual irregularity, and infertility.
40. Pseudocyesis is a rare psychiatric condition in which a woman presents with nearly all
signs and symptoms of pregnancy, however, ultrasound reveals a normal endometrial stripe
and negative pregnancy test. Pseudocyesisi is usually seen in women who have a strong desire
to become pregnant. It has been suggested that the depression caused by this need is behind
the occurrence of some hormonal changes mimicking those of pregnancy. This is a form os
conversion disorder, and management requires psychiatric evaluation and tx.
41. In false labor, progressive cervical changes are absent, contractions are irregular and
discomfort is readily relieved by sedation. All such pts need reassurance.
42. Systemic and topical corticosteroids can induce an acneiform eruption characterized by
monomorphous, erythematous follicular papules distributed on the face, trunk and extremities.
Comedones are characteristically absent.
43. Precocious puberty has both central and peripheral causes. Central precocious puberty
presents with increased FSH and LH, while peripheral causes present with low FSH and LH.
All pts with central precocious puberty should receive brain imaging with CT and MRI. Tx is
with GnRH analog therapy.
44. Hormone replacement therapy is now only recommended for the short-term use of
controlling menopausal symptoms. Large studies, such as the WHI, have shown that the long-
term use of combined HRT can slightly increase the risk of coronary heart disease and stroke.
Conclusions from the WHI (Women’s health initiative) trial: 1) there is no cardiovascular
benefit with either unopposed estrogen or combined estrogen-progestin therapy. 2) there is an
increased risk of stroke with long-term use of both unopposed estrogen or combined estrogen-
progestin therapy. 3) There is an increased risk of breast cancer with combined estrogen-
progestin therapy, but not with unopposed estrogen. The benefits of HRT are that it helps
relieve the symptoms of vaginal dryness, hot flashes and some of the mental changes seen in
menopause. It also protects against osteoporosis, and to a minor degree against colon cancer.
The risks of HRT include an associated increased incidence of venous thromboembolism and
endometrial hyperplasia/cancer (only with unopposed estrogen). This risk can be largely
reduced by the addition of progestin therapy.

1/24/10
1. A reactive NST is reassuring and it should be repeated weekly.
2. This pt has a female phenotype but lacks a normal vaginal and uterus, which narrows the
etiology of her primary amenorrhea to mullerian agenesis, androgen insensitivity, or 5-
reductase deficiency. The karyotype is the determinig test, with both androgen insensitivity
and 5-alpha-reductase deficiency being seen in pts with a XY genotype. This pt’s genotype is
XX, which leaves mullerian agenesis as the best explanation for her condition. The mullerian
duct normally leads to the development of the proximal vagina and the uterus; therefore pts
with mullerian agenesis normally have a blind ended vagina with little to no uterine tissue.

207
Androgen insensitivity pts have a male XY genotype but there is an abnormality in the androgen
receptor. The external genitalia develop as female, but mullerian inhibiting factor is still secreted
by the testes which prevents the development of internal female organs.
5-alpha-reductase deficiency cannot convert testosterone to the more potent DHT. They hava a
male XY genotype and female external genitalia, but typically show virilization at puberty.
3. Eclampsia is diagnosed when unexplained convulsion occur in the setting of preeclampsia.
4. The most effective agent used for tx and prevention of seizure in eclampsia is magnesium
sulfate.
5. Abdominal circumference is the most effective parameter for estimation of fetal weight in
cases of suspected FGR, b/c it is affected in both symmetric and asymmetric fetal growth
restriction.
6. Prolactin production is stimulated by serotonin and TRH and inhibited by dopamine.
Hypothyroidism may result in amenorrhea and galactorrhea.
7. Serum BUN and creatinine are usually decreased in pregnant pts due to an increase in renal
plasma flow and glomerular filtration rate.
8. Elevated prolactin levels suppress GnRH release thereby suppressing LH and FSH production
and ovulation. This is the reason for anovulation and amenorrhea in lactating mothers.
9. The superficial femoral vein is a deep vein of the thigh and a thrombus of this vein requires
anticoagulation. Pregnancy is a major risk factor for deep venous vein thrombosis, especially
during the peripartum period.
10. There is no role for medications in the suppression of breast milk production. Pts are advised
to wear a tigh-fitting bra, avoid nipple manipulation and use ice packs and analgesics to
relieve associated pain.
11. Weight gain has not been associated with the use of combination oral contraceptive pills in
controlled studies. Pap smear screening should be initiated in all women 3 years after
initiation of sexual intercourse but no later than 21 years of age.
12. Radiation levels used for most diagnostic exams are not associated with teratogenicity.
Exposure to ionizing radiation doses below 5 rad (5cGy) is not associated with known fetal
abnormalities. Therapeutic abortion is advised when the radiation level is between 5-10 cGy.
13. Pts with androgen resistance present with amenorrhea, normally developed breasts, absent
pubic and axillary hair, absent reproductive organs, and a 46 XY karyotype. Serum
testosterone levels are in a range typical for males. The internal reproductive organs do not
develop because the testes are still present and secret mullerian inhibiting factor.
14. Early decelerations are depressions in fetal heart rate that resolve by the end of the uterine
contraction. Early decelerations are not considered a nonreassuring heart rate pattern, and are
the result of fetal head compression.
Fetal sleep presents with decreased long-term variability.
Fetal cord compression presents with variable decelerations.
Uteroplacental insufficiency presents wht late decelerations.
Intrauterine infections may present with fetal tachycardia (HR>160).
15. In adolescent females, ASCUS or LSIS (low grade squamous intraepithelial lesions) are most
likely due to HPV infection. Because frank cervical cancer in this population is uncommon,
the best mx strategy is a follow-up Pap smear in 1 year.
LSIL (mild dysplasia or CIN 1)

208
-------------------------------------------------------------------------------------------------------
↓ ↓ ↓
Adolescents Premenopausal woman Postmenopausal woman
↓ ↓ ↓-------------------------↓
Repeat Pap in 12 months Colposcopy Reflex HPV resting Colposcopy
↓ ↑
If ‘positive’-------------------↑
16. Mx of placenta previa depends on the severity of bleeding and the age of the pregnancy.
Complete placenta previa requires delivery by cesarean section as the placenta completely
covers the os cervix and the fetus cannot be delivered vaginally. At 36 weeks gestation,
amniocentesis should be done in order to assess lung maturity. If the fetal lungs are mature,
elective cesarean section can be performed.
17. Congenital aromatase deficiency is a rare genetic disorder marked by either total absence or
poor functioning of the enzyme that converts androgens into estrogens. Its consequences are
numerous. In utero the placenta will not be able to make estrogens, leading to masculinization
of the mother that resolves after delivery. The high levels of gestational androgens result in
virilized XX child with normal internal genitalia but ambiguous external genitalia.
Clitoromegaly is often seen when excessive androgens are present in utero. Later in life pts
will have delayed puberty, osteoporosis, undetectable circulating estrogens, high
concentrations of gonadotropins and polycystic ovaries.
McCune-Albright syndrome (aromatase)↑ is marked by the triad of café-au-lait spots, polyostotic
fibrous dysplasia, and autonomous endocrine hyperfunction. The most common endocrine feature
is gonadotropin independent precocious puberty. Thus, pts have early puberty.
18. Both general and spinal anesthesia as well as sedation may reduce uterine activity if
administered in the latent phase (<2cm) thus prolonging this stage of labor.
19. All healthy pregnant women with uncomplicated pregnancies are encouraged to exercise for
30 minutes daily at a moderate intensity that allows the mother to carry on conversation while
exercising.
20. Renal calculi in pregnancy require special consideration because most of the standard
investigatory modalities will expose the fetus to radiation. The modality of choice in such pts
is abdominal or pelvic ultrasonography. If non-informative, transvaginal ultrasonography
should be performed next to help detect distal ureteral stones. If this examination is also
negative, but the pt continues to have discomfort or signs suggestive of nephrolithiasis, two
options are available for dx: limited intravenous pyelogram (IVP) and ureteroscopy. Limited
or single-short IVP delivers the maximum recommended monthly radiation exposure, and is
positive in most pregnant women with a symptomatic stone.
21. The most appropriate test to confirm the diagnosis of intrauterine fetal demise (IUFD) is real
time ultrasonography to demonstrate an absence of fetal movement and cardiac activity.
22. A search should be undertaken to determine the cause after the first episode of intrauterine
fetal demise. Autopsy of the fetus and placenta should be performed in all cases of stillbirth
with the permission of the parents.
23. Depression of the deep tendon reflexes is the earliest sign of magnesium sulfate toxicity. Tx
requires stopping the magnesium sulfate infusion and administration of calcium gluconate.
Oxytocin is analogous to ADH and may be responsible for hyponatremia and water

209
intoxication.
24. The combination of thrombocytopenia, microangiopathic hemolytic anemia and increased
liver enzymes in pt wit preeclampsia is defined as HELLP syndrome. Right upper quadrant
pain is typical of this condition sue to distention of the hepatic (Glisson’s capsule). Hemolytic
uremic syndrome (HUS) also causes thrombocytopenia and hemolytic anemia. As in HELLP
syndrome, both PT and PTT are normal; however, HUS also usually results in renal failure
and not necessary associated with preeclampsia.
25. Delivery is the definitive treatment of HELLP syndrome in women beyond 34-weeks
gestation.
26. Endometritis is characterized clinically be fever and uterine tenderness in the postpartum
period and is often associated with foul-smelling lochia. Risk factor include prolonged ROM,
prolonged labor, operative vaginal delivery and caesarian section among others.
27. Psotpartum endometritis is the MCC of puerperal fever on the 2nd and 3rd day postpartum.
Endometritis is a polymicrobial infection caused by a combination of gram positive and gram
negative organisms, aerobic and anaerobic organisms and occasionally other organisms such
as Mycoplasma and Chlamydia.
28. The tx of choice for postpartum endometritis, which is a polymicrobial infection, is
intravenous clindamycin and gentamicin.
29. A young woman who presents with a breast lump can be asked to return after her menstrual
period for reexamination (which may reveal regression of the mass) if no obvious signs of
malignancy are present.
30. The major source of estrogen in menopausal women is from the peripheral conversion of
adrenal androgens by the aromatase enzyme present in adipose tissue. This process is increase
is increased in obese women who may have decreaed menopausal symptoms.
31. A child’s blood group frequently differs from the mother’s blood group, but this fact does not
create a potential for alloimmunization-related fetal damage. Anbodies to ABO antigens to the
IgM class and do not cross the placenta. Anti-D antibodies that are responsible for Rh
alloimmunization belong to the IgG class and cross the placenta easily.
32. An intrauterine pregnancy should be seen with transvaginal ultrasonography at beta-HCG
levels of 1,500-2,000 mIU/mL. If the level is <1000 mIU/mL, both beta-HCG and
transvaginal ultrasonography should be repeated in 2-3 days. Beta-HCG should generally
double about every 48 hours in viable pregnancies, but increases at a slower rate in ectopic
and noninviable intrauterine pregnancies.
33. Dibetes screening is performed between 24 and 28-weeks of gestation. The proper initial
screening test is the one hour 50 gram oral glucose tolerance test. After one hour, if the blood
glucose value is less than 140 mg/dl, then gestational diabetes is ruled out. If >140mg/dl, then
a 3 hour OGTT is performed. Gestational diabetes is diagnosed if two or more of the serum
glucose values obtained during the 3 hour test are elevated above the values listed below:
Fasting serum glucose concentration >95 mg/dl, 1-hour >180, 2-hour > 155, 3 hour > 140.
34. Arrest disorder of dilation has not progressed for more than 2 hours. It can also be of descent
when the descent has not progressed for more than 1 hour. It can be caused by hypotonic
contractions, conduction anesthesia, excessive sedation, cephlopelvic disproportion or
malpresentation. In the present case, the arrest is resulting from a midpelvic contraction
indicated by the prominence of the ischial spines.

210
35. In the case of amenorrhea-hypoprolactinemia, first rule out hypothyroidism by measuring
serum TSH.
36. Mild preeclampsia is defined clinically by hypertension greater than 140/90mmHg and
proteinuria greater than 0.3/24h (300mg/24h) after the 20th week of gestation.
37. Methyldopa is a pregnancy category B agent and is the first-line medication for tx for
hypertension in pregnancy.
38. Lichen sclerosus most commonly affects postmenopausal women and manifests with vulvar
prutitus and discomfort. Exam shows porcelain-white atrophy. Biopsy should be considered to
rule out valvar SCC. Lichen sclerosus is considered a premalignant lesion of the vulva as
vulvar SCC occurs with greater frequency in these pts as compared to the general population.
39. High-potency topical steroids are considered first-line therapy for lichen sclerosus.
40. Premature ovarian failure is characterize by amenorrhea, hypoestrogenism, and elevated
serum gonadotropin levels in women younger than 40 years of age. It may be secondary to
accelerated follicle atresia or a low initial number of primodial follicles. It is most commonly
idiopathic but may also be due to mumps, oophoritis, irradiation or chemotherapy. It can be
associated with autoimmune disorders such as Hashimoto thyroiditis, Addison disease, type I
DM, and pernicious anemia, which supports wht hypothesis that at least some cases of
idiopathic premature ovarian failure are of autoimmune origin. Women present with signs and
symptoms that are similar to those seen in menopause. The dx is confirmed by demonstrating
increased serum FSH and LH levels and secreased estrogen levels. Pts with premature ovarian
failure lack viable oocytes, so the only option available ot allow pregnancy in such pts is in-
vitro fertilization.
41. Infertility in women with irregular menstrual cycles is often due to anovulation. Ovulation can
be tested for by measuring a midluteal phase progestone level.
42. Hypothalamic-pituitary-gonadal axis immaturity is a cause of irregular menstrual cycles in
women shortly following menarche.
43. In any woman of childbearing age with secondary amenorrhea, first rule out pregnancy.
44. Physician can refuse to perform elective abortions for personal or professional reasons. If you
refuse to perform any procedure then you should attempt to refer the pt to another physician
who can and will.

1/25/10
1. Granulosa cell tumors produce excessive amounts of estrogen, and can present with
precocious puberty in younger children and postmenopausal bleeding in elderly pts. This has
to be differentiated from heterosexual precocious puberty or virilizing symptoms which are
usully produced by excessive androgens.
2. Epidural anesthesia may cause overflow incontinence as a transient side effect. It is best
treated with intermittent catheterization. Oxybutynin has anticholinergic properties and
inhibits smooth muscle contraction. This drug is used for urge incontinence.
3. This is a case of hypergonadotropic hypogonadism, b/c the pt has a primary amenorrhea with
absent secondary sexual characteristics and increased FSH level. A differential dx of
hypergonadotropic hypogonadism includes conditions that present with primary amenorrhea
and either a male or female karyotype, making karyotype determination the most appropriated
next step in the work up. These conditions include Turner syndrome (45, XO), Savage

211
syndrome, or ovarian resistance syndrome (46, XX), male gonadal agenesis (46, XY), and
defects in testosterone production, such as in deficiency of 17-alpha-hydroxylase and 17, 20-
desmolase (46, XY).
Prolactin and TSH levels are indicated in secondary amenorrhea after ruling out pregnancy. They
usually do not have problems with secondary sexual characteristics.
4. An increase in blood pressure that appears before 20 weeks gestations is due to either chronic
hypertension or a hydatiform mole.
5. Risk factors for placental abruption include prior placental abruption, chronic hypertension,
pregnancy-induced hypertension, cocaine, alcohol or tobacco use and vascular diseases such
as diabetes and SLE. The most significant risk factor is hypertension of any kind. Sustained
maternal blood pressure in excess of 160/110 should be treated pharmacologically.
6. Idiopathic central precocious puberty, which is the most common type in females, results from
the premature activation of the hypothalamic-pituitary-gonadal axis. Pts with central
precocious puberty have pubertal levels of basal LH that increase with GnRH stimulation,
whereas pts with a peripheral source of precocious puberty, such as in certain ovarian
pathologies, have low LH levels with no response to GnRH. All pts with central precocious
puberty should have brain imaging to rule out an underlying CNS lesion. This pt has had a
fairly rapid progression of pubertal maturation, and should be treated with GnRH agonist
therapy in order to prevent premature epiphyseal plate fusion.
7. Kallmann’s syndrome consists of a congenital absence of GnRH secretion (ie.
Hypogonadotropic hypogonadism) associated with anosmia. Pts have a normal XX genotype
and normal female internal reproductive organs. They present with amenorrhea and absent
secondary sexual characteristics such as breast development and pubic hair; the addition of
anosmia to the presentation may suggest the dx. Abnormal development of the olfactory bulbs
and tracts result in hyposmia or anosmia (decreased sense of smell). The FSH and LH levels
are low, in contrast to the levels in primary ovarian failure which are usually elevated.
8. Galactorrhea presents as guaiac negative bilateral nipple discharge. Prolactonoma,
hypothyroidism, overstimulation of the nipple, oral contraceptive pills, and medications which
lower dopamine levels are common cause. Workup includes ruling out pregnancy, measuring
serum prolactin and TSH levels, and possible MRI of the brain to rule out prolactinoma.
9. Penicillin is considered to be the tx of choice for pregnant pts with syphilis. If a pt is allergic
to penicillin, the allergic shoud be confirmed with skin testing and the pt should then undergo
desensitization so that she can safely take the medication. It is typically accomplished using
incremental doses of oral penicillin V.
10. Postterm pregnancies are at an increased risk for oligohydramnios, which itself is associated
with increased fetal morbidity. Postterm pregnancies should be monitored for
oligohydramnios twice weekly. Oligohydramnios in these cases is defined as no vertical
pocket of amniotic fluid greater than 2cm or an amniotic fluid index of 5cm or less.
11. Hypertension in a pregnant female in the setting of massive proteinuria, a malar rash, and a
strongly positive ANA titer is most likely due to SLE. Glomerulonephritis in general will
cause proteinuria, hematuria and RBC casts.
12. The CDC recommends that all pregnant women without contraindications be vaccinated
against influenza.
13. It is strongly recommended that asymptomatic bacteriuria in pregnant women be treated,

212
especially women at 12-16 weeks’ gestation. Recommended antibiotics include amoxicillin,
nitrofurantoin, and oral cephalosporins. Trimethoprim-sulfamethoxazole is a class C
mediation in pregnancy. It may be used during the second trimester but is not recommended
for use during the first trimester because it interferes with folic acid metabolism or during the
3rd trimester b/c it increases the risk of kernicterus in the newborn.
14. The combination of primary amenorrhea, bilateral inguinal masses, and breast development
without pubic or axillary hair is strongly suggestive of androgen insensitivity syndrome(a
mutation of the androgen receptor gene). A blind vaginal pouch and karyocype of 46XY are
other clues. A gonadectomy should be performed to avoid the risk of testicular carcinoma.
15. Sudden onset abdominal pain, fetal heart rate abnormalities, and recession of the presenting
part during active labor indicated uterine rupture. Rish factors include a preexisting uterine
scar or abdominal trauma.
16. The requirement for L-thyroxine in pts receiving estrogen replacement therapy increases. The
potential causes may include induction of liver enzyme, increased level of TBG, and an
increased volume of the distribution of thyroid hormones. In pregnancy, also, thyroid
hormone requirements will be increased, and the pt should be monitored every 4-6 weeks for
dose adjustment.
17. Secondary amenorrhea is relatively common in elite female athelets and results from estrogen
deficiency (due to decrased LH and GnRH). These amenorrheic women are therefore at
increased risk for all conditions associated with estrogen deficiency, including infertility,
vaginal atrophy, vaginal atrophy, breast atrophy, and osteopenia.
18. Lithium is associated with congenital heart disease, classically Ebstein’s anomaly, and should
be weaned in pregnant women with stable bipolar disorder. When a woman is treated with
isotretinoin, she should receive strict contraception. Inhaled steroids can be used during
pregnancy.
19. Laparoscopy is the gold standard for the dx of endometriosis.
20. Pts with endometriosis are at an increased risk of decreased fertility or infertility. Up to 30%
of females being evaluated for infertility are found to have endometriosis.
21. Ovulation can be induced in pts with PCOS with clomiphene citrate tx as well as with
metformin tx.
22. Anabolic steroid use by a male can produce infertility by suppressing the productin of GnRH,
LH and FSH. Klinefelter syndrome (XXY seminoiferous tubule dysgenesis) in an inherited
disorder characterized by resticular fibrosis (primary hypogonadism), azoospermia,
gynecomastia, decreased intelligence and decreased axial skeletal growth. FSH and LH will
be high.
23. If maternal serum AFP levels are abnormal in a pregnant pt, the next step is ultrasonography
to confirm gestational age, detect fetal structural anomalies, detect multiple gestation and
confirm a viable prennancy.
Amniocentesis is indicated in cases where the MSAFP pr triple/quadruple screen is abnormal but
only after ultrasonography has ascertained gestational age accuracy and ruled out nonviable
pregnancy or multiple gestation. It is best performed between 16-20 weeks gestation.
Chorionic villus sampling is indicated for early screening in women with known genetic diseases,
or who have had children affected by a genetic condition. It is performed between 10-12 weeks
gestation, and therefore offers the advantage of a first trimester termination of pregnancy if the

213
fetus is affected.
Cordocentesis is used for rapid karyotype analysis, or when fetal blood dyscrasias 恶液质, such as
fetal anemia and Rhesus isoimmunization, are suspected. In the present case, ultrasound should be
performed first to rule out causes of MSAFP elevation other than genetic anomalies.
24. Abdominal pain in a young female in the middle of her cycle with a benign history and
clinical examination is most likely mittleschmerz (midcycle pain). Ovarian torsion is a
medical emergency. Pts present with sudden-onset lower quardrant abdominal pain that
radiates to the groin or back and it accompanied by nausea and vomiting. An adnexal mass is
usually present.
25. Female offsping of women who ingested diethylstilbestrol during their pregnancy are at
increased risk of developing clear cell adenocarcinoma of the vagina and cervix as well as
cervical anomalies and uterine malformations.
26. Lupus anticoagulant can sometimes be seen in pts with SLE and is a potential cause of
recurrent pregnancy loss due to thrombus development within the placenta.
27. Painful third trimester vaginal bleeding with normal ultrasound is most likely due to placental
abruption.
28. Women shoudn be screened for group B strep colonization with vaginal and rectal cultures
obtained between 35-37 weeks gestation. Women colonized with GBS receive prophylactic
antibiotics (penicillin or ampicillin) at the time of delivery. Women who have had GBS
bacteriuria during pregnancy or who have previously delivered a child that developed an
early-onset GBS infection are automatically given prophylactic antibiotics regardless of the
results of a rectovaginal culture.
29. A biophysical profile score of 2 or less is consistent with severe fetal asphyxia. In this setting,
the baby should be delivered immediately.
30. Primary ovarian failure results in decreased estrogen levels and increased FSH and LH levels
(the elevation of FSH is generally greater than that of LH due to slower clearance of FSH
from the circulation). FSH elevation in the setting of >=3 months of amenorrhea in a woman
inder age 40 confirms the diagnosis of premature ovarian failure.
31. OCPs offer both risks and benefits as outlined below. The risks and benefits should be
weighed carefully for each individual pts.
Serious side effect Protective against
Venous throboembolism Ovarian cysts and cancer
Cardiovascular events/stroke Endometrial cancer
Elevation of triglyceride levels Benign breast disease
Cholestasis or cholecystitis Dysmenorrhea (anemia)
DM
Hypertension

32. Fetal distress (repetitive late decelerations) is an indication for emergent cesarean section.
33. Gestational diabetes is most commonly first treated with a low suger diet, but if diet fails to
produce euglycemia, insulin is the tx of choice.
34. Gestational diabetes carries numerous risks for the fetus including macrosomia, hypocalcemia
(due to parathyroid hormone suppression), hypoglycemia (due to high baseline insulin
production), hyperviscosity due to polycythemia (increased erythropoietin), respiratory

214
difficulties, cardiomyopathy and congestive heart failure.
35. The most important complication of PPROM is pulmonary hypoplasia (immaturity). Steroids
are used to enhance fetal lung maturity when premature rupture of membranes occurs at less
than 34-weeks of gestation.
36. Hypotension is a common side effect of epidural anesthesia. The cause of hypotension is
blood redistribution to the lower extremities and venous pooling.
37. Septic abortion is managed with cervical and blood cultures follows by IV antibiotics and
gentle suction curettage. Vigorous suction curettage may cause perforation of the uterus and
should be avoid.
38. Intersitial cystitis (painful bladder syndrome) is a chronic condition associated with pelvic
pain worsened by bladder filling or intercourse accompanied by urinary frequency, urgency,
and nocturia. The negative urinalysis excludes a urinary tract infection. Stress incontinence
refers to involuntary leakage of urine with effort, exertion, sneezing or coughing. Pain is
typically not a symptom of stress incontinence.
39. Transvaginal ultrasonogram is more accurate than transabdominal ultrasonogram in
diagnosing ectopic pregnancy, and should be performed when beta-hCG levels are between
1500-6500 IU/L.
40. Pts with PPROM whose GBS status is unknown should receive antibiotic prophylaxis.
Corticosteroids are indicated if the gestational age is <32 weeks. If a woman is admitted to the
hospital after 34 weeks gestation with PPROM, delivery is recommended.
41. In the primigravida, cervical dilatation in the active phase progresses at a speed of 1.2cm/hr.
Cesarean section is the appropriate tx for active phase arrest.
42. The first step in a pt with secondary amenorrhea is to rule out common situations; that is
pregnancy, then hyperprolactinemia, and hypothyroidism. The subsequent step should be the
determination of the pt’s estrogen status with progestin challenge test.
43. Intraamniotic infection should be suspected in the setting of prolonged or premature rupture
of the membranes when maternal fever, leukocytosis, uterine tenderness, or tachycardia is
detected. Fetal tachycardia is another feature of chorioamnionitis.
44. The most appropriate tx of chorioamnionitis associated with PPROM is systemic, broad-
spectrum antibiotic therapy and expedited delivery of the fetus. Caesarean section should be
reserved for cases where fetal distress is evident. Fetal tachycardia is a sign of maternal
infection.

1/26/10
1. HCG is a hormone is a hormone secreted by the syncytiotrophblast and is mainly responsible
for the preservation of the corpus leteum in early preanancy. The hCG is composed of 2
subunits, alpha and beta. The alpha unit is common to hCG, TSH, LH and FSH. The beta
subunit is specific to hCG, and is used as the basis of virturally all pregnancy tests. Other
biological function of hCG include the promotion of male sexual differentiation and
stimulation of the maternal thyroid gland.
Progesterones is produced in large amounts during pregnancy, and helps to inhibit uterine
contractions; preparing the endometrium for implantation of a fertilized oveum.
Estrogen is responsible for induction of prolactin production during pregnancy.
2. By treating HIV-infected women zidovudine throughout pregnancy and labor, and treating the

215
newborn for the first 6 weeks of life, the risk of HIV transminssion is reduced by 70%.
3. At the first prenatal visit, blood typing and Rh antibody testing (in Rh-negative women)
should be performed. Unsensitized Rh-negative women potentially carrying an Rh-positive
fetus should have their Rh-antibody status retested between 24-28 weeks’ gestation. Pts at risk
of alloimmunization should be given anti-Rh immune globin at 28 weeks and again at the
time of delivery.
4. CIN in a low risk pt with a low-grade lesion on Pap smear can be observed with annual Pap
smear screening.
CIN I
↓----------------------------------------------------------------------↓
CIN I preceded by low grade abnormalities CIN I preceded by high grade abnormalities
(ASC-US, ASC-H, LSIL) (HSIL; AGC-NOS)
↓ ↓
Repeat cytology in 12 months Immediate diagnostic excisional procedure
↓ (post child bearing age)
Still abnormal after 24 months (Or)
↓ Close clinical follow-up
Diagnostic excisional procedure (Women planning future pregnancy)
5. The presence of dysmenorrheal, heavy menses, and an enlarged uterus is classic for uterine
fibroids.
6. The classic manifestations of acute abruption placentae include vaginal bleeding, abdominal
pain, uterine contractions, and uterine tenderness. The absence of blood on pelvic exam does
not rule out this condition.
7. Suction curettage is the tx of choice for inevitable abortion. Anti-D immunoglobulin should be
administered to all Rh-negative women following any form of abortion in order to prevent the
formation of Rh foctor antibodies that may complicate subsequent pregnancies.
8. Infant born to pts with Graves disease treated with surgery are at risk for thyrotoxicosis
because of the passage of thyroid stimulating Ig across the placenta.
9. Missed abortion involves a dead fetus that is still retained in the uterine. The dx is suspected
when there is disappearance of the nausea and vomiting of early pregnancy, and an arrest of
uterine growth.
Inevitable abortion manisfests with vaginal bleeding, lower abdominal cramps that may radiated
to the back and perineum and a dilated cervix. Ultrasonography demonstrates a ruptured or
collapsed gestational sac with absence of fetal cardiac motion. Incomplete abortion has the same
presentation except that there is an incomplete evacuation of the conceptus. Ultrasonography
reveals endometrial debris.
10. Progestin-only oral contraceptives are the preferred hormonal contraceptives in lactating
women as they do not affect the volume or composition of milk produced by the mother, they
have no known effects on the infant and they do not carry the risk of venous thrombosis
associated with combination pills.
11. Ectopic pregnancy should be suspected in any pt of childbearing age, presenting with a triad
of amenorrhea, abdominal pain and vaginal bleeding. In all cases of secondary amenorrhea,
administering a pregnancy test should be the first diagnostic step.
12. Pregnancy is associated with an increase in TBG, resulting in increased total T4 and T3, a

216
normal free T4 and T3, and a normal TSH.
13. Breast engorgement is a common problem associated with breast feeding. Charaterized by
bilateral breast tenderness and swelling, it typically presents 24-72 hours post partum, peaks
3-5 days after delivery, and resolves spontaneously. Plugged dusts present similarly to mastitis
but lack fever or systemic symptoms. They are treated by improving the quality of
breastfeeding. Persistently plugged ducts resulting in galactocele may be treated with
aspiration.
14. During pregnancy, the primary goal of colposcopy is the exclusion of invasive cancer. Any
woman with a cytologic specimen suggesting HSIL should undergo colposcopy and directed
biopsy. If the biopsy is negative, a second biopsy is recommended 6-8 weeks after delivery.
LEEP procedure is an excisional therapy and is recommended for all pts except adolescents
and pregnant woman with HSIL on Pap smear but without CINII or greater on biopsy since
the abnormal area may have been missed by the biopsy. Excisions in pregnanct women should
be considered only if a lesion suggestive of invasive cancer is detected at colposcopy.
15. Copious vaginal discharge that is white or yellow in appearance, nonmalodorous, and occurs
in the absence of other symptoms or findings on vaginal exam is referred to as physiologic
leucorrhea.
16. The dx of bacterial vaginosis (BV) is made when 3 of 4 Amsel criteria are met: 1) thin, gray-
white vaginal discharge. 2) Vaginal pH >4.5. 3) A positive “whiff” test upon addition of KOH
to the vaginal discharge. 4) “clue cells” (vaginal epithelial cells with adherent coccobacilli) on
wet mount.
17. External cephalic version (ECV) can be attempt in women with breech pregnancies at greater
than 37 weeks gestational age if there are no contraindications to vaginal delivery and fetal
well-being has been established. EVC has been shown to reduce the rate of cesarean sections,
but the maneuver has the potential to result in fetal distress, so it should only be performed
when arrangements have been made to allow for an emergent caesarian delivery.
18. The most appropriate first steps in the mx of variable decelerations (due to umbilical cord
compression) are oxygen administration and change in maternal position. If the variable
decerlerations persist, the pt should be placed in the Trendelenburg position and the presenting
fetal part should be elevated. Persist variable decelerations may require amnioinfusion, which
consists of the infusion of fluid into the amniotic cavity. Fetal scalp pH testing should be
performed to assess for fetal hypoxia if the abnormal FHR pattern persists after the initial
measures of position change, oxygen administration, and discontinuation of oxytocin have
been tried. Cesarean section is indicated when fetal distress is confirmed.
19. For acute preterm labor bed rest and tocolysis are continued as long as possible with a long-
term goal of reaching 34-36 weeks gestational age. In otherwise normal pregnancies, systemic
corticosteroids are administered when gestational age is between 24-34 weeks. Tocolysis
should then be attempt with the goal being to maintain the pregnancy for at least 48 hours in
order to realize maximum benefit from the steroids.
20. In the ovulatory phase of the menstrual cycle, cervical mucus is profuse, clear and thin (pH
6.5 or greater, more basic than at other pahses). It will stretch to approximately 6 cm and
exhibit ferning on a microscope slide smear preparation. In early follicular phase is thick,
scant and acidic. In the mid- and late-luteal phase, the cervical mucus becomes progressively
thicker and exhibits less stretching ability.

217
21. Suspect atrophic vaginitis in post-menopausal females with symptoms of vaginal dryness and
dysuria, and physical exam findings of pale, dry vaginal mucosa, diminished labial fat pad,
and scarce pubic hair. Moderate and severe cases require local estrogen therapy.
22. Bilateral edema of the lower extremities in pregnancy is most commonly a benign problem.
Preeclampsia and DVT should also be considered, but are unlikely in the absence of other
classic symptoms of these conditions.
23. Raloxifene is a mixed agonist/antagonist of estrogen receptors. In breast and vaginal tissue, it
is an antagonist, whereas in bone tissue, it is an agonist. It is a first-line agent for the
prevention of osteoporosis, and it decreses breast cancer risk. It increases the risk of
thromboembolism.
24. BPP includes the NST in addition to four parameters assessed by ultrasonography: 1/fetal
tone; 2/fetal movements (3/10min); 3/fetal breathing (30/10min); 4/amniotic fluid index, (5-
20). A total score of 8-10 is considered normal, and should only be repeated once or twice
weekly until term for high risk pregnancies.
1) If the presence of oligohydramnios (AFI<5) delivery is to be considered since it can
be result in umbilical cord compression and therefore fetal compromise.
2) If the score is 6 without oligohydramnios, contraction stress test should be ordered. If
this latter gives non-reassuring results, delivery is usually indicated; if it gives
suspicious results, repeat the next day.
3) If the score is 4 without oligohydramnios and fetal lungs are mature, delivery should
be considered. If fetal lungs have not yet reached their maturity, steroid injection
should be administered and BPP assessed within 24 hours.
4) If the score is <4, the fetus should be delivered.
25. An antepartum hemorrhage with fetal heart changes progressing from tachycardia to
bradycardia and finally to a sinusoidal pattern occurring suddenly after rupture of membranes
suggests the dx of vasa previa. The bleeding in this setting is fetal in origin, so maternal vital
signs will remain stable while the fetus exsanguiantes 放血.
26. Low doses of anti-D immune globulin postpartum is the most likely cause of anti-D
immunization in this pt. After the events that are associated with excessive feto-maternal
hemorrhage (eg. placental abruption), the failure to correct the dose of anti-D immune
globulin may result in maternal alloimmunization.
27. Primary dysmenorrhea is caused by increased levels of prostagladins and presents with lower
abdominal pain that occurs with menstruation. NSAIDs and oral contraceptive pills can be
used to improve symptoms.
28. PCOS is characterized by an unbalanced estrogen secretion that may result in endometrial
hyperplasia and carcinoma.
29. Premenopausal women with simple or complex hyperplasia without atypia respond to therapy
with cyclic progestins. All pts should undergo repeat biopsy after 3-6 months of tx. The risk of
pregression to endometrial cancer in pts with complex hyperplasia without atypia is low (1-
2%), and therefore even if this pt does not want more children, hysterectomy is not warranted.
The risk of pregression to endometrial cancer in pts with complex hyperplasia with atypia is
high (30%). Therefore, in premenopausal women diagnosed with complex hyperplasia with
atypia who have completed child bearing, total hysterectomy is the tx of choice. In women
who would like to preserve fertility and in those who are considered poor surgical candidates,

218
cyclic progestins with repeat biopsy in 3-6 months is the appropriate initial mx.
30. An arrest disorder of dilation and descent are defined by an arrest in dilation superior to 2
hours in the former, and an arrest in descent superior to 1 hour in the latter. Arrest disorder
resulting form midpelvic contraction (prominence of the ischial spines) is treated with
C.section.
31. Pts with placental abruption in labor must be managed aggressively to insure a rapid vaginal
delivery. Cesarean section is used only when there are obstetric indictions, or when there is a
rapid deterioration of the state of either the mother or the fetus and labor is in an early stage.
32. Aortic coarctaion presents with elevated blood pressure in the upper extremities but low BP in
the lower extremities. It is a common finding in Turner syndrome. Pts with Turner syndrome
have ovarian dysgenesis and poor ovarian function; FSH should be high due to negative
feedback.
33. Labor should be allowed to proceed in pts where the fetus has been diagnosed with a severe
congenital anomaly incompatible with life.
34. Threatened abortion is characterized by any hemorrhage occurring before the 20th week of
gestation with a live fetus and a closed cervix.
35. Reassurance and outpatient follow up is the standard of care for threatened abortion.
36. Tx for vaginismus is effective with success rates of 80% or better. Typically, tx includes
relaxation, Kegel exercises (to relax the vaginal muscles), and insertion of objects of
gradually increasing size to encourage desensitization. Self-stimulation techniques are useful
for primary anorgasmia. Referral ot a sex therapist is usually indicated for pts with hypoactive
sexual desire.
37. Paresthesias or pain in the distribution of the median nerve during pregnancy is likely to be
due to carpal tunnel syndrome (CTS). Initial txs for CTS involve neutral position wrist
splinting and NSAIDs. Local corticosteroid injection is indicated in cases where wrist
splinting is insufficient to relieve symptoms. Surgical decompression of the carpal tunnel is
reserved for cases when conservative mx fails.
38. In the presence antepartum hemorrhage, pelvic examination must not be done before ruling
out placenta previa. Placenta previa presents with painless third trimester vaginal bleeding.
39. The MCC of excess postpartum blood loss is uterine atony. Initial tx includes bimanual
uterine massage, fluid resuscitation, uterotonic agents (oxytocin, methylergonovine,
carboprost), and blood transfusion as needed. Uterine packing for tamponade is performed if
medical therapy fails and in conjunction with preparations for surgery.
40. The single MCC of postpartum hemorrhage is uterine atony. Risk factors for uterine atony
include multiparity, prolonged labor and any condition that results in uterine hyperdistention.
41. Maintenance of airway, breathing and circulation is always the first priority in any pt
presenting for emergent care. Hemodynamic resuscitation must be promptly initiated before
starting any measure ot diagnose the source of blood loss in pts with antepartum hemorrhage
who are hemodynamically unstable.
42. The MCC of mucopurulent cervicitic is Chlamydia trachomatis.
43. PID dx criteria: a fever >38C, leukocytosis, elevated ESR, purulent cervical discharge,
adnecal tenderness, cervical motion tenderness and lower abdominal tenderness.PID is most
commonly caused by N.gonorhoeae, C. trachomatis, and genital mycoplasmas. It is managed
with empirical wide-spectrum antibiotic therapy; in hospitalized pts, regimens include

219
Cefoxitin/Doxycycline, Cefotetan/ Doxycycline and Clindamycin/gentamicin (all iv). The pt’s
partner should also be treated. In non-hospitalized pts, Cefoxitin (IM) is concurrently used
with Probenecid (PO) plus Doxycycline (PO), or Ceftriaxone (IM) and Doxycycline (PO) are
administered.
44. Semen analysis is a simple test that helps to identify male factor as the cause of infertility.
Semen analysis should be performed early in the evaluation of the infertile couple, usually as
the initial screening test.

1/27/10
1. The maternal quadruple screen will return increased levels of beta-hCG and inhibin A,
decreases levels of maternal serum AFP and estriol in cases of anruploidy. These findings
carry an increased risk of Down syndrome but are not diagnostic of Down syndrome.
2. Cervical dysplasia in a high risk (post-adolescent, premenopausal) pt should be investigated
with colposcopy. (Ref figure 1/25 No.15)
3. Chorioamnionitis refers to infection of the amniotic fluid. The infection is typically
polymicrobial. Maternal fever, tachycardia (>100bpm) and leukocytosis, fetal tachycardia
(>160bpm), uterine tenderness and a foul uterine discharge indicate this dx. Subclinial
infection is confirmed by amniotic fluid analysis (amniocentesis for gram staining, culture and
sensitivities and other studies). Fetal complications of intraamniotic infection include preterm
labor and an increased risk of neurodevelopmental delay and cerebral palsy.
4. All pts who take metronidazole shoud abstain from drinking alcohol, as it is associated with a
disulfiram-like reaction.
5. Adenomyosis occurs most frequently in multiparous women above 40 years of age and
typically presents with dysmenorrheal and menorrhagia. The physical exam reveals an
enlarged and generally symmetrically uterus. In contrastm a fibroid uterus is usually
irregularly shaped.
6. Given the high rate of co-infection with Gonorrhea and Chlamydia and the poor sensitivity of
gram stain for Chlamydia, antibiotic mx must be aimed at eradiction of both organisms when
gram stain confirms cervical Gonorrhea infection (Ceftriaxone and azithromycin). Amipicllin
and gentamicin is indicated if there is a suspicion for polymicrobial infection such as post
partum endometritis.
7. Inevitable abortion is characterized by vaginal bleeding and fluid discharge, low abdominal
cramps and a dilated cervix through which the products of conception can occasionally be
visualized.
8. Thichomonas infection usually causes vaginal discharge, prutitus, dysuria, and dyspareunia,
though it can also be asymptomatic. Regardless of the presence of symptoms, both partners
should be treated with oral metronidazole to prevent recurrent infections.
9. The increased incidence of urinary tract infections in women is due to the shorter length of the
female urethra compared to males. Other predisposing factors for UTI’s include sexual
intercourse, recent antibiotic use, the use of spermicidal contraceptives, and a close proximity
of the urethra to the anus.
10. Fetal hydantoin syndrome presents with small body size, microcephaly, digital hypoplasia,
nail hypoplasia, midfacial hypoplasia, hirsutism, cleft palate and rib anomalies.
11. Cesarean delivery should be performed on women with active genital herpetic lesions

220
(primary or secondary) in order to reduce the risk of neonatal HSV, a severely morbid and
potentially fatal illness.
12. Ureine rupture presents with intense abdominal pain associated with vaginal bleeding which
can range from spotting to massive hemorrhage. Pts also typically exhibit vital signs
consistent with hypovolemia, retraction of presenting parts one pelvic exam, and palpability
of fetal extremities on abdominal exam.
13. A breech presentation diagnosed before the 37th week of gestation does not require any
intervention, as breech presentations often convert to vertex before the 37th week. External
cephalic version is indicated if breech presentation is persistent after 37 weeks, and if this
fails, then planned cesarean delivery is indicated.
14. The best mx of intrauterine fetal demise is delivery of the fetus to reduce the risk of infection
and coagulopathy. Tx options should be discussed with any pt prior to embarking on a plan so
the pt can make an informed decision about therapy with knowledge of the risks and benefits
of all options.
15. A hysterosalpingogram is a radiographic test used to identify structural abnormalities in the
uterus or fallopian tubes that may contribute to infertility. The pt has regular menstrual cycles
accompanied by midcycle pelvic pain (mittelschmerz) and breast tenderness. These facts
indicate that the pt is most likely ovulating, so testing for ovulation is not likely to identify the
cause of infertility.
16. Cigarret smoking is the most common preventable cause of fetal growth restriction in the US.
17. Fetal alcohol syndrome is characterized by growth restriction, midfacial hypoplasia, a smooth
philtrum, short palpebral fissure, a thin upper lip and CNS abnormalities.
18. Bilateral solid ovarian masses discovered incidentally in a multiparous African-American
pregnant woman most likely signify a pregnancy luteoma. This is a benign self-limited
condition and requires no tx.
Sudden onset hirsutism or virulization during pregnancy

Physical examination
Pelvic ultrasonogram
↓------------------------------↓------------------------------↓---------------------------↓
No ovarian mass Bilateral cystic Bilateral solid Unilateral solid
↓ ↓ ↓ ↓
Abdominal CT scan to Theca Lutein cysts Mostly pregnancy Laparptomy or
rule out adrenal mass (Rule out high-β -HCG states) luteoma Lapaoscopic biopsy to
rule out malignancy
Solid ovarian tumors (viewed by ultrasonography) are almost malignant and demand immediate
and aggressive evaluation and treatment in all age groups, except in pregnancy.
19. Hypogonaotropic hypoganadism is a decrease in circulating sec hormones due to decreased
concentrations of LH and FSH. This condition increases a pt’s risk of osteoporosis. Common
causes include steneous exercise, anorexia nervosa, marijuana use, starvation, stress,
depression, and chronic illness.
20. A menstrual diary for at least 3 cycles is useful aid for comfirning the dx in suspected cases of
PMS. Confirmation of the dx must be made before initiating tx. There is no universally
accepted tx. Reduction of caffeine intake may reduce breast symptoms. An exercise program

221
may be effective in improving the general well being of the pt. In women whose symptoms
are more severe and cause socioeconomic dysfunction, SSRIs are the drug of choice. When
SSRI fail to alleviate symptoms in such pts despite therapy over multiple cycyles, low dose
alprazolam is indicated. Relaxation techniques and bright light therapy have some proven
effect in mx of PMS, but cognitive behavior therapy and insight oriented and supportive
psychotherapy do not play a role. Tx should not be initiated until the dx is made.
21. The tx of choice for bacterial vaginosis in both pregnant and non pregnant pts is oral
metronidazole. Vaginal metronidazole and clindamycin are alternatives.
22. RhoGAM is indicated in unsensitized, Rh-negative women at 28 weeks gestation and withn
72 hours of any procedure or incident such as abortion, ectopic pregnancy and delivery.
23. Asymptomatic pts with Chlamydia detected by screening should be treated with azithromycin
or doxycycline, but concurrent tx for Gonorrhea is not needed if screening for Gonorrhea is
negative.
24. ACEIs and ARBs are C/I in pregnancy. The first-line agents for mx of essential hypertension
during pregnancy are labetalol and methyldopa.
25. Low back pain is a very common complaint in the 3rd trimester of pregnancy. It is believed to
be caued by the increase in lumbar lordosis 脊柱前弯症 and the relaxation of the ligaments
supporting the joints of the pelvic gridle.
26. PCOS is characterized by anovulation or oligo-ovulation, signs of androgen excess, such as
male-pattern hair growth and acne, and ovarian cysts.
27. Mx of placenta previa depends on the severity of bleeding and the age the pregnancy.
Regardless of gestational age, pts with active, uncontrolled antepartum hemorrhage, who also
typically exhibit unstable vital signs and unreassuring fetal heart rates, should undergo
emergent caesarian section delivery.
28. Voiding after intercourse has been shown to decrease the risk of urinary tract infection in
sexually-active females.

1/28/10
1. Acute bacterial parotitis presents with painful swelling of the paroid gland that is aggravated
by chewing. High fever and a tender, swollen and erythematous paroid gland are common.
This post-operative complication can prevented with adequate fluid hydration and oral
hygiene. The most common infectious agent is Stapylococcus aureus.
2. The rotator cuff is formed by the tendons of the supraspinatus, infraspinatus, teres minor and
subscapularis muscles. The supraspinatus is most commonly injured, due to repeated bouts of
ischemia near its insertion on the humerus induced by compression between the humerus and
acromion. A common cause of acute rotator cuff tears is a fall on outstretched hands. Pts
usually have severe shoulder pain and edema following the traumatic event and are unable to
abduct the arm past 90 degrees. The drop arm test is a maneuver that can help to dx a rotator
cuff tear. Here, the pt’s arm is abducted passively to greater than 90 degrees, and the pts is
then asked to lower the arm slowly. With a complete rotator cuff tear, the pt will unable to
lower the arm smoothly and it will appear to drop rapidly from near the 90 degree position.
Rupture of the tendon of the long head to the biceps produces a positive “Popeye sign” where the
biceps muscle belly becomes prominent in the mid upper arm. Weakness with supination is
prominent, and forarm flexion is typically preserved.

222
Injury to the long thoracic nerve causes a winged scapula due to paralsis of the serratus anterior
muscle. The most common cause is iatrogenic injury during axillary lymphadenectomy.
The low (inferior) trunk of the brachial plexus originates from the C8 and T1 cervical roots. Injury
to this trunk, which typically results from sudden upward pulling on the arm, produces Klumpke’s
palsy. This palsy primarily affects muscles innervated by the ulnar nerve, which supplies most of
the intrinsic muscles of the hand. Weakness and atrophy of the pothenar and interosseous muscles
characterize this palsy, and a “claw hand” deformity may also result.
A humeral neck fracture would be more likely to present with swelling, acchymosis and crepitus
over the fracture. Axillary nerve injury may present.
3. Atelectasis is a common post-surgical complication that results from shallow breathing and
weak cough due to pain. It is most common on post-op day 2 and 3 following abdominal or
thoracoabdominal surgery. Adequate pain control and incentive spirometry decrease the
incidence of post-operative atelectasis.
Diaphragmatic paralysis may occur as a consequence of phrenic nerve injury from thoracic
surgery, cervical manipulation, or tumor compression but is less likely during abdominal surgery.
Unilateral diaphragmatic paralysis is often asymptomatic, while bilateral diaphragmatic paralysis
causes hypoxemia, repid shallow breathing, orthopnea, and even respiratory failure.
4. Radial nerve is the most commonly injured nerve in association with fracture of midshaft
humerus.
5. When there is an intimal flap of the carotid artery, surgery is recommended to repair the
vessel. Intimal flap injury can lead to vessel occlusion and symptoms and signs of ischemia.
Today, with the availability of stenting, this may be an alternative option to surgery.
6. The femoral nerve innervates the muscles of the anterior compartment of the thigh, and is
therefore responsible for knee extension and hip flexion. The femoral never provides
sensation to the anterior thigh and medial leg via the saphenous branch.
7. Inflammatory breast cancer is an uncommon form of breast cancer that presents with an
erythematous and edematous cutaneous plaque overlying a mass on the breast commonly with
axillary lymphadenopathy. 1/4 of pts with this condition will have metastatic disease at the
time of presentation. Clinically, inflammatory breast cancer can not be differentiated from
infectious process, such as a breast abscess, with 100% certainty. Therefore, a biopsy for
histology should be done first to exclude or confirm that dx.
8. Extremities subjected to at least 4-6 hours of ischemia can suffer from both intracellular and
interstitial edema upon reperfusion (eg. removal of an embolus). When edema causes the
pressure within a muscular fascial compartment to rise above 30mmHg, compartment
syndrome occurs leading to further ischemic injury to the confined tissues. The clinical
findings are 5 “P’s”: Pain, Paresthesias, Pallor, Pulselessness, Paresis/Paralysis.
9. A pt, who is hypotensive with abdominal pain and has a CT scan showing an enlarged aortic
silhouette, has a dx of a ruptured abdominal aortic aneurysm. This pt needs immediate
surgery.
10. Peripheral artery aneurysms manifests as a pulsatile mass that can compress adjacent
structures (nerves, veins), and can result in thrombosis and ischemia. Popliteal and femoral
artery aneurysms are the most common peripheral artery aneurysms. They are frequently
associated with abdominal aortic aneurysms.
11. Pts who present more than five days after the onset of symptoms of appendicitis, and have

223
localized right lower quadrant findings, should be treated with IV hydration, antibiotics and
bowel rest. Antibiotics should cover enteric gram-negative organisms and anaerobes. A
second/third generation cephslosporin or a fluoquinolone plus metronidazole are usually used.
Cefotetan had a good coverage of gram-negative oraganisms and anaerobes.
12. Biliary colic occurs when the gallbladder becomes distended as it contracts against an
obstructed cystic duct. The pain is typically exacerbated by fatty meals and resolves
completely between episodes. Fever and leukocytosis are not present.
13. A breast mass that produces a non-bloody aspirtate and disappears completely on aspiration
does not need any further evalution other than observation for recurrence.
14. Fat necrosis shows clinical and radiographic findings similar to those seen in breast cancer
including skin or nipple retraction and calcification on mammography. Biopsy of the mass in
fat necrosis will reveal fat globules and foamy histocytes. No tx is indicated for this self-
limted condition. The calcification seen in breast malignancy tend, however, to
microcalcification wihile the clacifications seen in benign conditions such as fat necrosis tend
ot be coarse.
15. Pulmonary contusion represents parenchymal bruising of the lung, which may or may not be
associated with rib fractures. The clinical manifestations develop usually in the first 24 hours
(often with in few minutes); tychypnea, tachycardia, and hypoxia are characteristic. Physical
examination typically reveals chest wall bruising and decreased breath sounds on the side of
pulmonary contusion. Chest x-ray reveals patchy irregular alveolar infiltrate and a CT scan
may be employed to make an early dx. ABG typically shows hypoxemia and, itself, is an
indiction to suspect pulmonary contusion in trauma pts.
16. Acute shoulder pain after forceful abduction and external rotation at the glenohumeral joint
suggests an anterior dislocation, which may cause injury to the axillary nerve or artery.
17. Pts with Paget disease of bone are typically asymptomatic; the dx is made when an increased
bone-specific alkaline phosphatase is discovered on rountine laboratory testing. Pts who do
become symptomatic typically present with pain associated with fractures or with arthritis
resulting from bone deformity (hearing loss).
18. Osteogenic sarcoma (osteosarcoma) is the most common primary malignancy of bone. It most
commonly affects males in their second decade preferentially in the metaphyses of long
bones. Pts present with bone pain but do not have systemic symptoms (x-ray film shows an
osteolytic lesion of the distal femur along with periosteal inflammation).
Osteoclastoma (giant cell tumor of bone), is a tumor of adults that often involves the epiphysis of
the distal femur or proximal tibia. Radiology characteristically shows a soap-bubble appearance
due to the osteolytic quality of the tumor.
Ewing’s sarcoma is also most common in the second decade of life. It is s small, round, blue cell
(neuroectodermal) malignancy classically associated with systemic features such as fever, malaise
and weight loss. It typically affects the diaphyses of long bones as well as the spine and pelvis. X-
ray shows an osteolytic lesion with onion-skin appearance.
Chronic osteromyelitis may present with painful swelling, fever and laboratory abnormalities
including an elevated ESR and leukocytosis. Chronic osteomyelitis most commonly results form
contiguous spread of infection from an adjacent site, such as the skin. This is commonly seen in
diabetic foot ulces.
19. Spinal cord ischemia with lower spastic paraplegia is a rare complication of abdominal aortic

224
aneurysm surgery. The artery of Adamkiewicz arises from the aorta and supplies the anterior
circulation of the middle and lower spine cord. Diminished flow through this artery may
results from its thrombosis, ligation, or systemic hypotension. Resultant neurologic
dysfunction is due to ischemia of antero-lateral structures of the spinal cord. It includes lower
apastic paraplegia, pelvic organ dysfunction, and loss of temperature and pain sensation over
the lower extremities. Vibrotory and proprioceptive sensation is preserved because posterior
circulation of the spinal cord is not affected.
20. In cases of suspected child abuse, physicians are required to thoroughly examine the child for
documentation of all potential signs of abuse. This includes a complete skeletal survey. Child
protective services should be notified of all suspected cases immediately, and the pt should be
admitted.
21. Bowel ischemia may complicate up to 7% of procedures on the aortoiliac vessles and most
commonly affects the distal left colon. Pts report dull pain over the ischemic bowel as well as
hematochezia. Colonscopy shows a discrete segment of cyanotic and ulcrated bowel.
22. A retrograde urethrogram should be the first step in mx of a suspected urethral injury. Foley
catheterization in the presence of a urethral injury will predispose the pt to abscess formation
and worsening of the urethral damage. Classic signs of posterior urethral injury include blood
at the urethral meatus, inability to void and a high-riding porstate on digital rectral exam.
23. Pancreatic cancer classically presents insidiously with a combination of constant visceral
epigestric pain radiating to the back, jaundice due to extrahepatic biliary obstruction, and
anorexia with weight loss. A peptic duodenal ulcer typically causes periodic epigastric pain
relieve by meals.
24. The central cord syndrome classically occurs hyperextension injuries in elderly pts with
degenerative changes in the cervical spine. Such a traumatic injury causes selectively damage
to the central portion of the anterior spinal cord, specifically the central portions of the
corticospinal tracts and the decussating fibers of the lateral spinothalamic tract. Central cord
syndrome is characterized by weakness that is more pronounced in the upper extremities than
in the lower extremities. This unique motor deficit occurs because the motor fibers serving the
arms are nearer to the central part of the cortisospinal tract. Rarely, a pt may also have a
selective loss of pain and temperature sensation in the arms due to damage to the
spinalthalamic tract.
The anterior cord syndrome should be suspected when there is bilateral spastic motor paresis distal
to the lesion. It usually occurs secondary to occlusion of the vertebral artery.
Posterior cord syndrome is associated with bilateral loss of vibratory and propioceptive sensation.
25. Tachypnea, tachycardia, jugular venous distension and tracheal deviation following blunt
chest trauma all suggest tension pneumothorax. This life-threatening emergency requires
immediate decompression with needle thoracostomy. Subsequent tube thoracostomy may be
performed to maintain lung expansion. In a pt who remains hemodynamically unstable
following decompression of a pneumothorax, pericardial tamponade shoud be suspected.
26. Gastric outlet obstruction is a clinical syndrome characterized by early satiety, nausea, non-
bilious vomiting, and weight loss, all of which can be the result of many difference disease
processes. In a pt with a history of acid ingestion, pyloric stricture is the most likely cause.
Esophageal stricture/dysmotility tends to present with dysphagia, which is not a presenting
symptoms of this pt. In addition, abdominal succession splash 液波震颤 is not a finding in those

225
with esophageal stricture/dysmotility.
27. In frail chest, multiple contiguous ribs are fractured resulting in an isolated segment of the
thoracic wall that exhibits paradoxical motion during normal respirtation. Pain control and
supplemental oxygen pressure ventilation is required in many pts with this injury. The
institution of positive pressure mechanical ventilation causes the flail segment to move
normally.
28. When a wound fails to heal after a prolonged period, biopsies should be obtained to ensure
that the ulcer has not degenerated into a squamous cell carcinoma. When SCC arises within a
burn wound, these unclers are known as Marjolin ulcers.
29. In a young individual who presents with a fleshy immobile mass on the midline hard palate,
the most likely dx is torus palatinus. No medical or surgical therapy is required unless the
growth becomes symptomatic or interferes with speech or eating. A pt with torus palatinus
will usually give a history that the lesion has been present for some time and will deny
tenderness. The thin epithelium overlying the bony growth will tend to ulcerate and be slow to
heal due to poor vascularity.
30. Aortoilliac occlusion (Leriche syndrome) is characterized by the triad of bilateral hip, thigh
and buttock claudication, impotence and symmetric atrophy of the bilateral lower extremities
due to chronic ischemia. Impotence is almost always present in men with this condition, in the
absence of impotence, an alternate dx should be sought. The pulse is soft or absent bilaterally
from the groin distally in this condition. Men with a predisposition for atherosclerosis, such as
smokers, are at the greatest risk of this condition. Because impotence is not uncommon in this
age group, and the complaints of hip and thigh pain with walking may also be attributed to
osteroarthritis, there is a risk of missing this dx if a thorough vascular exam is not performed.
31. Injury to a number of different structures within the knee, including ligaments and
cartilaginous menisci, can cause a popping or snapping sensation. Most meniscal tears in the
knee jount occur during a distinctly recalled acute knee injury, often associated with a
popping sensation. Subsequent joint swelling develops gradually, and is often not noticed until
the following day. This is in contrast to ligamentous tears, which may also be associated with
a popping sensation, but which cause rapid joint swelling due to hemarthrosis.
An anterior cruciate ligment (ACL) tear wound cause immediate swelling due to rapid
development of hemarthorosis. Moreover, pts typically experience immediated inability to weiht-
bear and then lasting knee instability following such an injury.
Chronic overuse, typically related to strenuous athletic activities, can cause patellar tendonitis, or
“jumper’s knee”. Physical exam typically reveals point tenderness over the proximal patellar
tendon.
The anserine bursa underline the conjoined tendons of the gracilis and semitendinosus muscles
and separtates them from the head of the tibia. Anserine bursitis causes tenderness over the medial
aspect of the knee, and typically affects athletes and obese middle-aged to elderly women. A
popping sensation is not typically reported.
Stress fractures do not result from discrete injuries such as falls or other trauma. Instead, they arise
insidiously secondary to repeat stress. Athletes, especially long-distance runners, are most
commonly affected. The tibia and fibula are most vulnerable.
32. Nasopharyngeal cancer (NPC, undifferentiated carcinoma of squamous cell origin) is most
common in pts of Mediterranean and Far Eastern descent. It is strongly associated with

226
Epstein-Barr virus infection. Pts with NPC often present with recurrent otitis media (resulting
from Eustachian tube obstruction by tumor), recurrent epistaxia, and/or nasal obstruction.
33. The tibia is the most common bone is the body to be affected by stress fractures, and the pts
affected by ths condition are most commonly competitive athelets. A stress fracture can occur
even in conditioned athletes after only four to six weeks of intense training. Pts who participte
in running sports classically obtain stress fractures in the distal third of the tibia on the
posteromedial border while pts involved in jumping sports, such as dancing, classically abtain
stress fractures in the middle third of the tibia on the anterior side of the bone. Pts typically
complain of pain with activity that improves with rest. If activity is continued, the pain may
persist during rest. Point tenderness to palpation over the fracture is present on exam. X-rays
are usually normal, but they may reveal periosteal reaction in the site of the fracture. The
injury is best defined radiographically using CT or bone scan.
Epiphysitis of the tibial tubercle would cause pain in the proximal tibia, near the knee, where the
iliotibila tract inserts.
34. Urinary calculi present as the flank or abdominal pain radiating to the groin, along with
nausea and vomiting. A non-contrast spiral CT of the abdomen and pelvis is the imaging
modality of choice (best!) for diagnosis. IVP was the test of choice in the past for diagnosing
urinary stones but, because of the risk of contrast-related reactions with IVP, non-contrast CT
is now the preferred test.
35. Scaphoid is the most commonly fractured bone among all the carpals. It is commonly seen in
young adults following a fall on the outstretched hand. Pt generally complaints of pain at wrist
joint. Tenderness in anatomical snuffbox is a very sensitive marker of scaphoid fracture.
Fracure is most commonly located across the waist of scaphoid bone. Initial X rays may be
normal or may show fine radiolucent line in non-displaced scaphoid fractures. Scaphoid views
are necessary to avoid missing the scaphoid fractures. Cast immobilization is recommended in
the tx of all non-displaced scaphoid fractures (fractures <2mm displacement and no
angulation). Open fraction and internal fixation is required if initial X ray shows fracture
displacement.
36. Absent bowel sounds with gaseous distention of both the small and large bowels indicates a
paralytic (adynamic) ileus (X-ray film of abdomen). This classically follows abdominal
surgery but may also occur in cases of retroperitoneal hemorrhage associated with vertebral
fracture.
37. Anterior cord symptom is commonly associated with burst fracture of the vertebra and is
characterized by total loss of motor function below the level of lesion with loss of pain and
temperature on both sides below the lesion and with intact proprioception.
38. The immediate mx of splenic trauma caused by blunt abdominal injury depends on the pt’s
hemodynamic status and reponse to IV fluid. If the pt is initially hemodynamically unstable
but improves with fluid administration, the best next step is to obtain an abdominanl CT scan.
If the pt is initially hemodynamically unstable and unresponsive to fluid administration, then
emergent exploratory laparotomy is requires. In the case of CT scan-documentd splenic injury,
the decision for operative intervention is determined by the grade of the injury. If operative
imtervention is required, every effort is made to repair the spleen rather than remove it,
especially in children. If removal is inavoidable, post-operative immunization against
encapsulated bacteria is mandatory.

227
39. In hemodynamically unstable pts who have suffered blunt abdominal trauma and pelvic
fracture, both intraperitoneal and retroperitoneal bleeding must be ruled out. Focused
Assessment with Sonography for Trauma (FAST) and diagnostic peritoneal lavage are the
appropriate tests for establishing intraperitoneal bleeding. If thest rests are negative, then the
next step in mx would be a pelvic angiography to search for retroperitoneal bleeding. A pelvic
angiography is not only useful in identifying the source of the bleeding, but the bleeding can
also be resolved via embolization of the offending vessel.
40. In a pt with blunt thoracic trauma, suspect the dx of flail chest if he remains tachypneic and
hypotensive despite aggressive fluid resuscitation. Classical x-ray findings reveal multiple rib
fractures overlying a coutused lung. (X-ray film!)
41. In stable pts with abdominal trauma, CT scan with contrast is the single best study to evaluate
solid organ damage. Diagnostic peritoneal lavage is indicated only in unstable pts, where time
is limited to get CT of the abdomen.
42. Subluxation of radial head (Nursemaid elbow) is a common condition in preschool children
and needs closed reduction by flexion and supination of forearm. The child is usually not in
distress at presentation but would cry at any attempt to flex the elbow or supinate the forarm.
Dx is made clinically as radiographs are often normal.
43. It is important to rule out a fracture or dislocation of cervical spine as the first priority because
of grave consequences of missing a cervical spine injury. This pt has epistaxis suggestive of
fracture of base of skull. However, a CY scan is indicated only after cervical spine injury is
ruled out and pt is stable. An alternative could be to do a CT scan of head including the neck.
44. Esophageal rupture most commonly occurs following instrumentation of the esophagus. Less
commonly it may occur in pts with protracted vomiting who have been resisting the urge to
vomit. Rupture in these cases typically occurs into the mediastinum resulting in
pneunomediastinum.

1/30/10
1. Oliguria, azotemia and an elevated BUN/creatinine ratio of >20:1 in the post-operative state
most likely indicate acute pre-renal failure from hypovolemia, though urinary catherter
obstruction should be first be ruled out. The next step in the dx/mx of acute renal failure
manifesting as oliguria or anuria is an IV fluid challenge.
2. Suspect cardiac tamponade in an adult pt who has sustained blunt chest trauma if he has
persistent jugular venous distention, tachycardia, and hypotension despite aggressive fluid
resuscitation. CXR findings typically reveal a normal cardiac silhouette without tension
pneumothorax. DD: lung contrusion, aortic rupture, esophageal rupture, bronchial rupture
3. Esophageal perforation due to iatrogenic cause is very frequent and radiography with water-
soluble contrast is the best way to diagnose esophageal perforation.
4. For hemarrhagically unstable pts in whom blunt abdominal trauma is suspected, fluid
resuscitation should be initiated, followed by ultrasound examination. If ultrasound is not
definitive, then diagnostic peritoneal lavage is performed. Those pts with confirmed
hemoperitoneum whould then undergo laparotomy for surgical repair.
5. Gental traction to attempt alignment of the fragments of a fractured long bone is important to
prevent further vascular and neurological damage and it should be attempt immediately.
6. Severe blunt chest trauma can cause direct physical injury to the underlying pulmonary

228
parenchyma and pulmonary contusions. Signs and symptoms include dyspnea, tachynpea,
chest pain, hypoxemia worsened by iv volume expansion, and patchy, irregular alveolar
infiltrates on CXR. Aspiration of gastric contents has a history of having vomited.
7. Penile fracture is a surgical emergency that requires emergent urethral imaging and surgical
repair. The most common cause is a crush injury of the erect penis, most commonly during
intercourse where the female is on top of the male.
8. Femoral neck fractures are intracapsular fractures and are most commonly seen in
postmenopausal females and males over 70 years of age. The pt will classically present
following a fall with inability to ambulate and hip pain. On exam, the involved limb is
shortened and externally rotated. Active and passive range of motion of the affected limb
elicits pain. The dx can be made with AP and lateral X-rays of the hip, and classification of
the fracture using the Garden classification system may then be made based on radiographic
findings as follows: Type 1: Valgus 外翻的 impaction of femoral head. Type 2: complete but
non-displaced femoral neck fracture. Type 3: Complete fracture with displacement <50%.
Type 4: Complete fracture with displacement >50%. Type 1 or stress fractures can be
managed nonoperatively with toe-touch weight bearing on crutches until there is radiogical
evidence of complete healing. Garden types 2, 3 and 4 are unstable fractures and require open
reduction and internal fixation or a primary arthroplasty 关节形成术. Elderly pts and those
with severely displaced femoral neck fractures (type 4), are preferentially treated with
arthroplasty. Arthroplasty is preferred because the risk of avascular necrosis of the femoral
head is high, even following open reduction and intermal fixation, due to the tenuous blood
supply of the femoral head.
9. Mastitis associated with breast-feeding is treated with analgesics, antibiotics and continuation
of breast-feeding from the affected breast. The most commonly isolated organism is S. aureus.
The most commonly used antibiotics are dicloxacillin or cephalosporins.
10. The differential dx of a widened mediastinum as a solitary finding includes processes as
diverse as anthrax, various tumor, madiastinitis, aortic dissection, hemorrhage and large
pericardial effusion among others. Acute madiastinitis may occur following heart surgery.
Signs and symptoms include fever, chest pain, leukocytosis and mediastinal widening on
CXR. Mediastinitis is a serious condition that requires drainage, debridement and antibiotic
therapy.
11. The nature of this pt’s accident afn his scalp lacerations suggest that he has suffered blunt
head trauma, After initial fluid resuscitation, he lapses into a state of decresed consciousness
with right-sided hemiparesis. The mist likely explanation in this setting is a transtentorial
(uncal) herniation secondary to right-sided epidural hematoma. Uncal herniation mauy also
result from a subdural hematoma or an intracerebral mass.
Epidural hematomas result from rupture of the middle meningeal artery. Because they are under
arterial pressure, they expand rapidly. In this case, the fluid resuscitation likely increased the rate
at which the hematoma expanded, thereby precipitating the above neurologic signs. Focal
neurologic sings result form herniation of the parehippocampal uncus through the tentorial
incisure, which causes pressure on the ipsilateral oculomotor nerve and posterior cerebral artery as
well as compression of the contralateral cerebral peduncle against the edge of the tentorium. The
specific lesions and their respective signs are in the table.
Transtentorial (uncal) herniation

229
Lesions Signs
Comperssion of contralateral crus cerebri Ipsilateral herniparesis
against the tentorial edge
Compression of the ipsilateral oculomotor Loss of parasympathetic innervation causes
nerve (CN III) by the herniated uncus mydriasis (occur early)
Loss of motor innervation causes ptosis and
“down and out” gaze of the ipsilateral pupil due
to upopposed trachlear (CN IV) and abducent
(CV VI) action (occur later)
Compression of the ipsilateral posterior Contralateral homonymous hemianopia
cerebral artery
Compression of the reticular formation Altered level of consciousness, coma

12. Acute ruptured appendicitis may be complicated by pelvic abscess. Pelvic abscess typically
presents with lower abdominal pain, malaise, low-grade fever and a tender pelvic mass on
rectal examination.
13. Isolated duodenal hematoma most commonly occurs in children following blunt abdominal
trauma. It is treated conservatively with nasogastric suction and parenteral nutrition. Most
hematomas will resolve spontaneously in 1-2 weeks. Surgery may be considered to evacuate
the hematoma if this more conservative method fails.
14. The tx of pancreatitis is primarily supportive. Antibiotics may be used, but the evidence does
not presently support the use of antacids, antichoinergics, glucagan or octreoties. Identifying
the underlying cause may facilitate prevention of recurrent pancreatitis.
15. Legg-Calve-Perthes disease is characterized by avascular necrosis of femoral head. It
typically presents in children four to ten years of age with insidious-onset hip or knee pain and
an antalgic 镇 痛 的 gait. Physical exam shows marked limitation of internal rotation and
abduction at the hip joint. Proximal thigh atrophy may also be present. X-rays may be
negative initially, but serial studies will show changes in the femoral head consistent with
necrosis and recalcification. MRI and bone scans show findings suggestive of femoral head
necrosis much earlier than standard x-rays and may be used to make the diagnosis months
earlier. Tx is aimed at maintaining placement of the femoral head within the acetabulum so
that it may heal in the proper shape and position. This may be accomplished with splints or
surgery.
16. Unilateral suacute hip pain in a male child coupled with a progressive antalgic gait, thigh
muscle atrophy, decreased hip range of motion, and collapse of the ipsilateral femoral head on
plain pelvic x-rays are findings suggestive of idiopathic avascular necrosis of the femoral
capital epiphysis (Legg-Calve-Perthes disease.) (X-ray film!)
17. High-energy, blunt, rapid deceleration trauma to the chest commonly causes aortic injury. In
most cases of aortic rupture, death is the immediate result. In pts with a contained rupture, the
diagnosis must be made quickly. Widened mediastium and left-sided hemothorax are classic
CXR abnormalities pointing to this dx. In these cases, the diagnosis can be confirmed via CT
scanning. Mx of pts wht established aortic injury includes antihypertensive therapy where
appropriated and immediated operative repair.
18. Oxalate absorption is increased in Crohn disease and all other intestinal diseases causing fat
malabsorption. Increased absorption is the MCC of symptomatic hyperoxaluria and oxalate

230
stone formation. Under normal circumstances, calcium binds oxalate in the gut and prevents
its absorption. In pts with fat malabsorption, calcium is preferentially bound by fat leaving
oxalate unbound and free to be absorbed into the bloodstream. Failure to adequately absorb
bile slats in states of fat malabsorption also cause decreased bile salt reabsorption in the small
intestine. Excess bile salts may damage the colonic mucosa and contribute to increased
oxalate absorption.
19. Pts suffering esophageal perforation most commonly present with acute-onset substernal pain.
The diagnostic study of choice when esophageal perforation is suspected is a gastrografin
contrast esophagogram.
20. Pts with mild to moderate head injury can be discharged under the care of an adult if they
have a normal CT scan. The caretaker is given printed instructions detailing the signs and
symptoms that warrant immediate return to the hospital.
21. This pt presents with symptoms and signs suggestive of meniscal injury. The episode of injury
occurred one month ago, when acute pain and swelling were present. The current symptoms
suggest ‘locking’, a phenomenon that is due to impaired extension of the knee. In addition of
the pt’s history, a positive McMurry sign is present (popping sound on passive
flexion/extension of the joint), which is specific for meniscal injury. Persistent symptoms (eg.
mechanical symptoms) in pts with probable meniscal injury should be further evaluated by
MRI or arthroscopy. Surgery (arthroscopic or open) is often necessary to correct the problem.
22. Persistent pneumothorax and significant air leak following chest tube placement in a pt who
has sustained blunt chest trauma suggests traheobronchial rupture. Other findings include
pneumomediastinum and subautaneous emphysema. The dx can be comfirmed with high-
resolution CT scanning, bronchoscopy, or surgical exploration. Operative repair is indicated.
23. Bowel ischemia and infarction ar possible early complications of operation on the abdominal
aorta, such as abdominal aortic aneurysm reqair. This adverse effect can be minimized by
cheching sigmoid colon perfusion following placement of the aortic graft.
24. All pts with acute carbon monoxide poisoning should be treated with 100% oxygen via a
nonrebreather facemask. Early symptoms of carbon monoxide poisioning are typically
neurologic and include agitation, confusion and somnolence.
25. Positive pressure mechanical ventilation increases intrathoracic pressure, which decreases
venous return to the heart and thereby decreases the ventricular preload. In pts with
hypovolemic shock, this effect may cause circulatory collapse if the pt’s intravascular volume
is not replaced before mechanical ventilation is attempted.
26. Acute appendicitis is a clinical dx. Pts with a classic presentation should be operated on
immediately to remove the inflamed appendix and prevent appendiceal rupture.
27. The medial meniscus is typically injured during forceful torsion of the knee with the foot
planted. Pts complain of a popping sound and severe pain at eh time of injury; McMurray’s
sign is commonly positive on physical exam. A bucket handle tear of the medial meniscus
leads to locking of the knee joint during terminal extension. Because the meniscus is not
directly perfused, effusion following injury typically is not clinically apparent for many hours
following the injury.
In cases of anterior cruciate ligment tear, the pt gives a history of a forceful hyperectension injury
to knee or a noncontact torsional injury of the knee during deceleration. Effusion is seen rapidly
following injury. Lachman’s test, anterior drawer test and pivot shift test are used for clinical dx.

231
Posterior cruciate ligament injury is classically seen in the “dashboard injury”, which refers to
forceful posterior-directed force on the tibia with the knee flexed at 90 degrees. The posterior
drawer, reverse pivot shift and posterior sag 下垂 tests will help in clinical dx.
Medial collateral ligament injury is associated with abduction injury to the knee. The valgus stress
test will help in clinical dx.
Lateral collateral ligament injury is very rare and would be seen in adduction injury to the knee.
The varus stress test will help in clinical dx.
28. For hemodynamically unstable pts in whom blunt abdominal trauma is suspected, fluid
resuscitation shoud be initiated, followed by ultrasound examination. If ultrasound reveals
intraperitoneal blood, the pt should then undergo urgent laparotomy for surgical repair.
29. When a pt presents with a pulsatile abdominal mass and hypotension, a presumptive dx of
ruptured abdominal aortic aneurysm must be entertained and the pt should be taken
immediately to the operating room.
30. Postoperative pulmonary complications are common, particularly in pts with known risk
factors. These risk factors include smoking, preexisting pulmonary diasease, age>50, thoracic
or abdominal surgery, surgery lasting > 3 hours, and poor general health. Postoperative
measures such as incentive spirometry and deep breathing exercises are used to prevent such
complications and improve outcomes by promoting lung expansion.
31. A hydrocele is a fluid collection within the processus or tunica vaginalis—the peritoneal
projection that accompanies the testis during its descent into the scrotum. Hydrocele can be
differentiated from other testicular masses by transillumination; a hydrocele will
transilluminate while other masses will not. Most cases of hydrocele will disappear
spontaneously by the age of 12 months and can be safely observed during that period.
Hydroceles that do not resolve spontaneously should be removed surgically due to the risk of
inguinal hernia.
32. In a steady resting state the RQ (Respiratory quotient) depends upon the proportions of
metabolic fuels being oxidized for ATP production. A steady state RQ close to 1.0 indicates
predominant oxidation of carbohydrates and net lipogenesis. The RQ for protein and lipid as
sole sources of energy are 0.8 and 0.7, respectively. Assessment of the RQ is important when
attempting to wean pts from mechanical ventilation, as overfeeding, especially carbohydrates,
can cause excessive CO2 production and make weaning more challenging. This factor is
especially important in pts with preexisting lung disease.
33. A varicocele is a tortuous dilation of the pampiniform 蔓形的 plexus of veins surrounding the
spermatic cord and testis in the scrotum. A varicele results form incompetence of the valves of
the testicular vein and occurs most frequently on the left side, possibly because the left
testicular vein enters the left renal vein infeiriorly at a right angle thereby predisposing to
impaired drainage. Pts are commonly asymptomatic. Those who do complain of symptoms
may endorse a “dull” or “dragging” discomfort of the scrotum that is worse when standing.
On exam, the affected side of the scrotum will feel similar to a “bag of worms” and valsalva
maneuver will typically cause the mass to enlarge.
Cystic dilations of the efferent ductules (spermatocele) are painless fluid-filled cysts that contain
sperm. They are located on the superior pole of the testis in relation to the epididymis. These
masses are distinct from testix and classically transilluminate.
34. Complete small bowel obstruction usually present with nausea, vomiting, abdominal bloating,

232
and dilated loops of bowel on abdominal x-ray. Adhesions, typically post-operative, are the
most common etiology.
35. Arterial occlusion of an extremity will cause pain, pulselessness, pallor, paresthesias and
paralysis in the affected limb. Embolic occlusion classically causes sudden-onset severe pain
and asymmetric pulselessness.
36. Volkmann’s ischemic contracture is the final sequel of compartment syndrome in which the
dead muscle has been replaced with fibrous tissue.
37. Reflex sympathetic dystrophy is a vague painful condition seen as a sequel of infection or
trauma which may be minor. It is characterized by pain, hyperesthedia and tenderness, which
are out of proportion to the physical findings. It does not have an acute onset like in this pt
and is not associated with compartment syndrome. Sudeck’s atrophy is a radiographic term for
spotty rarefaction seen in pts with reflex sympathetic dystrophy.
38. Following splenectomy, pts are at increased risk for sepsis secondary to encapsulated
organisms like S. pneumoniae, N.meningitidis, and H.influenzae. Vaccinations against each of
these organisms should be administered. Pneumococcal vaccine boosters are required every 5
years.
39. An exploratory laparotomy is required in all pts with blunt abdominal trauma with signs of
peritonitis or hemodynamic instablility.
40. Three to four percent of pts with spinal cord injuries will develop post-trumatic
syringomyelia. Whiplash is often the inciting injury. Symptoms develop months to years later.
The condition involves enlargement of the central canal of the spinal cord due to CSF
retention, resulting in impairment strength and pain/temperature sensation in the upper
extremities or having a cape-like distribution, with preservation of dorsal column function.
MRI is used for definitive dx.
41. Drooping of the contralateral hemipelvis below its normal horizontal level during monopedal
stance constitutes a positive Trendelenburg sign. It is caused by weakness or paralysis of the
gluteus medius and minimus muscles, which are innervated by the superior gluteal nerve.
42. Blunt trauma to the upper abdomen can cause a pancreatic contusion, crush injury, laceration
or transaction. Pancreatic injuries may be missed by CT scan during the first 6 hours
following trauma. An untreated pancreatic injury can later be complicated by a retroperitoneal
abscess or pseudocyst.
43. Fat embolism presents with dyspnea, confusion and petechiae in the upper part of the body
and occurs after multiple fractures of long bones. Serial x-rays shows increasing diffuse
bilateral pulmonary infiltrates within 24-48 hours of onset of clinical findings.
44. Posterior dislocation of the shoulder may follow tonic-clonic seizures. In posterior shoulder
dislocation the arm is classically held adducted and internally rotated and fullness is palpable
on the posterior shoulder. Neurovascular impairment is unusual.

1/30/10
1. Pts treated with high-dose methylprednisolone within eight hours of spinal cord injury have
significant and sustained motor and sensory neurological improvement. Thus, its use is
warranted as the first priority after stablilizing a pt with clinical evidence of spinal cord injury.
All pts with clinical evidence of a spinal cord injury should be imaged with CT scan to assess
the bone and MRI to assess the cord and other soft tissues, such as the intervertebral disks. Tx

233
with iv corticosteroids should not be delayed for these imaging procedures.
2. A cystic lesion involving the pancreas on CT scan in the setting of acut pancreatitis with
systemic toxicity must be considered an abscess and should be treated with drainage. In the
absence of systemic toxicity, a pseudocyst may be presumed and magaged expectantly.
3. The first step in evaluating a pt with suspected peripheral artery disease (PAD) is to obtain an
ankle-brachial index (ABI). The ABI is calculated by dividing the systolic blood pressure
obtained by Doppler in the posterior tibial and dorsalis pedis arteries by that in the brachial
artery. Ratio of 1 to 1.3 are considered normal. An ABI less than 0.9 is highly sensitive and
specific for greater than 50% occlusion in a major vessel. ABIs less than 0.4 are consistent wit
limb ischemia. After PAD is diagnosed by ABI, a number of different imaging studies may be
performed to more accurately identify the occlude vessel.
4. TIG tetanus immunoglobulin, TT tetanus toxoid
History of tetanus toxoid Clean minor wounds All other woulds
immunization
< 3 doses of tetanus toxoid in TT: Yes TT: Yes
past TIG: No TIG: Yes
>= 3 doses TT: Yes if last does >10 years TT: Yes if last does >5 years
ago ago
TIG: No TIG: No
5. Diverticulosis is the MCC of hematochezia in an elderly pt. Chronic constipation resulting
from a low fiber diet is the most common predisposing factor for developing diverticulosis.
6. Any gunshot wound of the abdomen requires exploratory laparotomy. Any gunshot wound
belowt the 4th intercostal space (the level of nipple) is considered to involve the abdomen.
7. MRI is the investigation of choice for defining soft tissue injuries of the knee. Surgery is
rarely necessary for MCL (medial collateral ligament) tears; bracing and early ambulation is
the preferred tx. Injury to MCL leads to increased angulation (abduction) of the effected knee
on valgus stress.
8. Hypotension not responsive to fluid administration following trauma is suggestive of ongoing
occult blood loss. Pts in this scenario must be treated emergently with surgical intervention to
stop further hemorrhage.
9. Pregnancy test should be checked before ordering X-ray in any pt of childbearing age.
10. In a middle-aged adult, superficial unilateral hip pain that is exacerbated by external pressue
to the upper lateral thigh (as when lying on the affected side in bed) suggests trochanteric
bursitis.
11. In burn victims, clinical indicators of thermal inhalation injury to the upper airway and/or
smoke inhalation injury to the lungs include burns on the face, singing of the eyebrows,
oropharyngeal inflammation/blistering, oropharyngeal carbon deposits, carbonaceous sputum,
stridor, carboxyhemoglobin level > 10%, or history of confinement in a burning building. The
presence of one or more of these indicators warrants early intubation to prevent upper airway
obstruction by edema.
12. Pain relief should be the prime objective in the mx of rib fracture as it will allow proper
ventilation and prevent atelectasis and pneumonia. Oral agents, such as opiates and/or
NSAIDs are most commonly utilized, but an intercostals nerve block with a long-acting local
anesthetic can be used if oral or systemic analgesics are not sufficiently effective. Intercostal
nerve blocks provide pain relief without affecting respiratory function, as opiate analgesics

234
may, but it does carry some risk of pneumothorax.
13. All hemodynamically unstable pts with penetrating abdominal trauma must undergo
immediate exploratory laparotomy to dx and treat the source of bleeding, as well as to
diagnose and treat perforation of any abdominal viscus in an effort to prevent sepsis.
14. After blunt trauma to the chest, if an x-ray shows a deviated mediastinum with a mass in the
left lower chest, one should suspect a diaphragmatic perforation with herniation of abdominal
viscera. Typical symptoms include chest pain radiating to the shoulder, shortness of breath and
abdominal pai. Respiratory distress may follow. A barium swallow or CT scan with oral
contrast will be diagnostic. All diaphragmatic ruptures require operative repair.
15. Intraabdominal pathology causing pain in one or both shoulders suggests subdiaphragmatic
peritonitis. Among the possible blunt traumatic bladder injuries, only an intraperitoneal
rupture of the bladder dome could, by itself, cause a chemical peritonitis. The other lower
urinary tract structures are all extraperitoneal.
16. For any pt who suffers blunt deceleration trauma (motor vehivel accident or fall from >10
feet), blunt aortic trauma must be ruled out. CXR is the initial screening test, and widening of
the mediastium is the most sensitive finding.
17. Acalculous 非 结 石 性 cholecystitis occurs in critically ill pts. The clinical presentation is
vague and similar to calculous cholecystitis if the pt is able to interact. Imaging studies show
gall bladder distention, thickening of the gall bladder wall and pericholecystic fluid. The
emergent tx of choice is percutaneous cholecystostomy.
18. In pts with significant total body surface area burns, the major cause of morbidity and
mortality is hypovolemic shock. In the setting of adequate initial fluid resuscitation, bacterial
infection (usually bronchopheumonia or burn wound infection) leading to sepsis and septic
shock is the leading complication. Criteria for the systemic inflammatory response syndrome
(SIRS): 1. Fever to hypothermia 2. Tachypnea 3. Tachycardia 4. Leukocytosis, leucopenia or
bandemia.
19. If a pt develops a whistling noise during respiration following rhinoplasty, one should suspect
nasal septal perforation likely resulting from a septal hematoma.
20. The most commonly injured viscera in blunt abdominal trauma is the spleen, followed by the
liver and kidney. For hemodynamically unstable pts in whom blunt abdominal trauma is
suspected, fluid resuscitation should be initiated, followed by ultrasound examination (FAST-
focused Assessment with Sonography for Trauma).
21. The next investigational step in the work-up of a solitary pulmonary nodule detected on CXR
is usually high-resolution CT scan. The data obtained, when coupled with demographic data,
can be used to determine a pt’s percent risk of malignancy, which is necessary for determinig
further therapy.
22. All pts who have clinical signs of a scaphoid fracture following an injury likely to cause such
a fracture should be presumed to have a fracture (even if radipgraphs are negative) and treated
with thumb immobilization in a spica cast and reevaluation by x-ray in 2-3 weeks.
23. Hyperventilation helps to prevent and treat intracranial hypertension by causing cerebral
vasoconstriction and thus decreasing cerebral blood flow.
24. Tension pneumothorax is treated with immediate needle or tube thoracostomy.
25. Moving from supine to sitting can increase the functional residual capacity (FRC) by 20% to
35%. In normal adults, this can amount to several hundred cubic centimers of lung volume.

235
Increasing the FRC can help prevent postoperative atelectasis.
26. Following blunt trauma, cardiogenic shock with a high-normal initial pulmonary capillary
wedge pressure (PCWP) reading may be the result of a cardiac contusion. Tx is first with IV
fluid to bring the PCWP to between 15-20 mmHg. Following that, positive inotropic agents
should be used to increase cardiac output.
Major forms of shock in thoracic trauma pts MAP HR PCWP PCWP after fluid
challenge
Hypovolemic/Hemorrhagic ↓ ↑ ↓ U or ↑
Cardiogenic (LV dysfunction): contusion, MI, ↓ V ↑ ↑↑
traumatic valve or septal rupture, arrhythmia
Cardiogenic (RV dysfunction): contusion, MI ↓ V ↓, N U or ↑
Extracardiac obstructive: tension, ↓ ↑ ↓, N U or ↑
pneumothorax, massive hemothorax
Extracardiac obstructive: pericardial tamponade ↓ ↑ ↑ ↑ or ↑↑
Neurogenic ↓ ↑, N ↓, N ↑
Septic (Hyperdynamic) ↓ ↑ ↓, N ↑
MAP = mean arterial pressure; V=variable PCWP = pulmonary capillary wedge pressure
HR = heart rate; N = normal; U = unchanged

27. Non bleeding varices are managed with nonselective beta-adrenergic antagonists, such as
propranolol. Such therapy can reduce the risk of bleeding by up to half. Pt has cirrhosis and
PUD—conservative mx of his encephalopathy (oral lactulose), ascites (diuretics), portal
hypertension and PUD (PPIs). Sclerotherapy, endoscopic band ligation and surgery are
indicated after a pt has experienced a first episode of variceal bleeding, but these procedures
are not recommended for prophylaxis. A portosystemic shunt or TIPS procedure are
considered a last resort in variceal bleeding unresponsive to medical and endoscopic
interventions and may worsen the encephalopathy in this pt.
28. Deep vein thromboses occur when Virchow’s triad of stasis, endothelial injury, and
hypercoagulability are present. Major surgery is a significant risk factor for DVTs. Pts with
DVT should be treated with a heparin product acutely and warfarin for several months. The
goal of therapy is to prevent extension of the clot and development of future clots, not lysis of
the clot already present. Streptokinase and TPA are clot-lysing enzyme used to treat acute ST-
elevation MI. They are not generally recommended for the vast majority of DVT pts.
29. All pts with a clavicular fracture should have a careful neurovascular examination to rule out
injury to the underlying branchial plexus and subclavian artery.
30. Acute GI perforation requires emergent laparotomy. If an affected pt is on warfarin, then
reversal of anticoagulation must be rapidly achived pre-operatively by infusion of fresh frozen
plasma. VitK is not appropriate for emergency. In general, tissue oxygen delivery does not
become deficient until the hemoglobin drops below 7 g/dl. Transfusion (Packed RBC) is often
not required in pts with chronic anemia. Platelet counts greater than 50,000/mm3 provide
adequate hemostasis for most invasive procedures. Desmopressin (DDAVP) is given pre-
operatively to pts with mild hemophilia A in order to prevent excessive bleeding. It indirectly
increases factor VIII levels by causing VWF release form endothelial cells.
31. Orotracheal intubation and surgical cricothyroidectomy are preferred way to establish an
airway in apneic pt with head injury (obvious head and neck injuries). Needle
cricothyroidectomy is an excellent field procedure to establish an airway especially in
children. However, it is not suitable in adults due to risk of carbon dioxide retention especially
in pts with head injury where hyperventilation can be required to prevent or treat intracranial

236
hypertension.
32. The initial tx of both complete and partial small bowel obstruction is IV fluid resuscitation, IV
electrolyte replacement, NPO and NG tube gastric decompression. Surgical is typically
required ultimately in pts with complete obstruction in order to rule out and prevent
strangulation.
33. Venous insufficiency (valvular incompetence) is the MCC of lower extremity edema. It
classically worsens throughout the day and resolves overnight when the pt is recumbent.
34. Technetium-99 labeled erythrocyte scintigraphy should be used in cases of GI bleeding where
the source can not be identified. This study localizes the source of blood loss so that region
can be further evaluated and treated by colonoscopy or angiography.
35. Rupture of the duodenum is frequently diagnosed by noting retroperitoneal air on abdominal
x-ray. Duodenal injuries may then be better characterized using CT scan with oral contrast.
36. In case of amputation injury, amputated parts should be wrapped in saline-moistened gauze,
sealed in a plastic bag, placed on ice and brought to the emergency department with the pt.
37. When hemorrhage occurs, tachycardia and peripheral vascular constriction are the first
physiological changes. These responses act to maintain the blood pressure within normal
limits until severe blood loss has occurred.
38. Dumping syndrome is a common postgastrectomy complication. This process leads to the
fluid shift from intravascular space to the small intestine, release of intestinal vsoactive
polypeptides, and stimulation of autonomic reflexes. The symptoms usually diminish over
time and dietary changes are helpful to control the symptoms. In resistant cases, octreotide
should be tried. Reconstructive surgery is reserved for intractable cases.
39. A chest x-ray required following central line placement (ideally should lie in the superior vena
cava) to ensure proper positioning and absence of pneumothorax.
40. CT scan is the investigation of choice to dx an intraabdominal abscess. Musculoskeletal
infections, such as osteomyelitis or abscess, frequenly result from hematogenous spread of
organisms from another site, such as the skin. In such cases, S.aureus is the most common
offending pathogen.
AP and lateral lumbar films may be useful in making the dx of ureteral stones (if radioopaque),
bowel obstruction and perforated abdominal viscus. A lumbar x-ray may also disclose the
destruction of the vertebral bodies caused by spinal TB (Pott’s disease). Potts disease is a rare
cause of psoas abscess resulting from contiguous spread.
41. Edema, stasis dermatitis and veneous ulcerations result form lower extremity venous
insufficiency due to valve incompetence. Such disease classically occurs on the medial leg
superior to the medial malleolus.
Arterial thrombosis classically presents with a pt complaining of severe pain in a single ectremity.
The onset of pain is less acute than that seen in arterial occlusion due to embolus. The extremity
typically exhibits coolness to touch, pallor, pulselessness and paralysis on exam.
Raynaud disease/phenomenon is characterized by arterial spasm in response to cold or emotional
stress causing discoloration and discomfort of the distal digits. Raynaud phenomenon may
ultimately result in distal digital gangrene if severe.
Peripheral neuropathy predisposes to chronic ulceration resulting from unknown or neglected
injuries or chromic pressure on the soles of the feet. This most commonly occurs in the setting of
uncontrolled diabetes.

237
42. When circumferential full-thickness burns involving the extremities or chest are present, and
escharotomy may be necessary to prevent vascular compromise and respiratory difficulty,
respectively.
43. Perforated peptic ulcer causes sudden onset severe epigastric pain, which spreads over the
whole abdomen deu to chemical peritonitis. Upright abdominal x-rays classically show free
air under the diaphragm.
44. An acute blood loss in excess of 1500ml (25-30% in 70kg man) usually requires a blood
transfusion. The hemotocrit is a poor indicator of acute blood loss as it generally remains near
the normal level in the time period immediately following hemorrhage.

1/31/10
1. Parotid surgery involving the deep lobe of the parotid gland carries a significant risk of facial
nerve palsy resulting in facial droop.
2. In pts who have had at leasr 3 tetanus vaccinations (a complete childhood series), tetanus Ig is
not indicated following wounds, even if the length of time since the pt’s last vaccination is
greater than 10 years.
3. The possibility of aortic disruption must be considered in every victim of a severe
deceleration injury. Clinical signs can include hypertension of the upper extremities due to
pseudocoarctation (caused by compression of the aortic lumen by the aforementioned
hematoma) and a weak, hoarse voice due to pressure on the left recurrent laryngeal nerve by a
periaortic hematoma.
4. Posterior urethral injury is associated with pelvic fractures. Pts present with blood at the
urethral meatus, a high riding prostate, scrotal hematoma, inability to void despite sensing an
urge to void, and a palpably distended bladder.
5. Hymenoptera 膜 翅 目 昆 虫 stings account for more deaths in the US than any other
envenomation 毒液螫入 . Anaphylactic shock of any cause should be promptly treated with
subcutaneous epinephrine in order to acutely prevent respiratory distress and circulatory
collapse. Antihistamines are useful in facilitating relief of urticaria, angioedema and
bronchospasm as ancillary agents, but epinephrine (exerts both alpha and beta-adrenergic
effects resulting in vasoconstriction, bronchial smooth muscle relaxation and a decrease in
vascular permeability) should be given first.
6. This ileus is possibly due to a vagal reaction due to urethral colic. Needle shaped crystals on
urinalysis indicate uric acid stones. Uric acid stones, which are radiolucent, have to
beevaluated by either CT of the abdomen or iv pyelography. Ileus will be over when the
uretherolithiasis is treated. Stones less than 0.6 cm may pass spontaneously with hydration
and analgesia, otherwise instrumental intervention is needed.
7. The tx of choice for fracture of the femoral shaft is closed intramedullary fixation of fracture.
Plate and screw fixation is reserved only for pts who can not treated with intramedullary
nailing, such as victim of polytrauma who are unable to be placed on the fracture table.
8. The first step in the mx of atraumatic massive hemoptysis (TB) is bronchoscopy to
simultaneously localize and control the site of intrapulmonary bleeding. Bronchoscopy has the
additional benefit of offering increased control and protection of the pt’s airway. When the
underlying cause of hemoptysis is a vascular lesion, such as an arteriovenous malformation,
then pulmonary angiography and embolization might be necessary subsequent therapeutic

238
steps.
9. Acute adrenal insufficiency is a potentially lethal postoperative complication. Preoperative
steroid (lupus pt) use is the main cause. A high index of suspicion is required. Commonly,
they present with nausea, vomiting, abdominal pain, hypoglycemia and hypotension.
10. IV catherters are one of the MCC of nosocomial infections. Femoral central venous catheters
carry a higher risk of causing becteremia than subclavian catherters. IV catherter infections
are MCC by cutaneous organisms, such as Stapylococci, but femoral catheters may also cuase
G- bacteremia. Causes of postoperative fever—“5 W’s”: wind (po 1-2), water (3-5), walking
(4-6), wound (5-7) and wonder drugs (>7 days).
11. After blunt chest trauma, hemorrhagic shock associated with decreased breath sounds and
dullness to percussion over one hemithorax and contralateral tracheal deviation is most likely
due to large ipsilateral hemothorax.
12. CT scan of a diffuse axonal injury shows numerous minute punctuate hemorrhages with
blurring of grey-white interface. It is frequently due to traumatic deceleration injury. Pt loses
consciousness instantaneously and later develops persistent vegetative state.
13. Synovial fluid analysis is essential in managing monoarticular arthritis because the fluid
characteristics dictate treatment. For example, septic arthritis warrants antibiotic therapy and
surgical washout of the joint whereas other types of arthritis can be managed conservatively
with painkillers alone.
Synovial fluid Normal Noninflammatory inflammatory Septic
Appearance Clear Clear, yellow Clear ot opaque, Opaque, yellow
yellow to green
WBC/mcl <200 <2000 >2000 >2000, often
>50,000
Neutrophils (%) <25 <25 >=50 >=75
Glucose (mg/ml) Serum Serum Between 25 and <25
concentration concentration serum level
Gram stain/culture Negative Negative Negative Positive
Etiology No pathology Osteoarthritis, Rheumatic Bacterial
trauma disease, crystal infection
arthritis
In line with the table above, the indication for surgical washout are positive Gram stain or culture,
glucose <25 mg/dl, >75% neutrophils, or >50,000 leukocytes per mcl
14. Acute pain and swelling of the midline sacrococcygeal skin and subcutaneous tissues are most
commonly due to pilonidal disease. The acute presentation involves infection of a dermal
sinus tract originating over the coccyx. Tx is by drainage of abscess and excision of sinus
tracts.
15. Early recognition and tx of diaphragmatic hernia is extremely important since the mortality of
undiagnosed injury and subsequent bowel strangulation may be up to 30%. An elevated left
diaphragm may be the only abnormal CXR finding. Pts usually have respiratory distress and
deviation of the mediastinal content to the opposite side.
16. Stress (hairline) fractures of the metatarsals are not uncommon in athletes and military
recruits; the second metatarsal is most commonly injured. Tx is with rest, analgesia and hard-
sole shoe. Point tenderness over the affected metatarsal is present on exam. Fractures of the
2nd, 3rd, 4th metatarsals are managed conservatively because the surrounding metatarsals act as
splints and nonunion is uncommon. Plaster casting is used for pts with persistent pain

239
following more conservative tx. Surgical intervention is reserved for fractures of the 5 th
metatarsal, such as Jones fractures, or for displaced fractures not amenable to closed
reduction.
17. Tension pneumothorax is an emergency situation and needs immediate needle thoracostomy
based on clinical diagnosis.
18. Acute mesenteric ischemia classically presents with severe acute-onset midabdominal pain
out of proportion to physical examination findings. The MCC is an embolus from the heart.
Progression to bowel infarction causes peritoneal signs and the passage of bloody stool.
19. The risk factors for Developmental dysplasia of the hip (DDH) are Caucasian race, female
gender, first-born infants, breech position and a family history of DDH. The Barlow and
Ortolani tests (act to dislocate and relocate affected infant’s hips) are used to assess for hip
dislocatibility. Positive physical exam findings should prompt a radiologic dx with ultrasound
(< 4 months old) or radiographs (> 4 months old). Tx may involve use of a hip harness or
spica cast.
20. Pts with peripheral vascular disease typically have risk factor similar to those for heart disease
and present with claudication, impotence and skin changes. Ankle-branchial index (obtain
resting and post-exercise systolic BP in the ankle and arm) is the best tool for determining the
severity of disease.
Segmental volume plethysmography and Doppler ultrasound may be used to aid in determining
the location of a lesion causing peripheral arterial disease, but the dx should first be established
with ABI. Doppler ultrasound is also used in the dx of DVT.
21. CT guided percutaneous drainage is the standard tx approach for pelvic abscess. Surgical
drainage may be attempt if percutaneous drainage fails.
22. Diabetic foot ulcers result from neuropathy, microvascular insufficiency and
immunosuppression. They classically occur on the plantar surface of the foot under points of
greatest pressure, such as under the head of the first metatarsal bone.
23. History of a traumatic event and presence of crunching are compatible with fracture of
metatarsal.
24. A mammogram should be the first step in evaluation of all pts over age 35 with a palpable
breast lump. This is particularly important if there is a family history of breast cancer.
Physical examination alone is insufficient to differentiate benign from malignant lesions. All
suspicious lesions should then be biopsied.
25. Necrotized surgical infection is characterized by intensive pain in the wound, decreased
sensitivity at the edges of the wound, cloudy-gray discharge, and sometimes crepitus. Early
surgical exploration is essential.
26. Intermittent bloody discharge from one nipple is the classic presentation of intraductal
papilloma, a benign breast disease. Masses are generally not appreciated in this condition, as
the abnormality is small, soft, and located directly beneath the nipple. Ultrasound is best at
detecting masses greater than 1cm in diameter.
27. Postoperative ileus follows most surgeries where the peritoneal cavity is entered. Morphine
and other opiates compound this problem by decreasing GI motility. Signs and symptoms of
postoperative ileus include nausea, vomiting, abdominal distention, failure to pass flatus or
stool, and hypoactive or absent bowel sounds on physical exam. (In contrast, mechanical
bowel obstruction causes hyperactive “tinkling” bowel sounds).

240
28. Slipped capital femoral epiphysis (SCFE) typically occurs in obese early adolescent males.
Acute SCFE is an emergency condition and should be promptly corrected with external screw
in order to prevent avascular necrosis of the femoral head and chondrolysis.
29. Colicky or paroxysmal abdominal pain with episodic hyperactive bowel sounds attributable to
peristaltic rushes, nausea, vomiting, abdominal distention, diffuse abdominal tenderness and
obstipation are typical signs of complete small bowel obstruction (SBO). SBO is treated
surgically if the obstruction is complete or if signs of strangulation of a bowel loop are
present.
30. Hypotension, tachycardia, flat neck veins, confusion, and cold extremities despite iv fluid
resuscitation in a trauma pt indicates hypovolemic/hemorrhagic shock.
Hemorrhage Class I Class II Class III Class IV
classification
Blood loss (cc) 1000 1000-1500 1500-2000 >2000
Blood loss % <20 20-30 30-40 >40
Systolic BP Compensated Orthostatic Marked decrease Profound decrease
HR <100 >100 >120 >140
Respiratory 14-20 20-30 30-40 >35
rate
Urine output >30 20-30 5-20 Anuria
(cc/h)
CNS status Normal/anxious Agitated Confused Confused/obtunded
Capillary refill Normal Slight Significant Significant
delay/Cool delay/cool delay/cold

31. Air under the diaphragm indicates perforated viscus, which is a surgical emergency. Once
diagnosed, immediately obtain surgical consulation for emergent exploratory laparotomy.
32. Hemodynamic stability and the need for blood transfusion are the determinants that guide
surgical versus non-surgical mx of pts with splenic trauma. If a pt being treated with
observation experiences hemodynamic instability unresponsive to a 2L saline bolus or
otherwise requires a blood transfusion, then laparotomy is indicated for splenorrhaphy or
splenectomy. Following splenectomy, pts are at a greater risk of infections by encapsulated
organisms such as H.influenzae, S. pneumoniae, and N.meningitidis.
33. Acute epidural hematoma has a classic presentation of uncomsciousness followed by a lucid
interval followed by gradual deterioration of consciousness. CT scan is diagnostic and it
shows a biconvex hematoma.
34. Pts with compartment symdrome complain of severe pain, pain with passive range of motion,
paresthesias as well as pallor and paresis of the affected limb. The presence of pulses does not
rule out compartment syndrome. Fasciotomy is the tx and must be done urgently.
35. Acute cholecystitis presents with right upper quadrant pain, fever and leukocytosis. The
majority of pts with acute cholecystitis may be treated with observation and supportive care
initially, followed by elective cholecystectomy within a few days of the same hospitalization.
Emergent cholecystectomy is required for pts with biliary gangrene or perforation.

241

You might also like